You are on page 1of 533

http://www.mathematica.gr.

Leonardo da Vinci
(32-) . 30
. -
-
- quasiregular ,
( - ).
(0, 0, ),



12 ,
, 1+
,
2
2

1+ 5
.
2

:http://en.wikipedia.org/wiki/Icosidodecahedron
:

mathematica.gr
(http://www.mathematica.gr) .

mathematica.gr

11. (swsto)

12. (achilleas)

1. (Mihalis_Lambrou)

13. ( )

2. (nsmavrogiannis)

14. (Demetres)

3. ( )

4. (k-ser)
5. ( )

1. (spyros)

6. (m.papagrigorakis)

2. (vittasko)

7. ( )

3. (p_gianno)

4. (kostas.zig)

1. (grigkost)

5. (exdx)

2. (cretanman)

6. ( )

1. ( )

7. (mathxl)

2. (lonis)

8. (mathnder)

3. ()

9. (mathematica)

4. (nkatsipis)

10. (rek2)

5. ( )

11. (hsiodos)

6. (chris_gatos)

12. (A.Spyridakis)

7. (gbaloglou)

13. ( )

8. (R BORIS)

14. (bilstef)

9. (Rigio)

15. ()
16. (xr.tsif)

10. (dement)

http://www.mathematica.gr

14 ( )
5
3
2
P (x) = x + x + x 6x 12
x2 1

1. +
1 ( )

x1 , x2 , x3 . y1 , y2 , y3
x1 y1 + x2 y2 + x3 y3 .
y1 , y2 , y3
x1 , x2 , x3 .
;

7 ( )
:

1.


2.

2
52 6

4+

15 +

A= + +

4 ( qwerty) 417 530 ;

= 102450 , = 5200 , = 24360


6 ( )

360 36h = 123 (1 h) 34567 .

15 (
)

 = 90 .
E = 8, ZB = 6, ZB
AB.

9 ( )
x + y1 = 1 y + z1 = 1
xyz .
)

16 (
)

4
1. .

11 (
)
( ),

.
,,

,,.
12 ( )
= 15o ,
= 30o .
AB B
B , B =
.
A. BA

|x2 + x 2| = x3 1 |x + 1|.

3 ( ) 2001, 2002, 2003, , 2010


-
,
, 40
;

;




4 15 2 3 5

10 (

5 (
:

2
5+2 6

8 ( ) 2 =
2 + 2 , :
3

2 ( )

: ,
, 10
.

.

;

2.
P (x) x2 1

13 ( ) a >

0 a RQ,
1, a, a
(
).

17 ( )
.

  

AB A A = A B AB


.
18 ( )




2
+  = 6




+ 2  = 18 24




+  = 22 ,


1. 
+ .

2.

http://www.mathematica.gr


3.

22 ( )
z

|z 4i| 2
|z 2i| = 2 .


z .
19 ( )

t = 0 x(t) =
t(t 4)2 + 3 t sec
x(t) m. :
1. t
.
2.


.
3.

.
4.

.

f (x) =

2 (x )2 ,

x +
2

3
2

x [0, )
x [, 3]

1. f
2.

, ,

21 ( math68)
:

, ,
,

23 ( )
f : R R 1-1
:

f (x2 ) (f (x))2

1
4

24 ( )
x
0

1
1, f (x) + f 1x
.
= 2(12x)
x(1x)
f .

, ,

26 ( ) f
[a, b], f (a) = a, f (b) = b

f (x) 1 x
(a, b), f (x) = x x
[a, b].

27 ( )

1. z1 , z2
.
2.

25 ( )
f : R (0, +)
:
f = f f;

,
.

K = z12012 z22015 + z1 2015 z22012


3. a =
z12012 z22015 b = z1 2015 z22012

.

G(x) =

x
0x

0x

f (t)dt , x [0, 1]
g(t)dt , x [0, 1]
f (t)G(t)dt , x [0, 1]

1. x (0, 1]
:
H(x) < F (x)G(x) .
2. x (0, 1)
:
H(x)
< H(1)
< 1.
F (x)
F (1)

1

3. H(1)+ g(t) F (t)dt = F (1) + G(1),


0

(0, 1)
:
4.

f ()
g()

()
= 1F
1G()

H 2 (x)
F (x)G(x)
x0+

lim

30 ( )

f (x) = (x ) ( x) , x R,

, ,

z 2 ( + 2) 3z + 5 + 6 = 0

F (x) =
H(x) =

20 ( )

29 (
)
f, g
[0, 1] : f (x) > 0, 0 < g(x) < 1,
x [0, 1]. :
F , G, H :

/2
0

sin x
1 k2 sin2 x

< , .
1. x (, )

f  (x)
.
=
+
f (x)
x
x

2. f
Rolle
[, ] f 

= +
+ +
.
3. I(, ) =

I(, ) =

f (x) dx.

I( + 1, 1) .
+1

4. , ,
, , , ,
: Cf ,
x = , x =
x x.

, ,

dx

k (0, 1).
28 ( xgastone) f ,
(0, )

f (/2)
= 0

f (x) =
1 + f 2 (x) . f .

, ,

31 ( )
z1 , z2 C |z1 | = |z2 | = 0.

:



xy = z1
xy = z2

32 ( )
f : (0, +) R
f (x) > 0

f (x) = x +
x (0, +).

f (x)
x

t
dt
2t

http://www.mathematica.gr

. Juniors

46 ( )

xy + y = y x + x .
33 (
)

9

;

39 ( Putnam 2009) A, B, C
A . (A B)C = BA1
C(A B) = A1 B .

34 ( )
: a, b, c , : a+ 1b = b+ 1c =
c + a1 = t, t R, :
t = abc.

40 (
)

f : [0, 1] R :

. Seniors

(1) f

[a, b] [0, 1].


(2) c R, f 1 (c) = {x
[0, 1] : f (x) = c} .
f .

35 ( )

(
),

.
36 ( )

0, a1 a2 a3 ... .

a1 , a2 , a3 , ... ;

41 ( ) n
N, ()
 


n
.
6k
k=0
42 ( giannis1990)
A M33 (R)
A2 + I = 0.

(i) = 3

(ii) AZ = EO
(iii) EOZ
38 (
2010) ABC
(O) I .
AI, BI, CI
(O), D, E, F, .
(O1 ), (O2 ), (O3 ),
ID, IE, IF ,
BC, AC, AB,
A1 , A2 B1 , B2 C1 , C2 , .
,
(O ).

47 ( )

y2
x2
+ 2 = 1.
2
a


B(x2 , y2 )
:

A(x1 , y1 )

 


y1 y2
  x1 x2

 x 1 x 2 y1 y2
 2 + 2 1+ 2 + 2 + 1 = 2 .
a

48 (
:

(1 + tan 1 )(1 + tan 3 ) (1 + tan 43 ) <


211 <
(1 + tan 2 )(1 + tan 4 ) (1 + tan 44 )

37 (
2010) AB
AB = B = .
.
BA A
AE = AO B
B BZ = BO.
EZ , :

()

43 (


n
ln(n!)
n

,
(1)
ln
H
n+1
n=1
+


, Hn =
.

n
k=1

1
nk

44 ( MoV )
: N
N .

49 ( )


:

R,
,
5 , .
50 ( )
f
(, ) lim f (x) = lim f (x) =
xa+

k, k ,
(, )
f  () = 0.

51 ( ) z
C |z 1| = 1,
z 2 .
52 (
)

z1 , z2 , z3 C |z1 | = |z2 | = |z3 | = r > 0,


45 (
)

51985 1
,
5100 .

|z1 z2 |2 + |z2 z3 |2 + |z3 z1 |2 9r 2 .


,
z1 , z2 , z3
.

http://www.mathematica.gr

http://www.mathematica.gr/forum/viewtopic.php?f=44&t=5362

http://www.mathematica.gr/forum/viewtopic.php?f=44&t=5362

x1 , x2 , x3 .
y1 , y2 , y3
x1 y1 + x2 y2 + x3 y3 .
y1 , y2 , y3
x1 , x2 , x3 . ;


: ,
, 10 .

.
;

1
y1 = 106 , y2 = 103 y3 = 1.
9- a1 b1 c1 a2 b2 c2 a3 b3 c3
x1 = a1 b1 c1 , x2 =

1
x .
x 10
. : x + x
10 = 2x 10 .

a2 b2 c2 , x3 = a3 b3 c3 .

2
,
x1 , x2 , x3 .
, ! ,
, , .
..., , , ...
!
: y1 , y2 , y3
x1 , x2 , x3 (
, ),
x1 y1 + x2 y2 + x3 y3 , x1 , x2 , x3 .
y1 = log 2, y2 = log 3, y3 = log 5 .
x1 y1 + x2 y2 + x3 y3 = log 2x1 3x2 5x3
2x1 3x2 5x3 .
x1 , x2 , x3 ,
.
:
. :
, ..
G
odel
.


2x 10 10 .
: 2x 10 +
2x 10 10 = 4x 30 .
,
. 4x 30
.
: 4x 30 + 4x 30 10 = 8x 70 .
, ,
....( ..)
: 8x 70 = 0 x = 70/8 x = 8, 75 ..
8,75. (
), , :
2 8, 75 10 = 7, 5 . ,
: 4 8, 75 30 = 5 ..
2
:
3 10 , 0
2
5

http://www.mathematica.gr

10 3
5

15 2 7,5
1 10 7,5
17,5
8,75

2 10 5
1

3
http://www.mathematica.gr/forum/posting.php?mode=edit&f=33&p=54144

2001, 2002, 2003, ..., 2010


- ,
, 40
;
;

3
.
4 qwerty
http://www.mathematica.gr/forum/viewtopic.php?f=33&t=9362

417 530 ;

T-REX
10 , -
40 - 66.

417 530 = (22 )17 530 = 234 530 = 24 (230 530 ) = 24 1030 = 16.1030
30+2 = 32 .

 100
= 3300 = 33100 = 33
= 27100
25 < 27 < 32. 25100 < 27100 < 32100
<<

5
http://www.mathematica.gr/forum/viewtopic.php?f=34&t=230
Revista matematica din Galati
.

http://www.mathematica.gr/forum/viewtopic.php?f=34&t=2310

= 102450 , = 5200 , = 24360

360 36h = 123 (1 h) 34567


1 x 1.
1 (1 h) 1.
123 > 0


1024 = 210 , 243 = 35 .
:

 50
= 102450 = 210
= 21050 = 2500
200
=5
 60
= 24360 = 35
= 3560 = 3300

123 (1 h) 123
34567

123 34567 (1 h) 34567 123 34567


34690 (1 h) 34567 34444


 100

= 2500 = 25100 = 25
= 32100


100
= 5200 = 52100 = 52
= 25100

360 36h = 123 (1 h) 34567

http://www.mathematica.gr


, (1 h) 34567

-34690 -34444.
.

 


32
 

 2

2

5
1
5 1
z = 2 3 5 = 2
+

2
2
2
2 2 =

 2


5
1
5
1
2

=
2

2
2
2
2
2
   



5
3
5
3
5
1
x+yz =
+
+

2
+2
=
2
2
2
2
2
2
2

7
http://www.mathematica.gr/forum/viewtopic.php?f=35&t=8760

1.
2.

2
52 6

4+

2
5+2 6




15 + 4 15 2 3 5

1 Stavros11

5
2

 2
3
2

5
2


3
6)
5+2 6 52
=




(5+2 6)(52 6)

1:

x
=
4
+
15
+
4 15,

2( 5+2 6 52 6)

=
x2 = 10 x = 10 

2524
2 = 22 4 5
2: y = 10 2 3
5
,

y
= 2(5 + 2 6 5 2 6) =



 2

2
2
30

10
5
=
224
54
52 2 5 5 +
5 =
4
= 2( ( 3) + 2 3 2 + ( 2)





= 22 4 5 4 5 5 = 2,

( 3)2 2 3 2 + ( 2)2 ) =

y= 2


= 2( ( 3 + 2)2 ( 3 2)2 =


8
= 2( 3 + 2 3 + 2) = 2 4 = 4
1

52
6

5+2 6

2(

http://www.mathematica.gr/forum/viewtopic.php?f=35&t=218

2 = 2 + 2 , :

2 Stauroulitsa

A = 3 + 3 + 3

1: :


a

b=

a+c

a2 b =c2



x=

4+

15 =

4+1
2

41
2

ac
2


=

2


+

a3 + b3 + c3 = a(b2 + c2 ) + b3 + c3 =
ab2 + ac2 + b3 + c3 = b2 (a + b) + c2 (a + c) =

2

(a c)(a + c)(a + b) + (a b)(a + b)(a + c) =


41
5
3
4 15 = 4+1
2
2 =
2
2




(a + b)(a + c)(2a b c) .

12+8
z = 2 3 5 =
12 80 =
128
=
2
2

2
10 2
y=

c2 = (a b)(a + b) c3 = c(a b)(a + b).

x+yz =
+
+

10 + 2 = :


a3 + b3 + c3 = a3 + b3 + c(a b)(a + b) =
2 52 10 + 2 = 2


(a + b)(a2 ab + b2 ) + c(a b)(a + b) =



2 : x = 4 + 15 =
 
(a + b)(a2 ab + b2 + ac bc) =

 2
 2  2
2
2
2
5
3
5
3
+
+ 2 52 32 =
= (a + b)(b + c ab + b + ac bc) =
=
2
2
2 +
2
2


(a + b)[2b + c(a + c) b(a + c)] =
5
3
+
(a + b)[2(a2 c2 ) + c(a + c) b(a + c)] =
2
2
 

 2

(a + b)[2(a c)(a + c) + c(a + c) b(a + c)] =
2

5
3
5 3
y =
4 15 =
+

=
(a + b)(a + c)(2a c b)
2
2
2 2

5
2

3
2

5
2

3
2

http://www.mathematica.gr

:
1
1 x = y1 1 y = z1 yz
= (1 x)(1 y)
1
yz = (1 x) y(1 x).

9
http://whttp://www.mathematica.gr/forum/viewtopic.php?f=19&t=9469

x + y1 = 1 y + 1z = 1 xyz .

1
yz

1
y

=1xy

1
yz
= x. xyz = 1.

10
1
xy + y + z1 = xy + 1 = y(x + y1 ) = y , http://www.mathematica.gr/forum/viewtopic.php?f=19&t=4858
|x2 + x 2| = x3 1 |x + 1|.
xy + 1z = 0, xyz = 1.
2 ... y + 1z =
1 yz + 1 = z yz z = 1 z(y 1) = 1 , x3 1 = |x2 +x2|+|x+1| 0
y 1 = 1z x + y1 = 1 xy + 1 = y xy = x 1. x2 + x 2 = (x 1)(x + 2) 0
y 1 xy = 1z xyz = 1 : y + 1z = 1 |x2 + x 2| = x2 + x 2, |x + 1| = x + 1.
x + y1 = 1 xy + 1 = y xy + y + 1z = y xy + 1z = x3 1 = |x2 + x 2| + |x + 1| x 1
x3 1 (x2 + x 2) (x + 1) = x(x2 x 2) =
0 xyz = 1
3
x(x + 1)(x 2) = 0. , -1, 0, 2,
, :
2.

11

A1 = A2 = K1 = K2 =
= A = B.
(= )

http://mathematica.gr/forum/viewtopic.php?p=41418


( ),
.
,,
,,.

. 1 = 2 = 1 .

= B, O = O 1 = 1

1 = B1 . .

12
http://www.mathematica.gr/forum/viewtopic.php?p=44281

= 30o .
= 15o ,
AB B
B , B = A. .
BA
1
= 30
= 15 ,
B
, = .
, > ,
 = 120
. AE

 = 135 = 120 = 15 .
EAB

http://www.mathematica.gr

= 360 + 15 = 360 165 , Z


0 < < 135 = 15 .
3

= 30 AKB
= 60
. AK
.
.

= .
,
 = 15 .
BA
2
. : :

A
15
BE
A

=
=
.
15

BE

A
30
A
A

=
=
,
30
(15 + )
A
(15 + )

= 75 , .
AE
(--),
. (- = 15 .
), , BA
: : ,
(2 ), ,

30
1
215
15
:
=

(15 + )

(15 + )
15 + 15 = 2 15
15 15 = 0
(15 ) = 0 .
:

13
http://www.mathematica.gr/forum/viewtopic.php?p=57693

a > 0 a R Q,
1,
a, a
( ).


P (x) = x5 + x3 + x2 6x 12
x2 1

http://www.mathematica.gr/forum/viewtopic.php?p=57479

1. +
2. P (x)
x2 1

a1 + kd = 1 (1)

a1 + ld = a (2)
a1 + md = a (3).

1,
x2 1 =
(x 1) (x + 1) 1 1
x = 1 : x5 + ax3 +

(2) (1) (3) (1)

(k l)d = a 1 (m k)d = a 1.

kl
1, a .
mk

14

1, a, a k +
1, l + 1, m + 1 .

a=

a1
kl
=

mk
a1
9

http://www.mathematica.gr

x2 6x 12 = 0 1 + a + 6 12 = 0 a + =
17, (1). x = 1 : x5 + ax3 + x2 6x 12 =
0 1 a + + 6 12 = 0 a = 7, (2).
(1) (2) a + b = 17 b a = 7 b = 12
a = 5.
x5 + 5x3 + 12x2 6x 12 = (x2 1)(x3 + 6x + 12)
x3 + 6x + 12.

2,

(x) = ( 5)x + 12
(x) = x3 + ( + 1)x + .

, = 5
=
+
= 5 + 12 = 17
12.
(x) = x3 + ( + 1)x + = x3 + 6x + 12.

15
http://mathematica.gr/forum/viewtopic.php?p=45740

= 90 .
E = 8, ZB = 6, ZB
AB.





2 = 22 64 2

8  = 2  2 64

64 22 36 = 22 64 2 2 2 = 1152 (1).
:

2

2 + 2 = 64 2 + 2 = 80 (2).
(1), (2) 2 , 2
2 80t + 1152 = 0 2 < 2 :
: t
2
= 40 + 4 28.
3

x . B =

2x ZB
= B
= 45 ZB
= ZB
= .
ZB
.
ZH = H = 6 + y
Z = 2 (6 + y).
2
: x2 = y 2 + (6 + y) (1).

x
2 (6 + y)

= x2 = 8 (6 + y) (2).
8
2x
2
(1), (2)
: y + 2y 6 = 0
1
y =
7 1, (2)
, AE = = EBZ
2
E. = x = 40 + 8 7 ..
EB = 45 .
16

6
, (1)
(45 ) =
8 + ZE

http://www.mathematica.gr/forum/viewtopic.php?p=50444

4 1.
.

 =

ZE
, (2)
6

ZE = 2 7 4

E = a2 = 40 + 8 7

2


. =, =,
=.
BE = 2 AE = a 2.
: 1 Eukleidis
10

http://www.mathematica.gr

.
2

(AB) = 1 + 4 2


1
2 2 (AB) = 7
2

5
3
25 3
, (AB) =
,
+ = 7.
2 2
4
4

3

, .

( ).


, , 3, 3
. 1

= 27 . , = 7.
4

T1 = T2 , T3 = T4
7-

(AB)2 3

.
= 7 1 4 3 (AB) =
4

7.
6

. , ,
( ).
2, , 3, .
1

: 3 2 + = x x, 7a2 = x2 x = a 7.
7

.
.

2..120

= 2 3 , (1).
() =
2


() =
t(t a)(t 2a)(t x)
t t = 3a+x
2 ,

:
(9a2 x2 )(x2 a2 )

(AB) =(A) + (B) =

4 + 3 = 7 AB =

22

+ 2

2
3
2

5
.

() =
(2)
4
x > 3.
(1) (2)
x4 10a2 x2 + 21a4 = 0,
x2 = 3a2 x2 = 7a2 .

x2 = 7a2 , x = a 7.
8
,
Euler :

11

http://www.mathematica.gr


. .

.
2
( ) 3,
.
2 =

Euler : 22 + 2(2a)
2 2
2
2
2
2
2
(a 3) + x 10a = 3a + x 7a = x x = 7.
9
.

ABE, BCZ, CAD,



.
(ABE) = (BCZ) = (CAD) =
2(DEZ).

,
( =
),

(AB)2
(ABC)
=
= 7 =
(DEZ)
(DE)2

7, ( DE = 1 ) .
10

AB =

7
3
=
3
2 7
AP
( )
=
60

7
P
1
P
3

=
P = , = 2
3
3

). AP =

(ABC) = 7(DEZ),

2 7

http://www.mathematica.gr/forum/viewtopic.php?f=23&t=3019

   
AB A A = A B AB

.
-


= 0 A = AB A//AB
, = 0.

= 0, AB

A = 0 ABA A B = 0 AB.
.

17

18
http://www.mathematica.gr/forum/viewtopic.php?f=23&t=3569



AB A = A B =

: A = AB ( :

 = 
  , , = = 0).
12

2
+ = 6



+ 2 = 18 24



+ = 22 ,

http://www.mathematica.gr

4
2 + 4
+ 2 = 144 (1)
+ 2 = 12



2

+ 2
= 144
2 +
+ 2 = 144

4
+ 4 2 = 144 (2)

(1) (2): 4
2 + 4
+ 2 =
2 +

2 3
2 = 3 2
2 = 2
4
+ 4

2




2

|
| = |
| = (3)



2

+
=
+ = 8
+ = 64

64
2

2=

+ 2 = 64 2
= 64
2 +2
2 +2

= 32
2 +
2

2=

= 144
2 + 4
(1) 5

2 = 16
:
2

| = 4
|
| = 16 |



1.
+ .

2.

.
3.

1.








= 3
+ 3 = (2
+ ) + (
+ 2)






2
+  +
 + 2 24 24
62

2
62
24 +
24 0 6( 2) 0 = 2



 +  = 8











+ 2 = 12,
+ = 8 2
+ =
12,


2

+
= 144
12 2
+ = 144 2

2.

=2 : 2
+ =

3.

(3) |
| = 4 =
+ = 8 = |
| +

:
2. (t) = 0

19

... t =

http://www.mathematica.gr/forum/viewtopic.php?f=18&t=4737


t 0 x(t) = t(t 4)2 + 3
t sec x(t) m. :

: (t) =
 (t) = 6t 16, t 0,

 ,

1. t
.



= 8 m sec2 , (4) = 8 m sec2

4
4
t > 0 (t) < 0
< t < 4,
3
 3

4
t 0,
(4, +)
3


4
t
, 4 .
3

t <

3.
.
4. t0
.

4. (t) = 0

8
t = sec.
3

x (t) =

1. : (t) = x (t) = 3t2


16t + 16, t 0, :

(0) = 16 m/sec

4
3

3. v > 0
v < 0. : (t) > 0 0

2.

.


:
t3 8t2 + 16t + 3, t 0.

4
sec t = 4 sec
3

(4)
,
0, ,
() t 0.

13

http://www.mathematica.gr

20

1. .

http://www.mathematica.gr/forum/viewtopic.php?f=18&t=439

f (x) =

2. ,

1.

(x ) , x [0, )
x + 3 ,
x [, 3]
2
2

1. f
2.


-

a2
4

4+
4
+ 12 2a a = 1 a2 = 4+
a = 2 4+
2
E = a4
2
2
E = 12 2a a = a2 = 4a4 > a4 = E


[, 3]. x =
. 3

3a
2

1/2.
2
12 (3a ) = 12 ( 3a) = 2 =

3a

2 = 3a

2 =3

2 4+
4+

21 math68

1.
< 0.
z 2 3( + 2)z + 5 + 6 = 0.
= 32 8 12 = 1 = 4
. = 1

http://www.mathematica.gr/forum/viewtopic.php?f=51&t=9819

C :

z 2 ( + 2) 3z + 5 + 6 = 0
,
.
1. z1 , z2 .

z 2 3z+1 = 0, z1,2 =

3 1
2 2i

2. z1 = 23 + 12 i z2 = 23 12 i. (

z1 = 23 12 i z2 = 23 + 12 i ) z13 = i
z23 = i

K = z12012 z22015 + z12015 z22012 =

2. :

1
z12012 z22015

K = z12012 z22015 + z1 2015 z22012


3.
a = z12012 z22015 b = z1 2015 z22012

.

1
z12015 z22012

z13 +z23
z12015 z22015

ii
12015

=0

3. a = z12012 z22015 = z 20121z 2015 = z 2012 z12012 z 3 =


1
2
1
2
2
1
1
2012 = i = i
3
z2 (z1 z2 )

= z12015 z22012 =
1
z13 (z1 z2 )2012

1
i

1
z22012 z12015

= i

1
z12012 z22012 z13

A(0, 1)


14

http://www.mathematica.gr

B(0, 1) A, B . |z|max =
O(0, 0).
(OK
 1 ) = 4 z = 4i |z|
min = (OA) = (OB) =

(OK1 )2 R12 =

22

16 4 = 2 3

http://www.mathematica.gr/forum/viewtopic.php?f=51&t=10002

|z 4i| 2

|z 2i| = 2
z .
chris
|z 4i| 2 z

C1 K1 (0, 4)
R1 = 2 |z 2i| = 2
z K2 (0, 2) R2 = 2
. z

AB

23
http://www.mathematica.gr/forum/viewtopic.php?f=52&t=2571

f : R R 1-1
2
: f (x2 ) (f (x)) 14 ;


1
x 1x
(
)
:

giannisn1990




1 x2
x
1
f : R R 1-1 f (x2 )
+f
=2
(1)
f
1x
x1
x
f 2 (x) 14 , 0 1
1
f 2 (1) f (1) + 14 0 f 2 (0) f (0) + x 1x (1) (
1
1 2
) :
4 0 2 (f (0) 2 ) +
1 2
1
(f (1) 2 ) 0 f (0) = f (1) = 2


x(x 2)
x
1-1 1=0

x1

+ f (x) = 2

1x

(2)

24
http://www.mathematica.gr/forum/viewtopic.php?f=52&t=2052



 (2) (1)



 x 0

f (x)f

1, f (x) + f
f .

1
1x

2(12x)
x(1x) .

1
1x

x+1

= 2 xx(1x)

f (x) = x+1
x1
.

15

http://www.mathematica.gr

25
http://www.mathematica.gr/forum/viewtopic.php?f=53&t=1801

f : R (0, +)
: f  = f f ;
,
f (x) > 0 x R. x
f (x) f (f (x)) > 0, x R f  (x) > 0 f
R [1] f (x) > 0 [1]

f (f (x)) > f (0) f  (x) > f (0) (f (x) xf (0)) >


0 f (x) xf (0) R x < 0 f (x)
xf (0) < f (0) 0f (0) f (x) < (1 + x)f (0)
x < 0 [2] f (x) > 0 f (0) > 0.
x < 1 [2],

f [a, b], f (a) = a, f (b) = b


f  (x) 1 x (a, b),
f (x) = x x [a, b].
1,
g

g(x) = f (x) x x g ()

g (x) g () g (x) = 0 f (x) = x, x [, ]


2,
x0 f (x0 ) = k
= x0 . k > x0

[a, x0 ] f  () 1

k
x0

26

[x0 , b] f  () 1

f ()f (x0 )
x0

1
x0 [a, b]
f (x) = x

:
2

k
tan2 = 1k
2 sin = k

27
http://www.mathematica.gr/forum /viewtopic.php?f=54&t=8057&p=46212

 /2

0
k (0, 1).

sin x
1k 2 sin2 x

I=
dx,

 /2

sin x
dx
(1k 2 )+k 2 cos2 x

/2
sin x
1

dx
2
1k 2 0
1+ k 2 cos2 x

I=

1+k
1
ln
2k 1 k

2

1k

cos x = tan u, u [0, ],


1k 2
(0, /2)
k
tan = 1k
(0, +).
2


1
1
1k 2 1
I = 1k2 0 1+tan
du =
2u
k
cos2 u


 cos u

cos u
cos u
1
1
= k1 0 1sin
= 2k
2 u du
k 0 cos2 u du
0 sin u1
cos u
1
1sin
sin u+1 du = 2k ln 1+sin
k

16

1 k xo k < x0

k
x0 1 k x0
f (x0 ) = x0 ,

http://www.mathematica.gr/forum/viewtopic.php?f=53&t=2081

f (x0 )f ()
x0

k(0,1)
xdx
=
1k 2 +k 2 2 x
0


2
x=t 1k
2
k2
xdx
1

=

k
1k2
+ 2 x xdx= 1k2 dt
0
k2
k2

2
k

1k2
1k2

dt
1
k2

=
k
1k2
1k2 2
+ 2 t
0
k2
k


1
k
1
k

k2
1k2


0

dt
1+t2




ln t + 1 + t2

=

0

k2
1k2

http://www.mathematica.gr
1
2k

ln 1+k
1k

f (x)
= g
(x) g(x), x (0, )
 

=f
=0
g

28 xgastone



 (x) =
 (x) = 1 + 2 g (x) g  (x)
f

f
2

g(x)
f , (0, )


f  (x)

 f (/2) =
0 f  (x) = f  (x) = 1 + f 2 (x) g (x) 1+f
2 (x) = g (x)

2
1 + f (x) . f .
1 = g  (x) g (x) = x + c  x = /2
g 2 = 2 + c g 2 = 2 + c 0 = c c =

0

g


f (x) = g (x) f (x) = (x), 



f (x)
=
1
f  (x) = 1 + f 2 (x)
.
1 + f 2 (x)
http://www.mathematica.gr/forum/viewtopic.php?f=54&t=8003

g  (x)

x

29
http://www.mathematica.gr/forum/viewtopic.php?f=55&p=39775

f, g [0, 1]
: f (x) > 0, 0 < g(x) < 1, x [0, 1].
: F , G, H :

x

F (x) = 0 f (t) dt , x [0, 1]

x

G(x) = 0 g(t) dt , x [0, 1]

x

H(x) = 0 f (t) G(t) dt , x [0, 1]


1. x (0, 1] :

H(x) < F (x)G(x) .


2. x (0, 1) :

H(1)
H(x)
<
<1 .
F (x)
F (1)
3.

H(1) +

1

g(t) F (t)dt = F (1) + G(1),

1 F ()
f ()
=
.
g()
1 G()
x0+

x

H(x) =

[0, 1]
x

f (t)G(t)dt H  (x) = f (x)G(x) > 0


x (0, 1], H
H(0) = 0
(0, 1].
0

1. w(x) = H(x) F (x)G(x),


x [0, 1], : w (x) =
H  (x) F  (x)G(x)  F (x)G (x) = g(x)F (x).
x
F (x) = 0 f (t)dt
[0, 1], x > 0 F (x) > F (0)
0
F (x) > 0 f (t)dt F (x) > 0 0 <
g(x) < 1 x [0, 1]
w (x) = g(x)F (x) < 0 x (0, 1],
w
[0, 1] (0)=0 x > 0

w(x) < w(0) H(x) F (x)G(x) < 0 H(x) <


F (x)G(x) x (0, 1].

(0, 1) :

4. : lim

G(x) = 0 g(t)dt G (x) = g(x) > 0


x [0, 1] G [0, 1]
G(0) = 0
x (0, 1].

2. u(x) =
:

H 2 (x)
F (x) G(x)

H(x)
F (x) ,

x (0, 1].


 (x)
H  (x)F (x)H(x)F  (x)
= H (x)F (x)H(x)F
=
F 2 (x)
F 2 (x)
f (x)G(x)F (x)H(x)f (x)
f (x)(G(x)F (x)H(x))
=
> 0,
F 2 (x)
F 2 (x)

u (x) =
=

(0, 1]

x

F (x) = 0 f (t)dt F  (x) = f (x) > 0


x [0, 1] F [0, 1]
F (0) = 0
x (0, 1].
17

H(x)

, x < 1 u(x) < u(1) F (x) <


H(1)
F (1) x (0, 1). H(x) < F (x)G(x)
x (0, 1], H(1) < F (1)G(1)
.
x
F (x) = 0 f (t)dt

http://www.mathematica.gr

x


0 < 1 F (0) <
0
F (1) 0 f (t)dt < F (1) 0 < F (1).
H(1) < F (1)G(1) (

F (1) > 0)

H(1)
F (1)

M tdt H(x) M x2 , x [0, 1].


x (0, 1] : H(x) < F (x)G(x)
H(x)
: F (x)G(x) < 1
0

< G(1).


(1)1.
q(x) = G(x) x, x [0, 1].
q  (x) = G (x) 1 q  (x) = g(x) 1,
0 < g(x) < 1 g(x) 1 < 0
x [0, 1], q  (x) < 0,
q [0, 1], q(1) <

H(x)
F (x)G(x)

H(x) < H(x)

(H(x))2
F (x)G(x)

< H(x) ,
(H(x))2

: 0 < F (x)G(x) <


2
H(x) M x2 , x (0, 1] , :

0 <

(H(x))2
F (x)G(x)

M x2 im M x2 = 0
x0+

,
2

H (x)
lim F (x)G(x)
=0
q(0)
+
 0 G(1) 1 < G(0) 0 G(1) 1 <
x0
g(t)dt

G(1)
<
1
,

0
H(x)
H(1)
30
x (0, 1) : F (x) < F (1) < 1.

http://www.mathematica.gr/forum/viewtopic.php?f=55&t=429

3. v(x) = (1 F (x))(1 G(x)), x [0, 1].


 :

f (x) = (x ) ( x)
1
H(1) = 0 f (t)G(t)dt =
1 
x R, < , .
=
0 F (t)G(t)dt 
1
[F (x)G(x)]10 0 F (t)G (t)dt =
1. x (, )
1
F (1)G(1) 0 F (t)g(t)dt

f  (x)
=
+
1
f (x)
x x
F (1)+G(1) = H(1)+ g(t)F (t)dt F (1)+G(1) =
F (1)G(1)

1
0

F (t)g(t)dt +

1
0

g(t)F (t)dt

F (1) + G(1) = F (1)G(1)


F (1) + G(1) F (1)G(1) 1 = 1
F (1) (1 G(1)) (1 G(1)) = 1
(1 G(1)) (F (1) 1) = 1
(1 G(1)) (1 F (1)) = 1,
v(1) = (1 F (1))(1 G(1)) = 1
v(0) = (1 F (0))(1 G(0)) = 1.
4. v(x) = (1 F (x))(1 G(x)),
x [0, 1] Rolle,
(0, 1) :

2. f
Rolle [, ]

f  = + + +
.
3. I (, ) =

f (x) dx.

I (, ) =

I ( + 1, 1)
+1

4. , ,
, , , , :
Cf , x = , x =

x x

v  () = 0

v(x) = (1 F ()) (1 G()) + (1 F ())(1
G()) = 0
1. f (x) = (x ) ( x) ,


F ()(1 G()) (1 F ())G () = 0
, [, ]
F  () + F  ()G() G () + F ()G () = 0
(, ) :
f () + f ()G() = g() F ()g()
f  (x) = (x )1 ( x) (x ) ( x)1 .
f () (1 G()) = g() (1 F ())
x (, ) f (x)
= 0, :
f ()
1F ()
f  (x)
(x)1 (x) (x) (x)1
=
=
g() = 1G()
f (x)
(x) (x)

5. q(x) = G(x) x, x [0, 1]


[0, 1], x > 0
g(x) < q(0) G(x) x < 0 G(x) < x,
x [0,
1]. M [0, 1]

0 f (t) M
t [0, 1]

0 G(t) x
: f (t) G(t) M t
 x t [0, 1]
: 0 f (t) G(t)dt
18

(x)1 (x)
(x) (x)

(x) (x)

(x) (x)1
(x) (x)

2. f () = f () = 0,
[, ],
Rolle,
(, ) f  () = 0

( )1 ( ) ( ) ( )1 = 0
( )1 ( ) = ( ) ( )1

http://www.mathematica.gr

( ) = ( ) =
= ( + ) = ( + )

= +
+ = + + +
3. :


I (, ) = f (x)dx =
  (x)+1 


[(x

)
(

x)
]dx
=
( x) dx =
+1



(x)+1

x)

+1



 (x)+1
( x)1 ( x) dx =
+1

+1

( x)1 dx = +1
I ( + 1, 1)
+1 (x )

I(, ) = +1
I( + 1, 1) =

1
+1 +2 I( + 2, 2) = ... =

2
1
1
= +1
+2 +3 ... + I ( + , 0) =

+

2
1
1
( x)0 dx =
= +1
+2 +3 ... + (x )



(x)++1

2
1
1
=
= +1
+2 +3 ... +
++1

=
=

+1

+1

2
+3
2
+3

...
...

1
+
1
+

()++1
++1
1
++1 (

=
)++1

, f (x) 0,
:

E =

= +1

4.
:

1
+2
1
+2

|f (x)| dx
1 2
+2 +3 ...


= f (x)dx = I (, ) =
1
1
+
++1
( )++1

31
http://www.mathematica.gr/forum/viewtopic.php?p=52685#p52685

z1 , z2 C |z1 | = |z2 |
= 0.
:

xy = z1
xy = z2

2az2 z1 + z2
,
z1 + z2
2a

a R

32
http://www.mathematica.gr/forum/viewtopic.php?p=26850#p26850
f : (0, +) R f (x) > 0

 f (x)
t

1


xy = z1
, |z1 | = |z2 | = > 0, x, y, z1 , z2 =

0
x
y = z2
y
z1
y = z2 (1).

xy = z1
, |z1 | = |z2 | = > 0, x, y, z1 , z2 =

0
x
y = z2
z1
x
x
= z2 (2)

f (x) = x +

2t

dt

x (0, +).
1

(1) (2)
, x, y .
2

x, y 0,

z2
z1 x
,
=
:
x x

z1
.
z1 x
= x (x, y) =
 z z2 =
1
ai, i a R
a
.

z
z1
2
=

 :
x
z2
z1
=
y= ,

x x
+x
z1 + z2 

y=

2
, Rex = a : Imx = zz11 z
+z2 ai , a R


 : (x, y) =

f (x)

f (x)

sin t
dt = x
2t

x
1

sin t
dt.
2t

sin x
g  (x) = 1
> 0, x > 0.
2x

g 1 1 ( . )
g[f (x)] = g(x) x, f (x) = x, x > 0.
.
2
x
|x| < |x| , x
= 0 1 < x < 1, x
1
(0, +) x
2x < 2 < 1, x > 0, (1)
g (x) =

t
dt, a, t, x
2t

Imx
2
+ 2Rex
i = z1z+z
.
a
2


(0, +). f (x) =
z2
z2
Imx
Rex = z +z z +z i , x + g (f (x)) g (x) f (x) x = g (f (x)) g (x),
2
1
2
1
z1 z2
, Imx
(2)
Rex = z1 +z2 i

y=

z1
x,

1
2

x

19

http://www.mathematica.gr

x0 (0, +)
f (x0 )
= x0 ,
f (x0 ) , x0 ... g ,



f(x)


t
|f (x) x| =
dt

x 2t

f(x)
f(x)

1
1
g (f (x0 )) g (x0 )
t dt
2 dt 2 |f (x) x|
x 2t
, g () =
x
f (x0 ) x0
.
(2)
= 1, (1) f (x) = x, x > 0. : f (x) > 0
2


33
http://www.mathematica.gr/forum/viewtopic.php?f=49&t=8906&start=0

9
;
stavros11
A = abcd N .
: 9A = dcba 9abcd = dcba a
= 0, d
= 0

, A 1111 (
1112 9 ).
a d : 1001 A 1111
a = 1 d = 9.
b 0 1.
dcba
9 ( dcba = 9A A N ),
9.
9/a + b + c + d. :9/10 +
b + c. b = 1 c = 7 A = 1179. ,

A < 1112.
b = 0, c = 8 A = 1089.
1089 1089 9 = 9801.
34
http://www.mathematica.gr/forum/viewtopic.php?f=49&t=8079&start=0

: a, b, c , . : a + 1b = b + 1c = c + a1 =
t, t R, : t = abc
chris

ab2 c + ab + 1
1
+ abc =
=
b
b
1
abc(b + c ) + 1
abct + 1
=
(1)
b
b

t + abc = a +

abct + 1
abct + 1
abct + 1
=
=
b
a
c


a
= b
= c
= 0 abct = 1 (1)
.

35

,
,
http://www.mathematica.gr/forum/viewtopic.php?f=50&t=895&start=0
.
( ), ,
- ( ) , .
.
1

,

20

http://www.mathematica.gr

. , ,
(). ,

.

, ,
= = .

36
http://www.mathematica.gr/forum/viewtopic.php?f=50&t=51&start=0

0, a1 a2 a3 ... .

a1 , a2 , a3 , ...
;


2
Hilbert.
, ,
. a1 , a2 , a3 , . . .,
= = .
A = {0, 1, 2, 3, . . . , 9}. ,
,
AB + M N AM + BN (1).
, .
,
+<(+)+(+), :
+ < , , +, (1). . b1 , b2 . . . , bn b1 <
, , b2 < < bn
. , () ,
: b1 b2 . . . bn b1 b2 . . . bn b1 b2 . . . bn . . ..
.
.
() b1 b2 . . . bn
> 3.
.

37 2010
http://www.mathematica.gr/forum/viewtopic.php?p=33045

AB AB =
B = . .
BA A
AE = AO B
B BZ = BO . EZ
, :


a2 + 2 =

2
(2a) 2 = 3a2 = a 3
EO Z. (ii)
=, =
= OBA
.
AOB

(i) = 3
(ii) AZ = EO
(iii) EOZ
1
(=,
= Z O

= =). E A
===,
= B EO
ABO
=
. AB
=, .
OBA
== .
21

http://www.mathematica.gr

(i) : = , =
= , . ,
 = ZO
 ( = ).
EA

 + OZ
 = EA
 + OZ
 = ZE
 = 60 ,
ZO
 = 120 .
. ZO
 = OB
 = 30 . OB


= 30 =
= 3.

(ii) ,
 = 60 .
= AOB
 =
: = , = AOZ

 = 60 ,
EBO
.
, = ,
 EBO
.
AOZ
A1 A2 , XL ,
 = 60
(iii) OAB
O  OI, O



, EOA = 30 , EOB = (O),
30 + 60 = 90 , .
IXLO.

38 2010
http://www.mathematica.gr/forum/viewtopic.php?p=32836

ABC (O)
I . AI, BI, CI
(O), D, E, F, .
(O1 ), (O2 ), (O3 ), ID, IE, IF , BC, AC, AB,
A1 , A2 B1 , B2 C1 , C2 , .
,
(O  ).

,
B1 B2 , C1 C2 , (
Y M, ZN,
), O  .
P AI EF IEP +

A + B + C
= 90o = IP E = 90o
EIP =
2
IP F = 90o
P
(O2 ), (O3 ).
(O1 ), (O2 ),
B1 B2 , P I, C1 C2 A, (AB1 )
(AB2 ) = (AP ) (AI) = (AC1 ) (AC2 ) , (1)
(1)
B1 B2 C1 C2 (O )
O  , ,
1
L, M, N, BC, AC, AB B1 B2 , C1 C2 ,
X, Y, Z, - .

(I), .
A1 A2 B1 B2 , A1 A2 C1 C2 ,
ID (O1 )
(O  ) .
X, L

2
A1 A2 ,

A1 A2 , ,
http://www.mathematica.gr/forum/viewtopic.php?p=4568
XL
3
( ).
A, B, C u2 , v 2 , w2
ABC
22

http://www.mathematica.gr

D, E, F, I 2 2 ( wv, wu, uv, wv wu uv . )


z
zA A1 A2 K zA |zA zK | = zA I

w2 + v 2 zA
OI zA =
A1 , A2 BC
v 2 w2
IA1 , DA1 IA2 , DA2
zA zI
I |u| = |v| = |w| = 1
zA zD
m = m
m I

mathematica u, v, w

(u2 (v + w) + vw(v + w) + u(v 2 + vw + w2 ))/(uvw)


.

39 Putnam 2009

[a, b] [0, 1].

http://www.mathematica.gr/forum/viewtopic.php?f=59&t=2255

A, B, C A
. (A B)C = BA1
C(A B) = A1 B .

AC BC = BA1 AC BC + I =
BA1 + I , I .
A (A B)(C + A1 ) =
AC BC + I BA1 = I . A B
C + A1 . I =
(C + A1 )(A B) = C(A B) + I A1 B
C(A B) = A1 B .
40
http://www.mathematica.gr/forum/viewtopic.php?f=59&t=5832

f : [0, 1] R :
(1) f

(2) c R, f 1 (c) = {x [0, 1] : f (x) = c}


.
f .

x f .
> 0 (xn ), (n ) lim dn = 0,
|x xn | < n |f (x) f (xn )|  n.
n f (xn )  f (x) +
n f (xn )  f (x) .
f (xn )  f (x) + n.
(1) Dn x, xn
sn Dn f (sn ) = f (x) + .
lim sn = lim xn = x sn
f 1 (f (x) + ) (2)
lim sn = x f 1 (f (x) + ), f (x) = f (x) + ,
. f .

41
http://www.mathematica.gr/forum/viewtopic.php?f=10&t=941

( ), :

n N, ()


n
k=0

6k

 


n
n
n 2
n n
(1 + x) =
+
x+
x + +
x .
0
1
2
n
n

x x, x

2 x ,
23

http://www.mathematica.gr

.
1 + + 2 = 0.
xk k
3, 3 = 1.

42 giannisn1990
http://www.mathematica.gr/forum/viewtopic.php?f=10&t=1005

A M33 (R)
(1 +
+ (1 +
+ (1 +
=

n n 3 n 6
A2 + I = 0.
3 0 + 3 x + 6 x + .
x = +1 x = 1 .

. A2 = I , det(A2 ) =
det(I) = 1 (det(A))2 = 1,
2n + (1 + )n + (1 + 2 )n + (1 )n + (1 2 )n
.
(det(A))2  0. .
6

x)n

2 x)n

x)n

ln(n + 1)

(n + 1)sn

43
http://www.mathematica.gr/forum/memberlist.php?mode=viewprole&u=54


n
ln(n!)
,
(1) 
ln Hn+1
n=1
n

n

, Hn =

k=1

n+1

sn

(1).

x
,
 1sn
 1 1 ...,

n+1 , n n+1 , n1 ,


+


sn+1

n+1

n
sn

1
n-
k

n+1

s ln(sn )

sn = n
n(n + 1)

sn ln(sn )
ln(sn )

(2).
n(n + 1)
(n + 1)2

n+1

-


ln(s )

ln(n + 1)

(2) (1)

n
sn ln(sn ) (n + 1) ln(n + 1)

ln(n!)
(n + 1)2
(n + 1)sn
 , n N,
an := 
ln Hn+1

n s n+1 s n+1 s
n+1 s
n s n+1 s
,
n
n
n+1
n
n
n+1 .
n

n+

Hn+1 ln(n + 1)

Hn+1
1
ln(n + 1)

Stirling

lim an = lim

n+

n+

 
 n 
n
ln 2n ne

 ,
ln ln(n + 1)

() Bernoulli,
0 1.
44 MoV

LHospital 0.

Leibniz
. 

http://www.mathematica.gr/forum/memberlist.php?mode=viewprole&u=89

ln Hn+1

, an , n N,
, n ln(n!), : N N .
n N, n.
1
sn := ln(n!),


n+1
x = 0, 1 2 ... [0, 1)
n+1 s
sn + ln(n + 1) =
n+1 =
.
x
1


()

ln(n+1) n+1
n+1 s

:
n 1+
sn

n+1

sn

ln(n + 1)
1+
(n + 1)sn

ln(n + 1)
sn +
(n + 1)sn

n+1

21+1 , 21+1 31+2 , 21+1 31+2 51+3 , . . .


24

http://www.mathematica.gr

.
1-1
[0, 1)


.

[0, 1)
Cantor .
2
X .
g : X [0, 1] X .
g() = 0, (1) (2) . . ., n
n 10.
x = 0, x1 x2 . . . [0, 1], :
N N; (n) = 10n + xn X
g() = x, g .
3 MoV
Cantor :

: N N :

(0 )n := (0,0 , 0,1 , 0,2 , . . .)


(1 )n := (1,0 , 1,1 , 1,2 , . . .)
(2 )n := (2,0 , 2,1 , 2,2 , . . .)
.............................................
n := n,n + 1 + n1 0 = 0,0 ,
n = n +

n+1 n =

n


i,i
i=0
n+1,n+1 + 1

> 0 -

(ai )n . .
4
N.
, N
( N
).
N.
P(N)
N ,
.

x(x 5199 ) + 3x 5199 + 1 x + 0 + 1 5100

45
http://www.mathematica.gr/forum/viewtopic.php?f=63&t=4769

(51985 1) ,

5100 .


()
1985.
46

http://www.mathematica.gr/forum/viewtopic.php?f=63&t=3872

x5 1 = (x 1)(x4 + x3 + x2 + x + 1).

xy + y = y x + x .

x4 + x3 + x2 + x + 1 = (x2 + 3x + 1)2 5x(x + 1)2 .


x = 5397 , x4 + x3 + x2 + x + 1 = (x2 + 3x + 1)2

2
398
1)2 5199 (x + 1) =
5 (x + 1)2 = (x2 + 3x + 

x2 + 3x + 1 5199 (x + 1) x2 + 3x + 1 + 5199 (x + 1)
,



5 1 = (x 1) x2 + 3x + 1 5199 (x + 1)
x

 2
x + 3x + 1 + 5199 (x + 1)


 2
199
x 1, x + 3x + 1 + 5 (x + 1)
5100 .
,



x2 + 3x + 1 5199 (x + 1) =



(x, y) = (c, c), (c, 1), (1, c), c N
(x, y) = (2, 3), (3, 2).
.
, x = 2 y > x, xy y x (
f (x) = lnxx
x = e f (2) = f (4)) y > x, xy + y > y x + x,
.
3 x < y , f (x)
xy > y x y > x
.

25

http://www.mathematica.gr

47

Cauchy-Schwarz....



x2
a2

y2

+
11=1

= 1.

A(x1 , y1 )
B(x2 , y2 ) :

x x

x x
1 2 y1 y2

1 2 y1 y2
2 + 2 1 + 2 + 2 + 1 = 2 .
a

1
(*) :

(x2 , y2 )
1 xa1 x2 2 + y1b2y2 1.

x2
a2

(x1 , y1 ),
+
= 1
y2
b2

+
1 = 12 2xa12x2 + 2y12y2 2


x21
y12
x22
y22
2y1 y2
1 2x1 x2
+

2
2
2
a2
2
a2
2
a 2

2
y12 2y1 y2 +y22
1 x1 2x1 x2 +x2
+
=
2
2
a2


2
2
(x
x
)
(y
y
)
1
2
+ 1 2 2
0
12
a2

y1 y2
2

+10




x 1 x 2 y1 y2
y y
a2 + 2 1 + xa1 x2 2 + 1 22 + 1 =
xa1 x2 2 y1 y22 + 1 + xa1 x2 2 + y1 y22 + 1 = 2
x1 x2
a2

y1 y2
|

x21
a2

y12
2

x22
a2

y22
2

y y
y y
: | xa1 x2 2 + 1 22 | 1 1 xa1 x2 2 + 1 22 1
:
x1 x2
a2
x1 x2
a2

+
+

y1 y2
2
y1 y2
2

1
1

| xa1 x2 2 + y1 y22 1| + | xa1 x2 2 + y1 y22 + 1| =


1 xa1 x2 2 + y1 y22 + 1 + xa1 x2 2 + y1 y22 = 2
48
http://www.mathematica.gr/forum/viewtopic.php?f=27&t=9347

(*) 1 c 1 |c1|+|c+1| = (1c)+(c+1) = 2


(**) Ax + By = 1
( Ax + By > 1) ( Ax + By < 1).
2 manos66 

y1 y2
2

| xa1 xa2

:
x2 x2 , y2 y2 (
(0, 0)).

y y
(x1 , y1 ) xa12x + b12 = 1,
y1 y
x1 x
a2 + b2 < 1 (x, y)
(**) ( (0, 0)
(x1 , y1 ),
(x2 , y2 )).

x1 x2
a2

y1 y2
2 |

| xa1 x2 2

http://www.mathematica.gr/forum/viewtopic.php?f=27&t=8258

(1 + tan 1 )(1 + tan 3 ) (1 + tan 43 ) < 211 <


(1 + tan 2 )(1 + tan 4 ) (1 + tan 44 )
1

sin()+cos()
1 + tan () =
= 2 sin(45+)
cos()
cos() =

cos(45)
cos()

(1 + tan (1)) (1 + tan (3)) .. (1 + tan (43)) =


 22 cos(44) cos(42)
cos(2)
2
cos(1) cos(3) .. cos(43)
=
211

cos(2)
cos(1)

cos(4)
cos(3)

..

cos(44)
cos(43)

< 211

(1 + tan (2)) (1 + tan (4)) .. (1 + tan (44)) =


 22 cos(43) cos(41)
cos(1)
2
cos(2) cos(4) .. cos(44)
=
211

cos(1)
cos(2)

cos(3)
cos(4)

..

cos(43)
cos(44)

> 211

2 :

(1 + tan 1 )(1 + tan 3 ) (1 + tan 43 ) < 


211! <

!


(1 + tan 2 )(1 + tan 4 ) (1 + tan 44 )



!


0 < A < C .
:

44
1 + tan 1 = 1 + tan(45 44 ) = 1 + 1tan
1+tan 44 =
2
(1 + tan 1)(1 + tan 44 ) = 2
1+tan 44o 
 
A C = (1 + tan 1 )(1 + tan 44 ) (1 + tan 2 )(1 +
 

tan 43 ) . . . (1 + tan 22 )(1 + tan 23 ) = 222

A < C A2 < AC < C 2 A2 < 222 < C 2 A < 211 < C

26

http://www.mathematica.gr

49
http://www.mathematica.gr/forum/viewtopic.php?f=61&p=41303#p41303


:

R, ,
5 ,
.


,
.
...
. , , , , , ...
. ... ( ) ,

, .

1

mathematica
. :
R.
;
.. 5 ;
2
, !
1

1 2
x + (x), f  (x) = 1
2
1
(x) > 0 f . g(x) = x2 ,
2
f  (x) = 1 > 0 f . f (x) = 50
g(x) x = k k Z.
http://www.mathematica.gr/forum/viewtopic.php?f=61&p=16318#p16318
: f (x) =

2
( f
f g ):
(, ) lim f (x) = lim f (x) = k , k

xa+

(x + 1) x < 1
f (x) 0 g(x) = 0
1  x  1

(x 1)2 x > 1

,
(, ) f  () = 0.

3
: f (x) = x2 , g(x) =
x2 + sin2 x. . ..
g  (x) = 2x + 2 sin x cos x = 2x + sin 2x, g  (x) =
2 + 2 cos x 0. sin x = 0,
= .
g  (k) = 2k + sin 2k = 2k =

f  (k)


Rolle:
g : [, ] R g (x) = f (x) , x (, )
g () = g () = k . g (, )
[, ]
Rolle. -
.

27

http://www.mathematica.gr

51

4.
z

http://www.mathematica.gr/forum/viewtopic.php?f=60&t=5244

z C |z 1| = 1,
z2.

5. w =
z 2 w


z C |z 1| = 1,
z 2 . z = 1 + (cos(t) + i sin(t)) t[0, 2 ).

6.
( 4 )
w z .

t
t
t
2
2
2
t
z 2 = 4 cos2 ( ) (cos(t) + i sin(t)) 52
2

z = 2 cos( ) (cos( ) + i sin( ))

http://www.mathematica.gr/forum/viewtopic.php?f=60&p=47538#p47538

t
x = 4 cos ( ) cos(t)),
2
z1 , z2 , z3 C |z1 | = |z2 | = |z3 | = r > 0,
t
y = 4 cos2 ( ) sin(t)) t[0, 2 )
|z1 z2 |2 + |z2 z3 |2 + |z3 z1 |2 9r 2 .
2
2



Geogebra
:

=,
z1 , z2 , z3 .

1
, , ,
2
2
r . |z1 z2 | + |z2 z3 | +

|z3 z1 |2 = (AB)2 + (BA)2 + (A)2 =


 2  2  2
OB OA + O OB + OA O = 6r 2

2(OAOB + OB O + OOA) =
6r 2 2r 2 (a + b + c) =
r 2 [6 2(a + b + c)], a, b, c
, a + b + c = 360
, : a + b + c + (a + b + c) =
b+c
c+a
4 a+b
2 2 2
a + b + c = 1 4 a2 2b 2c
:

a2 2b 2c 18 4 a2 2b 2c 12
a + b + c 1 12 = 32 ,
- : a
b
c

2 + 2 + 2 = 180
1. z

(x 1)2 + y 2 = 1

.
, :

|z1 z2 |2 + |z2 z3 |2 + |z3 z1 |2 6r 2 + 3r 2 = 9r 2

2. . a = b = c = 120 ,
Geogebra .
3.

.
2
|z|2 = z
z |z1 z2 |2 +
2
2
|z2 z3 | +|z3 z1 | +|z1 +z2 +z3 |2 = 3(|z1 |2 +|z2 |2 +|z3 |2 ),

|z1 z2 |2 +|z2 z3 |2 +|z3 z1 |2 3(|z1 |2 +|z2 |2 +|z3 |2 ) =

28

http://www.mathematica.gr

9r 2 = z1 + z2 + z3 = 0, . B(, ) (, ). = ,
.
, r .
3
OA+OB >
A, B, C AB 2 > AB , 42 > (a )2 + 2 .
z1 , z2 , z3 , OB = 2 + 2 = p2 .
ABC O < 2 (1).
r . G ABC M OB +
, O > B, 2 > B,

42 > ( )2 + ( )2 . OB =
1 2
2
2
2
2
2
2
M A + M B + M C = 3M G + 3 (a + b + c ), BC = 2 + 2 = p2 . O = 2 + 2 = p2 .

a, CA = b, AB = c.
, M  < p2 (2).
O
< p2 (3).
3r 2 = 3OC 2 + 31 (a2 +b2 +c2 ) 9r 2 = 9OG2 +a2 +b2 +c2 , AB 2 + B2 + A2 .
9r 2 |z3 z2 |2 + |z3 z1 |2 + |z1 z2 |2 .
= OG = 0, 6p2 2a 2 2 2a
2
.
AB 2 + B2 + A2 6p2 + 3p = 9p2
4
: , , > , > > ,
(0, 0) A(, 0) > 0, > , (3).

29

http://www.mathematica.gr
. .

Leonardo da Vinci
(32-) . 30
. -
-
- quasiregular ,
( - ).
(0, 0, ),



, 1+
,
12 ,
2
2

1+ 5
.
2

:http://en.wikipedia.org/wiki/Icosidodecahedron
:

mathematica.gr
(http://www.mathematica.gr) .

mathematica.gr


1. (Mihalis_Lambrou)
2. (nsmavrogiannis)
3. ( )
4. (k-ser)

11. (achilleas)
12. ( )
13. (Demetres)

1. (spyros)

5. ( )
6. (m.papagrigorakis)
7. ( )

1. (grigkost)

2. (vittasko)
3. (p_gianno)
4. (kostas.zig)
5. (exdx)

2. (cretanman)
6. ( )

7. (mathxl)
1. ( )
8. (mathnder)
2. ()
3. (nkatsipis)
4. ( )
5. (chris_gatos)
6. (gbaloglou)

9. (mathematica)
10. (rek2)
11. (hsiodos)
12. (A.Spyridakis)

7. (R BORIS)

13. ( )

8. (Rigio)

14. (bilstef)

9. (dement)

15. ()

10. (swsto)

16. (xr.tsif)

1 ( maths-!!)
, a 1 a 10
a b 1 b 10.
...
100.

, ;
;

5 ( )

x
y .


110
.

11 ( )
.

, = + .

2 ( )
9 13 16
( 16) 22
9 6. 17
5 13 19. 15
24 27. 20
15.

6 (
)


20 20 cm.
.

15 .

7 ( )
x2 + y 2 = 3xy , x, y
,

x
xy

2


+

y
xy

2

8 ( )
0 < b < a a2 + b2 = 6ab

a+b
ab

4 ( )

170 330.
, 13
, 27 . ,
1
.
4
;

,
, , : =2=6,
3 5.

.

13 ( )

3 (
)



. , ,

,

.

,

(
);

12 ( )
==5.

|x 1| +

|x 5| = 2

x
9 ( )

:

x2
+ x1 +
x1
1
x2

+
x1
x1

x1
x2

x1
+
x2

10 ( )

14 ( )
,

.

| 1| x2 + |3 2| x + |1 | = 0
= 1 x
.
) .
)
.
) ( 1 )
( 2 ) 1 2 )
,
.

15 ( )

K . A :
A1 A
A2 A1 K
A3 A2
B A3 K
AB 4 K .

19 ( )
200
5 , .
4,
20, 60, 50, 40, 30
20.
1. .
2.
5

20 .

16 (
)
,
.
.

.
.

()
.

24 (
)

f, g
R .

3f (x) + 5g(x) + 8f (x)g(x) = 0 (1)


R.

, ,

()
( ),
5.
()
)
:
20, 60 , 50 , 40 ,30 .
20 ( )
2 + 2
2 2 ,
2.
.
1. : :
.

25 ( )
R

(f  (x))2 = f (x)
x.

26 ( )
f : R (0, +)
, :f  =
f f;

2. : P ( ) = P ( );
3.
.

, ,

, ,

21 ( )


17 (

 OA =
 OB =
,


 = 
 

 + |
|

72
1+ 3
31
+
i
2 2
2 2

18
()
+2

 3KA
 KB+A








 
=5K KA=6 , KB=8 , K=5 .
:
) .

v =KB+K A
)
= 32
7

x+

2x2

lim f (t) dt = 0

x+

x2

22 ( )

2z 2 = 6iz + 3, z C


AOB

27 ( )
f [0, +)
lim [xf (x)] = 0, :

28 ( )


, ,
,

23 ( )
f f (f (x)) = x2 x+1,
x R.
g : R R
g(x) + xf (x) x2 = 1, x R

I :=

cos(2x + 2 sin(3x)) dx

, ,

29 ( Math
Rider)

f : (0, +) R
f  (x) + x1 f (x) = x12 ,
x > 0



A e, 1e .
) f

.
)

ex dx

n


xe dx

) g(x) =

x
1

f (t)dt,

x > 0. h :
(0, +) R
 
1
ln2 x
h(x) = g(x) + g
x
(0, +).
)

2
ln t
dt + 2 2xf (x) =
t
x

1
x

35 ( 2010)
c1 (O1 , r1 )
c2 (O2 , r2 )
, .
c1 (O1 , r1 ), c2 (O2 , r2 )
r1 < r2 , M A < M B .
36 (

2010)

f : R

 R 

f (x)
f (y)

: f

1
f (f (x)),
y

x, y R
(0, +).

ln t
dt
t

41 ( )
f : R R ,
x0 R,

f (x0 ) lim f (x)


xx
0

f (x0 +) lim f (x)


xx+
0

R {}.

f .
42 ( )

+


x > 0.

30 ( )
f : R R
f 
R. x0 R
f  (x0 ) > 0,
lim f (x).

n
(k 1)k
lim k=1
n
k
n
k=1 (k + 1)

. Juniors
38 ( )

An
31 ( )

:
2
y
2
z
2
x

{1, 2, . . . , n}

an = |An |

32 ( )

lim

an
n


3n 1
n 8n

43 (
)



,
( ).
44 ( ) (

X Rn x
X .
Y X :
|Y |  n + 1
x Y .

2x + 10x + 19
]/x R}
x2 + 5x + 7

( )

. Seniors

39 ( )

45 ( )
()

K1 = {a + b p : a, b Q}

33 ( )
x, y, z > 0 x + y + z = 1

3xyz(xy +yz +zx)+2xyz (xy +yz +zx)

{k

2k 1}

n=0

37 ( ) ( ) :

x+

A = {[

2ik
n

k=1
(k,n)=1

x
y.

2 3

2x = y +
2y = z +

2z = x +

x > 0.

40 ( )

f (x + y) + f (x)f (y) = f (xy) + 2xy + 1

2 3

34 ( )
f : R R

K2 = {a + b q : a, b Q}

, p, q
, .

()|
()
3.


()

46 (
)

an = [n 2] n
N

A2 A + I = O,
B = A kI
k R
A.

2.
.
()

47 ( )
f : [a, b] R.
n

1 < 2 < ... < n


[a, b] :

49 ( )

(, ) limxa+ f (x) =
limx f (x) = ,
(, ) f  () = 0.
50 ( )
f : (0, +)
(0, +) g
g(x) = f 2 (x) + f (x), x > 0,

f (b) f (a)
f  (1 ) + f  (2 ) + .... + f  (n )
1
f (x)
=
=
lim
ba
n
x+ x
2

.
48 ( ) A

51 ( )
Lagrange
: ,
, 5, 1974
f (x)
1,
x1 , x2 , ..., x
x,
y1 , y2 , ..., y , :

f (x1 ) = y1 , f (x2 ) = y2 , ..., f (x ) = y

52 ( ) (
17 Hilbert).
f (x)
, f (x) 0
x R.
A(x), B(x) ,

f (x) = (A(x))2 + (B(x))2

x R.

1 (maths-!!) ,
a 1 a 10
a b 1 b 10.
...
100.
,
;
;

24 27. 20 15.

http://www.mathematica.gr/forum/viewtopic.php?f=44&t=9776

1 ( )
( 100),
89 (11 100).
89 11
89 ( 78) ... ,

.
:1,12,23,34,45,56,67,78,89
( ) ,

.


67 (
78) .
(
)
.

http://www.mathematica.gr/forum/viewtopic.php?f=44&t=8394

1 ( )
25 20 , 15.
16 13 , 9.
3 17 , 22
5.
2 ( )
5 .
.

10
21
11
27
26
14

2 ( ) 9
13 16 (
16) 22 9 6. 17
5 13 19. 15
5

20 13
5 17
18 7
15 9
24 22
19 6

1 25 16
4 23 3
12 2 8

3 ( )

. , ,
,


.
,
(
);

)
.
.
4 ( ) 170
330. , 13
, 27 . , 14
. ;

http://www.mathematica.gr/forum/viewtopic.php?f=33&p=62943

http://www.mathematica.gr/forum/viewtopic.php?f=33&p=62516

(chris t)
2 ,
1
4 ( 1 4
).
.
, 1 x1
x4 .
x1 + x4 = 1 x = 5.

15 , 25 , 35 , 45 , 4 . ( 55 ).
( )

. (

( ) :
1/3
,
3,
2/7
. ,
7,
1/4
,
4.

3, 7, 4 170 330,
(3,4,7)=84
170 330, 252.

5 ( )
x
y .


110 .


220 + a.

( 360
30
a
360/12=30 ): 360
= 220+a

220 + a = 12a a =

220
11

a = 20

20 240
60
x
, : 360
= 240
x = 40 .
6 ( )
20 20 cm.
.

15 .

http://www.mathematica.gr/forum/viewtopic.php?f=34&t=3620

()
, a
.

http://www.mathematica.gr/forum/viewtopic.php?f=34&t=5886

(Broly) 4
10 cm .

.

200
14.14cm < 15cm

:
7 ( ) x2 + y 2 = 3xy ,
x, y ,

x
xy

2


+

y
xy

http://www.mathematica.gr/forum/viewtopic.php?f=35&t=10378

( ) :

2

(x y)2
2xy
=

=
xy

x2 2xy + y 2

3xy
:

2 
2
x
y
x2
y2
+
=
+
=
2
xy
xy
(x y)
(x y)2
3xy
x2 + y 2
=3
=
=
(x y)2
xy


http://www.mathematica.gr/forum/viewtopic.php?f=35&t=9678

( )

8 ( ) 0 < b < a

I=

a+b
a2 + b2 = 6ab

8ab
a+b
a2 + b2 + 2ab
I2 = 2
=
=
2

I
=
2.
ab
a + b2 2ab
4ab

ab

9 ( )
:

x2
+ x1+
x1

http://www.mathematica.gr/forum/viewtopic.php?f=19&p=63839#p63839

x2
x1
1
x1

+
=
+
x2
x2
x1
x1

( )
) | 1| x2 + |3 2| x + |1 | = 0
:

http://www.mathematica.gr/forum/viewtopic.php?f=19&p=44884#p44884

( ) x > 1. a =

2
x
, b = x 1, c = xx1
2 . abc = 1, ,
x1

> 0 |3 2a|2 4|a 1||1 a| > 0 (3 2a)2


4(1 a)2 > 0 (3 2a 2 + 2a) (3 2a + 2 2a) >
5
:
0 5 4a > 0 a < , a = 1
4
1 1 1
a + b + c = + + = ab + bc + ca
a (, 1) (1, 54 )
a b
c
) 1 , 2 = 0
q = a + b + c, a, b, c = :
1
2

1 2 =

y 3 qy 2 + qy 1 = 0

|1a|
|a1|

|1a|
|1a|

= 1 > 0

1
.

1. a, b, c
1. a
1. c. ,

x4 + 1 = x. x4 + 1 2x2 > x, x > 1.
b = 1, x = 2.

1 + 2 = |32a|
|a1| < 0


.
2 =0

) 1 = 42 1 2 = 422 422 = 1 22 =
1
1
4 2 = 2 .

10 ( )

1 2 = 1 1 = 2


1
(1 , 2 ) = 2,
2
5
|3 2a|
|3 2a|

=
|6 4a| =
) 1 + 2 =
|a 1|
2
|a 1|
|5a 5|

11
5

6 4a = 5a 5
a = 9 < 4

a = 1
6 4a = 5a + 5


| 1| x + |3 2| x + |1 | = 0
= 1 x .
) .
)
. ) ( 1 )
( 2 ) 1
2 ) ,
.

11 ( )
.
,
= + .

A2 = 2 ( ), ,

= (3)
(1), (2) (3) : = +
= + = + .
3 ( )

http://mathematica.gr/forum/viewtopic.php?f=20&t=480

1 (. )
= .
=
, = AE

=
 = + H
=
 . ZH
AZ = + AE
E

ZH , . =
=+ =+.

Z = , AE = 2,

AE + Z = ( + 2) =



=

2 ( )



2 
2
2 
. + 1 2 = 2 + 1 2 =
2
2
,
,

= (1)
= E
2
P1 = 1 . P1 = A1 ( )
1 = 2 ( ), A1 = 2
,
= (2)
12 ( )
: = ==5.
9

3 5. .

http://www.mathematica.gr/forum/viewtopic.php?f=20&p=59752#p59752

(kostas136)

2
EZ cos =
5

3
4
2
2
cos2 = 2cos 1 = sin2 = 1 cos 2 =
5
5
EK

EK DC : sin2 =
ED
, ,
24
(ABCD) = DC EK = 24
, : =2=6, EK =
5

x 1+ 5 x + 2 x 1 5 x = 4 x 1 5 x = 0
: x = 1 x = 5

13 ( )

|x 1| +

|x 5| = 2

14 ( )
,
.

http://www.mathematica.gr/forum/viewtopic.php?f=21&t=11407

1 ( )


x .
a = |x 1|, b = |x 5|.

a, b 0 a + b = 2 (1)

http://www.mathematica.gr/forum/viewtopic.php?f=21&t=10482

1 ( ) b, c, a
a. : 2c = a+b, E = tr, t = a+b+c
2
E = bc
,
2tr
=
bc
,
(a
+
b
+
c)r
=
bc
3cr
=
bc
,
b
=
3r ,
2

w = a c = 2r + x (r + x) = r
x .
3, 4, 5

a2 + b2 = |x 1| + |x 5| |x 1 (x 5)| = 4
(1) a2 + b2 = 4 2ab 4.
a = 0 b = 0, x = 1 x = 5
()
2 ( ) )
x
 1 .
: |x 5| > 4
|x 5| > 2
5 .
) [1,5].
: 1 x 5 x 1 0 5 x 0

: x 1 + 5 x = 2

2 ( )
x ,
x, x + (x + )
:
(x + )2 = x2 + (x )2 x2 4x = 0
x(x 4) = 0 x
x = 4 :

E =

x+x+x+
1
x(x) =
x = 3
2
2

4 = 3 =

10

15 ( )
K .
A :
A1 A
A2 A1 K
A3 A2
B A3 K
AB 4-
K .

K (), AA1 = 2. ,
= = + , A2 A3 = 2A2 Z = 2( + 2) = 2 +4.
AB = A1 B A1 A= 2 +4 -2 =4.
2 ( ) P ( ) A1 A2
,
A, P, A3 K  , K ( ),
AP A3 .
, KA3 = KB K  A3 = K  A (
KA2 = KA1 ), AB  KK  .
, A3 AB,
AB = 2 KK  = 4 KQ, Q ( ) KK 
.
3 ( ) AA1 A3 A2
A1 BA2 A3 .
A3 A2 A1 A
1A
M K = K
M K = A1 BA
= AB
2 =
4
4
4
AB = 4M K .

http://www.mathematica.gr/forum/viewtopic.php?f=22&p=7575#p7575

1 ( )
...

16 ( )
, .
A1 , A, B .
K , ( ),
. A1 KB A3 KA2 , .
A1 B = A3 A2 . A .
11

1 ( ) 4 , , ,
:
,
K2 KN 2 = M 2 M N 2 .
, AB 2 AO 2 = EB 2 OE 2 .
2 O2 = E2 OE 2 .
=
AB 2 2 = EB 2 E2 .
AB 2 2 =
OB 2 O2 . EB 2 E2 = OB 2 O2 ..

.
2 ( )
+ OA)

OB
= 0 (EO
: EA





OB = 0. EO OB + OA OB = 0.(1)
http://www.mathematica.gr/forum/viewtopic.php?f=22&p=24231#p24231

O
+ O
O
= 0.(2) O
O
= O
2 =
EO
OB.
(1),(2)
2 = OA
OA

OB)

B
= 0
EO (O
= 0 EO
, .

17 ( ) OA =

OB =
=

( a,
)

 

=
|
||
|


|
|||+
.

| |



|
|||+

| |



) =

(,
=
(
, ) =
 
|||
|
+ |
|


( , ).

AOB


, .


, .

http://www.mathematica.gr/forum/viewtopic.php?f=23&t=3713


1 ( )  = |
|
| + |
| |||
,
, .
o =
2 ( Papel ) ao = |aa| ,

()

18

3KA+2KB+A =5K




KA =6 , KB =8 , K =5 . :
) .

)
v =KB+K A = 32
7

| |

1 (). :
= ... = http://www.mathematica.gr/forum/viewtopic.php?f=23&t=3606



|
|
o + o 1 , 2 ( ) )

o )
o
ao (ao +
3KA + 2KB + A = 5K
1+ao

=
=
: 2 =
o |
|ao ||ao +
|ao +o |

... = 1 . 1=2.
3KA + 2KB + A 3K 2K = 0


.
3 KA K + 2 KB K + A = 0
3 ( )
12






7
= 7 (3) (
K A K, A = 5 5 25




= 2 B
2 B
= 16 9 =
K, A = 2B
25
25


3A + 2B + A = 0

2A + 2B = 0

A = B
,
.

4K2 = 2AK2 + 2KB2 AB2


4 52 = 2 62 + 2 82 AB2
AB2 = 100
AB = 10
.
)

7
25 )

(1), (2), (3) 32 + 7 = 0 = 32


7

( ) 0 = (KB +

KB +
K)A = KB A + KA = Ao
A

K = A(o KB + o K) =
Ao
A
A
A

A(PB + P) .

A, PB + P ,

 


PB





, PB + P = 0 PB = P || = 


P
KB + K A = 0




PB 

KB A + KA = 0 (1)
PB = P < 0 =  }



P



8
KB A = KB A KB, A = 8 5 10
= , 82 62 = 2.10.(P) (P) = 7
5


32
(PB) = 75 + 5 = 32

5 : = 7

) K A =
32 (2) ( KB, A = B

19 ( )
200 5 ,
. 4,
20, 60, 50, 40, 30
20.
1. .
2. 5

20 .
() .
() (
), 5.

1. a .
: [a, a + 4), [a + 4, a + 8), [a +
8, a + 12), [a + 12, a + 16), [a + 16, a + 20).
20, :
20(a+2)+60(a+6)+50(a+10)+40(a=14)+30(a+18)
200

= 20
2a + 4 + 6a + 36 + 5a + 5a + 4a + 56 + 3a + 54 =
200 20a = 200 a = 10.
:

[10, 14), [14, 18), [18, 22), [22, 26), [26, 30).
2. X Y
: Y = (X +5)0, 8

Y = 0, 8X + 4
() Ry = Ymax Ymin = (0, 8 30 + 4) (0, 8 10 +

4) = 16

()
)
: 20, 60 , 50
, 40 ,30 .

() :
0, 8 10 + 4 = 12, :

http://www.mathematica.gr/forum/viewtopic.php?f=18&p=37954#p37954

() ( )

Y = 0, 8X + 4,

,

( )
13

[12, 16), [16, 20), [20, 24), [24, 28), [28, 32)

2. : P ( ) = P ( ) P ( ) = 1P ( )

2 2+2
2 = 1 = 4

3. : f (x) = x2xx+2
2 x , x
[2, +). , : P (A ) = f (), = 2, 3, ...
x 2, :

20 ( )
2 + 2 2 2
, 2.
.

f  (x) =

> 0, x > 2+3


x2 4x+1
= 0, x = 2+3
(2x2 x)2
< 0, x < 2+ 3

f


2, 2 + 3

2 + 3, + . , : 2 <

3 < 2 + 3 < 4 < 5 < ..., : f (2) > f (3)


f (4) < f (5) < ..., : P (A2 ) > P (A3 )
P (A4 ) < P (A5 ) < ...
8
1
: P (A3 ) P (A4 ) = 15
14
28 = 30 > 0
P (A3 ) > P (A4 ). : P (A4 )
P (A ), = 2, 3, 4, ....
= 4. = 4 P ( )

1. : :
.
2. : P ( ) = P ( );
3.
.

http://www.mathematica.gr/forum/viewtopic.php?f=18&p=34672#p34672

(.)
1. :

P (A ) =

2 +2
( 2 +2)+( 2 2)

(2x1)(2x2 x)(x2 x+2)(4x1)


(2x2 x)2

2 +2
2 2 .

21 ( )

72

31
1+ 3
+ i
2 2
2 2

http://www.mathematica.gr/forum/viewtopic.php?f=51&t=10381

22 ( )

2z 2 = 6iz + 3, z C

http://www.mathematica.gr/forum/viewtopic.php?f=51&t=10218

( )
z i

1 ( )

2 ( ) w = iz
: 2w2 = 6w + 3,
.

3 + 31

z = 1+
2 2

2 2
2

1+2 3+3
31
12 3+3
3 + 31
i
z 2 = 1+
=
+
2
i

8
8
8
2 2
2 2

3+i
2

3+i
3+2 3i1
3i+1
3i
4=
z
=
=
=
i
2
4
2
2

3i 3+i
3+1
6
4
2
z =z z =i 2
= i 4 = i..
2

 12
 3
z 72 = z 6
= i12 = i4 = 1

2z 2 = 6iz + 3 4z 2 = 12iz + 6 [(2z)2


2
] + 3 = 0 (2z 3i)2 ( 3i)2 = 0
12iz + (3i)
3i + 3i) = 0
(2z 3i
3i)(2z

=
(3+ 3)i

z =
2

z = (3 3)i
2

14

,
az 2 + bz + c = 0,
R, C (
, R -

, ,
C, ).
C ,
.

23 ( ) f
f (f (x)) = x2 x + 1, x R.
g : R R
g(x) + xf (x) x2 = 1, x R

http://www.mathematica.gr/forum/viewtopic.php?f=52&t=1591

( )
f g
.

f (f (x)) = x2 x + 1
x f (x) :

f (f (f (x))) = f 2 (x) f (x) + 1

f (x2 x + 1) = f 2 (x) f (x) + 1

x = 1 : f (1) = f 2 (1)
f (1) + 1 f (1) = 1.

g(x) + xf (x) x2 = 1

http://www.mathematica.gr/forum/viewtopic.php?f=52&t=3155

1 ( )

3f (x) + 5g(x) + 8f (x)g(x) = 0


(8f (x)+5)(8g(x)+3) = 15. f (x), g(x) < 0
8f (x) + 5 < 5 8g(x) + 3 < 3
x1 < x2 . f, g ,

8f (x1 ) + 5 < 8f (x2 ) + 5 8g(x1 ) + 3 < 8g(x1 ) + 3 (2)


8f (x1 )+5 > 0 v , (2) 15 < 15
.
8f (x1 ) + 5 < 0 8g(x1 ) + 3 < 0, :
- 8f (x2 ) + 5 < 0 8g(x2 ) + 3 < 0,
(2) 15 > 15,
- (5 >)8f (x2 ) + 5 > 0 (3 >)8g(x2 ) + 3 > 0,
15 < 15, .
2 ( ) (1) x1 , x2 x1 < x2 .
:
f (x1 ) < f (x2 ) < 0 g(x1 ) < g(x2 ) < 0 (2)
(1)

3f (x1 )+5g(x1 )+8f (x1 )g(x1 ) = 0 g(x1 )(5+8f (x1 )) =


3f (x1 )
3f (x1 ) g(x1 ) = 5+8f
(x1 ) , 5 + 8f (x1 ) < 0

x = 0

g(0) = 1
x = 1

(2)
3f (x )
g(x2 ) = 5+8f (x2 ) , 5 + 8f (x2 ) < 0
2
3f (x1 )
3f (x2 )
f (x1 )
g(x1 ) < g(x2 ) 5+8f (x ) < 5+8f (x ) 5+8f (x
) >

g(1) + f (1) 12 = 1

f (x2 )
5+8f (x2 )

f (x1 )(5 + 8f (x2 )) > f (x2 )(5 + 8f (x1 ))


5f (x1 ) + 8f (x1 )f (x2 )) > 5f (x2 ) + 8f (x1 )f (x2 ))
f (x1 ) > f (x2 ) , . (1)

g(1) = 1
.
24 ( ) f, g
R .

3f (x) + 5g(x) + 8f (x)g(x) = 0 (1)


R.

15

25 ( )
R

c d y
y

(f  (x))2 = f (x)


,



x.

y(x) =

http://www.mathematica.gr/forum/viewtopic.php?f=53&t=3568

y(x) =

( )
. y
. (a, b) y(x) > 0, x (a, b).
y
a, b
Rolle y  y
(a, b) (). y k

. y(k) = 0

x > k |y  | = y > 0 () y 

2 y = x + c1 , x > k

2 y = x + c2 , x > k
k c1 = c2 = k
c1 (-)=(+)

y(x) = 1/4(x k)2 , x k


x < k c2
1
y(x) = (x k)2 , x R
4
y(x) = 0, x R (
y ) ( y 

Darboux)
. (c, d) : y(x) = 0, x (c, d)

y(x) =

x < c

1
(x
4

c)2

1
(x d)2
4 1
2
4 (x c)

0
1
(x

d)2
4

x c
x > d
x d
x c
c < x < d
x d

c, d R
c<d

26 ( ) f :
R (0, +) ,
:f  = f f ;

http://www.mathematica.gr/forum/viewtopic.php?f=53&t=1801

( ) f (x) >
0 x R.
x f (x) f (f (x)) > 0, x
R f  (x) > 0 f R [1]
f (x) > 0 [1] f (f (x)) >
f (0) f  (x) > f (0) (f (x) xf (0)) > 0
f (x) xf (0) R
x < 0 f (x) xf (0) < f (0) 0f (0)
f (x) < (1 + x)f (0) x < 0 [2]
f (x) > 0 f (0) > 0.
x < 1 [2],

16

27 ( )
f [0, +) lim [xf (x)] = 0,
x+

(James Merryeld) cos(2x+2 sin 3x)


,

2x2

lim f (t) dt = 0

x+

x2

1
2

2x2

lim

x+ x2

1
2

g(x)
dx = 0.
x

2x2

x2





x > 0.

cos(2t 2 sin 3t) dt =

cos(2t 2 sin 3t) dt := I2 ,

x+

x2

1
2


/3

, x +, +
lim |g()| = 0. , .

G(y) :=

cos(2t 2 + 2 sin(3t 3)) dt =

2I = I1 + I2 =
cos(2x) cos(2 sin 3x) dx =

 5/3
cos(2x) cos(2 sin 3x) dx
(1).

1
|g(t)| dt = 2 x2 |g()| = |g()|,
x
(x2 , 2x2 ).


 2

 2x2
2x g(t)  2x2 g(t)
g(t)






dt

t dt 2 x2 dt =

x2
t
x2
x
1
x2

t=x+

:=I1

( ) xf (x) = g(x),
:

cos(2x + 2 sin 3x) dx




lim g(x) = 0,

cos(2x + 2 sin 3x) dx =


0

1
2


http://www.mathematica.gr/forum/viewtopic.php?f=54&t=7848

x+

I=

u=x+2/3

2I  =
5/3
4
cos(2 sin(3u 2)) du =
cos 2u
3
/3


|g(t)| dt [x2 , 2x2 ]

5/3

/3

4
cos 2u
3


cos(2 sin 3u) du

(2)

28 ( )

I :=

cos(2x + 2 sin(3x)) dx

2I

u=x+4/3

7/3

/3

5/3
/3

http://www.mathematica.gr/forum/viewtopic.php?f=54&t=8162

8
cos 2v
3

2
cos 2v
3

cos(2 sin(3v4)) dx =


cos(2 sin 3v) dv

(1) + (2) + (3)


17

(3).

6I =






4
2
+
cos
2x

cos
2x
+
cos
2x

cos(2 sin 3x)


dx =
/3
3
3
A+B




 5/3

0.

cos A + cos B =
AB
cos
.
2 cos
2
2
 2
: a, c Z c  a,
cos(ax +

=0

f : R R

T ,

a+T

f (x) dx a R.

f (x) dx =
a

b sin(cx)) dx = 0.

29 (Math Rider)
f : (0, +) R
f  (x) + x1 f (x) = x12 , x > 0 
A e, 1e .
) f
.
)

3

3

e dx

1
A Cf f (e) = 1e ln e+c
= 1e 1+c
e
e = e 1+c=
1c=0
f (x) = lnxx , x > 0. (
) f (0, +)
(ln x) xln x(x)
x
f  (x) =
= 1ln
x2
x2
f  ,

f :

xe dx

x > 0.
) g(x) =

x
1

f (t)dt, x > 0.

h : (0, +) R

 
1
ln2 x
h(x) = g(x) + g
x

(0, +).
)

x
1

x0+

2
ln t
dt + 2 2xf (x) =
t
x

f (0, e],
[e, +)
f (e) = 1e .

1
( lim f (x) = lim lnxx = lim
x ln x =

x
1

ln t
dt
t

x0+

x0+

(+) () = lim f (x) =


x+


(ln x)
+
1
+ = lim
(x) = lim x = 0 ).
(DLH)

x+

lim

x+

ln x
x

x+

) ) x > 0 :

f (x) f (e) lnxx lnee e ln x x ln e

x > 0.
ln xe ln ex xe ex (1) 0 < 3 < 2 < e
f
(0, e]

ln 3
ln 2
<

2
ln
3
<
3 ln 2
f ( 3) < f (2)
2
3
http://www.mathematica.gr/forum/viewtopic.php?f=55&t=6685

2
3
3
ln 3 < ln 2
3 < 2 3 (
ln ( 3) < ln 2
ln x ). (1)

x xe 0 x > 0. , (
1
1

e
) x > 0 : f (x) + x f (x) = x2

x
), 3 2 3 :
xf  (x) + f (x) = x1 xf  (x) + (x) f (x) = x1

2 3
2 3
2 3
,
(xf (x)) = (ln x) xf (x) = ln x + c f (x) = ln x+c
x
(ex xe )dx 0
ex dx
xe dx 0
x > 0.
3
3
3
18

2

2

2

2

x = 1 t(x) = 0 ( (3.
ex dx t (0, +)
3
3
3
3
t (x) = (x) ln2 x+x(ln2 x) +(2x) 2(x) ln x2x(ln x)
2 3
2 3
3
3


(2) =

ex dx
xe dx
ex dx
xe dx

3
3
ln2 x + 2x ln x(ln x) + 2 2 ln x 2x x1 0 =
2 3
2 3
x > 0.
ln2 x + 2x ln x x1 + 2 2 ln x 2 =
x
x ln t
ln2 x + 2 ln x + 2 2 ln x 2 = ln2 x > 0
f (t)dt =

) g(x) =
t dt , x > 0.
1
1
x > 0 x = 1.
g (0, +) t (0, +) ( t (x)
[ f (t) = lnt t (0, +) (0, 1) (1, +) t(x)
].
x = 1). x = 1
1
1
 1  x
x ln t
t(x) = 0.
g x = f (t)dt =
t dt

xe dx

ex dx

xe dx

(0, +)
(0, +)
(x) = x1 g(x). h
(0, +)

 




h (x) = g(x) + g x1 ln2 x =




1
x
x


ln t dt + ln t dt + ln2 x
=
t
t
1

ln x
x

ln

1
x

1
x

 1 
x

x+



2 ln x(ln x) = lnxx + x ( ln x) x12 2 ln x x1 =
ln x
ln x
2 ln x
2 ln x
2 ln x
x + x x = x x =0
h (x) = 0 x > 0. h
(0, +).
) ) h(x) = c, c R
x > 0.
 
1
h(1) = g(1) + g 11 ln2 1 = 2g(1) 0 = 2 f (t)dt =
2 0 = 0.

30 (
)

f : R R
f 
R. x0 R f  (x0 ) > 0,
lim f (x).

h(1) = c0 = c.
 
h(x) = 0 g(x)+g x1 ln2 x = 0 g(x)+g x1 = ln2 x,
x > 0 (2)
:

http://www.mathematica.gr/forum/viewtopic.php?f=55&t=7807&p=44481#p44481

( ) x > x0 .
f [x, x0 ]
,
(x, x0 ) f  () =

f (x)f (x0 )
. f 
xx0
> x0 f  () > f  (x0 )
xx0 >0
f (x)f (x0 )
> f  (x0 )
xx0
f (x) f (x0 ) > (x x0 ) f  (x0 )
f (x) > f (x0 ) + (x x0 ) f  (x0 )

x R :

R :

f (x0 ) + (x x0 ) f  (x0 ) 0 f (x0 ) + xf  (x0 )


x0 f  (x0 ) 0

0 )f (x0 )
,
f (x0 ) + xf  (x0 ) x0 f  (x0 ) 0 x x0 f (x
f  (x0 )
1
1
x
x
x
, R
x ln t



ln t
ln t
ln t
2
t dt + 2 2xf (x) = x
t dt
t dt +
t dt + f (x0 ) + (x x0 ) f  (x0 ) > 0 x (, +).
1
1
1
1

 
(2) f (x) > f (x0 ) + (x x0 ) f (x0 ) > 0,
2 2x lnxx = x2 g(x) + g x1 + 2 2 ln x = x2 x (, +),
1
ln2 x + 2 2 ln x = x2 xln2 x + 2x 2x ln x = 2 0 < 1 <
f (x)
f (x0 )+(xx0 )f  (x0 ) x (, +).
xln2 x + 2x 2x ln x 2 = 0 (3)
1
lim f (x )+(xx
= 0 ,

0
0 )f (x0 )
x+
t(x) = xln2 x + 2x 2x ln x 2
, x > 0. = 1 t(1) =
f (x)>0
1
1 ln2 1 + 2 1 2 1 ln 1 2 = 0 + 2 0 2 = 0.
lim f (x)
= 0 im f (x) = +.
x+

19

x+

31 ( ) :

32 ( )

2
2x = y + y
2y = z + 2z

2z = x + x2

A = {[

2x2 + 10x + 19
]/x R}
x2 + 5x + 7

( )

http://www.mathematica.gr/forum/viewtopic.php?f=49&t=10228

( )
x, y, z = 0 x, y, z > 0.
$
:

http://www.mathematica.gr/forum/viewtopic.php?f=49&t=11485

( ) : K(x) = 2xx2+10x+19
,x
= 2 2 2x 2 2 x 2
+5x+7
2

R : x + 5x + 7
: y 2, z :
2
5
D = 3 < 0 : K(x) = 2 + x2 +5x+7
,x R
:


2
: x2 + 5x + 7 = x + 52
+ 34 , x R :
2 2 2
3
1
2
x + 5x + 7 4 , x R : 0 < x2 +5x+7

x+y+z = + +
x y z
4
5
20
3 0 < x2 +5x+7 3 , x R : 2 <
5
20
2

2 
2  2 + x2 +5x+7 2 + 3 = 8 3 2 


x 2, x 2 , y 2, y 2 , z 2, z 2



y+

2
y

2
y

x = x2 , y = y2 , z = 2z : (x, y, z) =


( 2, 2, 2)



( 2, 2, 2)

(x, y, z)

5
2 + x2 +5x+7
= 2, 3, 4, 5, 6, 7, 8 :
A = {2, 3, 4, 5, 6, 7, 8}

. :

=
K(x) 2, 3, 4, 5, 6, 7, 8.

33 ( )
x, y, z > 0 x + y + z = 1

3xyz(xy + yz + zx) + 2xyz (xy + yz + zx)2

1
(
)
( :
xyz

(xyz)2 ). :


3

http://www.mathematica.gr/forum/viewtopic.php?f=50&t=454

20

1 1 1
+ +
x y z



2
1 1 1 2
+
(
+ +
xyz
x y z

34 ( )
1
f : R R
x .

a=

1 1 1
+ + = 1 ab + bc + ca = abc,
a b
c

f (x + y) + f (x)f (y) = f (xy) + 2xy + 1

3(a + b + c) + 2abc (a + b + c)2 .


( abc )

abc = ab + bc + ca

3(a + b + c)

(a + b + c)2
.
3

(a + b +
3

c)2

9 a + b + c,

: a + b + c = (a + b + c)(

1 1 1
+ + )9
a b c

- .
2 ( )

%
%
%
%
(
xy)2 =
x2 y 2 + 2
xyz(
x)
%
%
x + 2xyz
=
x2 y 2
%
=
(x3 y 2 + xy 2 z 2 + x3 z 2 ) + 2xyz.

x y .

http://www.mathematica.gr/forum/viewtopic.php?f=50&t=277

( ) y = 0
f , f (0) = 1.
x = 1, y = 1 f (1) = 1 f (1) = 0.
f (1) = 1, x = 1
f (x) = 2x1 . f (1) = a = 1,
f (1) = 0 (x, y) = (z, 1) (x, y) =
(z, 1) f (z + 1) = (1 a)f (z) + 2z + 1
f (z 1) = f (z) + 2z + 1. f (z + 1) =
(1 a)f (z 1) + a(2z + 1)

f (x) = (1 a)f (x) + a(2x 1) ()


x x

f (x) = (1 a)f (x) + a(2x 1).

%
%
xy)
(x3 y 2 + xy 2 z 2 + x3 z 2 ) 3xyz(

(a2 2a)f (x) = 2a2 x (a2 2a)

%
xy).
x3 (y 2 + z 2 ) 2xyz(

a 0 2, f (x) =
2ax
a2 1
a
= 2 f (x) = x 1. a = 2
-

%
%
%
%
3 2
2
3
2
2x yz = 2xyz(
x ) 2xyz
xy a = 0. a = 0 (*) f (x) = f (x)
x (y +z )
(x, y) = (z, z) (x, y) = (z, z) : f (2z) + f 2 (z) = f (z 2 ) + 2z 2 + 1
.
1 + f 2 (z) = f (z 2 ) 2z 2 + 1. :
f (2z) = 4z 2 1, f (x) = x2 1.
.
f (x) = 2x 1, f (x) = x2 + 1, f (x) = x2 1.

35 ( 2010)
c1 (O1 , r1 ) c2 (O2 , r2 ) , .

c1 (O1 , r1 ), c2 (O2 , r2 )
r1 < r2 , M A < M B .

21


x
f (x)
: f
= , x, y R (2).
f (y)
y

x
1 y f (y)
 
1
1
: f
=
, y R (3)
y
f (y)
1
(3)
(2) y
y
: f (xy) = f (x)f (y), x, y R . x =
y = 1 f (1) = 1 f (1) = 1.
f (1) = 1 f 1 1.
f (1) = 1 .
(0, +) f .
g(x) = ln f (ex ),
g(x + y) = g(x) + g(y)


http://www.mathematica.gr/forum/viewtopic.php?p=57805

( )
: O1 ABO2
O1 A, O2 B O1 A < O2 B . M
O1 A = O1 M O2 M =
O2 B , AM < M B .
O1 (0, 0), A(0, a), B(b, a), O2 (b, c) a, b, c > 0
M (x, y). |OA| = |OM | , |O2 M | =
|O2 B| : a2 = x2 + y 2 (1),
(b x)2 + (y + c)2 = (a + c)2 (2). (1)
(2)
2c(a c) = b(b 2c). M
a > y b > 2x (3).
, AM < BM
: x2 + (a y)2 < (b x)2 + (y a)2
a2 = x2 + y 2 ,
0 < b(b 2x),
(3).
: (O1 , r1 ) (O2 , r2 )
() , .


(
. ), g(x) = ax
a R. x > 0
ln f (ex ) = ax f (x) = xa a R.
f (f (x)) = x
a2 = 1 a = 1 a = 1 f (x) = x

g .

1
, x R . f .
x
(0, +) f (x) = x, x (0, +)
1
f (x) = , x (0, +).
x
f (x) =

36 ( 2010)

f : R R



1
=
f (f (x)),
: f
y
x, y R (0, +).
f (x)
f (y)

http://www.mathematica.gr/forum/viewtopic.php?p=35560

1 ( ) x = y

(i) f (x) = x, x (0, +)


f (x) = x, x R . x < 0 y < 0.
xy > 0 f (x)f (y) = f (xy)
f (x)f (y) = xy y = 1
f (x)(1) = x f (x) = x
x (, 0). ( x > 0 y < 0.
f (x)f (y) = f (xy)
xf (y) = f (xy) y = 1 : f (x) = x
x > 0 f (x) = x x < 0

1
.
f (f (x)) f (f (x)) = f (1)x, x
x
x < 0
R . a = f (1)

y > 0.)
1
f (f (x)) = ax,
x R .
x =

a
1
  
(ii) f (x) = , x (0, +)
1
x
f f
= 1 (1).
1
a
 
f (x) = , x R
1
x
(1)
y = f
.
a
1
1
: f (f (x)) =  1  f (f (x)), x R
f (x) = x, x R f (x) = , x R
f a
x
1
2 ( ) (1) y = x :
 1  ax, x R x = 1 :
ax =
f (f (x)) = x f (1) (2). (2)
  f a
1
= 1 (1): f (1) = 1 1 1 f . (2) x = 1 : f (f (1)) = f (1)
f
a
1 1 f
 f (1)= 1. f (f (x)) = x
a = 1. (1) : f (f (x)) = x, x R
x
f (x)
x, R (3) f
= , x, y R
( 1f (y)
y
1 R ) (4)
: f (1) =

22

f (1)
f (1)

f (0, +) :
x
>
1 f
(x) > f (1) = 1 > 0 0 < x < 1


1
1
1
= f (f (1)) ,
1 f
>1f
> f (1) = 1 > 0 (5) f (x) > 0.
f (1)
x
x
, f (1) = 1.
x > 0 f (x) > 0. f



(, 0)
f (x)
(4) y = x : f
= 1 = f (x) < 0 x < 0. , f
f (x)

f (1) f : 1 1 f (x) = f (x), R . ,


(3), :
f .
f (x) = x x R .


1
1
f (0, +)
(4) x = 1, y = x :f
=
f
(x)
x
1
& 
'
 
 
: f (x) =
1
1
1
1
x
f f
=f
f
=
(5).
x R .
f (x)
x
x
f (x)
(4) x = 1, y = 1 : f

37 ( ) (
) :

38 ( )
An = {k {1, 2, . . . , n} : 2k 1}

(n
(k 1)k
lim (k=1
n
k
n
k=1 (k + 1)

an = |An |

lim

n
http://www.mathematica.gr/forum/viewtopic.php?f=59&t=1354

an
n

( )
(
(
an = nk=1 (k1)k bn = nk=1 (k+1)k . http://www.mathematica.gr/forum/viewtopic.php?f=59&t=5619
(bn ) ,
k
. ,
( )
)
ln 10
ln210 
1 k ln 2 , ln 2 + 1  N.
an+1 an
lim
=

n

bn+1 bn

nn+1
2
= lim 1 +
n+1
n (n + 2)
n
n
lim


n

(k1)k


1

2
n+2

Cesaro-Stolz,

n+2

+
n ln 2
+ O(1)
an =
ln 10
*

= e2

ln 2
an
=
.
n n
ln 10

lim

e2 .

( ):
: Cesaro-Stolz

: ln 2/ ln 10
(an ), (bn )
(bn ) , . Weyl :
a
an
, 0  a  b  1 An = {k
limn bn+1
,
n+1 bn
|An |
an
limn bn .
= b a.
{1, 2, . . . , n} : a  {k}  b}, lim

lim

k=1
n

k
k=1 (k+1)

23

39 ( )

http://www.mathematica.gr/forum/viewtopic.php?f=10&t=2046

K1 = {a + b p : a, b Q}

( )
sn .

K2 = {a + b q : a, b Q}
n . , p, q , n- 1
sn = 1.
.
n = pk k > 1 p .
n-
http://www.mathematica.gr/forum/viewtopic.php?f=10&t=1881
pk1 - , 0,
sn = 0.
( )

n = pq gcd(p, q) = 1.
 
 
. : Q
p Q q k 1  s  q 1  r  p (s, q) = 1
, (r, p) = 1 k sp+rq mod pq . ,

=
()2 = p.
 p

= () Q q 2 = p. = x + y q
pq
q
p


%
%
%
2i(sp+rq)

 2 x, y 2 .
2ik

pq
x + y q + (2xy) q = p xy = 0 spq =
e
=
e pq
= sp + sq .


p, p/q

40 ( )

n
%

2ik
n

k=1
(k,n)=1

k=1
(k,pq)=1

s=1
r=1
(s,q)=1 (r,p)=1

sn
n
(1)r , n
r .
(n) Mobius.

41 ( )
f : R R , x0 R,

f (x0 ) lim f (x)


xx
0

24

|f (x)qy | > s > 0 s (


x). qx I
x0 (qx , qy ).

f (x0 +) lim f (x)


xx+
0

R {}.

f .

http://www.mathematica.gr/forum/viewtopic.php?f=9&t=444

1 ( ) x
f n N.
f (x) f (x+) = (x, n) :

,
x D (rx , ry ) :

/ I , qx = rx , x I {x0 },
x
qy = ry .
42 ( )


+ 
%
3n 1
n 8n
n=0

() y1 (x 2, x) f (y1 ) f (x) <


1/n

http://www.mathematica.gr/forum/viewtopic.php?f=9&t=2340



() y2 (x, x+2) f (y2 ) f (x+) <

 

1/n.
(1 + z)n
n
1
dz ,

=
2i C z k+1
k
z1 (x , x) y1 |y1 z1 | <
0.

min{, x z1 }. y1 (x 2, x)

n

%
%
(1 + z)3
(1 + z)3n
1

|f (y1 ) f (z1 )|  |f (y1 ) f (x)| + |f (x) f (z1 )| <


8n z n+1
z
8z
n=0
n=0
2/n.

1
z

z1 (x , x)

1
3
1 (1+z)
8z

8
8z(1+z)3

= z 3 +3z8
2 5z+1 =

8
(z1)(z 2 +4z1)

|f (z1 )f (z1 +)|  |f (z1 )f (z1 )|+|f (z1 )f (z1 +)| < |z| = 1/2.

4/n


%
1 %
(1 + z)3n
3n 1
z2 (x, x + ) |f (z2 ) f (z2 +)| < 4/n.
=
dz =
n n+1
2i
n 8n
|z|=1/2 8 z
2(x, n)
n=0
n=0

dz
x, z , ( 8
=
2i |z|=1/2 (z 1)(z 2 + 4z 1)
x)





4
1
f (z) f (z+) < 4/n (1).
; 2 + 5
=
,
8 Res
2
(z 1)(z + 4z 1)
3 55
An = {x : |f (x) f (x+)|  4/n}. (1)


1
1
x An z (x(x, n), x+(x,
n)) z = x, z
/ Res
;
2
+
5 = d (z 3 +3z 2 5z+1)|


(z1)(z 2 +4z1)
1
1
z=2+ 5
dz
An . x 2 (x, n), x + 2 (x, n)

1
1
x An  .

 x An =
.

= 
1
1
qx Q x 2 (x, n), x + 2 (x, n) .
3z(z + 2) 5 z=2+ 5 2 5 3 5
f : An Q

f (x) = qx 1-1 An . 

arcsin x
f
(x)
=
sin

3
f An


%
1
3n 4n x2n+1
f f (x) =
.
3 n=0 n 27n 2n + 1

.

2 ( ) D R
f .

x0 D ymax = max {f (x0 ), f (x0 +), f (x0 )}
ymin = min {f (x0 ), f (x0 +), f (x0 )}.
ymin = ymax qy
(ymin , ymax ) {f (x0 ), f (x0 +)}.

f (x)
 = x

sin arcsin
3
 arcsin
 f (x) =
1
x 1

3 cos
3
1x2



x

3 1 x2 f  (x) = cos arcsin


3


x


2
1 x f (x)
f (x) =
3
1 x2


x 1

13 sin arcsin
3
1x2

I (x0
, x0 + ) , x 0 < |x x0 | < 9 (1 x2 ) f  (x) 9x f  (x) = f (x).
25

9 (1 x2 ) f  (x) 9x f  (x) + f (x) = 0


f (0) = 0 f  (0) =

f (x) = sin

 arcsin x 
3

1

3

( 3n
n=0


cos


y 2n

= cos

 33 y  

arcsin

 33 y  

arcsin

3 3y
2

2 =

2
.
427y 2





%
1 % 3n 4n x2n+1
1
3n
1
, y =
g(x) =

=
3 n=0 n 27n 2n + 1
n 8n
8
n=0

, - 42
 33 
1

cos
arcsin
. 
,
3
4 2
5

 33 
 arcsin x 
4 2
4
1

cos

arcsin
=
sin
=
3
3
4
2
 n 2n+1
5
3 55

%

3n 4 x
1

 33 
1
.

2
cos
arcsin
=
n
3
n 27 2n + 1
3

4 2
n=0
 33  3+5
1
1
 n  2n+1 

cos
arcsin

= 42
%
3
  arcsin x 
x
3 5
4 2
1
3n 4
=
=
sin



3

3
n 27n 2n + 1
 33 
3+ 5
3+ 5 3
n=0




= ... =
cos arcsin 4 2
3
= 4

1 % 3n 4n 2n
4 2
4 2

5
3
n 27n
n=0






4 2
%
 arcsin x 
1
3n
2x 2n
,

 2

= cos
.


3

2
3 3
5
n
3
3
1

x
3
3

n=0

= ... = .
sin arcsin 4 2 = 1

4 2
4 2
2x
3 3y
y = x =

2
3 3

43 ( )

,
( ).

f (x)
(ax)2

b
= (ax)
2

f (x)
ax

b
ax

f (x)
ax

b
= ax
+c

f (x) =b + c (a x).
,

, .
2 ( )
( C 2 )
http://www.mathematica.gr/forum/viewtopic.php?f=11&p=54355#p54355
x
=x
(s) , s
. ,
1 ( ) :
M .
x
= m
x , m s.
, , ,
,
M .
x
 = m x
 + m
x .
f : D , D : .
x



xo D , N (a, b).
x
x
= 0,
: y = f  (x0 ) x + f (x0 ) x0 f  (x0 )m
. x = 0.
xo D, f  (x0 ) a + f (x0 ) x0 f  (x0 ) = b. x
 = 0, x
=a
+ sb, a
, b

: f (x) a + f (x) .

x f  (x) = b. f  (x) (a x) + f (x) =b

f  (x)
ax

+ .
26

44 ( ) ( ) X Rn
x X .
Y X :

1 m n + 1
2 m n + 2 t1 ....
tr < 0 tr+1 ... tm

|Y |  n + 1

x Y .

x=

http://www.mathematica.gr/forum/viewtopic.php?f=11&p=42317#p42317

m
(

i=1

i=1
m
(

i=1
m
(
i=1

ti xi = 0 p :

ci xi =

m
(
i=1

ci xi + p

m
(
i=1

t i xi =

m
(
i=1

(ci + pti ) xi

(ci + pti ) = 1 p

ci + pti 0 i.
p ct i ti > 0 p ct i ti < 0
i
i
p
ct i ti < 0 i r

( ) x
X . x1 , ...,xm X
c1 , ...,cm 1
x =

m
(

ci

p = t 0 ci +pti
i0

i0 x m 1 X
m .

ci xi

m x

m X .

45 ( )
()

()|
()


()

3.

n (n) = 9(11a + b) 0 (mod 9)



n (n) = (n) p (n) = (n)(p 1).
(n)(p1) 0 (mod 9)
((n), 9) = 1 p 1 0 (mod 9).
, p p 1 0
(mod 2)

p 1 0 (mod 18).

(3)

(1) (3) p
=
19, 37, 73, 109, 127, 163, 181, 199.
( ) n =
p = 163, 181, 199 (1) ( n
abc = 100a + 10b + c. (n) = a + b + c, ) (n) = 5
0 a, b, c 9 a = 0.
.
p = 109, 127 (n) = 5 7
http://www.mathematica.gr/forum/viewtopic.php?f=63&t=6385

.
p = 73 (n) = 5, 7, 11, 13
(1)
n = 7 73 = 511 n = 11 73 =
803.
p = 37 (n) = 5, 7, 11, 13, 17, 19, 23
(2) n = 11 37 = 407,

n = (n) p, p 3

(n) = a + b + c 3

27

n = 13 37 = 481 n = 17 37 = 629.
p = 19 (1)

( )

n = 19(n),
100a+10b+c = 19a+19b+19c, b = 9a2c.
a 4 . a = 3
b = c = 9 .
a = 2 a + b + c = 2 + 18 2c + c = 20 c
c = 1, 3, 7, 9
n = 13 19 = 247 n = 11 19 = 209.
a = 1 a + b + c = 10 c
c = 3, 5
n = 7 19 = 133.

,
1. M N m > M
(m + 1)
1 m
k .

1
k+1
k + 1/2
> >
.
m
2
2m
2

2m + 1
2m
2m
< 2<
<
.
k+1
k + 1/2
k+1

n = 133, 209, 247, 407, 481, 511, 629, 803


( k

1
1
2m1 , > )
2
2
46 ( )

(k
+
1)
2 = 2m (k + 1)

2m .
an = [n 2] n N
m k ,
2.
m,
, 2,
an = [n 2], n N .

http://www.mathematica.gr/forum/viewtopic.php?f=63&t=2591

47 ( )
f : [a, b] R.
n

d2 d1 = d3 d2 = ... = dn dn1 , n N
[a, d1 ] , [d1 , d2 ] , ..., [dn1 , b]
:

1 , 2 , ..., n (a, d1 ) , (d2 , d1 ) , ..., (dn1 , b)

1 < 2 < ... < n

(n

j=1 f

[a, b] :

1
dj dj1

f  (1 ) + f  (2 ) + .... + f  (n )
f (b) f (a)
=
ba
n

n
%

( ) [d1 , dn ]
n :
[d1 , d2 ] , [d2 , d3 ] , ..., [dn1 , dn ] :
28

i)

n
j=1 f

(n

f (dj )f (dj1 )
=
dj d
j1
)f (d1 )
f (dj1 ) = f (ddjnd
j1

j=1

(dj )

dj dj1 = dn d1 = n (dj dj1 )

j=1
http://www.mathematica.gr/forum/viewtopic.php?f=27&p=61378#p61378

 (

1% 
f (i )
n

j=1

f (dn ) f (d1 )
dn d1

. a = d1 , b = dn

f (dn ) f (d1 )
1

n
dj dj1

48 ( ) A
A2 A + I = O,
B = A kI
k R
A.

http://www.mathematica.gr/forum/viewtopic.php?f=27&t=11183

( ) A = B + kI ,
(B + kI)2 (B + kI) + I = 0

B(B + (2k 1)I) = (k2 k + 1)I (*).


k 2 k + 1 = (k 1/2)2 + 1/4

1
4

, (*),
, B 1 =
(B + (2k 1)I)/(k2 k + 1) = (A+ (k 1)I)/(k2 k + 1).
. 2 (
)
: B
k R, 0 = v Rn Av = kv .
(k 2 k + 1)v = 0, v = 0
k 2 k + 1 > 0 k R.

49 ( )
(, )
limxa+ f (x) = limx f (x) = ,
(, ) f  () = 0.

http://www.mathematica.gr/forum/viewtopic.php?f=61&t=3018&view=next

1 ( )
g :
g(x) = ef (x) , x (, ) g() = g() = 0.
Rolle [, ] g .
+, g : g(x) = ef (x) , x
(, ) g() = g() = 0.
2 ( )
limxa+ f (x) = :
( ba
> 1 > 0)(x (a, a + 1 ))(f (x) < 0)
2

x1 (a, a + 1 ) f (x1 ) = 2f (a + 1 ). ,
lim f (x) = :

[x3 , x2 ]. 2f (a + 1 ) < 2f (b 2 ).
1 ( )

, .
.
1) g ,
. ..
g(x) = 1+f12 (x) a < x < b g(a) = g(b) = 0.
( arctan)
g(x) = arctan f (x) a < x < b g(a) = g(b) =

/2.

2) :
y = c
y = f ((a + b)/2).
f , x = (a + b)/2. ,
x = (a + b)/2 f (x) > c
limxa+ f (x) c, . .
p, q p < (a + b)/2 < q
f (p) = f (q) = c. Rolle [p, q].
xb

( ba
> 2 > 0)(x (b 2 , b))(f (x) < 0) .
2
,
x2 (b 2 , b) f (x2 ) = 2f (b 2 ). :
2f (a + 1 ) = 2f (b 2 ) Rolle ( + - )
[x1 , x2 ] ( x1 , x2 .
1 , 2 3 ( )
a+b
) . .. .

2
2f (a + 1 ) > 2f (b 2 ), - : f 1 (a, x1 ) x3 1 ( ) (, )
f (x3 ) = 2f (b 2 ) . Rolle
29

f 1-1 c (**)
c
c
x1 , x2 (, ) x1 =

x2 f (x1 ) = f (x2 ) f  2c+1
, x f  12 2c+1
=
Rolle [x1 , x2 ] [x2 , x1 ] , () f (x) . ,
.
f (x) ,
(**) f  1/2.
.

50 (
)

f : (0, +) (0, +)
g g(x) = f 2 (x)+f (x), x >

1
f (x)
0, lim
=
x+ x
2
http://www.mathematica.gr/forum/viewtopic.php?f=61&p=56890#p56890

1 ( )
f = g = (f 2 + f ) = 2f f  + f  (*).
f (x) = 1/2 (*) 1/2 = 0.
(*)
f  = 2ff+1 > 0. (**)
f x
c +.


.
f
De lHospital (x) = f  ,
1/2.
2 ( ) , f
: ,
g g
, ,
lim g(x) = k > 0 g(x) < k, x > 0
x+

g , g  = f  = 2f +1 > 0,
g( 1+n
2 ) <

g(1)+g(n)
, n
2

g(1) + g(n)
)k
lim (
n
2

N lim g(

g(1)+k
2

1+n
)
2

k g(1), .

51 ( ) Lagrange
: , ,
5, 1974
f (x)
1,
x1 , x2 , ..., x x,
y1 , y2 , ..., y , :

y1
f1 (x) = (x1 x2 )(x1 x

3 )...(x1 x )
(x x2 ) (x x3 ) .... (x x )

y2
f2 (x) = (x2 x1 )(x2 x

3 )...(x2 x )
(x x1 ) (x x3 ) .... (x x )

..... :

f (x) = (x x1 )(x xy2 )...(x x1 )


(x x1 ) (x x2 ) .... (x x1 )
:

f (x1 ) = y1 , f (x2 ) = y2 , ..., f (x ) = y

f (x) = f1 (x) + f2 (x) + ... + f (x)


:

http://www.mathematica.gr/forum/viewtopic.php?f=60&p=11978#p11978

f (x1 ) = f1 (x1 ) + f2 (x1 ) + ... + f (x1 ) = y1


f (x2 ) = f1 (x2 )+f2 (x2 )+...+f (x2 ) = 0+y2 +0+...+0 =
y2
:

(
)

f1 (x), 1 :
f1 (x1 ) = y1 f1 (x2 ) = f1 (x3 ) = ... = f1 (x ) = 0.
:
f1 (x) = c (x x2 ) (x x3 ) ..... (x x )

f (x ) = f1 (x )+f2 (x )+...+f (x ) = 0+0+....+y = y


f (x) 1
. g(x) ,
:

f1 (x1 ) = y1 = c (x1 x2 ) (x1 x3 ) ..... (x1 x )

f (x1 ) = g(x1 ), f (x2 ) = g(x2 ), ....f (x ) = g(x )

:
30

f (x) g(x).

http://www.mathematica.gr/forum/viewtopic.php?f=60&p=55231#p55231

f (x)
.
: . , 1977

( )
, p(x) = a2n x2n + + a0
a2n > 0.

http://www.mathematica.gr/forum/viewtopic.php?f=44&t=9776

P (x) =

m
i=1

(x zi )(z zi )

(x i )2mi =

i=1




1 ( )
K
 

( 100), 
k
k

89 (11 100).
a2m (x zi )
a2m (x zi ) =
89 11
i=1
i=1



89 ( 78) ... ,
L
(A(x) + iB(x)) (A(x) iB(x)) = A2 (x) + B 2 (x)
.
1. K zi , zi
:1,12,23,34,45,56,67,78,89
i .
( ) ,

2. L zi , zi .
.


3. A(x) , B(x) R[x].
67 (
4. (x i )
78) .
f (x) = (x )2k+1 g(x) g() = 0,
(
g ,
)

f ,
.
52 ( ) (
17 Hilbert). f (x)
, f (x) 0
x R. A(x), B(x)
,

f (x) = (A(x))2 + (B(x))2


x R.

Prasolov Polynomials, Springer, 2004



.
1967 T. Motzkin
F (x, y) = x2 y 2 (x2 + y 2 3) + 1.

.

31

http://www.mathematica.gr
. .

Leonardo da Vinci
(32-) . 30
. -
-
- quasiregular ,
( - ).
(0, 0, ),



, 1+
,
12 ,
2
2

1+ 5
.
2

:http://en.wikipedia.org/wiki/Icosidodecahedron
:

mathematica.gr
(http://www.mathematica.gr) .

mathematica.gr


1. (Mihalis_Lambrou)
2. (nsmavrogiannis)

9. (swsto)
10. (achilleas)
11. ( )
12. (Demetres)

3. ( )
4. (m.papagrigorakis)

1. (spyros)

5. ( )
6. (Rigio)
7. ()

1. (grigkost)
2. (cretanman)

2. (vittasko)
3. (p_gianno)
4. (kostas.zig)
5. (exdx)
6. ( )
7. (mathxl)

1. ( )

8. (mathnder)

2. ()

9. (mathematica)

3. (nkatsipis)

10. ( )

4. ( )

11. (rek2)

5. (chris_gatos)

12. (hsiodos)

6. (gbaloglou)

13. ( )

7. (R BORIS)

14. (bilstef)

8. (dement)

15. (xr.tsif)

1 ( )

:

.
.
, ,
,

. , ,
,
.
.

-
,
. ,

.
,

.
,

. ,
,
.


. , ,

3 ( )
30% , 20
30% .

;

7 ( )
a, b, c

2 ( papel)

.

;

4 (
)

9
1, 2, 3, ..., 8, 9


.
.

x;

a+b+c = 0

1
1
1
+
+
=0
b2 + c2 a2 c2 + a2 b2 a2 + b2 c2

8 ( ) a b

a
a+1
a + 2009
+
+ ... +
= 2010
b
b+1
b + 2009
a = b

9 ( ) > 2
> 2 > + .
10 ( )
m

|2x |2x 1|| = m2 x

5 ( KARKAR )
, E1 .
, E2 .

E2
E1

6 ( )
.
7 .

2 .

: 1...6
.

11 (
)
B = 45
= 30 .
= 15 .
B
12 (
)

,
, ,

.

19 ( )
f : R R :

f (x) =
13 (
:

2
2
( 2 + 1)(x+1) + ( 2 1)(x+1) = 2
14 ( )

3x+1 9x + 3 5x 25x = 15x + 3

15 ( KARKAR)
AB a, O, M
O. M
A N . N N,
A . A
a .
16 (
)

28

.
.

ax2 +x+b
x+3

, x = 3
, x = 3

f
y  y
2 , .
. a = 1 b = 6.
. f x = 3.
. f

x x

20 ( )
x, y, z, x < y < z <
3, 3 4.
) x = 1 = 5
)

5
y z .
2

)
6
x1 , x2 , x3 , x4 , x5 , x6
6


xi = 38

i=1

6


x2i = 244

i=1


10 .

, ,

17 ( )
AB. E Z
AB B, ,
E Z A.
18 ( )


, ,
x

+x +
= 0
x2

0
) 2 4
)
2

=0

//

21 ( ) a, b, c 1
: a + b + c = 1
:
) ab + bc + ca = abc
) (1 a)(1 b)(1 c) = 0
1
1
1
) a2009
+ b2009
+ c2009
=1
) a, b, c,
, .
22 ( )
x2 x + = 0, , R
z1 , z2 C R.
, : z13 + z22 = 1

x, y R. :
) f (0) = 0 f (x) = f (x)

xR

) N x1 , x2 , ..... , x R
:

f (x1 + x2 + ..... + x ) = f (x1 ) + f (x2 ) +


..... + f (x ) (2)
 
f (1)
) f 1 = N .
) R, f (q) = q
q Q.

, ,

25 ( Stelmarg )

P (x) = x2n+1 2x + 1, n N, n 2
(0, 1)
( )
:
(0, 1)
(
, -
). xn ,
(0, 1), (xn ) limxn = 12 .
26 ( ) f : R
R f (x) > 0, x R
y R y(a) = k
R
y
, a, k, f (t)
,

y(x) f (x)y 2 (x) = 0, x R

, ,

27 (


F (t) =

sin x
dx
1 + x2

t > 0.
, ,
,

23 ( Iason Pap.)
f : R R
f (f (x)) = x3 x R
24 ( )
f : R R

f (x + y) = f (x) + f (y) (1)


2

28 (

1
I=
0

xn (2 x)n dx = 22n

1
0

xn (1 x)n dx = J

. Juniors

29 ( )
ABC, (AB = AC)
 = 20 .
A
AC D,

 = 60 AB
DBC
 = 30 .
E , ECB

EDB
30 ( ) f, g :
R ,
x, y [0, 1] ,

33 (
)

f : R R
f (f (x)) + f (x) = x x R.
34 (

1
4

. Seniors

31 ( ) a, b, c

a+b+c= 3
:

b
c
3
a
+ 2
+ 2

b2 + 1
c +1
a +1
2

41 ( )

111...111

3
(1)k
=
(2k + 1)3
32
k=0

91

.

:
,
, , .
, 1998.

[0, 1]

|f (x) + g(y) xy|

35 ( )
A
Q(x) = 12 (x, Ax) (x, b).
Q x0
Ax0 = b
21 (b, A1 b). ( n
n A

x Rn
(x, Ax) > 0.)
36 ( )
f Z[x] x1 , . . . , xn
M = max {|ai | : 1  i  n}.
m Z |m| > M + 1 |f (m)| .
f Z[x].

42 ( )

A = {2n 3|n N}

()

43 ( )

1+

1x 
= 1 x2 + x
2x


32 ( )
( , )
ABCD E, F,
AB, CD ,

AE
DF
=
=p
EB
FC
BC, EF, AD, K, L, M,

EL
AM
BK
=
=
=q
KC
LF
MD

KL
=p
LM

37 ( )

1
2

ln(1 x) ln x
dx .
x (1 x)

38 (

lim n2



1
0


2
1 + xn dx 1 =
12

39 (
)


.
40 ( )

(An )nN , (Bn )nN

An X = Bn n N

44 ( )
ABC
a2 cos2 A + b2 cos2 B = c2 cos2 C .

;

45 ( )
f R
f (x)sin3 x, f (x)cos3 x , f
.
46 ( )
P (x) n n
x1 , x2 , ..., xn . Q(x) n 1,

n

Q(xk )
xQ(x)
= lim
 (x )
x
P
P (x)
k
k=1

47 ( irakleios)
P (x) R[x]
:

(x 1)143 + (x + 1)2002 = [P (x)]13

48 ( ) p(x)
n,
n
,
xi , xj xi xj >
1.
( :
)

: p(x) n n 1
,
xi , xj p

xi xj > 1

:
1 ( )
: .
.
, ,
,
.
, ,
, . .

- ,
. ,
. ,
. ,

. ,
,
.

. ,
,

.)
[ 1 3 ,
2 1 3 2 .]

. 7 .

. (
.)
.
2 ( papel)

.
;

http://www.mathematica.gr/forum/viewtopic.php?f=44&t=875

( ) . 6
18 .

.

http://www.mathematica.gr/forum/viewtopic.php?f=44&t=7045

,,,,,,,,

( )
,
;.


. (

:
3 ( )
30% , 20
30% .
;

http://www.mathematica.gr/forum/viewtopic.php?f=33&p=65374

1 ( )
1:

http://www.mathematica.gr/forum/posting.php?mode=edit&f=33&p=65774

1 + 2 + 3 + ... + 9 = 45

(
70% 20
30% .
40% 20l. 20% 10l,
100% , 50l.

2 :
4 + 7 + 9 = 20.
3 : x
x x
20 23
45.
4 :
21, 1
45 21 21 = 3 x = 3 2, 5, 6, 8
x 22, 2
45 22 22 = 31 x = 1 3, 5, 6, 8
x .
1 :
, ,
2 ( ) A
,

4 ( )

9 1, 2, 3, ..., 8, 9


.
.
x;

A + A = 1 + 2 + 3... + 9 + x = 45 + x
x . x ,
1, 3, 5 ( 7, 9 ). x = 1 ,
x = 3 , x = 5 .

:
1 ( ) (K, R)
. E1

5 ( KARKAR )
, E1 .
, E2 .

E2
E1

E1 = 2R2

http://www.mathematica.gr/forum/viewtopic.php?f=34&t=12234

= R 2.
6

E2 =

4R2
5
2

4R
E2
2
= 52 =
E1
2R
5

6 ( )
.
7 . 2
.
: 1...6
.

http://www.mathematica.gr/forum/viewtopic.php?f=34&p=63518

1 ( )
. 2+7 6+2
. 9+9.
9+9=1..6. 9+9196 9+9106 .
, 1023. 9+9
6. 11 22 9+9
6 =13. 33
80.
2 ( ) ,
:
, 9+9, 9(+1).
9. 1..6
9,
2 (1+2+6=9 , ,
9)
126. 9+9=126 =13.


x.
KH,
.
H =

x
K = HZ = x HZK.
2

K2 + H2 = KH2
,

x2 +

x2
= R2
4

x2 =

4R2
5

:
(a + b)2 = c2 a2 + b2 c2 = 2ab

7 ( ) a, b, c

a+b+c=0

b2 + c2 a2 = 2bc

b2

a2 + c2 b2 = 2ac

1
1
1
+ 2
+ 2
=0
+ c2 a2
c + a2 b 2
a + b 2 c2

1
1
1
2 + a2 +b2 c2 =
b2 +c2 a2 + c2 +a2 b

2
2
1
1
1
1 a bc+b ac+c2 ab
+
+
=

2
2bc
2ac
2ab
2
(abc)
= 12 (a + b + c) = 0

http://www.mathematica.gr/forum/viewtopic.php?f=35&t=10253

1 ( () )

a+b+c=0
7

() a b. ( ),
b a, a = b.
2 ( )

8 ( ) a b

a a+1
a + 2009
+
+ ... +
= 2010
b
b+1
b + 2009
a = b

a + 2009
a a+1
+
+ ... +
= 2010
b
b+1
b + 2009
a a+1
a + 2009
b b+1
b + 2009
+
+ ... +
= +
+ ... +
b
b+1
b + 2009
b b+1
b + 2009
ab
ab ab
+
+ ... +
=0
b  b+1
b + 2009 
1
1
1
(a b)
+
+ ... +
=0
b b+1
b + 2009
ab=0
a=b

http://www.mathematica.gr/forum/viewtopic.php?f=35&t=10424

1 ( ) a > b,

1 k

a+k
>
b+k

= 0, 1, 2, . . . , 2009 ( 2010
). k = 0, 1, 2, . . . , 2009,

2010 =

a + 2009
a a+1
+
+ ... +
> 1 + 1 + + 1 = 2010
b
b+1
b + 2009

:
9 ( ) > 2 > 2
> + .

> 2 A = [0, +).


x1 , x2 A
f (x1 ) < f (x2 ) ( 1)x1 < ( 1)x2 (
1)(x1 x2 ) < 0 x1 x2 < 0 x1 < x2 ,
( > 2), f A.
f (2) = ( 1) 2 = 2 > 0 , x > 2 :
f (x) > f (2) > 0 = x x > 0 x > x +
x = , > 2
: > +

http://www.mathematica.gr/forum/viewtopic.php?f=19&t=10669

( ) 1 : > 2, > 2.
:

> 4 (1)

10 ( )
m

> 2 2 > 0, > 2 2 > 0


:

2 2 + 4 > 0 (2)

|2x |2x 1|| = m2 x

(1) (2) :

2 2 2 > 0 > +
2 = : 2 > 2
2 2 > 0 ( 2) > 0 , > 2.
> . :

http://www.mathematica.gr/forum/viewtopic.php?f=19&p=59936#p59936

( ) m = 0, 2x =
|2x 1| x = 1/4.
m = 0,

2 > + (1) ( )
:

m2 x = |2x |2x 1|| 0

> 2 > 2 (2) ( )


(1) (2):

x 0. , |2x 1| = 1 2x
|4x 1| = 1 4x

2 + > + + 2 > +
> .
3 > +
: 2 > 2 + 2 : + 2 2 > 0
: ( 2) + ( 2) > 0, > 2
> 2.
4

1 4x = m2 x

(4 m2 )x = 1
m2 4 , m2 > 4,
1
x = 4m
2.

f (x) = ( 1)x
8

:
11 ( )
B = 45 = 30 .
= 15 .
B


3 ( ) :

45

=
=

45
105

75
:

http://www.mathematica.gr/forum/viewtopic.php?f=20&p=32336#p32336

1 ( ) =45

2
2
6+ 2
4

= (1)
=60 =/2=
(1) =

= (2)

(150 )

(30 + )

2 2
4
2
=

(30 + )
6+ 2
+ 3

2 + 6 = 2 6 + 2 2

2 =
6 + 2 2 = 3 + 2
0 < < 45 , =
15 .

( : 15 =

15
=
15

6+ 2

4
6 2
4

= ... =

3 + 2)

4 ( ) .

==30 (3)
(2,3)
=15 .
2 ( )
= x :
B


,
.
 

B
b
=
2. sin x
sin 30

A(0, 1) B(1, 0)

 = 105 , B
 = 45
 = 30 .
A

B
b
=
2. sin(45 x)
sin 75

sin 75
sin(45 x)
=
,
sin x
sin 30

 =
B
(1)

x = 15 (1),

f (x) =

3, 0 .

sin(45 x)
sin x
9

1
2
3
2 +

3 1
,
. :
2 2

1
= 32
=
3+2
1

 < 45
0 < B
 = 15
B

12 ( )
, , ,
.

http://www.mathematica.gr/forum/viewtopic.php?f=20&p=66591#p66591

1 ( ) ABC ACD
: BAH1 = //2(OM ), DH2 = //2(OM ) AH1 =
//DH2 :

BD = //H1 H2


.
.

:
13 ( ) :

2
2
( 2 + 1)(x+1) = ( 2 + 1)(x+1) x = 1

2
2
( 2 + 1)(x+1) + ( 2 1)(x+1) = 2

4 ( )
:

http://www.mathematica.gr/forum/viewtopic.php?f=21&t=12239#wrapheader

1 () ( 2 1)( 2 + 1) =

21=1

av + ( a1 )v = 2 a2v 2av + 1 = 0 (av 1)2 = 0 av = 1

a = 2 + 1, v = (x + 1)2 :

2
( 2 + 1)(x+1) = 1 (x + 1)2 = 0 x = 1
14 ( )

3x+1 9x + 3 5x 25x = 15x + 3

2
2
1
)(x+1) = 2 az + az = 2
( 2 + 1)(x+1) + (
2
+
1

2
az = ( 2 + 1)(x+1)
2.
az = 1 (x + 1)2 = 0 x = 1.
2 ( )


2
a + b 2 ab, a, b > 0 : ( 2 + 1)(x+1) +

2
( 2
1)(x+1)

2 [( 2 1)( 2 + 1)](x+1)2 = 2

2
2
( 2 + 1)(x+1) = ( 2 1)(x+1)

2
( 2 + 1)2(x+1) = 1 = ( 2 + 1)0
(x + 1)2 = 0 x = 1

:
3 ()

http://www.mathematica.gr/forum/viewtopic.php?f=21&t=12190#wrapheader

1 ( )

3(3x + 5x ) = 9x + 25x + 3x 5x + 3 (1)

9x + 25x 2 9x 25x = 2.5x 3x


(1)
3(3x + 5x 3x 5x 1) 0
(3x 1)(1 5x ) 0
(3x 1 5x 1) (3x 1 5x 1)
x 0x 0 x =0
0
2 () 3x = w 5x = y .

( 2 1)( 2 + 1) = 1 ( 2 1) = ( 2 + 1)1


2
(x+1)2
=2
: ( 2 + 1)(x+1)
+ ( 2 + 1)
w2 + y 2 + wy 3y 3w + 3 = 0

a + b 2 ab, a, b > 0

y 2 + (w 3)y + w2 3w + 3 = 0
2
2
( 2 + 1)(x+1) + ( 2 + 1)(x+1)
y D = 3(w 1)2

2 ( 2 + 1)(x+1)2 ( 2 + 1)(x+1)2 = 2.
w = 1 y = 1. x = 0
10

( )
:
( + 2)2 + ( 1)2 + ( 1)2 = 0, (1) -

= = 1
(1) = 3x = 5x
.

:
15 ( KARKAR) AB a,
O, M O. M
A N . N N,
A . A a .

http://www.mathematica.gr/forum/viewtopic.php?f=22&p=67791#p67791

1 ( ) ,

A
=
2
4

, :

x
(1)


M


3 1
,
.
4 4

=

N

1
y = x N
3
: y = x



1
1,
.
3

1
:
3

=

N

1
= 3 (x 1)
3, : y
3


5 5
,

,
6 6


2
2
5
(A) = 2 1
=

,

6
6
2
, :
x
4
(2) (1), (2)
=
2

(A) =
2
2
6
+
4
3 ( )
2
2
OM :
.
A =
6
( 45o ) = 12  = 3
2 ( )
N :
( ).

a
2a
(90o ) = N
a N = 3 AN = 3
(0, 0)
AN :
 (1, 0), (1, 1), (0, 1).
A
AN
(90) = (135)
1 1

,
O
.
A
2a
a 2
2 2
= 3(45+) A = 6
11

16 ( )
28
. .

M 2 = (M )(M K) =

2 2
(3)
9

(2) (3) :

AZ = M = ( )

http://www.mathematica.gr/forum/viewtopic.php?f=22&p=64442#p64442

1 ( ) :

(2) :

(14 )2 =

+ + = 28 (1)

2 2 2 + 2 2 2
.
9
4

(1) (4) : 2 12+35 = 0


: 1 = 7, 2 = 5
: 1 = 7, 1 = 14, 1 = 7
2 = 5, 2 = 10, 2 = 13

1.
.
2.
14,
.


C(A, 0).
- .

.

AZ 2 = (AK)(A) ( )2 =

= 2 (4)
2

1 2
2
= 2 (2)
3 3
9

:
17 ( )
AB. E Z AB B, ,
E Z
A.

http://www.mathematica.gr/forum/viewtopic.php?f=23&t=4292

( )

AB =
, A = , AM = xA, M E = y E

AM = xA = x(
+ ), (1)

AM = AE M E = 12 AB y E = 12
y( + 12
) =

(1y)

+
y

,
(2)
2
(1), (2)

x = y = 13 AM = 13 A

CN

18 ( )


, ,
x

+x +
= 0
x2

) 2 4

12

)
( ) x = 0 x = 0
 = 0, .
 2 4
 2 4x2
 0 x2
  0
)

2
2
2
)2 0, .
( x
) 4x
  0 ( x2

2 2
2
2
2
x ) 4x
  = ( x2
) x = (
)2
2 2
2
...(
x a) = 0  = x a

=0

//

=0
 = 2x

2x2
 + x
.
 //

http://www.mathematica.gr/forum/viewtopic.php?f=23&t=1980

:
19 ( )
f : R R :


f (x) =

ax2 +x+b
x+3

, x = 3
, x = 3

f
y  y 2 , .
. a = 1 b = 6.
. f x = 3.
. f
x x


f x x, = 1, = 4 ,
[0, ) x (0, /2),
.
20 ( ) x, y, z,
x < y < z < 3, 3 4.
) x = 1 = 5
)

y z .
)
6 x1 , x2 , x3 , x4 , x5 , x6
6


xi = 38

i=1
http://www.mathematica.gr/forum/viewtopic.php?f=18&t=11229&p=61361#p61361

5

2

6


x2i = 244

i=1

10 .

( )
f y  y -2,

http://www.mathematica.gr/forum/viewtopic.php?f=18&p=66117#p66117
a 02 + 0 + b
= 2 b = 6 (I)
0+3
f R, -3, ( ) i. :
lim f (x) = f (3).
x = 3,
:
x = 4 (1)
x3
2
ax +x+b
2
:
f (x) =
f (x)(x + 3) = ax + :
x+3

lim [f (x)(x + 3)] = lim (x2 + x + b)


x + b,
+z
x3
x3
= 3 + z = 6 (2)
0 = (3)2 + (3) + b) 9 + b = 3 (II)
2
. () () : a = 1 b = 6.
:
x++z+
. a = 1 b = 6, :
= 3 x + + z + = 12(2) x + 6 + = 12
x2 +x6
4
, x = 3
x + = 6 (3)
x+3
f (x) =
5
,
x
=
3

(1) (3) : x = 1

=5
x 2 , x = 3
=
=x2
ii.
5
, x = 3

f R , f (x) =
(1 3)2 + ( 3)2 + (z 3)2 + (5 3)2
2


=
1, f (3) = 1.
4
. f
5
4 + ( 3)2 + (z 3)2 + 4
B(0, 2) A(0, 2).
=

2
4
f

f (0) = 2


OA = OB = 2,
(OAB) = 2..
13

5
( 3) + (z 3)
=
+2
2
4

6


( 3) + (z 3)
1
=

2
4
( 3)2 + (z 3)2 = 2

38 + 12
=
=5
10
10
2
2
2
2
1 + 2 + 4 + 5 = 1 + 4 + 16 + 25 = 46
10

xi = 50
=

(4)

(2),(4) :

i=1

( 3)2 + (z 3)2 = 2(2)


2

xi + 12

i=1

10

2

( 3) + (3 ) = 2 ( 3) = 1

x2i = 244 + 46 = 290

i=1

3 = 1 = 4 z = 2
3 = 1 = 2 z = 4
x = 1, = 2,z = 4, = 5
iii.

2
2500
10 )

10


1
x2i
10 (
i=1

10

i=1

x i )2

10

1
) = 10
(290

502
10 )

1
10 (290

1
= 10
(290

250) = 4

= 4 = 2

:
21 ( ) a, b, c
1 : a + b + c = 1
:
) ab + bc + ca = abc
) (1 a)(1 b)(1 c) = 0
1
1
1
) a2009
+ b2009
+ c2009
=1
) a, b, c, ,
.

http://www.mathematica.gr/forum/viewtopic.php?f=51&t=871

http://www.mathematica.gr/forum/viewtopic.php?f=51&t=1544

1 ( ) z12 z1 + =
0 z22 z2 + = 0 z2 = z1 z13 + z22 = 1
z12 z22 .
a2 z1 ab z1b + az1 b = 1 (1).
z1 , z2 C R D = a2 4b

z1 =

1 (giannisn1990)

1
1
1
b =
c =

a
b
c
1 1 1
a+b+c = 1 a + b + c = 1 a+b+c = 1 + + =
a b c
1 ab + bc + ca = abc
) a =

a
+
2

4b a2
i
2

(1)
a2 b a = 0 (2)
(
a2 )
a ab 1 = 0 (3)
(2), (3) (a, b) = (1, 0) (a, b) =
(1, 2). (1, 0)
(1, 2)
.
2 ( )

ab + bc + ca = abc
ab + bc + ca abc = 0
b(a + c) + ca(1 b) = 0
b(1 b) + ca(1 b) = 0
(1 b)(b + ca) = 0
(1 b)(1 a c + ca) = 0
z13 + z1 2 = 1 (1)
(1 a)(1 b)(1 c) = 0
) (1a)(1b)(1c) = 0 a = 1 b = 1 c = 1
(1)
1
1
a = 1 . 2009 + 2009 =
b
c
z1 3 + z12 = 1 (2)
0 b2009 = c2009 a + b + c = 1
b + c = 0 b = c b2009 = c2009 b2009 + c2009 = 0
) |b c|2 = |a c|2 + |b a|2 . a = 1
(1)-(2) (z1 z1 )(z12 + z1 2 + z1 z1 ) (z1 z1 )(z1 + z1 ) = 0
b + c = 0 |2c|2 = |c + 1|2 + |c 1|2
(z12 + z1 2 + z1 z1 ) (z1 + z1 ) = 0( z1 |c + 1|2 + |c 1|2 = 2(|c|2 + 1) = 4 |2c|2 = 4
22 ( ) x2 x + = 0,
, R z1 , z2 C R. , : z13 + z22 = 1

14

(z1 + z1 )2 z1 z1 (z1 + z1 ) = 0
a2 b a = 0 (3)

(1)+(2) (z1 + z1 )(z12


(z1 + z1 )[(z1 + z1 )2

0 (a 1)2 (a + 1) = 0 a = 1 a = 1

+ z1 z1 z1 ) + (z1 + z1 ) 2z1 z1 = 2
3z1 z1 ] + (z1 + z1 )2 2z1 z1 = 2
= a2 4b < 0 (5)
a(a2 3b) + a2 2b = 2
a = 1 (3) b = 0 ,
a(a + b 3b) + a + b 2b = 2
(5) a = 1 (3) b = 2 (5) a = 1 b = 2
( (3) a(a 2b) + a b 2 = 0

.
a = 1 b = 2
: x2 + x + 2 = 0 z12 + z1 + 2 = 0 z12 = z1 2 (6)
z13 = z12 2z1 = z1 + 2 2z1 = 2 z1 (
(6) ) z22 + z2 + 2 = 0 z22 = z2 2

a 2ab + a b 2 = 0 a + b 2ab + a b 2 = 0

2a 2ab 2 = 0 a ab 1 = 0 (4)
(3) b = a2 a (4)
a a(a2 a) 1 = 0 (a 1) a2 (a 1) = z13 + z22 = 2 z1 z2 2 = (z1 + z2 ) = (1) = 1

:
23 ( Iason Pap.)
f : R R f (f (x)) = x3

xR

= :

f (x1 + x2 + ..... + x ) = (x1 ) + f (x2 ) + ... + f (x ) (A)


= + 1 :
f (x1 + x2 + ... + x+1 ) = f (x1 ) + f (x2 ) + ... + f (x+1 )
:

http://www.mathematica.gr/forum/viewtopic.php?f=52&t=2985

f (x1 + x2 + ..... + x+1 ) =

( ) f ,
1 1 f (f (f (x))) = f (x3 ) f (f (f (x))) =
f (x)3 , f (x3 ) = f (x)3 f (0) = 0, f (1) =
f (1)3 , f (1) = f (1)3 . , f (1) = f (1)9 ,
f (1) = 1, 0 1. f (1) = 1, f (f (1)) = f (1) = 1,
f , f (f (1)) = 1, .
f (0) = 0 f 1-1, f (1) = 1.
f (1) = f (1)9 , f (1) = 1, 0 1,
f (1) = 1.
f (f (1)) = f (1) = 1 , f
f (f (1)) = 1, . .

(1)

f (x1 + x2 + ..... + x + x+1 ) =

(A)

f (x1 + x2 + ..... + x ) + f (x+1 ) =


f (x1 ) + f (x2 ) + ..... + f (x ) + f (x+1 ) ()
(2)
N .
) (2) x1 = x2 = ..... = x = 1 :


1
1
1
+ + ..... +
=

 
 
 
1
1
1
+f
+ ..... + f

24 ( ) f : R


 
 
1
1
R
1
f
= f
f (1) = f

f (x + y) = f (x) + f (y) (1)


 
1
f (1)
x, y R. :
(3)
f
=

) f (0) = 0 f (x) = f (x) x R


N
) N x1 , x2 , ..... , x R :

) q =
f (x1 + x2 + ..... + x ) = f (x1 ) + f (x2 ) + ..... + f (x ) (2)
, Z , N .
 1  f (1)

>
0
:
) f = N .



 (2)
 
) R, f (q) = q q Q.
f (q) = f = f 1 = f 1 + 1 + ..... + 1 =
 
 
  (3)
 

f 1 + f 1 + ..... + f 1 = f 1 = f (1)
= f (1) =
http://www.mathematica.gr/forum/viewtopic.php?f=52&t=1300
q f (1).
< 0:
1 ( )

> 0 ,

 

  = .
) (1) x = y = 0 : f (0) = f (0) + f (0) f (q) = f = f = f = ... = f (1) =

f (0) = 0.
f (1) = q f (1). ( ) = 0 q = 0 :
(1) y x :
f (0) = 0 R. = f (1) R
f (0) = f (x) + f (x)

0 = f (x) + f (x)

, f (q) = q q Q.
f (x) = f (x) x R.
) = 1: f (x1 ) = f (x1 )
15

:
25 ( Stelmarg )

P (x) = x2n+1 2x + 1, n N, n 2
(0, 1)
( ) :
(0, 1)
( ,
- ). xn
, (0, 1), (xn ) limxn = 12 .

(1/2, 1) xn
(1/2, 1) : p(xn ) = 0 xn
y = 2x 1 , y = xn+1 y = xn+1
y = xn [*] 0 < xn+1 < xn (1/2, 1)
A(1, 1)

http://www.mathematica.gr/forum/viewtopic.php?f=53&t=11726

1 ( ) xn+1 2x+1 = 0 x(xn 1) =


x 1 x = 1 x(xn1 + ... + x + 1) = 1
f (x) = xn + ... + x2 + x 1, f (0)f (1) = 1 n < 0
Bolzano P
(0, 1)
2 ( ) (
) Pn (x)
(1/2, 1).
n = 2, 3, ... rn (1/2, 1) : Pn (1/2 + rn ) =
0 (1/2 + rn )n+1 2rn = 0.
Qn (x) = (1/2 + x)n+1 2x, (
rn (0, 1/2)
rn limrn = 0. Qn (0) > 0
n = 2, 3, ... x0 (0, 1/2)
an = (1/2 + x0 )n+1 2x0
(1/2 + x0 ) < 1. rn (0, 1/2)
Qn (x) an (rn ) = (1/2 + rn )n+1 2rn = 0
an an+1 (rn ) < 0,
Qn+1 (rn ) < 0, Qn+1 (0) > 0,
rn+1 (0, rn ), rn+1 < rn ,
rn .  > 0
ln(2 )
 ( ln(1/2+ )
, n :
ln(2 )
ln(2 )
n+1 > ln(1/2+ )
. n+1 > ln(1/2+ ) (n+1)ln(1/2+) <
ln(2) (1/2 + )n+1 < 2 Qn () < 0 rn (0, ) :
Qn (rn ) = 0 
limrn = 0

3 ( )
( !) ! p(1) = 0 p(x) = (x 1)q(x)
q(x) = xn + xn1 + ... + x + 1

q(1) = n 1 > 0, q(1/2) = 1

1
2n+1

( )

<0

AB f
AB f (a, b)
(a, b) C
[*] y = 2x 1
B xn+1 < xn
0 < xn x1 < 1 (xn )n 0 2xn 1 = (xn )n 0
xn 1/2
26 ( ) f : R R
f (x) > 0, x R y R y(a) = k R
y
, a, k, f (t)
,

y(x) f (x)y 2 (x) = 0, x R

http://www.mathematica.gr/forum/viewtopic.php?f=53&t=9051

( ) y(x) ,
. ( ) h

h(x) = y(x)e

x
a

f (t)y(t)dt

,x R

h R :

p (x) = (n + 1)xn 2 < 0

h (x) = e
16

x
a

f (t)y(t)dt

(y  (x) f (x)y 2 (x)) = 0, x R

h . c = h(a) = y(a) = k
x
: h(x) = k y(x) = ke a f (t)y(t)dt x R
:
1) k = 0 y(x) = 0, x y(x)
( ).
2) k , y(x)
( , k )
k = 0 (
).
k .
y(x) . :

1
y  (x)
= f (x) =
=
y 2 (x)
y(x)

f (t)dt + c

x = a c = k1 :

y(x) =

1k

k
x
a

f (t)dt

,x R

.
2 ( ) y (

)
(
k ( lim f (x), lim f (x))
x

x+

y 2 (x) y(x) =

1
f (t)dt

x
1
k a

y x0 2
.
) k = 0 ) k = 0
) a < x0 ( k < 0), (, x0 ) [1]
y(x0 ) = 0 limxx0 y(x) =

 x0 .
( a f (t)dt) ,
a > x0
k = y(a) y
y(x) > 0 x > a x < a (, a)
1
y(x)
= F (x)+ c1 F f
x > a
2 F (a) + c1 ().
y 2
a

:
27 ( )


F (t) =

1

sin x
dx
1 + x2

I=

x (2 x) dx = 2
n

2n

t > 0.

1
n

xn (1 x) dx = J
0

http://www.mathematica.gr/forum/viewtopic.php?f=54&t=4056
http://www.mathematica.gr/forum/viewtopic.php?f=54&t=7713&p=44293

1
dx =
1 + x2
t  t 0

2x
1

(1 cos x)
dx
(1 cos x)
1 + x2 0
(1
+ x2 )2
0
 t
2x(1 cos x)
1 cos t
=
+
dx > 0.
2
1+t
(1 + x2 )2
0

(James M erryf ield)

sin x

28 ( )

[1]

( )


1 2 n
J=
(2x) (2 2x) dx =
y (2 y n )n dy =
2 0
0 
1
1 2 n
1
1
I+
y (2 y)n dy = I + K.
2
2 1
2
2


17

x=y1


0

2x=y

(1 + x) (1 x) dx
n

a
0

f (x)=

a

=0

f (ax)

I.

:
29 ( )
 = 20 .
ABC, (AB = AC) A

 = 60
AC D , DBC

 = 30 .
AB E , ECB

EDB

 = EM

B = 20 ,(CE BM ).
EBM
= DM
x =
AEM D (A

20 ).
 = EDM
 = EAM
 = 10 .
EDB
30 ( ) f, g : [0, 1] R
, x, y [0, 1] ,

http://www.mathematica.gr/forum/viewtopic.php?f=49&t=1706#wrapheader

|f (x) + g(y) xy|

( )

1
4

http://www.mathematica.gr/forum/viewtopic.php?f=49&t=9524#wrapheader

( ) :
|f (x) + g(y) xy| < 14 x, y [0, 1]
:
x = y = 0 :

|f (0) + g(0)| < 14


x = 1, y = 0 :
|f (1) + g(0)| < 14

|(f (0) + g(0)) (f (1) + g(0))|


|f (0) + g(0)| + |f (1) + g(0)| < 12
|f (0) f (1)| < 12 , (1)
x = 0, y = 1, |f (0) + g(1)| <
1, |f (1) + g(1) 1| < 14

1
4

x = 1, y =

|(f (0) + g(1)) (f (1) + g(1) 1)|


|f (0) + g(1)| + |f (1) + g(1) 1| < 12
|f (0) f (1) + 1| < 12 (2)
(1) (2) :
|(f (0) f (1) + 1) (f (0) f (1))| |f (0) f (1) + 1| + |f (0)
f (1)| < 1 1 < 1 ()
x, y [0, 1]
1
|f (x) + g(y) xy|
4

BCM .

:
31 ( ) a, b, c

a+b+c=3
18

b
c
3
a
+
+

b 2 + 1 c2 + 1 a2 + 1
2

http://www.mathematica.gr/forum/viewtopic.php?f=50&t=539

http://www.mathematica.gr/forum/viewtopic.php?f=50&t=516

( )

1 ( ) a, b, c, d, e, f, k, l, m
d+pc
, e = a+pb
1+p f = 1+p ,

b2

c2

ab
a
a
()
+1
2

k=

bc
ca
b
c
b
c .
2
+1
2
a +1
2

ab + bc + ca
ab + bc + ca
=3
.
2
2
ab+bc+ca
3
3
2 , ,
2
ab + bc + ca 3,

(a + b + c) 3(ab + bc + ca) ()


(

),
2

,m=

a+qd
1+q ,

( )

b + qc

(a+pb)+q(d+pc)
1+p

1+q
(1 + p)(b + qc) (a + pb) q(d + pc)
=
(1 + p)(1 + q)
(b a) + q(c d)
=
(1 + p)(1 + q)

e+qf
1+q

b + qc e + qf

1+q
1+q
b + qc (e + qf )
=
1+q

LHS (a + b + c)

= a(b b+1)ab
= a bab
2 +1
2 +1 .
2
b + 1 2b,
2
ab2
ab
a bab
2 +1 a 2b = a 2 .

l=

kl =

a
b2 +1

b+qc
1+q ,

l m =

32 ( ) ( ,
) ABCD E, F,
AB, CD ,

p((b a) + q(c d))


,
(1 + p)(1 + q)

l m = p(k l) p
K, L, M
p.

2 ( ).
DF
AE
=
=p
(
EB
FC
)
BC, EF, AD,

K, L, M,

, ,
. ,,, 1, p, pq, q , .
(1) (p) (1 + p). ..
(pq) (q) F (pq + q). ..
L EF EL : LF = q . .. ( ,
,) . ,
.
(

. )
, .

EL
AM
BK
=
=
=q
KC
LF
MD

KL
=p
LM

:
33 ( )
f : R R
f (f (x)) + f (x) = x x R.

http://www.mathematica.gr/forum/viewtopic.php?f=59&t=4366

( )

19

1-1 .

f (f (x)) = x f (x)
. 1-1 .
g
x + g(x) = g(g(x)).
s R, t = f (s)
(xn ), (yn )
yn = f (yn1 ) y0 = s xn = g(xn1 ) x0 = t.
(yn ) yn =
yn2 yn1 n 2
yn = asn1 + bsn2 a, b R, s1 , s2
x2 + x 1 = 0 s1 > 0 > s2 .
xn = as1n
+ bs1n
.
1
2
ab = 0. ab = 0. f
, yn+2 yn+1 = (yn yn+1 )(yn1 yn ) = (f (yn1 )
f (f (yn1 )))(yn1 f (yn1 ))
xn1 xn2 = (xn+1 xn )(xn xn1 ).
y
n+1
s2 xxn+1
s1
1 ,
yn
n
s1 s2 0 s1 s2 = 1. s, a = 0,
b = 0. f (s) = s1 s, f (s) = s2 s.
f (x) = s1 x x R
f (x) = s2 x x R. x > 0, f (x) = s2 x
f . 0 > y > xs2 /s1
f (y) = s2 y > 0 > f (x), f (y) = s1 y > xs2 = f (x).
. f
f (x) = s1 x x > 0.
f ,
1-1 f (x) = s1 x x 0.
f f (x) =
s2 x x.

.
34 ( )


k=0

(1)k
3
=
3
(2k + 1)
32

2 ( ) Laplace
 a yx
dx = (a+1)
y a+1 ( a = 2 y = 2k + 1)
0 x e



(1)k
1
k 2 (2k+1)x
x e
dx =
k=0 (2k+1)3 = 2
k=0 (1)
0
 x2 ex 
k 2kx
dx
k=0 (1) e
2
0
  x2 ex 
  x2 ex 
1
1
2
1+e2x dx = 2
1+e2x dx =
0
0
  x2 ex 
1
4
1+e2x dx f (z) =

2 z
z e
1+e2z

A(M, 0), B(M, i), C(M, i) D(M, 0).

f z 2 ez
2 i
z0 = i
2 Res f (z) = lim (z z0 ) 1+e2z = 8
z=z0

zz0

ABCD f (z) dz = 2i Res f (z) = 4 .

z=z0

z = M + iy z = M + iy y [0, ]

|f (z)|

ez (M 2 +1)
e2M 1

0 M
.

M ABCD f (z) dz

(x+i)2 ex+i
dx.
2x+2i

 x1+e
 xex
2 x
e
2 1+e2x
dx + 2i 1+e
dx
2x

ex
3
xex
2
1+e2x dx = 4 . 1+e2x
 xex
 ex
, 1+e
dx = 0. 1+e
dx =
2x
2x
 1
 ex
 (1)k

2 0 1+e2x dx = 1 1+u2 du = 2 . k=0 (2k+1)3 =


 3

  x2 ex 
3
1
1

3
dx
=

= 32 .
+
2x
4
1+e
8
4
2

x2 ex
1+e2x

dx +

3 ( ) Fourier

Edw http://www.mathematica.gr/forum/viewtopic.php?f=59&t=3614

1 ( )
f(x) = x( x) [0, ]
 Fourier

2
b
sin(nx)
,

b
=
n
n
n=1
f (x) sin(nx) dx =
4

1(1)n
n3

. x = /2 .

(1)n+1 sin(nx)

x2 =
<
n=1
n
0 t x < .

2
n+1
/n2 = 2 /12 t4

n=1 (1)
2
12

n=1

(1)n cos(nt)
. 0
n2
 (1)n sin(nx)
x3 2 x
=
.
n=1
12
n3

x
x = /2 .

:
35 ( )
A Q(x) = 12 (x, Ax) (x, b).
Q x0
Ax0 = b

12 (b, A1 b). ( n n A


x Rn (x, Ax) > 0.)

20

http://www.mathematica.gr/forum/viewtopic.php?f=10&t=1040

( )

0 x A1 b, A(x A1 b) =
x, Ax x, b A1 b, Ax + A1 b, b =
x, Ax 2x, b + A1 b, b = 2Q(x) + A1 b, b
x = A1 b.
36 (

f Z[x] x1 , . . . , xn M =
max {|ai | : 1  i  n}. m Z |m| > M + 1
|f (m)| . f Z[x].
http://www.mathematica.gr/forum/viewtopic.php?f=10&t=1749

( ) f (x) = g(x)h(x)
g, h Z[x]. |f (m)| = |g(m)||h(m)| |f (m)|
|g(m)| = 1.
g(x) = k(x x1 ) (x
xr ) k Z 1  r  n 1.
|m xi |  |m| |xi | > M + 1 |xi |  1
r
1  i  r. 1 = |g(m)| = |k| i=1 |m xi | > |k|  1,
.
( )
.
f Z[x] m Z |f (m)|
|m | > 1 f . f
Z[x].

:
1

37 ( )

1
2

ln(1 x) ln x
dx .
x (1 x)

1

http://www.mathematica.gr/forum/viewtopic.php?p=43790#p43790

( )

1/2
I=
0

1/2

ln (u) ln (1 u)
du
(1 u) u


ln(1 x) ln(x)
dx =
x
1

ln (1 x) ln (x) x=1u
dxdx=du =
x (1 x)

1

ln(1 x) ln(x)
dx
x

1

ln(x)
x



x3
x2
+
+ . . . dx =
x+
2
3


x x2
+ . . . dx =
ln(x) 1 + +
2
3



k=0

1

1
k+1

ln(x) xk dx =

k+1

=
ln(1 x) ln(x)
(k + 1)2
dx .
k=0
x (1 x)
0
0

 1

u = 1x  1
1
ln(1 x) ln (x)
ln(1 u) ln(u)
du = dx
= (3)
dx
du =

=
(k + 1)3
x
1u
k=0
0
0
 1
ln(1 x) ln(x)
dx.
1x
38 ( )
0


 1

2
1
n
2
n
lim
n
1
+
x
dx

1
=
ln(1 x) ln(x)
1
n
12
dx =
I=
0
2
x (1 x)
I=

1/2

1
ln(1 x) ln(x)
dx =
x (1 x)
2

1
0

1
ln(1 x) ln(x)
0

ln(1 x) ln(x)
dx +
x

1
1
+
x 1x

1
0

http://www.mathematica.gr/forum/viewtopic.php?p=43820#p43820

dx =

( )

ln(1 x) ln(x)
dx =
1x

I = lim n2

1

n+

21

(1 + xn )1/n 1 dx =

lim n

n+


1 
+
0

k=1

1/n
k


x

nk

lim

dx =


 1
+

1/n
2
xnk dx =
lim n
n+
k
k=1
0


+

1/n
1
lim n2
=
n+
nk
+1
k
k=1





1 1
1
1
+

n2 n n 1 n 2 n (k 1)
lim
n+
nk + 1
k!

n+

lim

i+

i


(1)

k=1

limn+ limi+

k1 (n

i

k=1



1)(2n 1) (k 1)n 1
nk2 k! (nk + 1)

(1)k1 (k1)! nk1 +ak2 nk2 ++(1)k1


(k1)! k2 nk1 +k! nk2

)


i

(1)k1 (k 1)! nk1 + ak2 nk2 + + (1)k1
lim
=
lim
i+ n+
(k 1)! k 2 nk1 + k! nk2
k=1
( k n k 1)

lim

i


i+

(1)k1

k=1

2
1
=
k2
12

k=1

:
39 ( ) .

a < an+1 < n = min{an , a +

1
n+1 }.

an

an a. A
{a1 , a2 , ..., an , ...}
.

http://www.mathematica.gr/forum/viewtopic.php?f=64&t=8313

1 ( ) A R ( ).
x A x
{y A : y > x}. (x, x
.
An x (x, x )
1/n. A = n An .
An ( An n
) A .
2 ( ) 1 A R
a A a A (a, a + a ) = ,
|A| 0 U = {(a, a + a )/a >
0, (a, a + a ) A = }. U
, |U | 0 .
f : U A f ((a, a + )) = a. , |A| = |f (U )| |U | 0 . 2
A R |A| > 0 , a A
an an a
a A
b > a, (a, b)
A. a1 A a < a1 < a + 1, a2 A
a < a2 < 1 = min{a1 , a + 12 }.
a1 , a2 , ..., an A a1 > a2 > ... > an
a < ak < a + k1 , k = 1, 2, ..., n an+1 A

40 ( )

(An )nN , (Bn )nN


X

An X = Bn n N

http://www.mathematica.gr/forum/viewtopic.php?f=64&t=10554

( )
An Bn


 n N.

(Bn An )

nN

Bn . , Bk Ak

nN

Bm , X
k - m.
,
X


nN

(Bn An ) X


nN

22

Bn

, 111...111
.


41 ( )

91

111...111
 

42 ( )

91

.
: , , , . , 1998.

A = {2n 3|n N}

.

http://www.mathematica.gr/forum/viewtopic.php?f=63&t=11074
http://www.mathematica.gr/forum/viewtopic.php?f=63&t=10742

1 ( )

111...111
= 1090 + 1089 + . . . + 10 + 1 =
 
91

1091 1
.
9

, 91 = 13 7 1013 1|1091 1. ,
1091 1 = (1013 1) , 1. ,

111...111
=
 
91

(1013 1)
1091 1
=
.
9
9

.
2 ( ) 91 = 7 13,

111...111
= 1111111
=
 
  1111111
  ... 1111111
 
91
7
7
7


13

( ) . , S
A
(
, S = {22 3, 23 3}). ,
p1 , p2 , . . . , pn
S ( , pi = 2). ,
Fermat, 2(p1 1)(p2 1)...(pn 1) 1
pi , i = 1, . . . , n. , 2(p1 1)(p2 1)...(pn 1) 3
pi , i = 1, . . . , n. ,
2(p1 1)(p2 1)...(pn 1) 3 S,
.

=
1
0000001
0000001
...
0000001
   
  .
  1111111


7
13

:
43 ( )

1+

1x #
= 1 x2 + x
2x

http://www.mathematica.gr/forum/viewtopic.php?f=27&t=12132

( )

1 cos a
= sin a + cos a
2 cos a

2 cos2 a 1 + sin 2a = 1 cos a

sin 2a + cos 2a = 1 cos a


1+

sin 2a + cos 2a = cos


2


2a + cos 2a =




cos 2a
4
4



= 1 cos a
2 cos 2a
4


2a > 0
cos
4
2 cos

( )



= 1 cos a
2 cos2 2a
4


cos 4a
= cos a
2

4a = 2k+( a) , k Z 4a = 2k( a) , k Z
2
2
2k 3
2k
+
,k Z a =
,k Z
a=
5
10
3
6
a a = 3
10

x = cos 3
10 = sin 5
23

sin2 (2A) =

44 ( ) ABC

a2 cos2 A + b2 cos2 B = c2 cos2 C .


;



sin(2C sin(2B) sin(2C) + sin(2B)

sin2 (2A) = sin(2A) sin(2B 2C), (sin(2A) = 0)


sin(2A) = sin(2B 2C)
2A = 2B 2C B = 90 2A =
180 2B + 2C C = 45

http://www.mathematica.gr/forum/viewtopic.php?f=27&t=11927

( )

a2 cos2 A + b2 cos2 B = c2 cos2 C sin2 (2A) + sin2 (2B) =


... A = 90 sin2 (2B) =
2
sin (2C)
sin (2C), -
2

:
45 ( ) f
R f (x)sin3 x, f (x)cos3 x
, f .

http://www.mathematica.gr/forum/viewtopic.php?f=61&p=65401#p65401

( )
, .
P, Q

P  (x) = f (x)sin3 x, Q (x) = f (x)cos3 x ()

(sin2 x + cos2 )3 = 1
,

( ),
.
.

,
.
,
.
46 ( ) P (x)
n n
x1 , x2 , ..., xn . Q(x) n 1,


((2P (x) cos x + 2Q(x) sin x) sin 2x (P (x) sin x Q(x) cos x) cos 2x)  =
f (x) + H(x) ()

n

Q(xk )
xQ(x)
= lim

P (xk ) x P (x)

H . ,
(*)

k=1

(2f (x)sin3 x cos x + 2f (x)cos3 x sin x) sin 2x


3

http://www.mathematica.gr/forum/viewtopic.php?f=61&p=64368#p64368

(f (x)sin x sin x f (x)cos x cos x) cos 2x + H(x) =

1 ( ) C
R >
(0, 0)


2f (x)(sin2 x + cos2 x) sin x cos x sin 2x

{|x1 |, . . . , |xn |}.

f (x)(sin2 x cos2 x)(sin2 x + cos2 x) cos 2x + H(x) =

= f (x) sin 2x sin 2x + f (x)(cos 2x) cos 2x + H(x) =


= f (x) + H(x).
: H ,
G G = H. (**)

.
1 : () x1 , . . . , xn

2i

((2P (x) cos x + 2Q(x) sin x) sin 2x (P (x) sin x Q(x) cos x) cos 2x G(x))  =

n


Res(z = xk ; Q(z)/P (z)) =

k=1

= f (x) + H(x) H(x) = f (x)

f . ...

( )
.
, (
) :

Q(z)
dz
P (z)

2i

n

k=1

 Q(xk )
x xk
= 2i
P (xk )
P (xk )
n

Q(xk ) lim

zxk

k=1

2 :

24

Q(z)  Ak
=
P (z)
(x xk )
n

k=1


2i

n


k=1

Ak .

xQ(x) 
=
Ak .
lim
x P (x)
n

k=1

2 ( ) :

(x1 , .., xn ) n 1.

xn1
.
i
4 ( ) :
Q(x)
Q(x)
P (x) , P (x) =
n


k=1

Ak
,
x xk

x
k <n1
(x r1 )(x r2 )...(x rn1 )

xQ(x)
P (x)

n

xAk
,
x xk
k=1

xQ(x) 
=
Ak (1)
x P (x)
n

lim

. x, r1 , r2 , ..., rn1
P .
3( ) :
q(x) = xr p .
(x1 , .., xn )
n2 xr1 , .., xrk
xi . r < n 1 ( )
, - ,
. r = n 1

k=1

(x xk )Q(x)
=
P (x)
Q(xk )
(x xk )Q (x) + Q(x)
lim
= 

xxk
P  (x)
P (xk )
Ak = lim

xxk

n

k=1

Ak =

n

Q(xk )
P  (xk )

(2)

k=1

(1) (2)

:
47 ( irakleios)
P (x) R[x] :

(x 1)143 + (x + 1)2002 = [P (x)]13

http://www.mathematica.gr/forum/viewtopic.php?f=60&p=65628#p65628

( )
[P (0)]13 = (0 1)143 + (0 + 1)2002 = 0, P 0.
[P (x)]13 13.
(x 1)143 + (x + 1)2002
( 143x + 2002x = 1859x). .
48 ( ) p(x)
n, n ,
xi , xj xi xj > 1.
( : ) : p(x)
n
n 1
, xi , xj p

xi xj > 1

http://www.mathematica.gr/forum/viewtopic.php?f=60&p=66536#p66536

1 ( ) :
x1 x2 ... xn x1 + 1.

1) [x1 , x1 + 1] m
2) m x1 , x2 , ..., xn (m, m + 1)
|n xi | < 1, i = 1, 2, ..., n,
p(m) Z |p(m)| = |m x1 ||m x2 |...|m xn | < 1
p(m) = 0, .

,
,

.

xi , xj |xi xj | 1, :
1) m x1 , x2 , ..., xn (m, m + 1),
.
2) m x1 x2 ... xi =
m xi+1 ... xn x1 + 1
p(x)
q(x) = xm
.
3) m x1 = m x2 ... xn =
m + 1.

p(x)
q(x) = (xm)(xm1) .

2, 2 ( )

25


. x1 , . . . , xk p.
p(x)
q(x) = (xx1 )(xx ) , n k
k

n k .
y, z y z > 1. y, z p
.

26

http://www.mathematica.gr
. .

Leonardo da Vinci
(32-) . 30
. -
-
- quasiregular ,
( - ).
(0, 0, ),



12 ,
, 1+
,
2
2

1+ 5
.
2

:http://en.wikipedia.org/wiki/Icosidodecahedron
:

mathematica.gr
(http://www.mathematica.gr) .

mathematica.gr


1. (Mihalis_Lambrou)

8. (gbaloglou)
9. (R BORIS)
10. (dement)

2. (nsmavrogiannis)
3. ( )

11. (swsto)
12. (achilleas)

4. ( )

13. ( )

5. (m.papagrigorakis)

14. (xr.tsif)

6. ( )

15. (Demetres)

7. ()

1. (spyros)
2. (p_gianno)

1. (grigkost)
2. (cretanman)

1. ( )

3. (kostas.zig)
4. (exdx)
5. ( )

2. (vittasko)

6. (mathnder)

3. ()

7. (mathematica)

4. (nkatsipis)

8. (rek2)

5. ( )

9. (hsiodos)

6. (chris_gatos)

10. ( )

7. (mathxl)

11. (bilstef)

6 ( ) ,
,
;

1 (
)
. 5
3.
2
4
2 (
)
1 6k + 2, k .


6

5
.
31

.
,
,

.

.
.



, 300
. , 14
15, ,
.

3 (
) :
103

A = 22011 (2100 + 2100 + 2101 + 2102 +


+ ... + 22010 )

4 ( )

 2
A = 34 + 310 : 9 + 35 33 2 39

-3,
.

x(x + y) + z = y(y + 1) 3z + 1 = 0

11 (
)
,
=.
.
12 (
)
120 .


,
=2 = 4. :
) 120 .
) // = 3 ,
.
) .


,
7 ( )
,
2

(x + 4x + 1)(x + 4x + 7) + 9

5 (
)
120
, ;

10 (

8 (
) a, b, c
ABC
2

2(ab c ) = (a + b)(a + b 2c)


.

9 ( KARKAR)
:


x2 = 1 y
y2 = 1 x

13 ( )
:

2+1 x + 2 x = 1 (1),

2.
14 ( )
:

 z
 1 x
= 25
+ 9 25

2
4
36

 



y
x
6 34 = 14 + 14

 9 y
 5 z

9
15 6 = 4 + 4 16

15 (
) AB (30 , 60 , 90 )
,


B
= B
= 30 E
B (O, O = OE)
, E .

x = AO.

19 (
) A, B

1
P (B) =
2

3
P (A B) =
4


P (X), X , X A = B
20 (
)

x1 , x2 , ..., x2009

22 ( .
) z1 , z2 , z3

, , .

z1 + z2 + z3 = 0

z12 + z22 + z32 = 0


) z1 z2 + z2 z3 + z3 z1 = 0
3
) z3 = z1 z2 z3
) z1 = 0 , z2 = 0 , z3

= 0

|z1 | = |z2 | = |z3 |

) .

, ,
,

f  (x) = (x s) (x CV )

16 (
)
AB(108 , 36 , 36 )
= 6 , B
= 30 , A
=
: BA



B = 18 . x = A.

s, CV
(s = 5).

1
.
. x
=2
.

() : y = x + 1

f 3
i. s = 4
ii.

23 (
)
f : (0, +) (0, +)
x R

f (x)f

f (x) +

1
x

1
2

f (1).
24 ( ) f : R R,
:

f (f (x) + f (y)) = x + f (y) + 2010


x, y R.

(x1 , y1 ) , (x2 , y2 ) , ..., (x2009 , y2009 )


().
iii.

17 ( -

)
, :

, ,

21 ( .
) z, w



+ +
=6

|z 2| = |z + 4i|

 9.
)


)
2 + 2 = 9,


, .
18 (

=
 0
)

|
x
|
x = |
x + 8
|

|w + 2| = |w 4i|
) z, w
.
)
|z w|.
)
|z + w|.
)
|z w 2i|.

, ,

25 (
)
x
 
2 , 2

ex
1x
x
26 (
) f
[0, 3]
(0, 3). f (0) = f (3) =
0 f (1)f (2) < 0,
k [0, 3)


2
f (k)f  (k) + f  (k) 2k = 0

, ,

27 (
)

f [a, b] f  (x) > 0


x [a, b].

F (x) =

b

|f (t) f (x)| dt.


a
F .
28 (
)

f : [0, +) R
.


|f (x)| |f (0)| +

x
0

|f (t) + f  (t)| dt

x [0, +).

32 (
) n
.
.



.

29 (
) ABAB

CCC 639027.
A + B + C
A, B, C ;
30 (
)
:

x+y 1
2
x2 + xy + y 2
3

(i) k > 2 1 L(k) 2.


1
(ii) 0 < k <
, L(k) = 0.
1986
(iii) k R L(k) = 1985.

34 (
- 1984)
(
)
.


f (x) =

39 (
) f :
[a, b] R.

lim

b
a

f (x)
dx.
3 + 2 cos(nx)

40 (
) a > 0. ,

lim

n+

1
ln n


1kna

1
k


1

1
n

k

35 (
-
.)

T n . n + 1

n ,
T = I .

41 ( Stuart
Clark)
A,B , C < 1, 1, 1 > , <
1, 1, 2 >, < 0, 2, 1 >.

ABC
< 1, 0, 1 >.

f 2 (x) dx

2a 1
.
a2 + 6a 3

Mihai Piticari.

42 ( )

f : R 2 R2

1) f (0) = 0
2) |f (u) f (v)| = |u v| u, v
f .

a1 x + b1 x + c1 , x Q
a2 x2 + b2 x + c2 , x R Q

() f 1-1
() f
() a1 = a2 = 0, b1 b2 = 0,
c1 c2
b2


a, b a + b = p ab 1 mod p.


31 (
) f : R R

x2 + 1 Zp [x], p ,

36 (
)
f : [0, +) [0, +) a
, f (f (x)) = xa
x [0, +).

. Seniors

38 ( )


33 (
- 1985) k R L(k)
[x] = kx 1985. :

([...]= )

. Juniors

p 1, f (A)
.
( 1425 Mathematics
Magazine.)

b1
b2

37 (
) p
A (p 1) (p 1)
Ap = I = A.
f (x) -

43 (
) 20801
2015 1.
44 (
)
n m
2m 1 (mod n) n2 2
(mod m).

()

46 (
) :

sin x + sin y = sin(x + y)


|x| + |y| = 1

45 (
)
2

f (x) = x + 6x + 1, x R
S (x, y)

:

f (x) + f (y) 0
f (x) f (y) 0

S .

n, m .
n, m (x) p(x);
48 (
)

ax2 bx + c = 0
.

47 (
)

p(x) = (x + 1)n xn 1

(x) = (x2 + x + 1)m

a, b, c .
a, b, c


(0, 1)

a+b+c
.

1 ( )
. 5
3. 2

4

3) 5
1 3 .
4) ,
5
4 .
2 ( )
1 6k + 2, k
.

http://www.mathematica.gr/forum/viewtopic.php?f=44&t=9089

( )
5 3 gcd(5, 3) = 1,
: ! :

5
.
31

2315 = 1

http://www.mathematica.gr/forum/viewtopic.php?f=44&t=9436

8345 = 4
( ) 6k + 2 k
6.

(8 5) 3 + (4 + 3) 5 = 4

k
6k + 2

3 3 + (1) 5 = 4


.
31
, ,

1)
5
k
5 .
=
6k
+
2
31
2)
5 k = 10. 62 .
.

3 ( ) - 4 ( )
:

A = 22011 (2100 +2100 +2101 +2102 +2103 +...+22010 )

 2
A = 34 + 310 : 9 + 35 33 2 39
-3, .

http://www.mathematica.gr/forum/posting.php?mode=edit&f=33&p=76486

http://www.mathematica.gr/forum/posting.php?mode=edit&f=33&p=75977

(T-REX) : 2100 + 2100 = 2 2100 = 2101


: 2101 + 2101 = 2 2101 = 2102

:
22010 + 22010 = 2 22010 = 22011 : A =

22011

22011

( ) : (34 )2 + 310 :
9 + 35 .33 2.39 = 38 + 310 : 9 + 38 2.39 = 38 + 310 :

32 + 38 2.39 = 38 + 38 + 38 2.31 .38 = 3.38 6.38 =


3.38 = 39 = (3)9 .

=0

5 ( )

120 , ;

1 (chris t)
.

http://www.mathematica.gr/forum/viewtopic.php?f=34&t=13081

=
6

180
(1)
2


.
.

+ = = - 120 (2)
(1) (2) 120 =

180

360

140

180
2

= 140

=
= 9.
.
 = 120
2 ( ) A



,
300 . , 14 15,
, .

AE = 240 AB = 120 .

http:// http://www.mathematica.gr/forum/viewtopic.php?f=34&t=13918

( )
: x ,

x x x
+ + +3=x
2
4
7

28

- -

120
360
= 40 ,
=9
3
40

x
x
x
+ 28 + 28 + 28 x
2
4
7

14x + 7x + 4x + 84 = 28x

, 9-.

6 ( ) , ,
;

84 = 28x 14x 7x 4x
84 = 3x

.
, ,

x = 28

7 ( )
,

1 ( ) a = x2 + 4x
:

(a + 1)(a + 7) + 9 = ... = a2 + 8a + 16 = (a + 4)2

(x2 + 4x + 1)(x2 + 4x + 7) + 9

(x2 + 4x + 4)2 = (x + 2)4


2 + 4x + 7) + 9 =
2 ( )
(x2 + 4x + 1)(x



(x2 + 4x + 4) 3 (x2 + 4x + 4) + 3 + 9 =
(x2 + 4x + 4)2 9 + 9 = (x2 + 4x + 4)2 =(x + 2)4

http://www.mathematica.gr/forum/viewtopic.php?f=35&t=10984

3 (Irakleios) a = x2 + 4x + 7, a(a 6) + 9 =

(a 3)2 = (x2 + 4x + 4)2 = (x + 2)4

( )

8 ( ) a, b, c
ABC

2(ab c2 ) = (a + b)(a + b 2c)


2ab 2c2 = a2 + b2 + 2ab 2ac 2bc

2(ab c ) = (a + b)(a + b 2c)

a2 + b2 + 2c2 2ac 2bc = 0

(a c)2 + (b c)2 = 0
http://www.mathematica.gr/forum/viewtopic.php?f=35&p=58341#p58341

a=b=c

9 ( KARKAR)

:


5
x = 1+
y = 1
2

x2 = 1 y
y2 = 1 x

y=

:
stavros11

x2 = 1 y (1)
y 2 = 1 x (2)

x4 2x2 + x = 0
x x3 2x + 1 = 0
x  x3 x x + 1 = 0
x x x2  1 (x 1)
 =0
2
x (x 1) x + x 1 = 0

( )

x2 + xy + z = 0,

3x2 + y 2 + 3xy + y + 1 = 0

1 5
2

( )

2

y
2 y
+1 =0
3 x+
+
2
2

x = 0 y = 1
x = 1 y = 0

5
x = 1
x = y = 1 12 5
2

y =1

y=

y 2 + y 3z + 1 = 0

x = 0 x = 1 x2 + x 1 = 0 x =

6+2 5
4

http://www.mathematica.gr/forum/viewtopic.php?f=19&t=12903

x 1 y 1
(1) y = 1 x2
(2) :

x=

y =

x(x + y) + z = y(y + 1) 3z + 1 = 0

( )

1+ 5
2

2+2 5
4


2
1+ 5
2

10 ( )
:

http://www.mathematica.gr/forum/viewtopic.php?f=19&t=13998

62 5
4 y
1+ 5
2

22 5
4

y=

1+2 5

y = 2 x = 1.
z = 1.
(1, 2, 1)
.

11 ( )

, =.
.

NM = 360 2 135 + 135 + = 90

MNM

http://www.mathematica.gr/forum/viewtopic.php?f=20&p=72965#p72965

( ) M B = M N ()
M B = M D ... ( CM B, M CD ...)
= 45o ... (
(M, M B) N BD
D = 90o .
N D )... ... N M

1 ( )
, =
= .

12 ( )
120
.
,
=2 = 4. :
) 120 .
) // = 3 , .
) .

http://www.mathematica.gr/forum/viewtopic.php?f=20&p=68651#p68651

1 ( ) AB = A = 2x

AE = 4x, A = 8x
, .

BM = M = .

MB = M = 135

MBN = BNM = 90

BMN = 2
9

 = 30
ABK
AK =

AB
=x
2

KE = 3x, = 6x

A
1
AK
=
=
KE

3
,
// .

 .
A
E (0, 3), = 4 = 2.
150 ,

 = AK
 = 90 = 33 :
AE
A
, (

3
)
y=

 = ABK
 = 30
AEB

3 y=

3
x+1
3

3
, 3 + 1 , > 3


 2


3

2

(A) = 8 +
=8
3

 = 90 + 30 = 120
BE

x
AK
3

=
BK = 3x
30 =
BK
3
BK

2
= 64 2 = 48
3



3 = 4 3, 4 3, 3 .
2 +

B = 2 3x

60 =

E
E = 4 3x
4x

, ) E//x x E//B

BO
3
 = 30

=
BEO
BEO =
EO
3

2 3x  = 30 + 90 = 120 .
BE


( = // ) BZ = 6x
Z 2 3, 3
BZ = 3AB. ,
Z

E
2

.
2 ( )








A (0, 1) , B 3, 0 , 3, 0 , -

 = 120
A
= = 2.


(BZ) =


2
3 3 + 9 = 36 = 6 = 3AB

) Z//B, Z = B
, . ( () ,
)

10

13 (
:

2+1 x + 2 x = 1 (1),


2.

http://www.mathematica.gr/forum/viewtopic.php?f=21&t=13747

, (1).
:




2+1 x = 2 x
1 =0
2 x 21 x


2 x = 2 x
2 x 22 x 1 = 0


(x = 0 x = 1)

(x = 0 x = 1)

1
1
=1
=1
x = 0 (2 x)
x = 0 (2 x)


(x
= 0 x = 1)
(x = 0 x = 1)


x = 0 2 x = 1
(x = 0 x = 0)

[x = 0 (x = 1 x = 0)] (x = 0 x = 1)

1 ( ) , x
cos x = 0 cos x = 1 .
.
x ,
. | sin x| < 1, sin2n x < 1,
cos2m+1 x > 0, cos x > 0.
, (1)

cos2m+1 x + sin2n x = sin2 x + cos2 x

cos2m+1 x cos2 x = sin2 x sin2n x 0


cos2m+1 x cos2 x

cos x 1, .
(1)

x = 2k, x = l +

, :

x=

+ x = 2, Z
2


.
3 ( )
x (cos x
= 0) (cos x
= 1),
:




a = cos2 x, sin2 x



b = cos2m1 x, sin2n2 x

a b =1

a | = cos4 x + sin4 x < cos2 x + sin2 x = 1

 |




2(2n2)
2
2(2m1)
2
b
=
cos
x
+
sin
x
<
cos
x
+
sin
x
=
1
 





|
a | b  < 1 =
a b

x = + x = 2, Z .
2

k, l Z.
2

2 ( )
x R (1), 1 <
(1) . :

a b



=
cos
a
,
b , .


|
a | b 

2+1 x 2 x, (2)

14 ( ) :

2 x 2 x, (3)

(2) (3)
, ,
:

2+1 x + 2 x < 1
11

 z
 1 x 25
= 4 + 9 25

2
36

 



y
x
3
1
1
6 4 =4+ 4






z
y
5
9
9
15 6 = 4 + 4 16


http://www.mathematica.gr/forum/viewtopic.php?f=21&t=13062

1
3
5
(a )2 + (2b )2 + (3c )2 = 0.
2
2
2

1 ( )

1
3
5
( )x = a, ( )y = b, ( )z = c
2
4
6

1
1 3
3 5
5
( )x = , ( )y = , ( )z =
2
2 4
4 6
6

a=

(x, y, z) = (1, 1, 1) ,
, .

25
1
9
+ 9c2 , 6b = + a2 , 15c = + 4b2 .
4
4
4

15 ( )
AB (30 , 60 , 90 ) ,
 = B
 = 30 E B
B
(O, O = OE) , E

. x = AO.

BEO :

a(2 3)

OE = BE tan 15 =
3

a( 3 1)
a

E = B BE = a =
3
3

a( 3 1)

OK A. OK = E =
.
3
AK = A K = A OE =

a(1 + 3)
a(2 3)

=
a 3
3
3

http://www.mathematica.gr/forum/viewtopic.php?f=22&p=76018#p76018

1 ( ) B = a.
A = a 3. B :

AOK :

OK
31
=2 3
=
tan x =
AK
3+1

B 2 = B2 + 2 2B cos 120 = 3B2

a
B = B 3 BE = B =
3

x = 15 .
2 ( )
.
12

3 (KARKAR) (BC) = 1. OZ
AB, OH AC OK ( DO ).


EB

BDK
:

0
 = OBE
 = 30 = 150
OB
2

BD =

 = ZBA
 + BAZ
 = 300 + 300 = 600
BZ

1
2
3
, DK = , BK =
3
3
3

BO ,

2 33
DO =
3


 = 600 = Z
BZ

Z = Z

0 ,ZB=90
0


BZ=30

2 2 3

BO =
3

B + Z
BZ
=

2
2

BZD ,

BOZ

3 3
2
... =
OZ = BO
2
3

 = ZB
 = 15
. . . B = Z BZO
0

3 3
3
=
OH = EC = 1
3
3

( OZ = OH AO
A.
)

 = 600 150 = 450 = OBA
OZ

 =x
 = 150
OA
 = ZBO
(
)

16 ( )
AB(108 , 36 , 36 )
 = 6 , B
 =
: BA




30 , A = B = 18 . x = A.

13

http://www.mathematica.gr/forum/viewtopic.php?f=22&p=74911#p74911

(
:

=
. , ,  = AE
 = 72 , EA
 = 36 , BEA
=
(EA

 = EAB
 = 36 , =
108 , EBA
. EN = EB
2 = EM ,
, 48 .


 = 24 + 72 = 96 .
M
A = M
E = 24 A

17 ( )

, :



 


=6

 

 
 
 |


|  

  9

)
  9.
2

2
) + = 9, )


 

, .

 

+   
  = 9

http://www.mathematica.gr/forum/viewtopic.php?f=23&t=3678

( ) )

2
=
 +  +
 

 
 


 
 

2 |
| = 
 +  +
   
 +   + 
   = 6

( 2i) 48
). 9
6. 3.


= 0

, .

18 ( )


= 0

|
x
|
x = |
x + 8
|

2 =
 + 
 + 
  
 
 

  
 
 

2   = 
 + 
 +   
 +   + 
 = 6

 
 

 
4 |
|   36 |
|   9

http://www.mathematica.gr/forum/viewtopic.php?f=23&t=1990

1 ( )
x =
, :

0 = 9|
|
|
| = 0
= 0
14


(
=

0)
:
x =

|x + 8
|

|x |

|x + 8
|
0

|x |

0 1 : 2 = 8. .

(1) = 4, :
x = 4
,

2 ( ) 
x =
 , > 0, ,

x =

:

|

 | 
= |
 +8
 |
| 1| = | + 8|

x =
(1)

, , ,
:



= 0.
:


 2
  = | + 8|

2
=+8

( > 0)
2
= 8

2 2 8 = 0

2
= 8

|
| | 1|
= |
| | + 8|

|
|
= 0
: | 1| = |l + 8|
> 1 : ( 1) = + 8 = 4 =
2(). = 4

= 4, = 2

19 ( )
A, B

P (B) =

1
2

P (A B) =

y B y
/ A A B =
2. :

P (A B) = P (A) + P (B)

3
4

1
3
= P (A) +
4
2
1
P (A) =
4

P (X), X ,
X A = B

3. P (X)

http://www.mathematica.gr/forum/viewtopic.php?f=18&t=13836

( ) 1. ,
. ,
,
,
,
, -
.
:

X AX
BX
P (B) P (X)
1
2 P (X)

y B y (X A) { y X y
/ A}
15

X A=B
X AB
P (X) P (A B)
P (X) 34

http://www.mathematica.gr/forum/viewtopic.php?f=18&t=12996

X A = B X = A B ... (1)

( )
)
1 :


(1):

x
=

yX
{ y (X A) y (X A)}
{ y B y A} y A B

1 + 1 + ... + 1
x1 + x2 + ... + x2009
>
=1
2009
2009

s
< s CV < s
x

f  (x) 0

X A B ,

y AB
{ y A y B}
{ y A y (X A)}
yX

f  (x) 0
(x s) (x CV ) 0
x (, CV ] [s, +)

A B X , A B = X
sxima

x

f (x)
f (x)

20 ( )

CV

x = CV
x = s

x1 , x2 , ..., x2009
f

s = 2CV

f  (x) = (x s) (x CV )
s, CV
(s
= 5).
1
.
. x
=2
.

s
=2 x
=2
CV

) i) () : y = x + 1
f
3 : f  (3) = 1 (3 s) (3 CV ) = 1

s2
3
= 1
9 3CV 3s + sCV = 1 9 s 3s +
2
2
s
=
5
s2 9s + 20 = 0 (s 4) (s 5) = 0 s = 4
ii)

y1 + y2 + ... + y2009
=
2009
f
(x1 + 1) + (x2 + 1) + ... + (x2009 + 1)
=
3
2009
i. s = 4
2009 (x1 + x2 + ...x2009 )
=1x
= 1 2 = 1
ii.
2009

2
(x1 , y1 ) , (x2 , y2 ) , ..., (x2009 , y2009 )
2009
2009
i=1 xi
1  2

2
iii)
S
=
x

x)2 =

= x2 (
x
i
().
2009
2009
() : y = x + 1

y =

iii.

i=1

x2 4 x2 = 4 + 16 = 20

16

21 ( . )
z, w

|z 2| = |z + 4i|

z, w
.
. :

|z w|min = d(1 , 2 ) =

|1 + 2(2) 3|
6
6 5

= =
d(M, 2 ) =
5
5
12 + 22

|w + 2| = |w 4i|
) z, w
.
) |z w|.
) |z + w|.
) |z w 2i|.

. w

2 : x + 2y 3 = 0
w 2 :
O(0, 0), 1 . , z
w . ,

|z + w|min = |z (w)|min = 0

http://www.mathematica.gr/forum/viewtopic.php?f=51&t=2661

( )
. v = w + 2i v = + i w = x + yi
. |z 2| = |z + 4i| (1) (1)
, , x, y . w
, A(2, 0)
2
B(0, 4) AB = 2,
+i = x+yi+2i x+2y 3 = 0,
1
v 3 : x + 2y 7 = 0

1 =

|z w 2i|min =

M (1, 2) .

|z (w + 2i)|min =

1
1 : y + 2 = (x 1) x + 2y + 3 = 0
2

d(1 , 3 ) = d(M, 3 ) =

10
|1 + 2(2) 7|

= =2 5
5
12 + 22

|w + 2| = |w 4i| (2)
(2)
, (2, 0) (0, 4),

= 2

22 ( . )
z1 , z2 , z3
, , .

z1 + z2 + z3 = 0

1
=
2

(1, 2) .

1
2 : y 2 = (x + 1) x + 2y 3 = 0
2

1 = 2

z12 + z22 + z32 = 0


) z1 z2 + z2 z3 + z3 z1 = 0
) z33 = z1 z2 z3
) z1
= 0 , z2
= 0 , z3
= 0 |z1 | = |z2 | = |z3 |
) .
17

) z1 = 0 z2 + z3 = 0 z22 + z32 = 0.

http://www.mathematica.gr/forum/viewtopic.php?f=51&t=2606

z2 + z3 = 0 z2 = z3

( )
) :

(z1 + z2 + z3 )2 = 0

(z2 )2 = (z3 )2 z22 = z32

z12 + z22 + z32 + 2 (z1 z2 + z2 z3 + z3 z1 ) = 0

z32 = z1 z2

z22 + z32 = 0 2z22 = 0 z2 = 0


z2 = z3 z1 , z2 , z3
. z2 = 0z3 = 0
3
z1
= 0, z2
= 0 , z3
= 0 . ) z32 = z1 z2 |z3 | =
2
|z1 | |z2 | z12 = z2 z3 |z1 | = |z2 | |z3 |
3
3
: |z3 | = |z1 | |z1 | = |z3 |
: z13 = z1 z2 z3 , z23 = z1 z2 z3 ,
z33 = z1 z2 z3 z13 = z23 = z33
.
) z32 = z1 z2 , z2 z3 = z12 . :

z33 = z1 z2 z3

z3 (z3 z2 ) = z1 (z2 z1 )

z1 z2 + z2 z3 + z3 z1 = 0
) :

z1 + z2 + z3 = 0
z1 z3 + z2 z3 + z32 = 0
z32 = z1 z3 z2 z3

z13 = z1 z2 z3

|z3 z2 | = |z2 z1 |

z23 = z1 z2 z3

23 ( )
f : (0, +)
(0, +) x R

"

f (x)f

1
f (x) +
x

1
2

1
1
+
2f (x) f (x) +

1
1
+
2f (x) f (x) +

http://www.mathematica.gr/forum/viewtopic.php?f=52&t=2712

( ) :

"

f (x) +

1
x

x : f (x) + x1 , x > 0 :


= f (x)

1
x

f 1-1

f (1).

1
x

=x

2xf 2 (x) f (x) =

1
x

, f (x) > 0,

1
2f (x)

 "

#
1
1
1

 =

+
f f f (x) +
1
x
2f f (x) + x1
f (x) + x

f (x) =

1
x

f (x) = x1 , x > 0
.

18

24 ( )
f : R R, :

f (f (x) + f (y)) = x + f (y) + 2010

f 1-1 .
(1) x f 1 (x) y f 1 (y),
:

f (x + y) = f 1 (x) + y + 2010, x, y R

x, y R.

y = x
http://www.mathematica.gr/forum/viewtopic.php?f=52&t=1350

f (0) = f 1 (x) x + 2010

( )

f 1 (x) = x + f (0) 2010, x R

f (f (x) + f (y)) = x + f (y) + 2010 (1) x, y R

x, y

f (f (y) + f (x)) = y + f (x) + 2010 (2) x, y R

f (x) = x + 2010 f (0), xstoR

x, y R f (x) = f (y )

x = 0 f (0) = 1005

f (x) + f (y) = f (y) + f (x)

f (x) = x + 1005 x R

f (1) (1),(2)

f (f (x) + f (y)) = f (f (y) + f (x))


(1) , (2)

y + f (x) = x + f (y) (3) x, y R

x + f (y) + 2010 = y + f (x) + 2010

y R f (x) = y x,
x = 2y f (y)
(3).

x=y

25 ( 
)


x 2 , 2

ex
x

ex > 0, x > 0,

1 x

0<x+1<
http://www.mathematica.gr/forum/viewtopic.php?f=53&t=13654

$

, 1
2

+1
2

ex x + 1(x + 1 > 0)
1
>0
x > 0 x

( )

<x< +1<x+1< +1
2
2
2
2

+1<x+10
2

(1, )
2
19


x ( , )
2 2
.
26 ( )
f [0, 3]
(0, 3). f (0) =
f (3) = 0 f (1)f (2) < 0,
k [0, 3)


f (k)f (k) + f (k)

2


:
f (1)f (2) < 0
0.
f  (0)
f (1)f (2) < 0

g(x) f (x)f  (x) 2x = 1/2(f (x) 2x2 )

(f (x) 2x2 ) > 0

(0, 3)

2k = 0

h(x) = (f (x) 2x2 )


. Bolzano f
[1, 2] p (1, 2) : f (p) = 0

http://www.mathematica.gr/forum/viewtopic.php?f=53&t=13312

f 2 (p) 2p2 < f 2 (3) 2.32

( ) g(x) = f (x)f  (x) +



2
(f (x)) 2x g [0, 3] Rolle
p2 > 9
f

m (0, 3) : f  (m) = 0 g(m) = 2m < 0

a < b : h(a) > h(b)

g(0) = (f  (0))2 0
f  (0) = 0 k = 0 f  (0)
= 0
Bolzano g [0, m]

q : h (q)
0 h h (x)/2 0 q
g(x) 0 q . Bolzano
g 0 q

27 ( )
f [a, b] f  (x) > 0

%b

x [a, b]. F (x) = a |f (t) f (x)| dt.


F .

f . ,
a t x,
x t , f , F

&x

&
(f (x) f (t)) dt +

F (x) =
a

http://www.mathematica.gr/forum/viewtopic.php?f=54&t=4746

(f (t) f (x)) dt
x


( )

&x

&x

|f (t) f (x)|dt +

F (x) =

F (x) = f (x)(x a)

&

|f (t) f (x)|dt
x

&
f (t)dt f (x)( x).

f (t)dt +
a

F , ,

F  (x) = f  (x)(2x a ).

x [a, b].
20

f  (x) > 0 x [a, b],


F x = (a + b)/2 .

( ) :

&

|f (t) + f  (t)|dt
0
& x
x
x
e |f (x)| e |f (0)| + ex
|f (t) + f  (t)|dt
& x 0
x
et |f (t) + f  (t)|dt
e |f (x)| |f (0)| +
|f (x)| |f (0)| +

28 ( )

f : [0, +) R

&

e |f (x)| |f (0)| +

&0

ex |f (x)| |f (0)| + 

&
|f (x)| |f (0)| +

x

0
x


[f (t)et ]  dt

x

t  
[f (t)e ] dt

ex |f (x)| |f (0)| + |e f (x) f (0)|

|f (t) + f  (t)| dt

x [0, +).

.
f : [a, b] R ,

 & b
& b



f (x) dx
|f (x)| dx

a

http://www.mathematica.gr/forum/viewtopic.php?f=54&t=4654

29 ( )
ABAB
CCC
639027.
A+B +C A, B, C ;

2 : C = 1 10A + B = 57,
A = 5 B = 7 ( A B
)
,
A + B + C = 13.
30 ( )
:

http://www.mathematica.gr/forum/viewtopic.php?f=49&t=12719

( ) :

(1000A+100B+10A+B)(100C+10C+C) = 111319101

x+y 1
2
x2 + xy + y 2
3

101(10A + B) 111C = 111 3 19 101,

http://www.mathematica.gr/forum/viewtopic.php?f=49&t=13221

(10A + B)C = 3 19

1 ( )
, :
1 : C = 3 10A + B = 19,
1
1
x = (a b) y = (a + b) .
A = 1 B = 9.
2
2
21

= 1 43 = 13 < 0 .
3 ( ) x, y

1
1
x + y 1 (a b) + (a + b)  1
2
2
1
a ,
2

x2 + y 2 + xy

x + y 1,

3
1
1
1
2
(a b)2 + (a + b)2 + (a b) (a + b) 
2
2
2
3
3 2 1 2 2
a + b 
2
2
3
x2 + y 2 + xy 

2
.
3

x2 + x (y 1) + y 2 y +

1
0 (1)
3

3
1
3 1
2
 a2 + b2  + b2 ,
3
2
2
4 2

"

1
= (y 1) 4 y y +
3
1
= 3y 2 + 2y
3
1
= (3y 1)2 .
3
2

.
().

,
4
2
, x2 + y 2 + xy 
3
M (x, y)
y2
x2
+
= 1,

1
1
4/9 4/3
< 0 x2 + x (y 1) + y 2 y + > 0,
y=

3
3
x + y  1 M (x, y)
1
x  . . . y = 1 . (1)
3
2

.
2 ( )
x, y
, :
,

1
x > 0 x + xy  x  1 y y y +  0,
3
2

1
1
2
1
x2 x + 0 (x )2 0 x = .
3
9
3
3
, x + y = 23
x + y 1.

:
2
() a1 = a2 = 0, b1 b2
= 0, bb12 Q c1bc
Q
2

31 ( )
f : R R


a1 x2 + b1 x + c1 , x Q
f (x) =
a2 x2 + b2 x + c2 , x R Q

http://www.mathematica.gr/forum/viewtopic.php?f=50&t=8128

() f 1-1

( ) (3) = (2), (1) :


(3). ,
, 1-1. x R

() f

22

x c1
x c2
Q

/ Q ( ,
b1
b2

b2 c2 c1
,
Q) 1-1
b1
b1
.

.
2 = 3 :
.
a2 = 0, b2
= 0,
(

).
a1 = 0, b1
= 0,

x c2
Q
b2

.
x

x c1
Q
b1

x c2

/ Q ( ).
b2
c2 c1
x = c2
Q.
b1
b2
x = c2 + b2
Q.
b1
1 = 3 :

http://www.mathematica.gr/forum/viewtopic.php?f=50&t=6120

( )

K + E = A + 2, K, E, A ,
.
K + E = A + 1, E = A E + 1.
[ ,
.

( ,
, )].

:
" #
() : n +

n
4


1-1.
[ 4
a2 = 0, b2
= 0, , ,

.
, (
1-1
- )].
" #
a1 r 2 + b1 r + c1 c2
n

, r Q
() : n +

b2

2
c1 c2
" #
Q,
n
b2
+ 2
.
4
a1 b1
r = 1
,
Q. , a1
= 0,
Euler
b2 b2
1-1
" #
" #
" #
n
n
n
a1 , b1 . , a1 = 0, b1
= 0.
E = n+
+2
n
+ 1,
2
4
4
Q. r = 0

32 ( )
n .

.

" # " #
n
n
E=
+
+ 1.
2
4

33 ( -
1985) k R L(k)
[x] = kx 1985.
:

(i) k > 2 1 L(k) 2.


23

(ii) 0 < k <

1
, L(k) = 0.
1986

(iii) k R L(k) = 1985.


([...]= )

1985
.
1986
x R

(iii) k =
http://www.mathematica.gr/forum/viewtopic.php?p=75041

( )

[x] = kx 1985.

(i) k > 2 x R
[x] = kx 1985.

2)

[x] x < [x] + 1


k 1985
1985
[x] <
1k
1k

1985 k
1985
< [x]
.
k1
k1
1985 k 1985
k
,


.
k1 k1
k1
k
k > 2 1 <
< 2.
'k 1
"
1985 k 1985
,

k1 k1

1985 1986 [x] < 1985 1985 1986.


1985
.
1985 .
[1985 1986, 1985 1985 1986).

1985
+

k
k
. [x0 ] = .

x0 < +1.

# [1985 1986, 1985 1985 1986).
"
1985
L
= 1985.
1986
x0 =

2 .
2
[x], x"
.
'

1985 k 1985
,
.
k1 k1

x0 =

1985
+
k
k

.
[x0 ] = .
x0 < + 1.

'
"

34 ( -
1984) (
)
.

1985 k 1985
,
.
k1 k1

http://www.mathematica.gr/forum/viewtopic.php?p=74877

1
(ii) 0 < k <
. 1 ( )
1986

x R
A1 A2 A3 A4 A5 . 
v1 = A1 A2 ,

[x] = kx 1985.
v2 = A2 A3 , v3 = A3 A4 , v4 = A4 A5 v5 = A5 A1 ,
5


vi = 0 (1), |vi | = r > 0 i = 1, . . . , 5

[x] x < [x] + 1


i=1
v1 v2 = v2 v3 = v3 v4 = v4 v5 = v5 v1 = 0.

a1 = 
v1 v3 , a2 = v1 v4 , a3 = v2 v4 ,
k 1985
1985
a4 = v2 v5 , a5 = v3 v5 . (1) ,
[x] <
.

5

1k
1k
5r 2
ai =
(2). (1)

1
2
i=1
0 < k <

1986

vi , i = 1, . . . , 5

ai
k 1985
1985
3r 2
[x] <
< 1985
1986 <
. , (2),

1k
1k
2
ai r 2 , :
.
r2
a1 = a2 = a3 = a4 = a5 = .
.
2
24

CD . ABC = DEA = 90 ,

r2
, .
v1 v3 =
= r 2 ).
,
( 
2

v1 , v2 , v3 R3 ,


x1 , x2 , x3 , v4 = x1v1 + x2v2 + x3v3 (3). ,
(3) 
v1 , v2 , v3
x
12 = x1 23
,

12 = x1
,

0 = x21 + x3

#
"
2 1 1
, ().
(x1 , x2 , x3 ) = , ,
3 2 3
,
2
1
1
v4 = v1 v2 v3 (4).
|AB| = |AE| = 1, CD//BE ABE =
3
2
3
AEB :
r2 r2
(4) 
+ , EAB = 60 . (
v5 , 0 =
4
6
ABCDE (BCDE )
r = 0, . , .
(EBA) .)
2 ( ) ABCDE
():
, |AB| = |BC| = |CD| =
,
|DE| = |EA| = 1 ABC = BCD = CDE =
, (
DEA = 90 . EAB = 60.
),
BCD = CDE = 90 , |CD| = 1, |BC| = |Y Y  |2 = |Y Y  |2 + |Y  Y  |2 , |Y  Y  |
|DE| = 1, ( ) |Y Y  |
, 1 Y XY  = Y XX  ( |Y X|
C , D , , 1 ).

v1 , v2 , v3
, 
v1 v2 , v2 v3 v1 , v3

35 ( .)
T n . n + 1
n
, T = I .

T
n+1

i=2

1 ai xi = 1 x1 =

n+1


i ai xi

i=2


1
.

http://www.mathematica.gr/forum/viewtopic.php?f=59&t=5627

1 ( )
x1 , . . . , xn+1 1 , . . . , n+1 .
a2 , . . . , an+1
:

x1 =

n+1

i=2

ai xi .

2 ( ) A = Pk Dk Pk 1 k = n + 1
n
k = 1, 2, . . . , n
k - k - (n+1)- (n+1)-
.

25

Mihai Piticari.

.
3 ( ) ,
E = {v : T v = v}.
E
.

,
.

http://www.mathematica.gr/forum/viewtopic.php?f=59&t=6112

( )
f , f (0) = 0
f (1) = 1. Stieltjes

&

&

dim(E )  n.

n + 1 ,
dim(E ) = k k +1
E . n
, k = n.
E = V , V n- ,
T = I .
36 ( )
f : [0, +) [0, +) a
, f (f (x)) = xa
x [0, +).
1
0

f [f (x)] df (x) =
xa df (x)
0
0
& 1
& 1
a
f (x) dx = 1 a
xa1 f (x) dx,
=1

f (x) dx =

&

&

&

(1 + axa1 )f (x) dx = 1,

Cauchy-Schwarz

&
0

2a 1
.
f (x) dx 2
a + 6a 3
2

f 2 (x) dx % 1
0

1
(1 +

axa1 )2

dx

2a 1
.
a2 + 6a 3

37 ( ) p A (p1)(p1)
Ap =
I
= A. f (x) -

p 1, f (A) .
( 1425 Mathematics Magazine.)

p-
1 ( A = I
).

p 1, f (A) .
2 ( ) Ap I = 0,
p(x) A xp 1.

xp 1 = (x 1)(1 + x + x2 + + xp1 ).

http://www.mathematica.gr/forum/viewtopic.php?f=10&t=10347

1 ( )
p- Q, p ,

p1

+x

p2

+ + 1.

A (p 1) (p 1), -

1 + x + x2 + + xp1
Q ( Eisenstein) A
= I
deg(p(x)) p 1,

p(x) = 1 + x + x2 + xp1 .
f (x)
p 1,

26

f (x) p(x)
.

. a, b Z a + b = p
ab 1 mod p,

g(x)f (x) + h(x)p(x) = 1

f (x) = x2 + 1 = x2 + px + 1 = x2 + (a + b)x + ab =

g(x), h(x) Q.
, g(A)f (A) + h(A)p(A) = I .
p(A) = 0, g(A)f (A) = I ,
f (A) g(A).

= (x + a)(x + b),
f (x) .
a, b Z

38 ( )

x2 + 1 Zp [x], p ,

p = a + b ab 1 mod p.
f (x) = x2 + 1 Zp [x], r, s,

a, b
a + b = p ab 1 mod p.

r + s 0 mod p rs 1 mod p.
a, b {0, 1, . . . p 1} a r mod p
b s mod p. a + b r + s 0 mod p
a + b = p ab rs 1 mod p, .

http://www.mathematica.gr/forum/viewtopic.php?f=10&t=11962

( )

f (x)
, .

f (x) = x2 + 1 Zp [x]

&/2
&
u=x
cos2k+1 (x) dx +
cos2k+1 (u) du =

39 ( )
f : [a, b] R.

&
b

lim

n a

f (x)
dx.
3 + 2 cos(nx)

&/2

cos2k+1 (x) dx

&
a0 +

http://www.mathematica.gr/forum/viewtopic.php?f=9&t=4428

/2
&/2

cos2k+1 (x) dx = 0,

ai cos(ix) dx = . . . = a0 = 0 a0 = 0 .

i=1

m = 2k :
( ) (x) = &
&
m
cos (x) , m = 1, 2, 3, . . .
(x) dx = cos2k (x) dx =
.
m

. (x) = cos (x) = a0 +


a0 .
:
m = 2k + 1 :

&

&

(x) dx =
0

ai cos(ix)

0
&/2

0
2k

cos (x) dx +

i=1
0

&/2

cos2k (x) dx +

cos2k+1 (x) dx =

27

&

cos2k (u)du

/2
&/2

u=x

cos2k (x) dx =

/2
%
0
&1
0

k+ 21 1

&b

cos(x)=u

u= y
u2k

du =
1 u2

2
&1

cos2k (x) dx

(1 y)

1
1
2

&1
0

yk
dy =

y 1y

1
lim
3 n



dy = B k + 12 , 12 =

(2k)!
  


k + 12 12
k
(2k)!


.
= 4 k!
= k
k!
4 (k!)2
k+1
&
&


a0 +
ai cos(ix) dx = a0 dx = a0 ,

i=1

(x) = cosm (x) = a0 +

&b

1
3

&b

1
f (x)
dx =
3 + 2 cos(nx)
3

f (x)

"


(1)

k=0

2k
3k

&b
a

lim

lim

f (x) cosk (nx) dx =

(2m)!

.
9m (m!)2


(2m)!

1
3

1
m
x
.
=
2
(m!)
1 4x

m=0

f (x) dx


m=0

1
1

4
9

(2m)!

=
9m (m!)2

1
=
5

&b
f (x) dx,
a

1
f (x)
dx =
3 + 2 cos(nx)
5

&b
f (x) dx
a

40 ( )
a > 0. ,

1
lim
n+ ln n

#
cos (nx) dx =

k = 2m + 1 : ,

&b

&b

f (x) dx (

2
3

k &
1 
k 2
f (x) cosk (nx) dx .
(1) k
3
3
k=0

&b

&b

f (x)
dx =
1 + cos(nx)


m=0

k=2m
=

1
3

&b
a

f (x) cosk (nx) dx

&b


2m
2
(2m)!
f
(x)
dx
=
(1)2m 2m m
3
4 (m!)2

f (x) dx

ai cos(ix) ,

Riemann Lebesgue
n

k=0

f (x) cos(nx) dx = 0 .

&b

2k
(1) k
3
k

i=1

m = 2k + 1
a0 = 0 ,
(2k)!

, m = 2k
a0 = k
4 (k!)2

lim

m=0

&b

1
3

(2k)!
.
a0 = k
4 (k!)2

1
3

f (x)
dx =
3 + 2 cos(nx)

lim


1kna

1
k

"

1
1
n

#k

http://www.mathematica.gr/forum/viewtopic.php?f=9&t=10924

( , )


1 k
x = ln(1 x) |x| < 1,
a 1:
k
k=1

,
n,





1
a
1
ln 1 1 n1 +
O (1 1/n)n =
ln n
ln n
f (x)
ai cos(inx) dx = 0
lim

n
1
a
n
i=1
O (1 1/n)
.
1+
a
ln n
k = 2m : ,
, a 1,
&b
a
(1 1/n)n (1 1/n)n 1/e,
lim
f (x) cosk (nx) dx =
n

1
a
a
O (1 1/n)n 1.
1+
ln n
&b
&b


#
 1"
f (x) a0 +
ai cos(inx) dx = a0 f (x) dx =
lim
1 k
1
n
1

0
<
a
<
1
:
=
i=1
a
a
ln n
k
n
a
1kn
&b
(2m)!
"
"
##
[na ]
f (x) dx .
k
k(k 1)
1 1
4m (m!)2
1 +O
=
a
ln n
k
n
n2
&b

k=1

28

[na ]

1 
ln n
k=1
[na ]

"

1
1
+O
k n

"

k1
n2

##

[na ]

[na ]

1 1
1 
1 1
O

+
ln n
k ln n
n ln n
k=1

k=1

[na ]

[na ]
1 1

+O
ln n
k n ln n
k=1

ln[na ]
[na ]

+O
ln n
n ln n

"

"

1
ln n

"

k=1

1
ln n
#

1 + 1/2 + + 1/n n+
1
ln n
1 n+
1
1 + + +
ln n,
2
n
[na ]
na + 1
na

na [na ] na + 1
.
n ln n
n ln n
n ln n
[na ]
0

n ln n
ln[na ]
ln(na + 1) DLH
ln na

a=
a.
ln n
ln n
ln n
#
 1"
1 k
1
= min{1, a}.
1
lim
n+ ln n
k
n
a

k1
n2

[na ]<n

n+

,
) *+

n+

)
*+ , a.

1kn

Cesaro - Stolz

41 ( Stuart Clark)
A,B , C
< 1, 1, 1 > , < 1, 1, 2 >, < 0, 2, 1 >.

ABC < 1, 0, 1 >.

(
1.)
42 ( )

f : R2 R2

1) f (0) = 0
( ) , - 2) |f (u) f (v)| = |u v| u, v
, f .
1, 0, 1
ABC ,
http://www.mathematica.gr/forum/viewtopic.php?f=11&t=13048

AB AC =

http://www.mathematica.gr/forum/viewtopic.php?f=11&t=4493

(1, 1, 2 1, 1, 1) (0, 2, 1 1, 1, 1) =


0, 2, 1 1, 1, 2 =
3, 1, 2
1, 0, 1.

1, 1, 2 1, 0, 1
1, 0, 1 3, 1, 2 = 1, 5, 1.
1, 5, 1

12 + 52 + (1)2 = 3 3

1 ( ) [ ]
f
.

f ((a, b) + (c, d)) = f (a, b) + f (c, d)

f ((a, b)) = f (a, b)


: :

.
3 5 3
3
,
,
9
9
9

f ((a, b) + (c, d)) = f (a, b) + f (c, d)


f ((a, b) + (c, d)) f (a, b) f (c, d) = 0
29

f 2 (a + c, b + d)) + f 2 (a, b) + f 2 (c, d)

2 ( ) [ ]
f .
=.
1) , , .
2)

2f (a, b)f (a + c, b + d) 2f (c, d)f (a + c, b + d)

+= ()

|f ((a, b) + (c, d)) f (a, b) f (c, d)| = 0


|f (a + c, b + d) f (a, b) f (c, d)|2 = 0
(f (a + c, b + d) f (a, b) f (c, d))2 = 0

+2f (a, b)f (c, d) = 0 (1)


()
=, =, =

|f (a, b) f (c, d)| = |(a, b) (c, d)|


|f (a, b) f (c, d)|2 = |(a, b) (c, d)|2
f 2 (a, b) + f 2 (c, d) 2f (a, b)f (c, d) =
(a, b)2 + (c, d)2 2(a, b)(c, d) (2)

|f (a, b) f (0, 0)| = |(a, b) (0, 0)|


|f (a, b) (0, 0)| = |(a, b)|

+= ()
f
. .
3) f
. .
4) ,,,
.

f (u + v ) = f (u) + f (v )

|f (a, b)|2 = |(a, b)|2


|f (a, b)|2 = (a, b)(a, b)
f 2 (a, b) = a2 + b2 (3)
(2) (3)

f (a, b)f (c, d) = (a, b)(c, d) = ac + bd (4)


(1) (3),(4)

5) OA =

u, OB = v > 0 ( ).

OA1 = OA, OB1 = OB


( f
) OAB = OA B , OA1 B1 = OA1 B1
A1 B1 = AB

(a + c)2 + (b + d)2 + a2 + b2 + c2 + d2

f ( (u v )) = f (u v ) ()

2(a+c)a2(b+d)b2(a+c)c2(b+d)d+2ac+2bd = 0
f (O) = O
a2 + c2 + 2ac + b2 + d2 + 2bd + a2 + c2 + b2 + d2
. w

2a2 2ac2b2 2bd2ac2c2 2bd2d2 +2ac+2bd = 0
w
 = u v
0=0
f (w)
 = f (w)
 ()
.

: (3), (4)

f 2 (a, b) + 2 f 2 (a, b) 2f (a, b)f (a, b) = 0

. (), () .

30

43 ( )
20801 2015 1.

a = 1, bm = (2m + 1)2 + 2 = 4m2 + 4m + 3,


m 3. (3, 7).
a > 1, a 3, n < m. , ab2n
1

http://www.mathematica.gr/forum/viewtopic.php?f=63&t=10981

n2 + 2 + 4m
(2m + 1)(n2 + 2)
2n =
=
nm
nm
2
4
2 4
n
+
+ < 1 + + < 3.
=
m nm n
9 3

( ) 20801 = 11 31 61,
20801 .
 11 2015 1,

ab 2n =


3
2015 = (25 105 )3 (1)(1) 1 mod 11.

 31 2015 1,
2015 = (43 53 )5 (2 1)5 1

ab 2n =

mod 31.

 61 2015 1,
2015 (34 )1 5 = 360 1

mod 61

( Fermat).
11, 31, 61
2015 1,
11 31 61|2015 1, 20801|2015 1.
44 ( ) n
m
2m 1 (mod n) n2 2 (mod m).

n = b + 4, m = n + 2a ab = 2n + 1 =
2(b + 4) + 1 = 2b + 9. (a 2)b = 9,

(a, b) = (3, 9), (5, 3), (11, 1).
, (m, n) =
(19, 13), (17, 7), (27, 5).
2 :
m = k < 0 2k 1 = n n2 +2 = k
.
, 2n

n2 + 2 4k
(2k 1)(n2 + 2)
=
=
nk
nk
n 2
4
=
.
=
n
k

2n = 2n

http://www.mathematica.gr/forum/viewtopic.php?f=63&t=10352

( )
m n .
|n| = 1, (m, n) =
(1, 1), (1, 1) (3, 1), (3, 1), |n| = 3,
(m, n) = (1, 3), (11, 3).
|n| > 3
|m| 3.
(3, 7), (19, 13), (17, 7), (27, 5) (3, 5).
, n
m:
1 :
m > 0 2m + 1 = an n2 + 2 = bm
a b.

b+4
n + 2a
n2 + 2 + 4m
=
=
nm
n
m

9, n
4, n > 3 . ,

2
4
< < 2.
n
n

, n = 4, k = n 2
= 2n 1 = 2 9. , (2 ) = 9
(, ) = (1, 9) ,
(m, n) = (3, 5).
0 < 2n =

: 1676 Mathematics Magazine. 10 ,


77, . 3,
2004. 10 4 .
.

31

45 ( )

{x y 0 x + y + 6 0} (3)

f (x) = x2 + 6x + 1, x R
S (x, y)
:

f (x) + f (y) 0
f (x) f (y) 0

(2)
(3)
S
KB AKB.

E=


S .

R2
= 8
2

46 ( )
:
http://www.mathematica.gr/forum/viewtopic.php?f=27&t=14191

sin x + sin y = sin(x + y)


|x| + |y| = 1

( )

f (x) + f (y) 0 (x + 3)2 + (y + 3)2 42


S
K(3, 3) = 4

http://www.mathematica.gr/forum/viewtopic.php?f=27&t=14216

( )

f (x) f (y) 0 (x y)(x + y + 6) 0(1)


2 sin

1 , x y = 0

x+y
x+y
xy
x+y
cos
= 2 sin
cos
2
2
2
2

sin

C : (x + 3)2 + (y + 3)2 = 42

A(3 2 3, 3 2 3) , B(3 + 2 3, 3 2 3)

cos

x+y
=0
2

xy
x+y
= cos
2
2

2 , x + y + 6 = 0

x + y = 2k x = 2k y = 2k, k

C : (x + 3)2 + (y + 3)2 = 42

(3+2 3, 32 3), (32 3, 3+2 3) .


1 = |x| + |y| |x + y| = |2k| = 2|k|
1 , 2
k = 0, x + y = 0.
C, K(3, 3)

, 2

(1) {x y 0 x + y + 6 0} (2)

x = y =
32

1
1
x = y = .
2
2

1 = |2k| + |y| 2|k|

"

k = 0, x = 0 y = 1
,
3
x = 1, y = 0.

1 1
,
2 2

# "
#
1 1
, ,
, (0, 1), (1, 0), (0, 1), (1, 0)
2 2

47 (

n((r + 1)n1 r n1 ) = 0

(n 1)((r + 1)n2 r n2 ) = 0

p(x) = (x + 1)n xn 1

r n2 (1 + r) r n1 = 0 r = 0 ().

m = 1, m = 2.
(x) = (x2 + x + 1)m
- m = 2 n = 6k + 5

4
n, m . n, m p (r) = 0 ... r = 1 r = 1 ().
-
m = 2 n = 6k + 1
(x) p(x);

(1)n1 r n1 = 1
n 1 3
http://www.mathematica.gr/forum/viewtopic.php?f=60&p=66638#p66638 m = 1, n = 6k + 1 n = 6k + 5
n = 6k + 1, m = 1 m = 2.
( ) r
48 ( )
p. r 2 + r + 1 = 0 r 3 = 1, r
= 1

n = 6k + v, v = 0, 1, 2, ..., 5
ax2 bx + c = 0
p(r) = 0

a, b, c . a, b, c
(0, 1)

(r + 1)6k+v r 6k+v 1 = 0
(r 2 )6k+v r 6k+v 1 = 0

a+b+c

r 3 = 1

(r 2 )v r v 1 = 0

http://www.mathematica.gr/forum/viewtopic.php?f=33&t=9362

v = 1, v = 5

( ) f (x) = ax2 bx + c

n = 6k + 1, n = 6k + 5
p
r

(1)

p(r) = p (r) = p (r) = 0


x1 , x2

(r + 1)n tn 1 = 0

f (0) > 0

b2 4ac > 0



33

(1)

k2 > 4a

f (1) > 0 a b + c > 0 (2)

(4)

2a > k (4) k > 2


x1 + x2
- k = 3 a k = 3 4a 12 > k 2 ,
0<
< 1 b < 2a (3)
2
(4)
(3) b = 2a k (2)
- k = 4 a k = 4 4a 16 = k 2 ,
c > a k
(4)
- k = 5 a k ,
c =
a, a = 5
ak+1

34

http://www.mathematica.gr. .
-- : . : . LaTEX.
.

Leonardo da Vinci
(32-) . 30
. -
-
- quasiregular , ( ).



(0, 0, ), 12 ,
, 1+
,
2
2

1+ 5
.
2

:http://en.wikipedia.org/wiki/Icosidodecahedron
:

mathematica.gr
(http://www.mathematica.gr) .

mathematica.gr

12. (achilleas)

13. ( )

1. (Mihalis_Lambrou)

14. (xr.tsif)

2. (nsmavrogiannis)
3. ( )
4. ( )
5.

(m.papagrigorakis)

6. ( )
7. ()

1. (grigkost)
2. (cretanman)

15. (Demetres)

1. (stranton)
2. (spyros)
3. (p_gianno)
4. (kostas.zig)
5. (exdx)
6. (skap)

1. ( )

7. ( )

2. (vittasko)

8. (matha)

3. ()

9. (mathnder)

4. (nkatsipis)

10. ( )

5. ( )

11. (mathematica)

6. (chris_gatos)

12. ( )

7. (mathxl)

13. ( )

8. (gbaloglou)

14. (rek2)

9. (R BORIS)

15. (hsiodos)

10. (dement)

16. ( )

11. (swsto)

17. (bilstef)

1 ( )
.

.
. .
,
, ,
.
.
. .
;

6 ( )
S , 2011 :

2 ( )
.
( ...). , , ,
:
().


183, 186, 187, 190, 191, 192, 193, 194, 196, 200
; (
).

S = 1 2 + 3 + 4 5 6 + 7 + 8 ...


7 ( )
.
. = 8 = 2 ,
.

3 ( qwerty)
, .
2 .
100, .
4 ( )
,
:

LI SARI BLO
GSP OT LI SA

8 ( )





(x + 1) (x 1) x2 5 (x + 2) (x 2) x2 2 4

5
,
1

,
9 ( ) :

( )
,
. 40 ,
;

x2
x3
x4
x1
+
+
+
=4
2+3+4
1+3+4
1+2+4
1+2+3

10 ( themiskant) :

x4 + 5x3 6394x2 32000x 38400 = 0


x > 0.

,
11 ( ) ,
. , Ox, Oy
xOy
, ++ .
12 ( )
,

. :
1. .
2. .
,

3. .
4. .

17 ( . ) :

x +

4 2 y 2 = 0 (1)

1. R,
(1) .
.

13 ( )

+1
ln x
5 = 11, 13 = 35

 
g (x) = 2x 1, x 0,
2
f (x) = ln( ln x) +

2. ,
,
.
18 ( )
AB K, AB ,

, f , ,
.

AK = AB
=

14 ( ) f

0 < < 1. B, K A
.

f (x) = ln (ex + 1)

1. f .
2. f R.
3. f


3x = f (x).

4. f (2x) > f (x).

19 ( )

5. f (x) = f (x) x x R.

= {1, 2, 3, 4, }

P (1) =

15 ( ) C
AB (O, R) Cx.
CA CB (K) , (L)

   (O, R).
K 
(K), (O)


Cx L
   (L)
, (O)

Cx K L .

P (2)
P (3)
P (4)
=
=
2
3
4

A = { |
, 2, 4, 5 10}
B = {x ln(x2 x + 1) > 0}
1. .

16 ( ) ABCD
(O).
D DC, DA, DB
AB, BC, CA O1 , O2 , O3 .
O1 , O2 , O3
O (O).

2. A = {3, 4} B = {2, 3, 4}
3. P (A B) P (A B)
4. X ,
P (X) A X =

20 ( )

1. f .
2. g R.

= {1, 2, 3, ..., 2010, 2011}

3. f (g(x)) = x2 x R.


.
. f (x) = x2 + (1 x)2 x R
.
. A N (A)
(P (A))2 + (P (A ))2
.
. f (x) 1 x [0, 1].
. B,
2

, ,
25 ( )

x5 5x6 + 13x3 19x2 + 16x 8 = 0

(P (B)) + (P ()) + (P (B )) + (P ( )) = 2

26 ( )
f : [, +) R [, +)
lim f (x) = R. f .

B = B = .

x+

, ,

, ,

21 ( barsakis) z ,
:

27 ( )
f : [0, 1] R x, y
xf (y) + yf (x) 1.

1. i, z, zi

2. 1, z + i, iz + 1

f (x) dx

i)

3. 1, z, z 2 + 1

.
4

ii) .
22 ( g.liolios ) M M1
z = 2 + 2i R z1 = 3 i M
OM M M1 (
).

28 ( )

I(a) :=
a > 0.

1. M
z = x + yi.
2. w
w = 1z .

f (x) =

1
, x (0, )
sin x

) f .
) f .
) f .
) Cf
x x x = 3 , x = 2 .

f (x y) = f (x) f (y) , x, y R
1. f
.

30 ( ) f :

2. f .

R R R f (0) = 0 f (2) = 4.
) x > 0 : x f  (x) < f (x).
x f (t)
) h(x) = 2 t dt x > 0 ,
i) h(x) (0, +)
ii) h(x) 2x 4 x > 0.

3. f (x) = 0 x = 0.
() f .
() z |z| = 1,






z + i  + f (3) = f (7)

z
z
z

|a (2x) 2 x| dx,

29 ( ) f
:

23 ( )
f : R R

Im

, ,

, ,,

, ,

31 ( )

f : R R

24 ( ) f, g
R :

f (x) f (y)
max{f  (x), f  (y)}
xy

g(f (x)) = x3

x, y R, x = y.

x R.

32 ( ) f

2

Seniors,

f (x + y ) = xf (x) + yf (y), x, y R
39 ( )  ABC , (AB =
AC ), AD, BE, CZ H
. M BC N
ZE BC , (C) AH
(C  ) M N .

) f (0)
) f .
) f (x+y) = f (x)+f (y) :
i) x, y
ii) x, y .
) f , ,
:

f (x) =

x+1

f (t)dt

40 ( ) (O)
P, . AB
(O), AB , Q.
A B  , A (O) P A B  (O) P B,
R, P Q.

f (t)dt, x R

, f (1) = 2012

Juniors, - -

33 ( )
f, g, h : R R

41 ( - 28 )
x, y, z x + y + z = 0. :

(x y)f (x) + h(x) xy + y 2 h(y)


(x y)g(x) + h(x) xy + y

y (y + 2)
z (z + 2)
x (x + 2)
+
+
0
2x2 + 1
2y 2 + 1
2z 2 + 1

x, y R.
42 ( )
p p3 + p2 + p + 1
.

34 ( )

n





1
1
1
1
1+ 2
1+
1 + 3 ... 1 + n < 3
2
2
2
2


Juniors,
43 ( ) f1 , . . . , fn
[0, 1]

35 ( )
 AB A.
B ( ) E , E = A.
B = E .


j=1

37 ( )
a1 , a2 , . . . , an

k=1

1
1
1
+
+ ... +
a1
a2
an


fj2 (x)

dx  n


j=1

1
0

2
fj (x) dx

a1 + a2 + ... + an = n
1
1
=
a2k + a2k+1 + 2
4

44 ( )
f : R R
F f . an = F (n) n ,
f (x) = 1 x R.

Seniors, - -

fi (x)fj (x) dx = 0

i = j .

36 ( ) 
ABC A BC L
N . L
AB, AC K, M .
AKN M ABC .

45 (

R.
k = xk = x x R.

( an+1 = a1 ).
38 ( ) ( Turkevici)

46 ( ) G
. T
G , T (x) = x1 x G
G .

x, y, z, t > 0, ,

x4 + y 4 + z 4 + t4 + 2xyzt
2 2

x y + y 2 z 2 + z 2 t2 + t2 x 2 + z 2 x 2 + y 2 t2

47 ( )

53 ( )

+

ln(1 + 1/k)
.
ln k
k=2

f (x) =

48 ( )

2x
,x 0
1 + e2x

, n . (
(n)
: f (0) Z, n 0)

1 + sin z
dz = ln(2)
n
z
2
2n
n=1

54 ( )

f : [0, +) R

lim f (x) = lim f  (x) = 0

x+

49 ( Mulder)
4 + 3 ( 4 1),
.

x+

lim f  (x) = lim f  (x) = 0

x+

x+

50 ( )

x2 = y 7 + 7

,
()
55 ( ) a, b, c C
:

51 ( ) a, b, c R
a + b + c = 0 :

a7 + b7 + c7
=
7

a2 + b2 + c2
2

a5 + b5 + c5
5

(a b)5 + (b c)5 + (c a)5 = 0


a, b, c
.

52 ( )

x2 + y 2 = r 2

56 ( ) (
M obius) a, b, c, d
w = az+b
w
cz+d
z
) ;
) ;

() A(x1 , y1 ), B(x2 , y2 ).

K = (1 x1 )(1 y1 ) + (1 x2 )(1 y2 )

1 ( )
.
.

. .
, ,
,
.
.

.
.
;

.
.

!

http://www.mathematica.gr/forum/posting.php?mode=edit&f=44&p=52498

1 ( )
.
,
.
,
, .
.
2 (nickthegreek)
...
:
( )
; ; , ,

.
,
( )

2 ( )
. ( ...).
, , ,
:
().




183, 186, 187, 190, 191, 192, 193, 194, 196, 200
; (
).
http://www.mathematica.gr/forum/viewtopic.php?f=44&t=9653

( ) 5

54
= 10
2
, .
.

(183 + 186 + 187 + 190 + 191 + 192 + 193 + 194 + 196 + 200)
= 478
4

<<<<
,

+ = 183

+ = 200
6

, ,

= 478 ( + + + ) = 478 183 200 = 95

= 196 = 196 95 = 101

+ = 186

= 200 = 99

= 186 95 = 91
91, 92, 95, 99, 101.

( )
. ,
...
.

= 183 = 183 91 = 92

+ = 196

4 ( )
,
:

3 ( qwerty)
,
.
2 . 100,
.

LI SARI BLO
GSP OT LI SA

http://www.mathematica.gr/forum/viewtopic.php?f=33&t=16302&p=84664#

( )
2,
3, 4 6.
3, 4, 6, 12.
( 100) 12 96.
98.

http://www.mathematica.gr/forum/viewtopic.php?f=33&t=

( )
,
, , 0 9.
0 .
,
0. 0

5 ( )
, ,
1 .
40 ,
;

6 ( ) S ,
2011 :

S = 1 2 + 3 + 4 5 6 + 7 + 8 ...

http://www.mathematica.gr/forum/viewtopic.php?f=34&t=16916

( )
4 ( )
+, -.
( 4
2008) S

S = 1 2 + 3 + 4 5 6 + 7 + 8 ...
+2007 + 2008 2009 2010 + 2011 =

http://www.mathematica.gr/forum/viewtopic.php?f=34&t=15002

1 ( ). xm
(x + 2) m
.
E = x2 m2

( )

= (2011 2010) + (2009 + 2008) + (2007 2006)+

EM = (x + 2)2 =
2

(2005 + 2004) + ... + (7 6) + (5 + 4) + 3 2 1


2

(x + 2) (x + 2) = x + 2x + 2x + 4 = x + 4x + 4 m


0,
0.
2011 4 2011 4 + 1 = 2008
, 2008 : 4 = 502 ,

E = 40
EM E = 40 :
x2 + 4x + 4 x2 = 40
4x = 36

x = 9m

S = 502 0 + 3 2 1 = 0

E = 92 = 81 m2

7 ( )

. . = 8
= 2 ,
.

AM 2 = AE2 + EM2 (2)


(1) (2) :

A2 + M2 = AE2 + EM2
x2 + 42 = 82 + (x 2)2
x2 + 16 = x2 4x + 68
4x = 52 x = 13
:

E=

AE EM
13 4 8 11
A M
+
=
+
= 70.
2
2
2
2

2(AB) 2E = 2 8 13 2 70 = 68.

http://www.mathematica.gr/forum/viewtopic.php?f=35&t=12064

2 ( ) 2
, ( x ) :

64 + (x 2)2 = x2 + 16
x = 13
:


2 (13 8) 2

8 11 4 13
+
2
2

8 (

= 68..





(x + 1) (x 1) x2 5 (x + 2) (x 2) x2 2 4

1 ( ) .
. , .
A = x EM = x 2 AE = 8
:

http://www.mathematica.gr/forum/viewtopic.php?f=35&t=11077

1 ( )





x2 1 x2 2 x2 4 x2 5 4 =




 2
x 3 + 2 x2 3 + 1 x2 3 1 x2 3 2 4 =


AM 2 = A2 + M2 (1)
:
9

x2 3 = :

(x4 6x2 + 5)(x4 6x2 + 8) 4


x4 6x2 =

( + 2) ( + 1) ( 1) ( 2) 4 =


 2
4 2 1 4 =
4 52 + 4 4 =
4 52 = 2 (2 5) =
(x2 3)2 ((x2 3)2 5) = (x2 3)2 (x4 6x2 + 4)

( + 5)( + 8) 4 = 2 + 13 + 36 =
( + 9)( + 4) = (x4 6x2 + 9)(x4 6x2 + 4) =
(x2 3)2 (x4 6x2 + 4) = ...

2 ( )

(x2 1)(x2 5)(x2 2)(x2 4) 4 =

9 ( )
:

x > 0.
http://www.mathematica.gr/forum/viewtopic.php?f=19&t=12903

x2
x3
x4
x1
+
+
+
=4
2+3+4 1+3+4 1+2+4 1+2+3

( alexandropoulos)

x4 6394x2 38400
http://www.mathematica.gr/forum/viewtopic.php?f=19&t=16597

1 ( )
. , .. x = 0
.
, x = 10 ,
.
12 ( )

( x2 = y 0)

x4 6394x2 38400 = (x2 + 6)(x2 6400)


,

x4 + 5x3 6394x2 32000x 38400 = 0

x2
x3
x4
x1
1+
1+
1+
1=0
9
8
7
6


1 1 1 1
+ + +
=0
(x 10)
9 8 7 6
x = 10
10 (
:

5x(x2 6400) + (x2 6400)(x2 + 6) = 0


(x2 6400)(x2 + 5x + 6) = 0
(x 80)(x + 80)(x + 2)(x + 3) = 0

themiskant)

x4 + 5x3 6394x2 32000x 38400 = 0

x = 80 x = 80 x = 2 x = 3
x = 80.

10

11 ( )
. ,
, xOy
Ox, Oy , ++
.

2. .
3. .
4. .
http://www.mathematica.gr/forum/viewtopic.php?f=20&p=73992#p73992

http://www.mathematica.gr/forum/viewtopic.php?f=20&p=#p72741

( )
, ,
, , Ox, Oy .

()

A + + B = A + + B
A 1 + 1 1 + 1 B = A B

( ).
1 , 1 A B

xOy .
12 ( )
,
.
:
1. .
11


1. AB
= A
(B A AB A
=
2. AB
o
=

B A = 90 B . AB

, AP B = + ,
= P A
+ AB
= +
= AP B ,
P AB
.
3. BK P A
,
, A P B

.

ZE (3), 3

4.

( , ) = (1), = (2) AP

,
.

13 ( )

f (x) = ln( ln x) +

Df = (0, 1) .
x1 , x2 (0, 1) x1 < x2 :

+1
ln x

5 =
11, 13 = 35
 
g (x) = 2x 1, x 0,

2
, f , ,
.
http://www.mathematica.gr/forum/viewtopic.php?f=21&t=16118

( ) , :
:


a5 = a1 + 4 = 11
a13 = a1 + 12 = 35

a1 = a = 1 , = 3

g(x) : x 1 2x

2 2x 1 1 g(x) 1 gmax = = 1
f : f (x) =
ln ( ln x) + ln1x + 1 :

x
>
0
x
>
0

ln x > 0

ln x < 0

ln x = 0
ln x = 0

x>0

x>0
ln x < ln 1

x<1
0<x<1

x
=

1

ln x = ln 1

x1 < x2 lnx1 < ln x2


1
1
>

ln x1
ln x2
1
1
+1>
+ 1 (1)
ln x1
ln x2
x1 < x2 ln x1 < ln x2

ln x1 > ln x2 ln ( ln x1 ) > ln ( ln x2 ) (2)


(1) (2)
x1 < x2 f (x1 ) > f (x2 ) f
x (0, 1).

1
+1=0
ln ( ln x) +
ln x
1
ln x = u ln(u) + + 1 = 0
u
u = 1.
f
:

u = 1 ln x = 1
1
e
f
ln x = ln e1 x = e1 x =

 


1
1
f (x) < 0 x
,1 .
e
e
 


1
1
1
f (x) > 0 x 0,
x < f (x) > f
.
e
e
e
1
x > f (x) < f
e

14 ( )
f

f (x) = ln (ex + 1)
12

1. f .

3) f

2. f
R.
3. f


3x = f (x).

2 x0 + 2 x0 =0

2 x0 + 2 x0 =1

1 = 0 ().

:x

: 3x = x 3 =

x
x =
3 x = 6
x

x = k + , k Z
6

4. f (2x) > f (x).


5. f (x) = f (x) x xR.

f (.o)

http://www.mathematica.gr/forum/viewtopic.php?f=21&t=15299

4) f (2x) > f (x)


2x > x x >
0
5) : f (x) = ln (ex + 1) = ln e1x + 1 =

ex >0,xR

( ) 1)
+1>0
x R Af = R, Af f
ex x
e 1


f (.o)f :11

3 x = f (x)

3 x = x : (1)
x0 R x0 = 0

x0 =0
3 x0 = x0 x0 =
0

ex

+1

2) x1 , x2 Af = R x1 < x2
< ex2 ex1 +
1 < ex2 + 1 f (x1 ) < f (x2 ) f
R.

ln

 ex +1  ln
ex

1
2

=ln 1 ln 2 ,1 ,2 (0,+)

ln (ex + 1)ln ex

f (x)=ln(ex +1),ln ex =x

f (x) = f (x) x

15 ( )
C AB (O, R)
Cx.
CA CB
(K) , (L)
(O, R).

K  
 (K), (O)

Cx L
(L)
 , (O)
 
 Cx
K L .
http://www.mathematica.gr/forum/viewtopic.php?f=22&p=83913#p83913

( )

K  S AB, AC = a, CB = b, OA =
OK  = OA r
a+b
a
r , KK  = + r ,
OK  =
2
2
OK = OA KA =

a+b
2

a
2

OK =

a+b
2

( :) )

b
2

OS = OC + r = OA a + r OS =

 OKK  (KK  )2 =
(OK)2 + (OK  )2 2 OK OS

ba
+r
2

r=

13

ab
2(a + b)

 OLL : ABCD .
1.
- ABC P,
ab
.

r =
2(a + b)
P A, P B, P C P,
BC, AC, AB ,
.
D, E, Z,
BC, AC, AB ,
16 ( ) P AP, BP, CP.
,
ABCD (O).
DB EC ZA
D DC, DA, DB DC EA ZB = 1 , (1) P DB, P DC
AB, BC, CA O1 , O2 , O3 . DB = (P DB) , (2)
DC
(P DC)
O1 , O2 , O3
(P EC)
(P ZA)
ZA
=
,
(3)

EC
EA
ZB = (P ZB) , (4)
(P EA)
O (O).
(P DB) (P EC) (P ZA)
EC ZA
(2), (3), (4), = DB
DC EA ZB = (P DC) (P EA) (P ZB) , (5)
AP P D BP P E = BP D = AP E
DB)
(P D)(P B)
= (P
(P EA) = (P E)(P A) , (6) ( ,
). BP P E CP P Z
EC)
(P E)(P C)
= BP Z = CP E = (P
(P ZB) = (P Z)(P B) , (7)
CP P Z AP P D = AP Z + CP D = 180o
(P ZA)
(P Z)(P A)
= (P
DC) = (P D)(P C) , (8) ( ,

).
EC ZA
(5), (6), (7), (8) = DB
DC EA ZB = 1
1 .
2. - ABC
(O) P BC
A.
P B, P C P,
AC, AB, E, Z, .
EZ O ABC .
http://www.mathematica.gr/forum/viewtopic.php?f=22&p=83444#p83444
H, K, E, Z P C, P B
1 ( ) - M, N, P B, P C,
.
:
EP P K ZP P C = CP E = ZP K
1. ABC P,

, HP E, KP Z,
.
PH
EH
P A, P B, P C P, P K = ZK , (1)
ABP C, = ACP =
BC, AC, AB , .
. ABK ,
. ( , HCE, KBZ,
CH
EH
1973 ), 1019, 87
ZK = BK , (2)
PH
CH
PH
PK
(1), (2) = P
4 .
K = BK = CH = BK =

PH
PK
2. ABC P H  = BK (3) P H = CH

(O) P BC
M
(3) = P HP H = P KBK
= 2HN
= 2P
BK
BK
PH
P H
A.
M
= PHN
= PBK
, (4)
H
P B, P C P,
HN
PM
HN
PM
(4) =
 = P M +BK = N P = M K , (5)
AC, AB, E, Z, .

EZ O ABC .
,
1, O1 , O2 , O3 ,
, ().
D, ABC,
(O) ()
O1 , O2 , 2 ,
O

HN +P H

(5), H, N, P P, M, K,

, E, O, Z,
,
2 .
, ,

14

,
.
2 ( )
1. ABCD
(O).
E F
AB CD . EB F C
K , EA F D L.
KL O . S
AE F C T EB F D .
= ( ),
= . = ( ABCD ),
= EST F .
ST BC AD LO

= = K CB
EB K K  CK 
AE S  (). . Desargues ADL CK  B
S  T BC AD , S  S
K  K .
2. () ABCD (O). E F
AD CD . EB
F C K , EA F D L.
KL O . S
BE F D T EA

1 2 3 4
F C .
= = = = , 1 : ,
2 : , 3 : ABCD 4 :
, SEF T : ,
ST AD BC .
S TE = S F E = E AD

KO EA L L
DL BE S  ().
. ADL CKB
S  T BC AD , S 
S L L .
.
E , F H A , B C , ACEH
ABEF .
O1 , O O2 .
O1 , O O3
. O1 , O , O2 O3 .
: . Desargues

,
,
( )

.

17 ( . )
:


x +

y 2 = 0 (1)

1. R,
(1) .
.

: = [2, 2].
2) , ,
K (, ) > 0 , :






 + 4 2 2

d(K, ) =
=
2 + 4 2





 + 4 2 2 = 2 (2)

2. ,
,
.

http://www.mathematica.gr/forum/viewtopic.php?f=23&t=5498

( ) 1) R
(1) , :

, ,
(2) .
) (2) . (2)
= 2, = 2 = 0 :

0
4
 2 4 2 2


= 0 4 2 0
15

|2 2| = 2
|2 2| = 2

|2 2| = 2


| 1| =
| + 1| =

| 1| =

1 ( ) ,
, :

| + 1| = | 1|
= | + 1|

| 1| =

=0

=1

( = 2 = 0)


1
1
(AB + )
N M = (KB + ) =
2
2

1

P N = (AK + ) = (AB + )
2
2

) .
= 0, = 2 = 1. ,
, (2) .
= = 0 = 1. .
, (2) . .
, ,
(0,0) =1. :
x2 + y 2 = 1.
18 ( )
AB K, AB
,

, ,
2 ( ) M, O N
B, K A, .

AK = AB
=

(1 )AK = BK

0 < < 1, :

OK = (1 ) OA + OB

0 < < 1. B, K
A .

(1 ) =

AK = AB
=


N M //P N

http://www.mathematica.gr/forum/viewtopic.php?f=23&t=4213

O = (1 ) O + O

OK = O

(1 ) OA OB = (1 ) O + O


(OB + O) = (1 )(OA + O)

2 ON = 2(1 ) OM

1
OM
ON =

B, K A
.

19 ( )

= {1, 2, 3, 4, }

P (1) =
16

P (3)
P (4)
P (2)
=
=
2
3
4

k . k = 3, 4 A = {3, 4}
B x2 x + 1 > 0 x R
= 3 < 0 ln(x2 x + 1) > 0

A = { |

ln(x2 x+1) > ln1 x2 x+1 > 1 x(x1) > 0


x x > 1 x = 2, 3, 4
B = {2, 3, 4}

3. A B = P (A B) = 0 A B =



{1, 2} {2, 3, 4} A B = P (A B) =
1

, 2, 4, 5 10}
B = {x ln(x2 x + 1) > 0}
1.
.
2. A = {3, 4} B = {2, 3, 4}

4. P (X)

X A X =
B .
A = {3, 4} B = {2, 3, 4}
2
X = {2} P (X) = P (2) P (X) = 10
= 15

3. P (A B) P (A B)
4. X ,
P (X) A X =
http://www.mathematica.gr/forum/viewtopic.php?f=18&t=14672

20 ( )

( )
P (2)
2

1. P (1) =

P (3)
3

P (4)
4

= {1, 2, 3, ..., 2010, 2011}

= :

P (1) =
P (2) = 2
P (3) = 3
P (4) = 4

P (1)+P (2)+P (3)+P (4) = 1 10 = 1 =


:

1
10


.
. f (x) = x2 + (1 x)2
x R .
. A N (A)
(P (A))2 +
(P (A ))2 .
. f (x) 1 x [0, 1].
. B,

(P (B))2 + (P ())2 + (P (B  ))2 + (P ( ))2 = 2


B = B = .

1
10
2
P (2) =
10
3
P (3) =
10
4
P (4) =
10
P (1) =

http://www.mathematica.gr/forum/viewtopic.php?f=18&t=15407

( ) ) :

f  (x) = 2x 2(1 x) = 2(2x 1)


x R. f  ( 12 ) = 0. f  (x) < 0

1
2

x (, ) f

2. :

1
s =
v
2

 v

i=1

1
s =
4
2

x2i



( vi=1 xi )2

144k2
46k
4
2

x (, ]. f  (x) > 0 x ( 12 , +)
2
f x [ 12 , +).
f x = 12 f ( 12 ) = 12 .
)

(P (A))2 + (P (A ))2 = f (P (A))

5
s = k2
2

P (A)

0,

5
s2 > 10 k2 > 10
2
k2 > 4 k > 2

2
2010
1
,
, ...,
,1
2011 2011
2011

(P (A))2 + (P (A ))2

17

f (0), f (

2
2010
1
), f (
), ..., f (
), f (1)
2011
2011
2011

2
1005
1
,
, ...,
2011 2011
2011
1
x (, ]
2

. : 2x(x 1) 0 x [0, 1].


x 1 0 x 0
x [0, 1]. x = 0
x = 1.
)

0,

f (P (B)) + f (P ()) =
(P (B))2 + (P (B  ))2 + (P ())2 + (P ( ))2 = 2
P (B) P () [0, 1]
()

2
1005
1
,
, ...,
0,
2011 2011
2011

f (P (B)) 1(1)

1005
2011 .

2010
1006 1007
,
, ...,
,1
2011 2011
2011

f (P ()) 1(2)

[ 12 , +),

x

.

f(

f (P (B)) + f (P ()) 2(3)

1007
2010
1006
), f (
), ..., f (
), f (1)
2011
2011
2011

1005
1006
f ( 1006
2011 ). f ( 2011 ) = f ( 2011 ).
(P (A))2 + (P (A ))2
N (A) = 1005 N (A) = 1006.
) :

f (x) = 2x2 + 2x + 1 = 2x(x 1) + 1

(3)
(1) (2) . P (B) = 0, 1,
P () = 0, 1. N (B) = 0 N (B) = 2011 N () = 0
N () = 2011. B,
 .
B = B =

21 ( barsakis)
z ,
:

yi)i = y + xi (y, x). :


 = (x, y 1), A
 = (y, x 1)
AB
, ,


AB//A

1. i, z, zi
2. 1, z + i, iz + 1


 x y1

 y x 1

3. 1, z, z 2 + 1

x2 + y 2 x y = 0 (I)

http://www.mathematica.gr/forum/viewtopic.php?f=51&t=14329

( ) z = x + yi, x, y R.
1. : i
A(0, 1), z
B(x, y), zi = (x +
18



=0


() : (1)2 + (1)2 4 0 = 2 > 0,



1 1
,
2 2

2
.
2

2. 2. : 1
w w = 1z .
D(1, 0), z+i
E(x, y + 1), zi + 1 =
y + xi + 1 Z(y + 1, x). :
http://www.mathematica.gr/forum/viewtopic.php?f=51&t=15367
 = (x 1, y + 1), DZ
 = (y, x)
DE
( ) M (2, 2),
, ,
M1 (3, 1), M (x, y).


DE//DZ


 x1 y+1

 y
x

1. OM M M1
 = M1 M x3 = 2, y +1 = ,
: OM
x 3 = y + 1 y = x 4
y = x 4.



=0


x2 + y 2 x + y = 0 (II)
() : (1)2 + 12 4 0 = 2 > 0,

1 1
,
2 2

2
.
2

3. :
1 D(1, 0),
z B(x, y),
z 2 + 1 = x2 y 2 +
1 + 2xyi H(x2 y 2 + 1, 2xy).
 = (x 1, y), DH
 = (x2 y 2 , 2xy)
: DB
, ,







DB//DH

x1
y 
=0
x2 y 2 2xy 

2. w = x+yi, x, y R z = a+(a4)i, a R.
:

z
1
a (a 4)i
= 2 x + yi = 2

z
|z|
a + (a 4)2
4a
a
,y = 2
(I)
x= 2
2
a + (a 4)
a + (a 4)2
w=

a = 0, () :
4a
y
=
ay = 4x ax a(x + y) = 4x(II)
x
a
x + y = 0, ()

=0, =0,

1
= 0.
z
x + y = 0, () :
4x
a = x+y
, ():
w=


x

y(x2 + y 2 2x) = 0
y = 0 x2 + y 2 2x = 0(III)



(2)2 +02 40

() :
= 4 > 0,

()

(1,
0)

4
= 1.
2

4x
x+y

x x (1, 0)
1.

22 ( g.liolios ) M
M1 z = 2 + 2i R
z1 = 3 i M OM M M1 ( ).
1.
M z =
x + yi.

19

2
+

4x
x+y
x

2

2 
4x
4x
4

=
x+y
x+y


4y
x+y

2 
=

4x

x+y

4x
16x2 + 16y 2

=
(x + y)2
x+y

16x(x2 + y 2 ) 4x(x + y) = 0
4x(4x2 + 4y 2 1) = 0 x2 + y 2 =

1
4

x = 0 . w

1
.
2
1
a = 0, () x = 0, y = ,
4
O(0, 0)

23 ( )
f : R R

zi zi
+
+ 1 = 16
z
z


z z

i = 14

z z
z
=7
Im
z

f (x y) = f (x) f (y) , x, y R
1. f
.
2. f .

24 ( )
3. f (x) = 0
f, g R :
x = 0.

g(f (x)) = x3

() f .
() z |z| = 1,

x R.
1. f .


i 
+ f (3) = f (7)
z
z



f z +

2. g R.
3. f (g(x)) = x2
x R.

Im z .

http://www.mathematica.gr/forum/viewtopic.php?f=52&t=3754

http://www.mathematica.gr/forum/viewtopic.php?f=52&t=3840

( )

( )

1. x = y = 0 f (0) = 0
2. f
x = 0 f (y) = f (y) ,

3.

() x1 , x2 f (x1 ) = f ( x2 ).
f (x1 ) f ( x2 ) = 0
f (x1 x2 ) = 0.
f (x) = 0 0 ,
x1 x2 = 0 x1 = x2 .



() f z + zi  = f (7) f (3)
f z + zi  = f (7 3)
f z + zi  = f (4) 

f 1 1 z + zi  = 4



i
i
= 16
z
z+
z
z
zz

1. x1 , x2 R, f (x1 ) = f (x2 ) g(f (x1 )) =


g(f (x2 )) x1 3 = x2 3 x1 = x2 , f
1-1.
2.

g(x) = y f (g(x)) = f (y)


x3 = f (y)



3
f (y), f (y) 0
x=
x = 3 f (y), f (y) < 0
, g(R) = R.
3. f (g(x)) = x2 f (g(f (x))) = f 2 (x)

f (x3 ) = f 2 (x)
x = 0, f (0) = 0 f (0) = 1
x = 1, f (1) = 0 f (1) = 1
x = 1, f (1) = 0 f (1) = 1
f (0) = 0,f (1) = 1, f (1) = f (0) f (1) =
f (1)
f (0) = 1, .

1
zi zi
+
+
= 16
z
z
zz
20

25 (

5

5(x 1)4 + (9x2 18x + 11) > 0 (2)

x
:

x 5x + 13x 19x + 16x 8 = 0

5 (x 1)4 0, x R

.
http://www.mathematica.gr/forum/viewtopic.php?f=53&t=14780

1 ( )

9x2 18x + 11 > 0, x R

f (x) = x5 5x4 + 13x3 19x2 + 16x 8


f f

f  (x) = 5x4 20x3 + 39x2 38x + 16


f 

f  (x) = 20x3 60x2 +78x38 = (x1)(20x2 40x+38)

= 72 < 0
(1) (2) . x1
.
3 ( ) :

(x 1)5 + 3(x 1)3 + 2x 4

( < 0

20x2 40x + 38 > 0


) f  (1) = 0 x > 1
f  (1) > 0 x < 1 f  (1) < 0. f 
1 xR

f  (x) f  (1) = 2 > 0

f R
f (x) = 0 R
2 ( )

f (x) = x5 5x4 + 13x3 19x2 + 16x 8

,
.
.
:

5(x 1)4 + 9(x 1)2 + 2 2 > 0


, .
26 ( )
f : [, +) R
[, +) lim f (x) =  R.
x+

f .

, http://www.mathematica.gr/forum/viewtopic.php?f=53&t=14506
x1 . ( ), ( )
x2 ... :
x0 : f  (x0 ) 0
1. f  (x0 ) > 0 f (x) > f (x0 ) + f  (x0 )(x
f (x1 ) f (x2 ) = f  ()(x1 x2 ) f  () = 0 (1)
x0 ), x > x0 (Cf ). f (x) +
:
2. f  (x0 ) = 0 x1 > x0 : f  (x1 ) > 0 f 

4
3
2
f (x) = 5x 20x + 39x 38x + 16 =
.
21

27 ( )
f : [0, 1] R
x, y xf (y) + yf (x) 1.

1
0

|a (2x) 2 x| dx, a > 0.

.
4

f (x) dx

i)

28 ( )
I(a) :=

http://www.mathematica.gr/forum/viewtopic.php?f=54&t=6836

( ) I(a)
ii) 
/2
.
|a (2x) 2 x| dx,

x=/2u

http://www.mathematica.gr/forum/viewtopic.php?f=54&t=7522

( )

1

/2

2I(a) =
0

x=y

f (x) dx =

I :=

yf (y) dy

1
I(a)
2

y=/2t

1
2

/4

/2
tf (t) dt

1
2

/2

2I =

/2

dt =
0

()

/2

|a (2x) 2 x|+|a (2x)2 x| dx

()

tf (t) + tf (t) dt

/2
0

()

| 2 x 2 x| =

1
x x dx
2
2

/2

(2x) dx

1
2

/2

/4

2 x 2 x dx =

/2

(2x) =
/4

1
.
2

()
() x [0, /2]

I .
2
4

2 x 2 x 0 x x x [0, /4]

() x = t

y = t.




2 x 2 x 0 x x x [/4, /2].

x2

1
[0, 1].

, I a = 1/2 :
O(0, 0) 1, I = 4 .
 


f ,

 

1 /2  (2x)
2   (2x)
2 
x, y [0, 1]

I(a) =
x
+
x
 
 dx =

f (x) =



(x2 1 y 2 )2 0

( y 2 1 x2 )2 0
 2


x +1y 2
2 1 y 2 = x 1 y 2 (+)

x
2


y 2 +1x2
2 1 x2 = y 1 x2

y
2

1
2
1
2

xf (y) + yf (x) 1

22


0

/2

0
/4

x|x x| + x|x x| dx =

1
xxx dx+
2
2

/2
/4

2 xxx dx =

/4
1
1 + (2x) (2x) dx+
4

(2x) = 2xx

1
4

2 x =

/2
1 (2x) (2x) dx =
/4

1
2

1 (2x)
2

1 + (2x)
2

2 x =

1/2

:
x(0,)

29 ( )
f :

f (x) =

x < 0
 
,
x
2

1
, x (0, )
sin x

) f .
)
f.
) f .
)
Cf x x x = 3 , x = 2 .

x
f  (x)
f (x)

/2

0
-

f 0,


2

,
2 
f

f  (x) =

http://www.mathematica.gr/forum/viewtopic.php?f=55&t=14744

( ) ) f
f  (x) =

2 x + 2 2 x
=
3 x
1 + 2 x
> 0, x (0, )
3 x

x
, x (0, )
2 x
:

f  (x) = 0
x
=0
2 x
x = 0
x=

f .
) f (0, )
.

x(0,)

lim f (x) = lim

:
f  (x) > 0
x
2 x>0
>
0

2 x

3 x 2x x (x)
=
4 x

x0+

x0+

1
= +
x

1
= +
x
x = 0 x =
Cf .
lim f (x) = lim


f ((0, )) = f

23

0,

 

f
,
=
2
2

   

  

, lim f (x) f
, lim f (x) =
f
2 x0+
2 x
[1, +) (1, +) = [1, +)

) ) h

h (x) =
)


2 
2
 1  x>0
1
 dx =
E = 
dx =
x 
x

2 x2

+ 2 x2
2 x2 x2


f h

3

2
dx =

x2

2 x2

2
dx +

x2
2 x2

h (x) =
dx =

xf  (x) f (x)
, x>0
x2

() h (x) < 0, x > 0 h


(0, +)
) g(x) = h(x) 2x + 4, x > 0



x  2
ln3
x
ln 2
=
ln 2

2
2 3
2

30 ( )
f : R R R f (0) = 0
f (2) = 4.
) x > 0 : x f  (x) < f (x).

x

f (x)
, x>0
x

f (t)

) h(x) = 2 t dt x > 0 ,
i) h(x) (0, +)
ii) h(x) 2x 4 x > 0.

g (x) = h (x) 2 =

f (x)
2
x

g (2) =

f (2)
2=22=0
2

g  (x) = h (x) < 0 g 


(0, +) x > 2 g  (x) < g  (2) = 0
( ) )
g [2, +) 0 < x < 2
[0, x], x > 0 f g (x) > g (2) = 0 g (0, 2]
(0, x)
x0 = 2 g
f
(x)
f
(x)

f
(0)
g(2) = h(2)4+4 = 0 g(x) g(2) = 0

http://www.mathematica.gr/forum/viewtopic.php?f=55&t=15210

f () =

x
h(x) 2x + 4 0, x > 0

< x f R f 
f  () > f  (x)

f (x)
> f  (x) f (x) > xf  (x), x > 0
x

31 ( )

f : R R

http://www.mathematica.gr/forum/viewtopic.php?f=56&p=82858

1 ( )

g(x) =

f (x) f (y)
max{f  (x), f  (y)}
xy

f (x)f ()
x
f  ()

, x =
,x=

( R)

x, y R, x = y.

lim g(x) = lim

24

f (x) f ()
= f  () = g ()
x

g g
R {}

g (x) =

f  (x)(x ) (f (x) f ())


=
(x )2

i) x, y
ii) x, y .
) f , ,
:

1
f (x) f ()
(f  (x)
)
x
x

x+1

f (x) =
x


f (t)dt

1
0

f (t)dt, x R

, f (1) = 2012



f (x) f ()
max f  (x), f  () f  (x), x =
x
:

f  (x)

http://www.mathematica.gr/forum/viewtopic.php?f=56&p=82979

( ) )
x = y = 0, f (0) = 0.
) y = x y = x,

f (x) f ()
0, x =
x



2xf (x) = f 2x2 = xf (x) xf (x)


1) g (x) 0 , x >
2) g (x) 0 , x <

xf (x) = xf (x)

g g
. x R
:

x R
x = 0

f (x) = f (x)

g(x) g()

x = 0, f
.
) (i) x, y > 0. ,

f (x) f ()
f  (), x =
x

f (x + y) =

 

2
x + ( y)2 =
f
 

xf
x + yf ( y) =



f (x) f ()
max f  (x), f  () f  (), x =
x
x = :

f (x) + f (y)

f (x) f ()
= f  ()
x

f (x) =

 
2
x + 02 =
f
 
xf
x

f (x) = f ()(x ) + f () : (1)


(1) x = :

f (x) = f  ()(x ) + f (a), x R

x, y < 0. ,

: f (x) = x + , x R , R
. ( )

f (x + y) =
f ((x) + (y)) =

32 ( )
f

[f (x) + f (y)] =

f (x2 + y 2 ) = xf (x) + yf (y), x, y R

f (x) f (y) =
f (x) + f (y)

) f (0)
) f .
) f (x+y) = f (x)+f (y)
:

) (ii) x > 0
y < 0.
f (a b) = f (a) f (b) a, b > 0 ,

25

G (x + y) f (x) G (y) = f (x + y) f (x) f (y) = 0

a b > 0. :
x + y > 0 ,
f (x + y) = f (x (y)) =

y R. , g R,

f (x) f (y) =

g (0) = G (x) G (x) G (0) + G (0) = 0

f (x) + f (y)

, , x, y R,

x + y < 0 ,

G (x + y) G (x) yf (x) G (y) + G (0) = 0

f (x + y) =

y = 0,

x R,

f ( (x + y)) =
f ((y) x) =

f (x) =

[f (y) f (x)] =
f (y) + f (x) =

1
[G (x + y) G (x) G (y) + G (0)] (1)
y

 f
R. F (x) =

f (x) + f (y)

f (t)dt,

G (x) = F (x) + c, c R . x = 0,
c = G (0) , y = 1
(1)

x + y = 0
f (x + y) = f (0) = 0

f (x) = G (x + 1) G (x) G (1) + G (0) =

= f (x) f (x) =
f (x) + f (x) =

F (x + 1)+G (0)F (x)G (0)F (1)G (0)+G (0) =


 1
 x+1
f (t)dt
f (t)dt (2)
= F (x + 1)F (x)F (1) =

f (x) + f (y)

) G f . x R
(2) ,
, y R,
x R, f  (x) = f (x + 1) f (x) = f (1) = 2012
f (x) = 2012x + k, k R . , ,
g (y) :=
f (0) = 0, k = 0,

G (x + y) G (x) yf (x) G (y) + G (0)


f (x) = 2012x

g ( y ) R,

g (y) =

x R.

26

,
,

33 ( )
f, g, h : R R

f (x) = f (0) x, x R

(x y)f (x) + h(x) xy + y 2 h(y)

(x y)g(x) + h(x) xy + y 2

y x + 1 x 1

x, y R.

g(x) = f (0) x, x R

http://www.mathematica.gr/forum/viewtopic.php?f=109&t=16177

( ) x 0
:

yf (0)+h(0)+y 2 h(y) yg(0)+h(0)+y 2 , y R (1) 34 ( )


n




 

1
1
1
1
1+ 2
1+
1 + 3 ... 1 + n < 3
yf (0) + h(0) + y 2 yg(0) + h(0) + y 2 , y R
2
2
2
2

http://www.mathematica.gr/forum/viewtopic.php?f=109&t=16099

yf (0) yg(0), y R

1 ( ) ln, :

y 1

n




ln 1 + 2i < ln 3

i=1

f (0) = g(0) (2)

:
(1) (2)

n


h(y) h(0) yf (0) + y 2 , y R

i=1


n


1  i
ln 1 + 2i  n ln 1 +
2
n
i=1

h(y) h(0) yf (0) + y 2 , y R

1  i
2
1+
n

<3

i=1

h(x) = h(0) xf (0) + x2 , x R (3)


(3)
y
x + 1 x 1

n


2i = 1 2n

i=1

f (x) f (0) x, x R

!

1
n
12
n ln 1 +
< ln 3
n


!

1
1 2n  1 2n < ln e < ln 3
n ln 1 +
n

f (x) f (0) x, x R
27


2 ( )
:


1+

1
2


1+

1
22


1+

1
23


... 1 +

1
2n


<3

<

n+

1+

1
22

1+

<e

n


1
ln 1 + k
2

ln

1
2

n

ln(1 + x) < x

( ).
3 ( )
! - :

1+

n+1
n

4 ( ) x > 0

k=1



<

2n+1 (3n 1)(n + 1)



1
i=1 2i

1
n

.
n = 1, 2, 3, 4, 5 .
n 6

n

1
23


... 1 +

1
2n

n 
"

1+

k=1

n 
"
k=1

1
1+ k
2

<

n

1
1
=1 n
k
2
2
k=1

1
2k




1
< ln e1 2n

< e1 2n < e < 3

35 ( )
 AB
A. B ( )
E , E = A.
B = E .

E 2 = (ax)2 +x2 2x(ax) cos 120o = a2 +x2 ax , (2)

(1), (2) = B = E .
2 ( ) K
B K = .
 K .  AK
http://www.mathematica.gr/forum/viewtopic.php?f=110&t=5101
 E ( E = AK = 120o
A = E K = ) AK = E , (1)
 AK  BK ( AK =
BK = 120o K BK = B K =
A = A ) AK = B , (2) (1)
(2) .
3 ( ) Z
AB, Z B.  BZ
 E ( Z = A = E
ZB = E ZB = E )
B = E .
4 ( ) K

B
1 ( ) A = E = x
AB = B = A = a = a x.
K = 30o =
 B
 E :

B 2 = (ax)2 +a2 2a(ax) cos 60o = a2 +x2 ax , (1)


28

K =

(AKN M ) = 2(AKN ) =
(AK)(N T ) , (2) T N
AB .
 ABC (ABC) = (ABL) +

BK = A K = A

A A
A A
=
+
, (1)
2
2
2

1
(ACL) = (LK)(AB + AC) , (3)
2
LK
AL
=
, (4)
KL T N =
AN
NT

KE = K + E =

A A
A A
+ A =
+
, (2)
2
2
2

(1), (2) = BK = KE
 BE .
36 ( )
 ABC A
BC L
N . L AB, AC
K, M .
AKN M ABC .

(1),

(2),

(3),

(4)

(AKN M )
(ABC)

AL N T
2(AK)(N T )
=

= 1 = (AKN M ) =
(LK)(AB + AC)
AN LK
(ABC) 2 ( )
AC CD = AB 
ABN,  DCN , AB = CD BN = N C
= N CD

ABN
(
ABN C ).

http://www.mathematica.gr/forum/viewtopic.php?f=110&t=12810

1 ( ) AC
CP = AB AK KS = AK =
AM .

A
 AN C
2
A
2

 KLM

A
( AKLM ).
2
 ABN  P CN (
LM
AN
,
=
= (AN )(KM ) = (LM )(AB +
AB = P C BN = CN ABN = P CN )
AD
KM
 AM L  SKL AC) = (AN )(ZM + ZK) = (AB)(LK) + (AC)(LM )
1
1
1
( AM = AK = SK M L = KL ).
= (AN )(ZM ) + (AN )(ZK) = (AB)(LK) +
2
2
2
1
A
(AC)(LM ) = (AKN M ) = (ABC)
CP N = KSL =
2
2
3 ( )
,  AN P
AL
2(AK)
AL
AS
bc
=
=
=
, (1)
 ALS
AN =
(1)
AP
AN
AB + AC
AN
w
a
 AKN,  AM N ,
AN AK = AM KAN =

A
M AN =
 ABL,  AN C , wa AL.
2

29

AKN M
AL AK = AM

1
(AKN M ) = (AN )(KM ) (2)
2
AKLM
AL

KM = wa sin A (3)
(1), (2), (3) = (AKN M ) =

(ABC)

, ,

37 ( )
a1 , a2 , . . . , an

n


a1 + a2 + ... + an = n

n

k=1

1
1
=
4
a2k + a2k+1 + 2

1
1
1
+
+ ... +
a1 a2
an

1
bc
wa sin A
=
2
wa

j=1


1
1

.
2
2
2
aj + aj+1 + 2 j=1 2aj + 2

1
1
(ai 1)2  0
 2
4ai
2ai + 2
i {1, 2, ..., n} .
n
1 1
.
: 
4
aj

( an+1 = a1 ).

j=1

http://www.mathematica.gr/forum/viewtopic.php?f=111&t=12332

1 ( )
Andreescu ( CS) :

a1 = a2 = = an .
(1) :

1
1
1
+
 2
,
4a21 + 4 4a22 + 4
a1 + a22 + 2
1
1
1
+ 2
 2
,
2
4a2 + 4 4a3 + 4
a2 + a23 + 2
..
.

1
1
1
+ 2
.
 2
+ 4 4an + 4
a1 + a2n + 2

(a1 , a2 , ..., an ) = (1, 1, ..., 1)


1 ( )
-
i = 1, 2, . . . , n, an+1 = a1 ,

a2i + a2i+1 + 2 2 (ai + ai+1 )

4a21

30

1
ai

8
1
+ ai+1

ai = ai+1 .
,

a2i

1
1

2
8
+ ai+1 + 2

1
1
+
ai ai + 1

x y z t > 0.
:

 2
2 
2 
2 
2
x z 2 + y 2 z 2 + x2 t 2 + y 2 t 2
2 (xy zt)2 (1)

i = 1, 2, . . . , n
1
1
1
+ + 2

2
2
2
4
a1 + a2 + 2
an + a1 + 2

a2 + b2


1
1
+ +
,
a1
an

(a + b)2

a := x2 t2 b := y 2 t2 , :

 2

2 2t2 2
 2



x
+
y
2
2
x t2 + y 2 t2
.
2

a1 = a2 = = an .

()
x + y 2 2xy xy t2 xy zt 0, (2)

(a1 , a2 , . . . , an ) = (1, 1, . . . , 1).

38 ( ) (
Turkevici) x, y, z, t > 0, ,

x4 + y 4 + z 4 + t4 + 2xyzt

2

 2
 2


2xy 2t2
2 2
2 2
+ y t

x t
2
2

= 2 xy t2
2(xy zt)2 ,


x y +y z +z t +t x +z x +y t
(1), .
: (1)
.
http://www.mathematica.gr/forum/viewtopic.php?f=111&t=15643
(V
o Quoc Ba Can, (2)
) , .
2 2

2 2

2 2

2 2

2 2

2 2

31

39 ( )
 ABC , (AB = AC ), AD, BE, CZ
H . M
BC N ZE
BC , (C) AH
(C  ) M N .
http://www.mathematica.gr/forum/viewtopic.php?f=112&t=15908

1 ( )
X AD
(C  ) M N O AH.
BCEZ, AEHZ

MB = MC = ME = MZ =
AH
OE = OZ =
2

BC
OA = OH =
2

(C  ), (M ) (C)
AH ( O
), (M ) ( OE EM ),
(C  ).
2 ( ) Z, E, C, B,
(C1 ) M, M Z = M E = M C =
M B AB = AC , (C  ) AM ,
K .

N K AM , (1)
A (C1 )
N H

N H AM , (2)
(1) (2) N, H, K

HKA = 90o , K
(C).
F AH P M N .

P KF = 90o K1 + K2 = 90o A1 + M2 = 90o ,


.

OEM = OEH + HEM =


OHE + HBD = 90o = OE EM
OZ ZM  XM N
(M X)2 = (M D)(M N ) , (1)
AEZHBC N, B, D, C, Newton, (M B)2 = (M C)2 = (M D)(M N ) , (2)
(1), (2) = M X = M B = M C ,
X AD (C  ), (M ) BC. 40 ( )
, AD (O) P, .
32

AB (O), AB
, Q.
A B  , A (O) P A B  (O) P B,
R, P Q.
http://www.mathematica.gr/forum/viewtopic.php?f=112&t=16476

( )
P u2 = (P O)2 r 2 ( =
P (O) ),
(O) .

R A B 
C  D  P R ,
(K), (L)  P AB,  P CD,
A B  , C  D  ,
.
P, R, R ,
(K), (L)
Q
, (QA)(QB) = (QC)(QD)
ACBD.
R (K) (L)
P Q,

(QR )(QP ) = (QA)(QB) , (1)


Q (K).
, ,

(P R)(P R ) = (P A )(P A)
R
P Q .

41 ( -
28 ) x, y, z x + y + z =
0. :

2x y + z

z (z + 2) 1
+
2z 2 + 1
2


1=

()
(1 x)2

y2 + z2 + 1

( )
: x2 y 2 z 2 :
2

[(2y + 1) + (2z + 1)]2


1=
4 (y 2 + z 2 ) + 4

http://www.mathematica.gr/forum/viewtopic.php?p=80514

(2z + 1)2
(2y + 1)2
+
1
2(2y 2 + 1) 2(2z 2 + 1)

x (x + 2) y (y + 2) z (z + 2)
+
+
0
2x2 + 1
2y 2 + 1
2z 2 + 1

y (y + 2) 1
+
2y 2 + 1
2

x (x + 2)
(1 x)2
1= 2
2
2x + 1
2x + 1

()

C S :

y (y + 2) z (z + 2)
+
=
2y 2 + 1
2z 2 + 1

x (x + 2) y (y + 2) z (z + 2)
+
+
0
2x2 + 1
2y 2 + 1
2z 2 + 1
33

m = 2k (1) :

p + 1 p2 + 1

= k2 (4)
2
2

(x, y, z) = (0, 0, 0) ,
 
 


1
1
1 1
1 1
, , 1,
, , ,1
1, ,
2 2
2
2
2 2

(3) (4)

q, rN p + 1 = 2q 2 (5)
42 ( )
p p3 + p2 + p + 1 .
http://www.mathematica.gr/forum/viewtopic.php?p=68902

( ) m N :

p2 + 1 = 2r 2 (6)
(6) (5)
:p(p 1) = 2(r q)(r + q).
rq < p :

p|r + q r + q p


p+1
p2 + 1

+
p
2
2


( p + 1 + p2 + 1)2 2p2

2 (p + 1)(p2 + 1) p2 p 2

p 3 + p 2 + p + 1 = m2
(p2 + 1)(p + 1) = m2 (1)
(p2 + 1, p + 1) = d.

d|(p + 1)2 (p2 + 1) d|2p


(p, p + 1) = 1 (p, d) = 1

4(p + 1)(p2 + 1) p4 + p2 + 4 4p2 2p3 + 4p


6p3 + 7p2 p4

d|2 (2)

6p + 7 p2

p = 2, . p
2|p + 1, 2|p2 + 1 (2)

p + 1 p2 + 1
,
2
2

p7
p = 7,
(73 + 72 + 7 + 1 = 202 ).

= 1 (3)

43 ( )
f1 , . . . , fn [0, 1]

1
0

http://www.mathematica.gr/forum/viewtopic.php?f=59&t=15825

(
-
)
1 2
0 fj (x) dx = 1
j = 1, . . . , n. :

fi (x)fj (x) dx = 0

i = j .
n 
"
j=1 0

fj2 (x) dx  nn

n 
"
j=1 0

n
"

j=1

fj (x) dx

1

1/n
|Ij |

1
 ,
n

Ij = 0 fj (x) dx. n
,
j=1 |Ij | 

n.
34


|Ij | =

n(m1)r

1
0

Ij > 0,
Ij = 0,
Ij < 0.

n


j=1

n(m1)r

g(s + mr) ds =
0

g(s) ds,

Cauchy-Swartz

,

2

n


|Ij | =

g(s + (m 1)r) ds +

1
j = sgn(Ij ) = 0




j fj (x) dx

j fj (x)dx

j=1

2

n


j fj (x) dx =

1 2
fj2
j
0

j=1

= n.

44 ( )
f : R R
F f .
an = F (n) n , f (x) = 1
x R.
http://www.mathematica.gr/forum/viewtopic.php?f=59&t=6112

g .
G(n + kr) G(n) = kD k, n Z,
r
D = 0 g(s) ds.
A = G(R) .
.
: r , x R
kn , mn mn
xn = mn + kn r x.
a A x G(x) = a.
kn , mn
mn xn = mn + kn r
x. G
G(mn + kn r) = G(mn ) + kn D G(x) = a
kn D a. B = {kD : k Z}
A. A
B
D - .
,
Kronecker  > 0,
a1 , b1 Z |x a1 b1 r| < . ,
i  2 ai , bi Z ai = 0
|ai + bi r| < /i2 . ai 1.

( )
F (n) n 0. g(x) = f (x) 1



 k
k
k
G(x) = F (x) x,





2 
1
<

.
a
+
b
r

x

g r ,
j
j


j2
6


j=1
j=1
j=1
G G(n) 0, g
.
x
6
G(x) = 0 g(t) dt + G(0). n Z  = 2 > 0,
k
k
m Z n < mr < n + r .
q = j=1 aj + j=1 bj r |q x| < q = a + br ,

G(n + r) G(n) =


mr

g(t) dt =
n
n+r

g(t) dt +
n

n+r

g(t) dt =
mr


a, b Z a = kj=1 aj  k 1 + a1 .
k a = k 1 + a1

35

45 ( )
R.
k =  xk = x x R.

http://www.mathematica.gr/forum/viewtopic.php?f=10&t=14131

1 ( ) x R. R

x R x2 R xn R

http://www.mathematica.gr/forum/viewtopic.php?f=10&t=13894

 ( )
n G

S = x G : T (x) = x1 , |S| > 3n
4 .
x S . xS = {xy : y S} K (x) =
S xS . |S xS|  |G| = n
|S| + |xS| |K (x)|  n |S| = |xS| > 3n
4 ,
n
|K (x)| > 2 . z K (x).
z S T (z) = z 1 . , z = xy
T (y) = y 1 , T (x1 z) = z 1 x.

n, R
z 1 = T (z) = T (xx1 z) = T (x)T (x1 z) = x1 z 1 x,
,
x ,
kx = x xkx = x x .
xz = zx.

, C(x) kx > x .  = max {x : x R}
'
k =  + xR (kx x ) xk = x x R. x G, K(x) C(x).
C(x)
,

G, , Lagrange,
xk = x +(kx x ) yR{x}(ky y )
C(x) = G. x

( x )+kx +(kx x )[ yR{x} (ky y )1]
Z(G) G. S Z(G)
=x
n
Z(G) = G |Z (G)|  3n

4 > 2 . , G
( x )+ x +(kx x )[ yR{x} (ky y )1]
=x
.

= x +(kx x )[

yR{x} (ky y )1

] = = x .

2 ( ) R = {x1 , . . . , xs }
Rs = R R
x = (x1 , . . . , xs ). Rs

k,  k =  xk = x .
xki = x i i .
46 ( )
G . T
G ,
T (x) = x1 x G G .

U : G G x  U (x) = x1 .
G , U
G T (x) = U (x) x S .
S G, G S
G, G.
, , T (x) = U (x) = x1 x G.
3n
4 .
, D8
8 a = (1234)
b = (13), T (x) = bxb1
(12) (34) (14) (23) (
), 6
D8 .

36

47 ( )
+

ln(1 + 1/k)

ln k

k=2

k ln k
an
= 1.
lim
= lim k=1
n bn
n ln (ln n)
n
n

1
n
ln (ln n) +. 
k ln k
k=1

http://www.mathematica.gr/forum/viewtopic.php?f=9&t=15001

n


48 ( )

x
f (x) = ln(1 + x)
2

+
[0, 1] f (0) = 0.


1 + sin z
dz = ln(2)
f (x) > 0 (0, 1] k = 2, 3, 4, . . .
n
z
2
2n


n=1
1
1
ln 1 + k 
.
2k





http://www.mathematica.gr/forum/viewtopic.php?f=9&t=12954
ln 1 + k1
1 
1
1

=
= + .
( )
ln(k)
2k ln(k)
2
k ln(k)
n=2
n=2
k=2



1
sin(z) z=2nx  1
sin (2nx)

1
dz =
dx =
= +

n
z
n
x
k ln(k)
n=1 2n
n=1

n=2
Cauchy. 
 

1
sin (2nx)
dx =
2 ( ) mini
x
n
n=1


!








1
1
1
1
1

1
1
1
ln 1 + n =

+
n =
sin (2nx) eny dy dx =
ln n
ln n n 2n2 n2
x
n=1



0
1
1
1
1


 
+ 2
n , (x) 1


1
1
2n n ln n
n ln n
en(y2ix) en(y+2ix) dy dx =
: R+ R : lim (x) = 0.
2i
x
x0
:



1
1
1
1
2n 1
= 1


, 1
2
2n ln n
2n n ln n
2n ln n
2n
1
.
2

. 

3 ( ) an =

n

k=2

k ln k

bn = ln (ln n) : bn
. ( , -

an+1 an
=
) lim
n bn+1 bn
1
=
lim
n (n + 1) ln (n + 1) (ln (ln (n + 1)) ln (ln n))

1
2i
1
2i

1
2i
1
2i
1
2i

`
= 1 . Stolz-Cesaro
37

1
x

1
x

1
x

0 n=1
 

e(y+2ix)
e(y2ix)

1 e(y2ix) 1 e(y+2ix)

 

dy dx =





ln 1 e(y2ix) ln 1 e(y+2ix) dy dx =

ln

1e(y2ix)
1e(y+2ix)


dy dx =

1  1e2ix   1
ln 1e2ix dx =
x
2i


2x
1
i 2x + 2
x
2

1  i (2x+)  
ln e
dx =
x

!
dx =

1
2 n=1

1
2 n=1

(n+1)

n
(n+1)

n

N 


 x 
1
2x + 2
dx =
x

n=1
N


1
( 2x + 2n) dx =
x




1 
(2n + 1) ln n+1
2
n
2


ln

n=1


ln

(n+1)n+1/2
enn+1/2

 n+1 
n


1 =


=


(N + 1)1/2 =


N
1/2

(N
+
1)

ln e1N (N +1)
N!







1
ln n+1
1 =
n+
n
2
n=1


N
1/2

(N
+
1)
=
lim ln e1N (N +1)
N!
N


N
lim ln e1N (N +1) N (N + 1)1/2 =
2N (N/e)
N



N  N +1 1/2 

e
lim ln 12 1 + N1
=
ln
N
2
ln

n=1








 n+1 
1
1
sin(z)
n+
dz =
ln n 1
n
z
2
n=1
n=1
2n




1cos(2x)isin(2x)
1e2ix
 ln 1e
=
ln
2ix
1cos(2x)+isin(2x) =






sin(x)eix
ln 2i
= ln e2ix = ln ei (2x+)
2i sin(x)eix

 !
 i (2x+) 
2x
 : ln e

= i 2x + 2
2

1
n+
2

1
eN

21 32 ...N N1 (N +1)N
11 22 33 ..N N










1
sin(z)
1
dz =
ln n+1

1
=
n+
n
n
z
2

n=1 2n
n=1





1
e
(, ] .

= 1 ln (2) . 
ln
2
2

49 ( Mulder)
4 + 3 (
4 1), .

.
( ) Dirichlet a, d
,
a + nd, n .
http://www.mathematica.gr/forum/viewtopic.php?f=63&t=15425
.
( )


a = 3, d = 4
. 4 + 3
.
p1 , p2 , . . . , pn .
A = 4p1 p2 . . . pn 1. 50 ( )
A 3 (mod 4). , A
4 + 3 (
x2 = y 7 + 7
4 + 1 A 4 + 1, )
4 + 3
, pi
i = 1, 2, . . . , n.
http://www.mathematica.gr/forum/viewtopic.php?f=63&t=11177
, pi |4p1 p2 pn , pi |A, pi |1, ( ) x2 +112 = y 7 +27 .
. 4 + 3 modulo 4,
38

x 0 mod 2 y 1 mod 4. y + 2 3 mod 4.


p p 3 mod 4 p|(y + 2).
p|(y + 2) p|x2 + 112 p 3 mod 4
p|x p|11. (*) p = 11
y + 2 = 11n k n
k
11. q q 3 mod 4 k . k 1 mod 4 y + 2 3 mod 4,
n , n = 2m + 1.
x2 + 112 = (112m+1 k 2)7 + 27 . 112 ( 11|x) 113 (

1 modulo 11).
112 , m  1.
113 , .
(*)
p p 3 mod 4 p|a2 + b2 , p|a
p|b. p  b c
bc 1 mod 11. (ac)2 + (bc)2 0 mod p
(ac)2 1 mod p. -1
modulo p
p 3 mod 4.

51 ( )
a, b, c R a + b + c = 0 :

a7 + b7 + c7
=
7

a2 + b2 + c2
2



a5 + b5 + c5
5

P = ab+bc+ca, Q = abc (
). ,
, sk = ak + bk + ck .

s2 s5
s7
=

s1 = 0.
7
2 5
, Taylor ln(1 + ax) +
ln(1 + bx) + ln(1 + cx).

http://www.mathematica.gr/forum/viewtopic.php?f=27&t=16543

1 ( )
.
, ,
,
.
Vieta, , ,
3
x + Ax
+ B = 0 (*).

a2 = a2 +b2 +c2 = (a+b+c)2 2(ab+bc+ca) =


0 2A.
(*)

,
,

s1 x

s2 2 s3 3
x x + ...
2
3

ln(1 + ax) + ln(1 + bx) + ln(1 + cx)


=
ln [(1 + ax)(1 + bx)(1 + cx)] =
= ln(1 + s1 x + P x2 + Qx3 ) = ln(1 + x2 (P + Qx)) =
(x2 (P + Qx))2 (x2 (P + Qx))3

+
= x2 (P + Qx)
2
3
... = ...

,
x5 = x3 x2 = (Ax + B)x2 = Ax3 Bx2 =
2
2
2

sk P, Q
= A(Ax B) Bx = Bx + A x + AB
x = a, b, c . ..
P, Q.

 2
 5

2

a = B a + A
a + 3AB = B(2A) + 0 +
: P, Q
3AB = 5AB
 7
.

a
7
3
4
4
5
4
x = x x = (Ax B)x = Ax Bx = P, Q ....
.
Ax5 B(Ax B)x = ...
,
() . 52 ( )
.

2 ( ) ,
.
x2 + y 2 = r 2
39

() A(x1 , y1 ), B(x2 , y2 ).

( ) :

r 2 = M B + M A  E + ()

K = (1 x1 )(1 y1 ) + (1 x2 )(1 y2 )

P  PM.

http://www.mathematica.gr/forum/viewtopic.php?f=27&t=16735

1 ( )
,

L (1 + ) (1 + ) + (1 + ) (1 + )

x1 = r sin a, y1 = r cos a, x2 = r sin b, y2 = t cos b

2 + 2 = 2 + 2 = r 2


2r (sin a + cos a + cos b + sin b)+r2 (cos a cos b + sin a sin b) = , , , . :
 



2 r 2 sin a + 4 + sin b + 4
+r2 cos (b a) 2 +

2



2

L 2+r 2 +( + )+( + )  2+r 2 +2 2r =


2+r
2
2 + r .
2 2r + r =

2+r

2

(*). :

r 2
x1 = x2 = y 1 = y 2 =
2

Kmax = Lmax =

2 ( ) .

2+r

2

, o = = = = r

2
2

(**)
1 1x 1+|x| =
1 + a, a = |x|.

53 ( )

f (x) =

http://www.mathematica.gr/forum/posting.php?mode=edit&f=61&p=78835

1 ( )

2x
,x 0
1 + e2x

f (x) =

, n
. ( : f (n) (0) Z, n 0)

2x
xex
xex
,x 0
= ex +ex =
2x
1+e
cosh
x
2


cosh x = 12 ex + ex

40

lim g(x) = lim g (x) = 0

f (x) cosh x = xe

0


f (x) cosh x + f (x) sinh x = (1 x) e

g = f f  + f 

, f (0) = 1

g + g = f + f  0

f  (x) cosh x + 2f  (x) sinh x + f (x) cosh x


= (x 2) ex , f  (0) = 2

Hardy, g 0

f  f  = f g 0 ()

f (3) (x) cosh x + 3f  (x) sinh x+


3f  (x) cosh x + f (x) sinh x

= (3 x) ex , f (3) (0) = 0
1

1.

0 0 1.
,
.
2 ( )

f (x) + f (x)e2x = 2x [1]


f n (x) + (f (x)e2x )(n) = 0, n 2
0

(e2x )k = 2k , k = 0, 1, ..., n
Leibniz







n
n
n
20 f (n) +
21 f (n1) + ...+
2n f (0) = 0
fn+
0
1
n
n > 1
 
 
n 0 (n1)
n n1 (0)
fn +
2 f
+ ... +
2
f =0
1
n

(*)

h + h = f f  + f  f  =
f f  + (f  f  ) 0 + 0 + 0 = 0
Hardy

h 0, h 0
f f  0 f  f  0.
f 0 f  0
f  0.
2 ( )
Taylor: Taylor
x x + h

f (x + h) = f (x) + hf  (x) +

54 ( )
f : [0, +) R
:


lim f (x) = lim f (x) = 0


x+

h2 
h3
f (x) + f  ()
2!
3!

x x + 1

f (x + 1) = f (x) + f  (x) +

f m n > 1 0.
.

x+

h = f f 

1 
1
f (x) + f  ()
2!
3!

x 1 x

f (x 1) = f (x) f  (x) +

1 
2! f (x)

f (x + 1) f (x 1) = 2f  (x) +

x+

lim f (x) = lim f (x) = 0

x+

x+

lim f  (x) = 0

x+
http://www.mathematica.gr/forum/viewtopic.php?f=61&p=79315#p79315

(*).

1 
1
f () + f  ()
3!
3!

0 = 2 lim f  (x) + 0


1 
3! f ()

x ,
, ,

:


(*), ,

1 ( )
Hardy

lim (g(x) + g (x)) = 0

lim f  (x) = 0

x+

41

55 ( )
a, b, c C
:

2 ( ) (1):


a, b, c .

a3 + b3 + c3 = 3abc

(a b)5 + (b c)5 + (c a)5 = 0

http://www.mathematica.gr/forum/viewtopic.php?f=60&t=9781

1 ( )
:
a, b, c = 0
a5 + b5 + c5 = 0, a + b + c = 0

|a| = |b| = |c| .

a2 + b2 + c2
:

a5 + b5 + c5 + a3 (b2 + c2 ) + b3 (c2 + a2 ) + c3 (a2 + b2 )


= 3abc(a2 + b2 + c2 )
:

b2 + c2 = (b + c)2 2bc = a2 2bc

5
a5 + b5 + c5 = abc(a2 + b2 + c2 ) (1)
2

c2 + a2 = (c + a)2 2ca = b2 2ca


a2 + b2 = (a + b)2 2ab = c2 2ab

a2 + b2 + c2 = 0

2(a5 + b5 + c5 ) 2abc(a + b + c) = 3abc(a2 + b2 + c2 )

ab + bc + ca = 0
( a + b + c = 0).

bc = a(b + c) = a2 abc = a3 |a|3 = |abc|


,

|b|3 = |c|3 = |abc|

(1): x3 + px q = 0
a, b, c,
p = ab + bc + ca q = abc.

56 ( ) (
M obius) a, b, c, d
w = az+b
cz+d
w z
) ;
) ;
http://www.mathematica.gr/forum/viewtopic.php?f=60&t=8876

a3 + pa q = 0

1 ( )

b3 + pb q = 0

w=

c3 + pc q = 0

d w + b
az+b
z=
cz+d
cwa

a2 b2 ) M z .
c2 ,
5

a +b +c = q(a +b +c )3pabc = q(a +b +c )3pq


( a3 + b3 + c3 = 3abc = 3q ). a + b + c = 0
a2 + b2 + c2 = 2p (1).
42

|z z0 | = R > 0



 d w + b


 c w a z0  = R

|(d + c z0 ) w (b + a) z0 | = R |c w a|





b + a z0 


= R |c| w
|d + c z0 | w
d + c z0 

a 

c

|d 
+ c z0 | = 
R |c|
 



. :
z

b + a z0
a
B
d + c z0
c
|d + c z0 | = R |c|
 (

a
|z z1 | = |z z2 |
b + a z0
) M A = M B A
,B

d + c z0
c
z1 = z2
R |c|
=
.
z |z z1 | = |z z2 |
|d + c z0 |
) z z1 = z2 .
A


( ), z1 z |z z1 | =
z2 |z z1 | = |z z2 |. |z z2 | , z1 = z2 , > 0 = 1
= 1 .
d w + b

z =

cwa

.
1)

2)
3) w =

az+b

cz+d

|z z1 | = |z z2 |





 dw b

 dw b





z
=

1
2
 cw a
 cw a
|(cz1 + d) w (az1 + b)| = |(cz2 + d) w (az2 + b)|

cz1 + d,
w ,
cz
+d

2
( w ).
2 ( )








 cz2 + d  

az2 + b 
az1 + b 




. ,
w cz1 + d  =  cz1 + d  w cz2 + d 
,



. .


+
b
az
2
 = |az1 + b|
w
:

cz2 + d  |cz2 + d|
c = d = 0





z . |c| + |d| = 0.
w az1 + b  = |az2 + b|

ad bc = 0 c = 0
cz1 + d 
|cz1 + d|




.

(adbc)
a
c2
.

+ 
c
z + dc
:

ad bc = 0 c = 0 d = 0
ad = 0 a = 0 .
az+b
b
( )
cz+d = d

;
ad bc = 0
) c = 0 z = dc . z z + a
w = ac z az -
1
.
z
z
) c = 0 z
( )
w . :

az + b
az + b
=
= 
cz + d
c z + dc


a z + dc + b ad

 c =
c z + dc

a
1
az + b
= + p.
cz + d
c
z+

dw b
az + b
=wz=
cz + d
cw a
43

d
c

p =

(adbc)
c2

:

1
d
1
1
a
+
p.
xz+

p.

.
1
c 2 z + dc 3 z + dc 4 c
z + dc
.
1 4 , 2 :
3 +. .

44

6
2011

www.mathematica.gr

http://www.mathematica.gr.
http://www.mathematica.gr.
LaTEX: , - : , : . LaTEX.
.

Leonardo da Vinci
(32-) . 30
. -
-
- quasiregular , ( ).



(0, 0, ), 12 ,
, 1+
,
2
2

1+ 5
.
2

:http://en.wikipedia.org/wiki/Icosidodecahedron
:

http://www.mathematica.gr
.

http://www.mathematica.gr

12. (R BORIS)

13. ( )

1. (Mihalis_Lambrou)

14. ( )

2. (nsmavrogiannis)
3. ( )

15. (dement)

4. ( )

16. (swsto)

5.

17. (achilleas)

(m.papagrigorakis)

6. ( )

18. ( )

7. ()

19. (xr.tsif)
20. (Demetres)

1. (grigkost)
2. (cretanman)

1. (spyros)

1. (stranton)

2. (p_gianno)

2. ( )

3. (kostas.zig)

3. (vittasko)

4. (exdx)

4. ()

5. ( )

5. (s.kap)

6. (mathnder)

6. (nkatsipis)

7. (mathematica)

7. ( )

8. ( )

8. (chris_gatos)

9. (rek2)

9. (matha)

10. (hsiodos)

10. (mathxl)

11. ( )

11. (gbaloglou)

12. (bilstef)

6 ( ) 9


1 ( ) .
120 , ... ,
70
. :
36 ,
35 ,
40
42 .

;

2 ( )

2.
.
x , x
x. 1089.
..
572. 752 257 x = 752257 =

1
cm 1cm.
2

.
12cm , .

297

7 ( )

.

, 792 + 297 = 1089


361. 631 136 x =

631 136 = 495

, 594 + 495 = 1089


8 ( KARKAR) a > b > 0 x =

ab
,y =
a+b

a 2 b2
) x, y ) () a 2 + b2
x.

3 ( )
.


72
.
,
,
50 ;

,
9 ( )

|x|

|x y|
= 1
x
|2x y| + |x + y 1| + |x y| + y 1 = 0

4 ( KARKAR) ,
, 300 ,
.
20 ,
24

10 ( )

30.


3


3

2
+
9


3

4
=
9


3

3
21


5 ( KARKAR) ,
( 2011) 12.
3 24, 2
.
1) ;
2) ,
;

,
11 ( KARKAR) M , ABC , (K, R) , AB, AC Z, H .

ZK, HK BC T, S .
: ZH = ST .

17 (
 )
 
a, b, c |a| = 1, b = 2, |c| = 3. 2a +3b +c =
0.

 12 ( ) B

30 45 . M B,
 .
MA

18 ( )

x2
y2
+ 2 = 1 E(, 0). K
2
a

C , (EK) + (E) = a,
K , .
C:

,
13 ( )
(an ) (bn ) ( )
a1 = b1 , a2 = b2 , bn > an n 3.

,
19 ( ) f, g : R R f (x) = (a2 + b2 )x2 2ax g(x) = 4bx + 5.

14 ( KARKAR ) : (x

x + 1)x < 1

i) lim f (x) + lim g(x) = 0 a = 1


b = 2.

x1

x1

ii) a, b
f (x) 35
lim
.
3
x 5 g(x) + 13x 8

15 ( ) ABC AB = AC = 1. AB
K, L, M AK = KL = LM = M B AC P, Q
AP = P Q = QC . D CK, P M , E
CK, QL Z P M, QL
DEZ .

iii) a, b
f  ( 35 )ln2 = g  (2012).
20 ( ) k , c.
fi .
. c k .
. c, k .
. f1 = 25, fi

16 ( KARKAR) S
, ABC , ST, SP ,
CA CB .
P T AS .

, ,

21 ( )
z 2 z + = 0 , R,z C z1 , z2

, ,
27 ( )
f : [a, b] R

z1 = 2 + i
i. , R
ii. z12008 + z22008 R. A(z1 ), B(z2 ), (z3 )
z1 , z2 z3

a+b
2

z1
1
z3 =
+ (17 + i)
z2
5

f (x)dx <

b
a

xf (x)dx

28 ( ) f, g
R
1) f (x) =
0, x Z,

, :

iii. AB .

iv. |w z1 | = |w
z1 | w R.

2) f (x) =

v. w
|w z2 | + |w
z2 | = 10,

f (t)eg(t) dt, x R.

f (x) = 0, x R.

, ,
29 ( Christiano ) f : R R
2f 3 (x) + 2f (x) = sinf (x) + x2 |f (x)| |x|.

22 ( ) z = x + yi x, y R : (1 i)z = 4 2i, R
) : (x + y) + (x + y)i = 4 2i
) (x, y) z .
) (x, y) O(0, 0);

30 ( )
f : R R f  (x) = 0 . f 
A (1, 0) B (2, 1).

, ,,

A f  R.

23 ( )
f : R R x R :

e1f

(x)

B f .
u.

f  2 + f  (|i u + 1|) < 0

f 1 (x) = x + 1

1. f .

u.

2. :
. f .
. f, f 1
.
3.

f 1 (x)
.
x
x

i. lim

z, z1 , z2 , w

|z z1 | = f (1), |w z2 | = f (2) |z1 z2 | = 2f 32 .


|z w|.

1
f (x) + x

, ,

ii. lim

31 ( )

24 ( ) f, g , f 2 (x) + g 2 (x) = 1
x .

ex x1


>

ex 1
x

xln(x+1)

x (0, +)

f 4 (x)
g 4 (x)
1
, +
=

+
0, + = 0
)

x
ln (x + 1)

32 ( )
f : R R <
< f (f () + b) + f (f (b) + ) = f () f  (b) + f  ()
f (b) + f () + f (b) f ,
.

g (x)
1
f (x)
+
=
.
3
3
( + )3

) |f (x) + g(x)| 2 + 2 .

, ,

33 ( ) AB
AA1 , BB1 , 1 . BA1 + B1 + A1 = A1 + B1 A + 1 B ,

25 ( ) : f, g :
[0, 1] R

1) f (1) = g(0) + 1

34 ( ) a R b, c R 4ac < (b 1)2 f : R R


f (ax2 + bx + c) = af 2 (x) + bf (x) + c, x R.
f (f (x)) = x
.

2) f (x) g(x), x [0, 1]


3) f  (sin2 x) + g  (cos2 x) = 2, x R
: f = g
26 ( )
f : R (0, +) f  (x) = 3 ln f (x) x R f (1) = 1.

Juniors, - -

35 ( )
,

f (x) = [x] + [2x] +

 5x 
3


43 ( )
:

+ [3x] + [4x],

9x + 10y = 12z + 1

x [0, 100].
36 ( )
0, 1, 2, ..., 100 . , A, 50 + 50
( .)
a.
B + +
A ,
b. a + b 2011,
.

44 ( ) (n)
(n) n,

(n)
n+1

(n)
2

;

45 (

Juniors,

n=1

37 ( )
ABCD. AC, BD
O.
 OAB,  OBC,  OCD,  ODA r1 , r2 , r3 , r4
E ABCD. :
r12 + r22 + r32 + r42 E. ;


ln 1 +

ln 1 +

1
2n

IMC

ln 1 +

2011/B3)
1
2n+1

46 ( IMC 2011/A4) A1 , A2 , ..., An ,


, . :

f (t) =

n


(1)k1 t|Ai1 Ai2 ...Aik | .

k=1 1i1 <i2 <...<ik n

=
38 ( )  ABC, A
90o , AB = AC, D BC . DE AB, DZ AC H BZ CE .
DH EZ .

f [0, 1].

47 ( ) A
M3,2 (C), B M2,3 (C)

Seniors, - -

1
AB = 0
0

39 ( ) f : Z Z

1
0
0

2
2
2

det(BA).

f (x y + f (y)) = f (x) + f (y)

48 ( ) n n A = (aij ), aij {0, 1} i, j {1, 2, . . . , n},


t
AA = mIn + Jn , m , Jn
n n 1, In n n t
A A.
A , AAt = At A.

x, y Z.
40 ( ) x, y, z > 0
xyz = 1, :

1
1
1
1
+
+

2
2
2
2
2
2
(x + 1) + y + 1
(y + 1) + z + 1
(z + 1) + x + 1
2


49 ( ) ,
,

Seniors,
41 ( )  ABC

M .
A


lim

n+

BC AM, B AC BM, C AB CM E1 =






(BA M ), E1 = (CA M ) E2 = (CB M ), E2 = (B AM )



1
1
1
E3 = (AC M ), E3 = (C BM ).
+
+
=
E1 E2 E3
1
1
1
 +
 +

E1
E2
E3

n2
ln2 n 
1
.
n
ln k ln(n k)
k=2

50 ( )

42 ( )
ABC , (1 ) O (2 )
AOC . OQ
(2 ) M, N
AQ, AC , AM BN
.
M N, BQ (2 ).

cos x 1
dx , a > 0 .
x2 (x2 + a)

= A, 
51 ( ) A

. A = B + C , B

C
. ( A, A  = B, B  + C, C 


x A y (x B y x C y (x B y C)) ).

52 ( )
X X
.
.

x0
( ),
( lim

xx0

f (x)
) g(x)

de l Hospital,

f, g ( x0 )

53 ( Atemlos) ,
,
.

lim

xx0

,
;

54 ( )
a, b, c R3 . x + (x a)b = c.

60 ( ) f : R
R

R .
;



55 ( ) an = 24n + 1, n = 1, 2, . . .
2 3.

,
61 ( ) [. ,
, ( ,
1976)]
P K
E Z
.

56 ( )

, .
( 83 73 = 512 343 = 169 = 132 13 = 22 + 32 )

62 ( )
.

()
57 ( ) ,

sin2 50
sin 1 + sin 3 + sin 5 + + sin 99 =
sin 1

f  (x)
.
g  (x)

63 ( ) P (x)
.
P (1) = 6 P (7) = 3438,
x = 3.

58 ( )
3a, a
E1 , E2 .

64 ( )

2
E : E = (E1 + 9E2 ).
9

P (x) = xn + a1 xn1 + + an

,
59 ( pla.pa.s )

Q(x) = xn + b1 xn1 + + bn

C.
P
x1 , x2 , ..., xn ,

Q x21 , x22 , ..., x2n . a1 + a3 + a5 + R
a2 + a4 + a6 + R, , b1 + b2 + bn R.

f  (x)

de l Hospital g (x) . ;
f (x)
g(x)

:
1 ( ) . 120 ,
... , 70
.
:
36 ,
35 ,
40
42 .

;

.
32.
,

50 35 = 15
,

32 15 = 17
32, 17
.
17.

http://www.mathematica.gr/forum/viewtopic.php?p=55933

1 ( )

50 36 = 14

36 + 35 = 71 > 120 70 = 50

17 14 = 3

21 2
. :

17.
,
, 3 .
3 ( )

40 + 42 = 82 > 50
32 .

36 + 35 + 40 + 42 = 153

32 + 21 = 53 > 50

50 3 ,
150.
3 .

3 4
2 ( )

2 ( )
2.

.
x , x
x.
1089.
..

120 70 = 50
, 42 .

50 42 = 8
, 40 ,

40 8 = 32
6

572. 752 257

x = 752 257 = 297


, 792 + 297 = 1089
361. 631 136
x = 631 136 = 495
, 594 + 495 = 1089

99a = 90a + 9a = 100a 10a + 9a = 100a a =


100a 100 + 100 a =
(a 1)100 + 9 10 + (10 a)

http://www.mathematica.gr/forum/viewtopic.php?p=84884

( )
xyz zyx

(10 a)100 + 9 10 + (a 1)

99(x z)
99 a99 a 2 8.
a99 .

1089.

:
3 ( )
.



72 .
, ,
50 ;

20 ,
24
30.
;
http://www.mathematica.gr/forum/viewtopic.php?p=85871

( ) ,
300 ,

http://www.mathematica.gr/forum/viewtopic.php?p=87192

( )
72 .., 72 : 2 = 36
..

1 1 1
, ,
20 24 30
a, b, c , :

1
20

b
1
24

c
1
30

a+b+c
1
1 =
+ 24
+ 30

1
20

300
15
120

a
1
20

,
6 . 62 = 36.
6 . 2 6 = 12 .,

b
1
24

= 2 12 + 2 6 = 36

c
1
30

, , 3650 = 1800 .

= 2400

= 2400 a = 120
= 2400 b = 100
= 2400 c = 80

:

.

4 ( KARKAR) ,
, 300 ,
.

:
5 ( KARKAR) ,
( 2011)
12.
3 24, 2 .
1)
;
2)
, ;

) 720, , , 720 2011,


365+355 -
2012 - 11 ,
20 2012!
6 ( )
9

http://www.mathematica.gr/forum/viewtopic.php?p=72306

(KARKAR) (
), 12 24 . (24-)
2 5 .
, 12
( = 720 ). 720 : 5 = 144 .
144 3 = 432
(= 7 12 ),
144 2 = 288 (= 4 48 ).
12 ( ),
7.12 (12+7.12, 124.48)
, 144
2011,
(31+28+31+30+24) , 24 .
, ,
12
720 : 3 = 240 .
,
12 720 : 2 = 360 .
( 240, 360

1
cm 1cm.
2

.
12cm ,
.
http://www.mathematica.gr/forum/viewtopic.php?p=85516

( )
4 ,

( )


 2
1
5
cm. ,
1+
=
2
2

5
+ 1 + 2 12 = 8 5 + 25 cm
16
2

:
7 ( )

.

)
- .
8 ( KARKAR) a > b > 0

x=

http://www.mathematica.gr/forum/viewtopic.php?p=60737

1 ( ) AB
AB//.
B A, B AE, Z .
ABE Z.
-

ab
a2 b2
) x, y
,y = 2
a+b
a + b2

) ()
x.
http://www.mathematica.gr/forum/viewtopic.php?p=66599

ab
a2 b2
a2 b2
x=
=
, x < y ,
=
a+b
(a + b)2
a2 + 2ab + b2

 = Z
 = 90 ) ABE
 = Z

(AEB
.
: AE = Z
( ) , AB =
,
. .
2 ( )

.
ABD, BCD
BD . (


(a2 b2 ),
(a2 + 2ab + b2 > a2 + b2 ).
)

a2 b2
a2 b2
<
2
a2 + b2
a2 + 2ab + b2
1
2

< 2
2
2
a +b
a + 2ab + b2
2
a + 2ab + b2 < 2a2 + 2b2

y x < x y < 2x

a2 2ab + b2 > 0 (a b)2 > 0,


, .

10

9 ( )

10 ( )

|x|
= 1
|x y|
x
|2x y| + |x + y 1| + |x y| + y 1 = 0

2
+
9

4
=
9

21

http://www.mathematica.gr/forum/viewtopic.php?p=94586

http://www.mathematica.gr/forum/viewtopic.php?p=94028

1 ( ) (x, y) ( x = 0).

( ) a =

0 = |2x y| + |x + y 1| + |x y| + y 1

b=

|2x y (x y)| + |x + y 1| + y 1
= |x| + |x + y 1| + y 1 |x + y 1| + x + y 1

.
(2x y)(x y) 0, x > 0, x + y 1 0. (1)
x > 0,
|x y| = 0, x = y.
, (1) x

1
.
3
1

2
+
9

4
9

a3 + b3
a+b

3
2
1
+
3
3
3

=
3
3 2
3 1
2+1
3 +
3

= a2 ab + b2 =
=

1
.
2

(a, a)

 1
a 0,
.
2
2 ( ) x 0 , |x y| = 0 x = y .
: x + |2x 1| + x 1 = 0
1
|2x 1| = (2x 1) 2x 1 0 x
2
x < 0
|x y| + 1 = 1 |x y| = 2.


21 =
=
=
=

1
: (x, y) = (a, a), 0 < a .
2

11


3
21 3 2+1

3
2+1



3
3
21 3+33 4+33 2

3
2+1

3
 



3 3 3
3
3

1
4
+
2
+
1
3
2+1

3
3
3
3
3

.
2

1
=
3
3
2+1
2+1
3

:
B  , C 
AC, AB

11 ( KARKAR) M
, ABC ,
(K, R) ,
AB, AC Z, H .

ABC , : AB

AC

.
.  
AA BC
=

AB 
C B //BC
AC 
.

=
AA C  B  : (1). ZC 
ZM


C KM KH
  . M KB

KM H, KA T , KM = KA =
oo
   
 o
= M KB
=

 = M KH
BCM
o . (1)

 , ,
=
A KT

M H = A T : (2). ZKM, SA K , ZM =
A S : (3). , (2) (3)
: ZH = ST .
2 ( ) D, N, E (K, R)
AB, BC, CA . N
BC : BD = BN =
CN = CE : AD = AE .
AE
AD

=
DE//BC (1).
BD
CE

ZK, HK BC
T, S . : ZH = ST .
http://www.mathematica.gr/forum/viewtopic.php?p=83176

1 ( ) ABC
(AB = AC), AK () BC A .

F KZDM G KH M E . KDZM, KM HE , F, G DM, M E


(1)

 = x
M DE : F G//DE // BC . F GK
12

 = x ( ), F M
K = x ( T SK

 = 90
KF M G, Z F M = M GK

) F ZM = x( F ZM
F = 90 x).
ZM
KM Z, KN S (

AM .
:

45 + x =

180 30
x = 30 .
2

2 ( )

M M B , E
BA.

- ),
KZ = KS . KZH, KST
, ZH = ST .
12 ( )  30
 45 . M
B
 .
B, MA
http://www.mathematica.gr/forum/viewtopic.php?p=84016

1 ( )
BA .
 = 45 ( AB)
A
 = 45 ( B
A
 = 15 ).
 = 30 AB
B


B EC
= 90 , M EB
M EC .
1 

AEM ACM
2
E EA,
 = 1 60 = 30 .
 = 1 M EC
M AC
2
2

=
ACM

A ,
M ( M)

13

:
: y > x n 3. :

13 ( )
(an ) (bn ) (
) a1 = b1 , a2 = b2 ,
bn > an n 3.

y>x

q n1 > pn1 + (n 1)(q p)pn2

http://www.mathematica.gr/forum/viewtopic.php?f=21&t=17827

(q n1 pn1 ) (n 1)(q p)pn2 > 0

1 ( ) , . , . . .
:

= a2 a1 , = bb21 =
= a1 ( 1)

q n1
> p + (n 1)(q p)
pn2

(qp)(q n2 +q n3 p+...+pn2 )(n1)(qp)pn2 > 0


(qp)(q n2 +q n3 p+...+pn2 pn2 pn2 ...pn2 ) > 0

a2
a1

(qp)[(q n2 pn2 )+p(q n3 pn3 )+...+pn3 (qp)] > 0


(qp)2 [(q n3 +...+pn3 )+...+p(q n4 +...+pn4 )+...+pn3 ) > 0

bn = b1 .n1 , an = a1 + (n 1)
bn > an n 3

.
14 ( KARKAR )
: (x4 x + 1)x < 1

a1 .n1 > a1 + (n 1)a1 ( 1)

http://www.mathematica.gr/forum/viewtopic.php?f=21&t=17040

n1

( ) :

> 1 + (n 1)( 1) ( a1 > 0)


Bernoulli :
n1

n1

= [1 + ( 1)]

x4 x + 1 = x4 x2 + x2 x + 1 =

1 + (n 1)( 1)

1
1 1
+ x2 x + + =
4
4 2
1
1
1
= (x2 )2 + (x )2 + > 0
2
2
2

x4 x2 +

( 1 > 1,
> 0 ).
n 1 = 1 , n = 2
n 3
.
2 ( )
p, q () x, y
. :

(x4 x + 1)x < 1 :

x ln(x4 x + 1) < 0, (1)


x = 0 (1)
x > 0 (1) : ln(x4 x+1) < 0 x4 x+1 <
1 x4 < x x3 < 1 x < 1 0 < x < 1.
x < 0 (1) : x4 x + 1 > 1 x4 > x
0
x3 < 1 x < 1. x < 
x ( , 0) (0, 1).

x = p + (n 1)(q p)

 n1
q
q n1
y =p
= n2
p
p

14

15 ( ) ABC AB = AC = 1.
AB K, L, M AK = KL = LM =
M B AC P, Q AP = P Q = QC .
D CK, P M , E
CK, QL Z P M, QL DEZ .

: (DEZ) =

1
3
=
2
12
48

2 ( )
AM P KDC :

1 MD 2
AK M D P C

=1
= 1 MD =
KM DP CA
2 DP 3
3DP (1).
AM P L, Z, Q:
2 MZ 1
AL M Z P Q

=1
= 1 MZ =
LM ZP QA
1 ZP 2
ZP (2)
(1) :

http://mathematica.gr/forum/viewtopic.php?p=95051

1 ( ) ,
12 (...
3, 4).
A (0, 0)

(2)

M D = 3P D M Z + ZD = 3P D ZP + ZD =
3P D
ZP P D + ZD = 2P D 2ZD = 2P D
ZD = P D (3)
D ZP .

AKC P DM
QEL :

K (3, 0) , L (6, 0) , M (9, 0) , B (12, 0) , P (0, 4) ,


Q (0, 8) , C (0, 12). ABC
AK = KL = LM = M B, AP = P Q = QC
CK y = 4x + 12, P M : y =
4
4
x + 4 QL : y = x + 8 , 9
3


 

 :
3
9
9

, 3 , E
, 6 , Z
, 2 DE =
D
4
2
2





9
1
3
, 1 (DEZ) = |6| = 3
, 3 , DZ =
4
4
2

DE = KD (4)
(3) (4)
KZEP .
EZ
ALQ :

AL =
15

2
5
1
4
, AQ = LQ = a =
2
3
6
10

/ : =

2
a
=
2
10

2 ( )
 = B CS
 = x (1), CABS : BAS


1
1
E (KZEP ) = (EZ) =
4
4
1 5/6 2
1
1 LQ a
..
=

=
=
4 2 2
4 2 10
48

CT P S

E(DEZ) =


 =
C TS = C PS = 90 : (B CS)
= P CS

(1)
P TS = x
 = B AS
 (1)
= x.
T S, AB : T SA

16 ( KARKAR)
S , ABC ,
ST, SP , CA
CB . P T
AS .

M T S, M T A (
) M T , M AS .
3 ( )
http://www.mathematica.gr/forum/viewtopic.php?f=22&t=18701&p

1 ( ) SK AB
T, P, K ( Simson).


: F SP (ABC) ,

AM = M S
SP = P F

P M  F A P M S = F AM =

2BAM = 2M ST = AM T.
T, M, P .

AT SK AS, T K
M.

16

17 ( )

 

 
a, b, c |a| = 1, b = 2, |c| = 3.
2a + 3b + c = 0.


1 (b, c) 1

2a + 3b + c = 0

http://www.mathematica.gr/forum/viewtopic.php?p=93638

1 ( )

2a + 3b + c = 0

18 ( )

x2
y2
+
= 1 E(, 0). K
a2
2
C , (EK) + (E) = a,
K

3b = c 2a

C :

 
 
3b = |c 2a| |c| + |2a|
6 2+3

, .

, .
2 (parmenides51) :

http://www.mathematica.gr/forum/viewtopic.php?p=70751

( ) K (xk , yk ) , (xl , yl ).
:

2a + 3b + c = 0

EK = a xk

3b + c = 2a
(3b + c)2 = (2a)2

9b2 + 6bc + c2 = 4a2

E = a xl
EK + E = a


9|b|2 + 6|b||c| (b, c) + |c|2 = 4|a|2

(xk + xl ) = a


9 2 + 6 2 3 (b, c) + 32 = 4 12
2

xk + xl =


36 + 36 (b, c) + 9 = 4

xm =

41

(b, c) =
36

a2

a2
2
2

a
M x = 2
.

17

19 ( )
f, g : R R f (x) = (a2 + b2 )x2 2ax
g(x) = 4bx + 5.

i) lim f (x) + lim g(x) = 0


x1

a = 1 b = 2.

x1

ii) a, b
f (x) 35
lim
.
x 53 g(x) + 13x 8
iii) a, b

f  ( 35 )ln2 = g (2012).

http://www.mathematica.gr/forum/viewtopic.php?p=90978

( )
. 5, 45 .
x1 , x2 , ..., xk . ,
c
c
40
2 + ck + 2 = 45 5 c = k+1 .

http://www.mathematica.gr/forum/viewtopic.php?p=90309

( )

i. 0 = lim f (x) + lim g(x) = lim [(a2 + b2 )x2


x1

x1

x1

2ax] + lim [4bx + 5] = a2 + b2 2a + 4b + 5.

. ,
, c. ,
b = 12 + k + 12 = k + 1.

100,
:E = 100 12 50b =
100 12 50(k+1) = 100 k = 3 c = 10.

x1

:a2 + b2 2a + 4b + 5 = 0 (a 1)2 +
(b + 2)2 = 0 a = 1 b = 2.

f (x) 35
5x2 2x +
= lim
5x 3
x 53 g(x) + 13x 8
x 3


 5 
1
(5x 3) x + 5
4
1
= lim x +
= .
lim
3
x1
5x

3
5
5
x 5

ii. : lim

3
5

. k = 3, f2 = 50
f3 = 100 (25 + 50) = 25
.

iii. f  (x) = 10x 20 g (x) = 8


f  ( 35 ) = 8 g  (2012) = 8
f  ( 35 )ln2 = g (2012) ln2 = 1 =
e = 1e
20 ( )
k , c.
fi .
. c k .
. c, k .
. f1 = 25, fi

18

21 ( )
z 2 z + = 0 , R,z C z1 , z2
z1 = 2 + i

22 ( )
z = x + yi x, y R :
(1 i)z = 4 2i, R
) : (x + y) + (x + y)i = 4 2i
) (x, y) z
.
) (x, y) O(0, 0);

i. , R

ii. z12008 + z22008 R.


A(z1 ), B(z2 ), (z3 ) z1 , z2 z3

z3 =

http://www.mathematica.gr/forum/viewtopic.php?p=86918

z1 1
+ (17 + i)
z2 5

( ) :

, :

(1 i)z = 4 2i
(1 i)(x + yi) = 4 2i

iii. AB
.

(x + y) + (x + y)i = 4 2i
x + y = 4, x + y = 2, R (I)

iv. |w z1 | = |w
z1 | w R.
v.
w |w z2 | + |w
z2 | = 10,

) (I) : (x + y) + (x + y)i =

4 2i

)
x, y :

  (I)


http://www.mathematica.gr/forum/viewtopic.php?p=27051

(x, y) =

( )

i. z2 = 2 i Vietaz1 + z2 = = 4
z1 z2 = = 5.

Dx Dy
D , D

2+4 424
,
2 +1 2 +1

, R. :

(x 2)2 + (y + 1)2 =
2 
2

4 2
2 + 4
2 +
+1 =
=
2 + 1
2 + 1
2  2
2

+ 4 1
22 + 2 + 2
+
= ...
=
2 + 1
2 + 1
.... = 5

ii. z12008 + z22008 = z12008 + z1 2008 = 2Re(z1 ) R.


iii. z3 = 4 + i, A(2, 1), B(2, 1), (4, 1). :
AB = A = 2 ABA,
.

iv. |w z1 |2 = |w
z1 |2 (z1 z1 )(w
w) = 0
w
= w w R.

(x, y)
z (2, -1) 5.
) (x, y) = (0, 0).

(1 i)z = 4 2i 0 = 4 2i, .
(x, y)
O(0, 0).

v. : |w z2 | + |w
z1| =
10 |w z2 | + |w z1 | = 10, |z1 z2 | =
|2 + i (2 i)| = |2i| = 2 10 .

19

M f 1 .
Cf , Cf 1
M ( y = x)
: Cf , Cf 1
A(x1 , y1 ) , x1 = y1
( y = x). :
f (x1 ) = y1 f 1 (x1 ) = y1 f (y1 ) = x1 , :
f (x1 ) f (y1 ) = y1 x1 e1x1 x1 1 (e1y1
y1 1) = y1 x1 e1x1 e1y1 = 0 e1x1 =
e1y1 x1 = y1

23 ( ) f : R R
x R :

e1f

1 (x)

f 1 (x) = x + 1

1. f .
2. :
. f .
. f, f 1
.
3. : i. lim

ii. lim

f 1 (x)

f 1 (x) + x

.
1
3. i) (1) f 1 (x) = e1f (x) x 1 f 1 (x) >

x 1

lim (x 1) = +

lim f 1 (x) = +

http://www.mathematica.gr/forum/viewtopic.php?p=57607

( )

e1f

1 (x)

(1) f 1 (x) + x = e1f

f 1 (x) = x + 1 (1)

x R .
f, f 1 ( )
R.
1. (1) x f (x) , x R
1
: e1f (f (x)) f 1 (f (x)) = f (x) + 1

lim




1
f 1 (x) + x = lim e1f (x) 1
x

u=f 1 (x)

e1x

x 1, x R
2. ) x1 , x2 R x1 < x2 x1 < x2 x1 >
x2 x1 1 > x2 1 (), , x1 < x2
x1 > x2 1 x1 > 1 x2 e1x1 > e1x2 ().
(), () :
1x
e 1 x1 1 > e1x2 x2 1 f (x1 ) > f (x2 )
f .
f (x) =

1 (x)

lim e1u 1 = 0 1 = 1

u+

f 1 (x)
=
x
x
f 1 (x) + x x
f 1 (x) + x
= lim
1=
lim
x
x
x
x

1
1
1 = (1) 0 1 = 1.
lim f (x) + x lim
x
x x

ii) lim

)
f, f 1 : f (x) = x (2) f (x) x =
0 e1x 2x 1 = 0 (3) , x R g(x) =
e1x 2x 1 ( )
[0, 1] g(0)g(1) = (e 1)(2) < 0 .
Bolzano (3) (2)
(0, 1). g
(
f x , )
1 1 (2) (0, 1),
f
y = x M (, ) , (0, 1).

24 ( ) f, g ,
f 2 (x) + g 2 (x) = 1 x .

g4 (x)
1
f 4 (x)
+
=
,

+
0, + = 0
)

1
f 8 (x) g8 (x)
+
=
.
3
3

( + )3
) |f (x) + g(x)|
20


2 + 2 .

http://www.mathematica.gr/forum/viewtopic.php?p=18988

1 ( ) ( ) , :

f 2 (x) =

( + ) f 4 (x)+ ( + ) g4 (x) = f 2 (x)+g2 (x)

2 4
2 4
f (x)
g (x)
g8 (x)
f 8 (x)
+
=
+
= ... =
3
3
3

3
+
1
4 =
( + )
( + )3

f 4 (x) + 2 f 4 (x) + g4 (x) + 2 g4 (x) =


f 2 (x) + g2 (x)

2 ( ) ( )
Lagrange:

, :

2 f 4 (x) + 2 g4 (x) =

2 + 2 = (f 2 (x) + g2 (x))(2 + 2 ) =





f 2 (x) 1 f 2 (x) + g2 (x) 1 g2 (x)

(f (x) + g(x))2 +(g(x) f (x))2 (f (x) + g(x))2

, :

2 f 4 (x) + 2 g4 (x) = 2f 2 (x) g2 (x)

2 + 2 |f (x) + g(x)|

3 ( ) ( ) CauchySchwarz

, g2 (x) =
+
+

f 2 (x) ag2 (x) = 0 f 2 (x) = g2 (x)

|f (x) + g(x)|

f 2 (x) + g2 (x) = 1


2 + 2

: f 2 (x) + g 2 (x) = 1

21

2 + 2


f 2 (x) + g2 (x)

25 ( )
: f, g : [0, 1] R

m M h (u) h (1 u) M m

1) f (1) = g(0) + 1

mM 0M mm=M

2) f (x) g(x), x [0, 1]


3) f  (sin2 x) + g  (cos2 x) = 2, x R

: f = g

h (u) = c f (u) u = c f (u) = u + c

http://www.mathematica.gr/forum/viewtopic.php?p=89293

u = 0 f (0) = c. f (u) = u + f (0),


.

( )

f  (sin2 x) + g (cos2 x) = 2

26 ( )

f : R (0, +) f  (x) =
3 ln f (x) x R f (1) = 1.

sin 2xf  (sin2 x) + sin 2xg (cos2 x) = 2 sin 2x




 2
f sin x g cos2 x = ( cos 2x)




f sin2 x g cos2 x = c cos 2x
x = 2 1 = c + 1 c = 0.



f sin x g 1 sin2 x = cos 2x = 2sin2 x 1, x R
2

sin2 x = u [0, 1]

f (u) g (1 u) = 2u 1 (1) u = 1 u f (1 u) g (u) = 1 2u (2).


f () > g (). x =
(1), (2)

http://www.mathematica.gr/forum/viewtopic.php?p=94525

( )
ln t t 1 f  (x) 3(f (x) 1)
x, (1).

g(x) = e3x (f (x) 1),


x R. g (x) = e3x (f  (x) + 3f (x) 3). ,
(1), g  (x) 0 x R. g R. , x (1, +)
g(x) g(1) = 0, f (x) 1,
.. f  (x) 0. f (1, +). f (x) f (1) = 1 x (1, +). , f (x) = 1 x (1, +). (, 1)
f (x) = 1 x (, 1).
f (1) = 1, , () ,
f (x) = 1.

f () g (1 ) = 2 1
f (1 ) g () = 1 2

f () g () + f (1 ) g (1 ) = 0
.
u [0, 1] f (u) = g (u)

, f  (x) = a ln f (x)
f (1) = 1 a < 0.
2 ( )
.

f (u) f (1 u) = 2u 1
f (u) u [f (1 u) 1 + u] = 0
h (u) = f (u) u, u [0, 1] [0, 1]
M m.
m h (u) M, u [0, 1]. u = 1 u

: t ln t t + 1 0 t > 0.
t = 1 . , .

m h (1 u) M M h (1 u) m
22

(f  (x))2 = 3f  (x) ln f (x).

x = 1 c = 0.
x

3f (t) ln f (t)dt
1

x
= 3 f (t) ln f (t)
1
 x
f  (t)
+ 3
f (t)
dt
f (t)
1
= 3f (x) ln f (x) + 3f (x) 3

(f (t)) dt =

ln f (x) e1

4 ( ) ln(y(1)) = 0. x.

x0 : lny(x0 )) = 1
1 y
. ln(y(x)) = 0x (1, x0 ) [1].
. Rolle (1, x0 ) : y  () = 0
3ln(y()) = 0  [1]. x > 1

x 1. (
). , ,
f (x) = 1 x 1.
x < 1;
3 ( )

x


(ln f (x)) e1
(ln f (x)) +
x


(ln f (x)) e1

x
y
dy
= 3
= 3(x 1) [2]. lny
lny
1
[2] x 1.
(x1)1/lnu = 3(x1)
1 < u < x x < u < 1. x 1 u 1 1/0+ = 3
lnf (x) = 0 f (x) = 1, x > 0.

3
ln f (x) = 0
f (x)
x


3
dt
f (t)

+ e 1

3
dt
f (t)

ln f (x) = 0

3
ln f (x) = 0
f (x)
x


3
dt
f (t)

x

ln f (x) e1

+ e 1

3
dt
f (t)

3
dt
f (t)

ln f (x) = 0
x

= 0 ln f (x) e1

3
dt
f (t)

=0

ln f (x) = 0 f (x) = 1

= 3(f (x) ln f (x) f (x) + 1 0

(ln f (x)) +

3
dt
f (t)

=c

23

27 ( )
f : [a, b] R

a+b
2

f (x)dx <

28 ( )
f, g R
1) f (x) = 0,
 x Z,
x

2) f (x) =

f (t)eg(t) dt, x R.

f (x) = 0, x R.

xf (x)dx

http://www.mathematica.gr/forum/viewtopic.php?f=54&t=19032

http://www.mathematica.gr/forum/viewtopic.php?f=54&t=18241

( )

( ) f
R

a+x
2

g(x) =

x
a

f (t)dt

x
a

tf (t)dt, x [a, b]

f (x) = f (x)eg(x)

g(b) < 0. g
(a, b) g(a) = 0 x (a, b)

k Z . f [k, k + 1]
.


1 x
xa
f (x) =
g (x) =
f (t)dt
2 a
2
x
a

x a  a f (t)dt a f (t)dt

f (x)
2
xa
xa
(f () f (x)) < 0
g  (x) =
2
(a, x) g [a, b]
g(b) < g(a) g(b) < 0.


f (k) = f (k + 1) = 0
f
r (k, k + 1) Fermat


f (r) = 0 f (r)eg(r) = 0 f (r) = 0

(*) 

y

f (t) dt [a, x].

. f (x) = 0 x [k, k + 1]
k Z . f (x) = 0 x R.

24

:
29 ( Christiano ) f : R
R 2f 3 (x) + 2f (x) = sinf (x) + x2
|f (x)| |x|.

http://www.mathematica.gr/forum/viewtopic.php?p=77273

( ) ) f  (x) = 0
f  , R
f  (x) > 0 f  (x) < 0

http://www.mathematica.gr/forum/viewtopic.php?p=94879

[1, 2]

( )

f  (2) f  (1)
= 1 < 0
21
f  (x) < 0 f  R
1-1 f  (1) = 0 1
f  R.
) f  R
x < 1 f  (x) > f  (1) = 0 f
(, 1] x > 1
f  (x) < f  (1) = 0 f
[1, +) x0 = 1 f (1).
) f  (2 + f  (|iu + 1|)) < 0 f  (2 +

f (|iu + 1|)) < f  (1) f 
R
2 + f  (|iu + 1|)) > 1 f  (|iu + 1|)) > 1
f  (|iu + 1|)) > f  (2) |iu + 1| < 2
|i(u i)| < 2 |u i| < 2
u
K(0, 1) 2.
) z A(z1 )
1 = f (1) w B(z2 ) 2 = f (2).

 f

3
3
1,
,
, 2
2
  2
3
1 + 2 < AB
f (1) + f (2) < 2f
2


 ,
3
+ f (0) + f (1)
max |z w| = AB + 1 + 2 = 2f
2
x0 (0, 1) f  (x0 ) =

2f (x)(1 + f 2 (x)) = sin f (x) + x2


2|f (x)|(1 + f 2 (x)) = | sin f (x) + x2 |, 2|f (x)|(1 + f 2 (x)) | sin f (x)| + x2 |f (x)| + x2 .

|f (x)| = a 0.

2a(1 + a2 ) a + x2 a + 2a3 x2 .
a2 a + 2a3 .

2a3 + a a2 = a(2a2 a + 1) 0.

30 ( )
f : R R
f  (x) = 0 .
f  A (1, 0)
B (2, 1).

A f  R.
B f .

u.


f  2 + f  (|i u + 1|) < 0
u.

z, z1 , z2 , w
|z z1 | = f (1), |w z2 | = f (2)
|z1 z2 | = 2f 32 .
|z w|.

min |z w| = AB 1 2 = 2f ( 32 ) f (0) f (1).

25

31 ( )

x
ln (x + 1)

ex x1

>

ex 1
x

32 ( )
f : R R < < f (f () + b) +

xln(x+1)

f (f (b) + ) = f () f  (b) + f  () f (b) + f () + f (b)


f , -

x (0, +)

http://www.mathematica.gr/forum/viewtopic.php?p=83034
http://www.mathematica.gr/forum/viewtopic.php?p=83034

( ) f (t) = ln t ln(x + 1) x, x 0 , x > 0 [ln(x + 1), x]


[ln(x + 1), x]

f  () =

( )
(
).
,
.
(a, f (a)) :

f (x) f (ln(x + 1))

x ln(x + 1)

f (x) f (ln(x + 1))


1
=

x ln(x + 1)

f (x) f  (a)(x a) + f (a), x R

x = f (b) + a :

0 < ln(x + 1) < < x

f (f (b) + a) f  (a)(f (b) + a a) + f (a)

1
1
1
< <
x

ln(x + 1)

f (f (b) + a) f  (a)f (b) + f (a) : (1)

(b, f (b)) :

f (x) f (ln(x + 1))


1
1
<
<
(1)
x
x ln(x + 1)
ln(x + 1)
x ex 1 , x > 0 [x, ex 1]
(x, ex 1)

f (x) f  (b)(x b) + f (b), x R


x = f (a) + b :

f (f (a) + b) f  (b)(f (a) + b b) + f (b)


f (f (a) + b) f  (b)f (a) + f (b) : (2)

1
f (ex 1) f (x)
f (ex 1) f (x)
=
f () =
x
e 1x

ex 1 x


(1) (2) :

f (f (b)+a)+f (f (a)+b) f  (a)f (b)+f (a)+f  (b)f (a)+f (b)

x < < ex 1

ex

1
1
1
< <
1

,
: f (a) + b = b f (b) + a = a (
).
: f (a) = 0 f (b) = 0
Rolle (a, b) f  () = 0.
( )
.


[a, ] f (x) < 0 .
: a < f (a) > f () f () < 0

f (ex 1) f (x)
1
1
<
< (2)
ex 1
ex 1 x
x
(1) (2)

f (x) f (ln(x + 1))


f (ex 1) f (x)
<
x
e 1x
x ln(x + 1)

(x ln(x + 1)) ln

ex 1
x
< (ex x 1) ln
x
ln(x + 1)

26

:
f (ax2 +bx+c) = af 2 (x)+bf (x)+c, x R.
f (f (x)) = x
.

33 ( ) AB
AA1 , BB1 , 1 . BA1 + B1 + A1 =
A1 + B1 A + 1 B ,
.

http://www.mathematica.gr/forum/viewtopic.php?p=75995
http://www.mathematica.gr/forum/viewtopic.php?p=86990

(Eukleidis) ax2 + bx +
c = x (1) ax2 + (b 1)x + c = 0,
(b 1)2 4ac,
, (1) x1 , x2 .
x = x1

( ) a, b, c
B, A, AB .
:
1)
BA1 + B1 + A1 = A1 + B1 + AB1

f (ax21 + bx1 + c) = af 2 (x1 ) + bf (x1 ) + c

ccosB + acos + bcosA = bcos + acosB + ccosA


a2 + b2 c2
c2 + b2 a2
a2 + c2 b2
+a
+b
c
2ac
2ab
2cb
c2 + a2 b2
b2 + c2 a2
a2 + b2 c2
+a
+c
=b
2ab
2ac
2bc
2
2
2
2
2
2
bc(a +c b )+ac(a +b c )+ab(c2 +b2 a2 ) =
= bc(a2 + b2 c2 ) + ab(c2 + a2 b2 ) + ac(b2 + c2 a2 )
bc(c2 b2 ) + ac(a2 c2 ) + ab(b2 a2 ) = 0
... ()
(c b)(c a)(b a)(a + b + c) = 0 a = b b =
c c = a, .

f (x1 ) = af 2 (x1 ) + bf (x1 ) + c


af 2 (x1 ) + (b 1)f (x1 ) + c = 0
f (x1 ) (1). (1)
x x2 f (x2 )
(1). {x1 , x2 } = {f (x1 ), f (x2 )},

. f (xi ) = x1 , i = 1, 2
. f (x1 ) = x2 f (x2 ) = x1
)
f (f (xi )) = xi , i = 1, 2, .
)
f (f (x2 )) = f (x1 ) = x2 , ,
.

.
34 ( ) a R
b, c R 4ac < (b 1)2 f : R R

27

,
,

35 ( )
,

f (x) = [x] + [2x] +

 5x
3

36 ( )
0, 1, 2, ..., 100
. , A, 50
+ 50
( .)
a. B +
+ A
, b. a + b 2011,
.

+ [3x] + [4x],

x [0, 100].
http://www.mathematica.gr/forum/viewtopic.php?f=109&t=18361

( )

g (x) = [x] + [2x] + [3x] + [4x]



f (x) = g (x) +

http://www.mathematica.gr/forum/viewtopic.php?f=109&t=15584&p=88219#p88219

( )
. , 51
.


.
.
+.
a ,
1, 3, . . . , 99
+ A. x++
+, x+ +
x+ x . a
x++ .
b + , b
x .
2x++ + x+ + x+ = 50 50 +. 2x + x+ + x+ =
50 . x++ = x . a
b a + b .
a + b
1
.

5x
3

g ,
x R
g (x + 1) = g (x) + 10. x [0, 1), g -

1 1 1 2 3
4 3 2 3 4

0, , , , , ,

g ([0, 1))


         
1
1
2
3
1
,g
,g
,g
,g
g (0) , g
4
3
2
3
4

g ([0, 1)) = {0, 1, 2, 4, 5, 6}


, g 6 [0, 1).
f ,
x R f (x + 3) = f (x) + 35. x [0, 3),
f

3 6 9 12
g 4 , , , . , 5 5 5 5
f 3 6 + 4 = 22
[0, 3). , f 33 22 = 726 [0, 99).

28

37 ( )
ABCD . AC, BD
O .  OAB,  OBC, 
OCD,  ODA r1 , r2 , r3 , r4 E
ABCD . : r12 + r22 + r32 + r42 E. ;

http://www.mathematica.gr/forum/viewtopic.php?p=89586

( )
2r1 AB 2r2 BC 2r3 CD

2r4 DA.

4(r12 +r22 +r32 +r42 ) (AB)2 +(BC)2 +(CD)2 +(DA)2


(AB)2 + (BC)2
+
2
(BC)2 + (CD)2 (CD)2 + (DA)2 (DA)2 + (AB)2
+
+
2
2
2
2
2
2
2
= 4(r1 +r2 +r3 +r4 ) (AB)(BC)+(BC)(CD)+
(CD)(DA) + (DA)(AB), (1).
=

4(r12 + r22 + r32 + r42 )

 ZCD,  DEK

ZC
ZD
KZ
=
=
, (1) ZC = ZD .
KD
DE
DE

,  DZL, 

EBL

ZD
ZD
LD
=
=
, (2)
LE
BE
DE

 DEZ

= 4(r12 + r22 + r32 + r42 ) 2[(ABC) +


(BCD) + (CDA) + (DAB)] = 4E
(1)

DS
SZ
DS
, (3)
=
, (4)
ZD
ZD
DE

SE
=
DE

ABCD .
(3), (4), =

(DE)2
, (5)
(ZD)2

38 ( ) 
ABC, A = 90o , AB = AC, D
BC . DE AB, DZ AC
H BZ CE . DH EZ .

DE
SE ZD

=
SZ DE
ZD

(1), (2), (5), =

SE
=
SZ

KZ LD SE

= 1, (6)
KD LE SZ

(6), Ceva, DS EK ZL H

DH EZ.

http://www.mathematica.gr/forum/viewtopic.php?f=110&t=17545

1 ( ) DS  DEZ K
DZ CE L DE BZ.

2 ( ) M, ABM C
K CE M Z L BZ M E.
29

(CZ)2 , (1)

,  BM E
(M E)2 = (M B)2 + (BE)2 , (2)
(1), (2) = (M Z)2 (M E)2 = (DZ)2
(DE)2 , (3) M C = M B DZ = CZ
DE = BE .
(3) M D EZ, (4)
 ABZ,  BM E,
AZB = BEM AELZ .
, ZL M E, (5) EK
M Z, (6).
 M EZ, (4), (5), (6), M D EK ZL H
DH EZ.

 CM Z,
, (M Z)2 = (M C)2 +

30

, ,

39 ( )
f : Z Z

40 ( )
x, y, z > 0 xyz = 1, :

1
1
1
1
+
+

2
2
2
2
2
2
(x + 1) + y + 1 (y + 1) + z + 1 (z + 1) + x + 1

f (x y + f (y)) = f (x) + f (y)

x, y Z.

http://www.mathematica.gr/forum/viewtopic.php?f=111&t=17554

http://www.mathematica.gr/forum/viewtopic.php?f=111&t=18966

( )

( )
.
f . , x = y , f (f (x)) =
2f (x), f
.
y 2y f (y) = a = 0.
x = a f (a) = 0. y = a ,
f (x a) = f (x), f
, .

f f (x) = 2x x Z f 0.
:
.

(x +

1)2

1
1

2
+y +1
2xy + 2x + 2

!
cyc

1
1.
xy + x + 1

b
c
a
,y =
,z =
=

b
c
a
!
bc
1
1
=
= 1.

xy + x + 1
ab + ac + bc
xy + x + 1
cyc

31

41 ( )  ABC M



. A BC AM, B AC BM, C



AB CM E1 = (BA M ), E1 = (CA M ) E2 =





(CB M ), E2 = (B AM ) E3 = (AC M ), E3 =


(C BM ).
1
1
 +

E2 E3

http://www.mathematica.gr/forum/viewtopic.php?p=92583

( )
Ceva
,
.



 ABC AA BB CC M,


42 ( ) ABC , (1 )
O (2 ) AOC .
OQ (2 )
M, N AQ, AC , AM BN
.
M N, BQ (2 ).

1
1
1
1
+
+
=  +
E1
E2
E3
E1

Ceva,

E1 E2 +E2 E3 +E3 E1
2 E3 +E3 E1
= E1 E2 +E



E1 E2 E3
E1 E2 E3
1
1
1
1
1
1
E1 + E2 + E3 = E1 + E2 + E3

http://www.mathematica.gr/forum/viewtopic.php?p=96244

( ) X
M N BQ Y ,
 AOC, BQ
Y X.
Z AC BQ K, H
AC,
 AOC.

AB BC CA

=
A C B  A C  B




E1 E2 E3


 = 1 E1 E2 E3 = E1 E2 E3
E1 E2 E3

:


E3
MC
E3
=

 =

MC
E1 + E1
E2 + E2




E2 E3 + E2 E 3 = E1 E3 + E 1 E3 , (1)

E2
E2
M B
=

 =

MB
E3 + E3
E1 + E1



E1 E2 + E1 E2 = E2 E3 + E3 E2 , (2)


E1
E1
MA
=

 =

MA
E2 + E2
E3 + E3




E3 E1 + E1 E3 = E1 E2 + E2 E1 , (3)
(1), (2), (3)






E1 E2 + E2 E3 + E3 E1 = E1 E2 + E2 E3 + E3 E1

 AQB -

MA
BX
=
, (1)
XQ
MQ
BZ
MA
=
, (2)
AZ  M B =
MQ
BQ
 BHZ,  QY O, BH
HZ
=
= (HZ)(Y Q) = (OY )(BH) , (3)

OY
YQ

M N,

32

 OBY,  KBH BOY = BKH ,


BQ -
 ABC.
( BKH = C + KBC = C + ZBA =
BCE = BOY , E (1 ) BQ ).
BY
OY
=
= (OY )(BH) =

KH
BH
(BY )(KH) , (4)

(3), (4) = (HZ)(Y Q) = (BY )(KH) =


HZ
BZ
BY
=
=
, (5)
YQ
KH
BQ
(1), (2), (5), =

BX
BY
=
, (6)
YQ
XQ

(6) X, Y ,
BQ ( B 90o ),
( B 90o ).

33

:
r, s, t pr , ps , pt
ab, n an bn . r+s = t.
p = 2, a, b a
b mod 2 r, s, t
2r , 2s , 2t a2 b2 , n an bn
r + s = t + 1.

43 ( )
:

9x + 10y = 12z + 1

http://www.mathematica.gr/forum/viewtopic.php?p=71915

44 ( )
(n) (n)
n,

1 ( )
lift the exponent (
).
x, y z .
z > x z > y . 9x + 10y = 12 12z1 + 1 >
2 12z1 , .
: x z .
3z ||10y 1. 32 ||10 1 3z2 ||y
z 4 3z2 > 3z .
.
2
y z . 22z ||9x 1. 24 || 9 21
22z4 ||x z 4 22z4 > 2z .
. .
x = y = z = 3 (
Ramanujan)
.
2 ( ) mod5:
1 mod 5.
x .
mod16: y  4 z  2
x
9 1 mod 16 x .
y < 4 z < 2. z = 1
x
9 + 10y = 13 . y < 4.
y = 1 9x + 9 = 12z 9
27 z = 2 . y = 2
9x + 99 = 12z z = 2
. , y = 3 9x + 999 = 12z . x = 1
. x  2 27
81 . z = 3
x = 3 .


.
: p , a, b
a b mod p n .

n+1
(n)

(n)
2

;
http://www.mathematica.gr/forum/viewtopic.php?p=91457

( ) (n) = 1, n = 1
. (n) > 1: n
, (n) :

1 = d0 < d1 < d2 < ... < d (n) = n n.


i N di d (n)i = n - :

(n)
(n)

"
# (n)
#%
!
(n)
di $
di

(n)

1
(n)
i=0
i=0
"
# (n)/2
# %
(n)
$
di d (n)i

(n)

i=0

(n)
2

-
, .
n = 1. n , n = d2j , ,
-:

(n)
(n)

=
=
=
=

34

1
(n)

(n)

(n)

(n)

"
# (n)
#%
!
(n)
di $
di

(n)
i=0

i=0

(n)1
2

(n)1
2

(n)
2

dj
1

n2

= n

: n = 1 . n > 1:
n
, (n) :

.
:

n (n) = 2

n+1
(n)

(n)
2

2d0 d (n) + 2

(n)
2
(n)

2d0 d (n) + 2

n (n) (n)
+
2 (1)
(n)
(n)

1 = d0 < d1 < d2 < ... < d (n) = n

i=0

(di + d (n)i ) =
(di + d (n)i ) 2d0 2d (n) =

n (n) (n)
+
(n)
(n)

d0 d (n) + d1 d (n)1 + + dk d (n)k +


+d0 d (n) + d1 d (n)1 + + dk d (n)k =

2
(n)
+
(n) (n)
(n) 2
= 2+
(n)
2

(1)
:

(n)/2

i=1
(n)/2
&

di d (n)i

2(n) 2
2
n (n)

=2
.
(n)
(n)
(n)

(n) = n + n + n + + n =

i=1
(n)/2
&

2n + 2(n) 2 2n = 2(n) 2

n di d (n)i = n i N.

di d (n)i
i=0
(n)/2
&

2d0 d (n) + 2

(n + 1) (n) 2(n)
(n + 1)

(n)/2
&

di d (n)i

2+

i=0


x, y :

(n) 2
(n) 2
2
0 (n) 2
(n)
(n)

, n > 1
n = 1, n .
n ( n = d2j ), : dj 2 d2j 2dj .
:

xy x + y
xy x y + 1 1
x(y 1) (y 1) 1

n (n) = 2(d0 d (n) + d1 d (n)1 + + dj1 d (n)j+1

(x 1)(y 1) 1

+ dj+1 d (n)j1 + + dk d (n)k + d2j


2 2 ,
.
2,
, 1,
. ,
2,

2d0 d (n) + 2(d0 + d1 + d2 + + dj1


+ dj+1 + + d (n) ) + 2dj 2(d0 + d (n) )
= 2n + 2(n) 2n 2 = 2 2
n
.

35

45 ( IMC 2011/B3)

ln 1 +

n=1


ln 1 +

1
2n


ln 1 +

1
2n+1

. :

.
f (t) =

( ) cn = ln(1 + 1/n)
cn = c2n + c2n+1 . L
:

3L =
=
=

3cn c2n c2n+1 =

n=1

http://www.mathematica.gr/forum/viewtopic.php?f=59&t=17717

( ) x A1 An
t . ( .) E
Ai . Pr(E)
t ( [0, 1]).
I [n], ( [n] = {1, . . . , n}) EI
x iI Ai .
Pr(Ei ) = t|iI Ai | .

3(c2n + c2n+1 )c2n c2n+1

[(c2n + c2n+1 )3 (c32n + c32n+1 )]

n=1

[c3n (c32n + c32n+1 )] = c31 = (ln 2)3 .

n=1

ln(2)3
].
3


.
46 (

IMC

2011/A4)

(1)k1 t|Ai1 Ai2 ...Aik | .

f [0, 1].

n=1

k=1 1i1 <i2 <...<ik n

http://www.mathematica.gr/forum/viewtopic.php?f=59&t=17717

n
!

Pr(E) =

(1)|I|1 Pr(Ei ) = f (t)

I[n]

f (t)

A1 , A2 , ..., An , , -

[0, 1].

36

At Jv, ej  = v, J(Aej ) Aej -

47 ( )
A M3,2 (C), B M2,3 (C)

v . 1 =
(1 1 1)ei = (1 1 1)At v = (m + 1)(1 1 1)v = 0,
. .

(i, j) 2 i, j
2

.
.
((i1 , j1 ), (i2 , j1 )) i1 = i2 2
A . ,
,
n(n 1). , i, xi (xi 1)
xi .

1 1 2
AB = 0 0 2
0 0 2
det(BA).
http://www.mathematica.gr/forum/viewtopic.php?f=10&t=17629

( )
AB 0, 1, 2, : pAB () = ( 1)( 2) = 3 32 + 2.
BA
: pBA () = 23 pAB () = 2 3 + 2,
det(BA) = 2.
48 ( ) n n A = (aij ), aij {0, 1} i, j {1, 2, . . . , n},
AAt = mIn + Jn , m , Jn n n
1, In n n At A. A ,
AAt = At A.

n
!

xi (xi 1) = n(n 1).

(1)

i=1

, ((i1 , j1 ), (i1 , j2 ))
j1 = j2 2 A .
nm(m + 1)
n(n 1) j1 , j2 i1 .

http://www.mathematica.gr/forum/viewtopic.php?f=10&t=18518

m(m + 1) = n 1. (2)

( )

A ,
m + 1
2
.
At A
1.
i, j At Aei , ej  = 0. i = j A . v
At v = ei At Aei = At AAt v =
At (mv + Jv) = mei + At Jv 0 = At Aei , ej  =

(1),(2)

n
!
i=1

x2i = n2 (n 1) + n

n
!

xi

i=1

= n2 (n + m) = (n(m + 1))2
' n
(2
!
=
xi
i=1

Cauchy-Schwarz xi m + 1 ,
At A.

37

n
+O
ln n

49 ( ) , ,

k=2

n/2

2 ln2 n !
1

.
n
ln k ln(n k)

http://www.mathematica.gr/forum/viewtopic.php?f=9&t=15160

k=2

1 ( ) (an ) . ln(ln x) ,

1
1
1
=
=
ln(1+k/n)
ln(n k)
ln n 1 +
ln n





ln(1 + k/n)
1
1
1
1+O
=
+O
.
ln n
ln n
ln n
ln2 n


1
1
1
1
=
+
O

ln k ln(n k)
ln n ln k ln k
ln2 n
n/2
2 ln2 n !
1
=

n
ln k ln(n k)

ln k ln(n k) (ln(n/2))2

k=2

n3
1

ln k ln(n k)
(ln n ln 2)2

, Chebyshev,
n2
!
k=2

1
1

ln k ln(n k)
n3

'n2
(2
! 1
.
ln k

k=2

k=2

Cesaro-Stolz),
`

lim an = 1 . 

n+

k=2

n/2

ln n
2ln2 n !
1
=
+O
n
ln k ln(n k)
ln(n/2)

1/(ln x)
,

k=2

1
1
1
1

=
=
ln x
ln n 1 + ln(x/n)
ln n
ln n

1
dx.
ln x


1+O

ln(x/n)
ln n

(1 +

x)1

n2
1
dx =
+O
ln x
ln n

k=2

1
ln n


1



cos x 1
dx , a > 0 .
x2 (x2 + a)

http://www.mathematica.gr/forum/viewtopic.php?p=83560#p83560

1 ( )

50 ( )

ln(x/n)
< 0.
1 <
ln n


k=2

k=2

n+

k=2



n/2
!
n/2
n
1
=
+O
ln k
ln(n/2)
ln2 (n/2)

2 ( ) -

n
!
1
ln k

n/2

n+

n
!
1
.

ln k

n/2

! 1
2 ln n ! 1
+ O(1/n)
.
n
ln k
ln k

(ln n)2
n3
, an bn =

n (ln n ln 2)2
'n2
(2
! 1
(ln n)2
an cn =
.
n(n 3)
ln k
k=2
lim bn = lim cn = 1 (
n+


n
.
ln2 n

, x = n/2,

n2

ln2 n !
1
lim
.
n+ n
ln k ln(n k)

n2
!



ln(x/n)
dx
ln2 n

eiz 1
.
z 2 (z 2 + a)

i a
i a .
f (z) =


=
38

2
(z 2 (z 2 + a)) = 2a , (z 2 (z 2 + a)) = 0 , z = 0

eiz 1 = ieiz = 0,
z = 0 . 1 .

f (z) = 2Res (f, ai) + iRes (f, 0)

e a

1
1
.
 =
2ia
a
+ a))
(
'


i
1 e a

+ i =
f (z) dz = 2i
a
2ia a
C



1 + a + e a

z = R eit =

a a
Res (f, 0) =


[1]
ay (cos x 1) eyx dx dy =

2
1
dy+
y
2
y (y + 1)
a a

+
a a a

sin


0


0


0


0


sin


ay

1
dy =
+ 1)

y (y 2

sin ( ay) y= x
=
dy
y (y 2 + 1)

sin ( ax)
dx =
x (x + 1)


1
sin ( ax)
sin ( ax)
dx
dx =
x
a a
x+1

sin y
dy
y

y=x
ax e(x+1)t dt dx =

+
a a a

a a


0

sin (x)
dx
x




[2]
et sin ax ext dx dt =

+
a a a a a

e
0

t= x1
a
a
e 4t dt =
2t t


[3]
a

1
(1 a)
e 4t et dt =

2a
a a
t t

2 ( )

sin

(cos x 1) eyx dx dy

2
+
a a a




1
+
a
+
e
cos x 1

dx =
. 
x2 (x2 + a)
a a

cos x 1
dx = 2
x2 (x2 + a)

a a

yeyx dy dx

+
a a a

cos x 1
dx =
x2 + a


yx
(cos x 1) sin ay e
dy dx =

+
a a a


 iR cos R sin


e
e
1
it 

iRe
dt


R2 e2it (R2 e2it + a)
0

  iR cos R sin
e

e
1
it 

 R2 e2it (R2 e2it + a) iRe dt 
0
  iR cos R sin 
e
e
+1
Rdt 0 .
2 (|R2 a|)
R
0

Rei cos R sin 1
iReit dt = 0
lim
R 0 R2 e2it (R2 e2it + a)



a
 +

1
+
ix
a
+
e
e 1

dx =
.
2
2
a a
x (x + a)

R ,

(cos x 1)

a a

(x2 (x2

R (cos + i sin ) ,
 iR cos R sin
 R
eix 1
e
e
1 it
dx +
Re dt =
2
2
2
2it
2
2it
R e (R e + a)
R x (x + a)
0



1 + a + e a

a a

2
a

e a

2

a a

R a

2
cos x 1
dx
2
x
a

2
a

1
eiz 1
Res (f, 0) =
lim
=
0! z0 z (z 2 + a)
iz

e 1
i
= .
lim

2
z0 (z (z + a))
a

2
a

cos x 1
dx =
x2 (x2 + a)




(1 a)

2 e a =
.
a a
2a
a
a a

39

yx

(cos x 1) e

[1] :
0


[2] :
0

[3] :

1
1
y
=
dx = 2
.
2
y +1 y
y (y + 1)

m2

1
my
yx
e 4x e
e
dx = 2
.
x x
m

(. - ... 3
- . 350 - 355
).


m m2
sin m x eyx dx = e 4y
2y y


sin

40

xy
ax e dx =

a
.
y2 + a

:
.
.

51 ( )

A = A,  .
+ C , B

A = B

C . (
A, A  =
) B, B )+ C, C  x A y
(x B y x C y (x B y C)) ).

http://www.mathematica.gr/forum/viewtopic.php?p=91760

1 ( )
.
a, b {a, b}
. Q
. x. y
X . x X {y}
x {t X : x  t}. x
x
x, x .
(xn ) x1 = x xn = y
xn+1 = xn
{x1 , ..., xn , ...}
.
n xn = y . X = {x1 , ..., xn }
Q . :

.
2 ( ) X .
x X I(x) = {y X : y  x}. A
x X I(x)
. X , A. A ,
a. I(a) \ {a}

, a .

a
/ A,

I(a ) = I(a) \ {a} .

X .

http://www.mathematica.gr/forum/viewtopic.php?p=91864

( ) A
B = . A ,
C = . , .
.
C x y y x.
C
m C .
m, C C , m,
.
p C q p q C (
y q p.
B = A C B, C
A. x A y .
B C , .
B C .
x C y C . , x B, y C .
. , x B
)
)
y x C y (x B y C). , . x B y C ,
C x A y , x A y .
52 ( ) X
X

41

S Q
/ , . , T (S) = S

53 ( Atemlos) ,

, .

.
54 ( )
a, b, c R3 .
x + (x a)b = c.

http://www.mathematica.gr/forum/viewtopic.php?p=93446

( ) S R3 , S
S . S ( :
).

http://www.mathematica.gr/forum/viewtopic.php?p=58082

( )



a
x + (x a) b = c (x a) + (x a) a b = (a c)

p , p S = C , C K . ,
p K .
P ,
. P C A, B S .
, P S
. O
AB , . , ,
 = {M, N },  S = {M, N }.
T (S) (
O )
P .
T (S) = S , S
.
T (S) S .
Q S
Q
/ T (S). Q , Q {M, N },
Q T (S), . , Q
/ ,
Q  ,
T (S), T (S) . S ,

x a =

a c
1 + a b

x = c

a c
1 + a b

b

1 + a b = 0, 
x = m b + n c

m b + n c + (m + n (a c)) b = c
n c + n (a c) b = c, :
a c = 0 n = 1 
x = m b + c, m R

a c = 0 b, c
.

a c = 0 b, c


k b + k b a b = b b = 0 b = 0 c = 0.

a c

b
1 + a b
1 + a b = 0 a c = 0 x = m b + c, m R
1 + a b = 0 a c = 0 b, c 1 + a b = 0 x = c

( ), .
1 + a b = 0 c = 0 x = m b, m R.

42

55 ( )

an = 24n + 1, n = 1, 2, . . .
2 3.

http://www.mathematica.gr/forum/viewtopic.php?f=63&t=18715

( )

(x + 1)3 x3 = y 2

http://www.mathematica.gr/forum/viewtopic.php?f=63&t=18906

( )

p2 1
24

3x2 + 3x + 1 y 2 = 0


= 12y 2 3
.
,

p > 3.

12y 2 3 = z 2

p 1 = (p 1)(p + 1)

3|z , z = 3z1 .

p 1, p + 1 ( p = 2),

4y 2 1 = 3z12

8|(p 1)(p + 1)

, p 1, p, p + 1 3
. 3. p p = 3,
p 1, p + 1 3.
, 3|(p 1)(p + 1).
,

(2y 1)(2y + 1) = 3z12

(2y 1, 2y + 1) = 1,

) 2y 1 = 3y12 2y + 1 = y22 y1 y2 = z1
y22 3y12 = 2,
y22 2 (mod 3), 2
mod 3.
) 2y1 = y12 2y+1 = 3y22 y1 y2 = z1
2y = y12 +1, y1 , y1 = 2k+1.

..(3, 8) = 1

3 8|(p 1)(p + 1)

2y = 4k2 + 4k + 2

56 ( )

, .
( 83 73 = 512 343 = 169 = 132
13 = 22 + 32 )

y = 2k2 + 2k + 1
y = k2 + (k + 1)2

43

57 ( ) ,

sin 1 + sin 3 + sin 5 + + sin 99 =

sin2 50
sin 1

http://www.mathematica.gr/forum/viewtopic.php?p=95017

sin 1 sin 1 +
sin 1 sin 3 + sin 1 sin 5 + .... + sin 1 sin 99 = sin2 50
1
(2 sin 1 sin 1 + 2 sin 1 sin 3 +
:
2
2 sin 1 sin 5 + .... + 2 sin 1 sin 99 ) = sin2 50
1
: (1cos 2 +cos 2 cos 4 +cos 4 cos 5 +
2
.... + cos 98 cos 100 ) = sin2 50
1
: (1 cos 100 ) = sin2 50 .
2

(A1 , B1 , C1 ) (

) (


ABB A ) A1 B1


: A1 B1 =

58 ( )
3a, a E1 , E2 .
E :

E=

2
(

) E =

2 3 (3)

2
(E1 + 9E2 ).
9

( ) (

(3)2
E1 =
4

2 3
(2)
E2 =
4

E=

92 3
E1 =
(1)
4

(2)2
4

E =

42 3
E1 =
4

http://www.mathematica.gr/forum/viewtopic.php?p=93378

k :

3 +
AB + A B
=
A1 B1 = 2
2
2

2
9
2
9
2
9

(1),(2)

(E
9E2 ) = 
 1 2+

9 3
2 3
+
9
=
4
4

18 2
4 3=
(3)
2 3 = E

k2 3
4

E =

44

2
(E1 + 9E2 )
9

59 ( pla.pa.s )

2 ( )

de l Hospital
. ;
f (x)
g(x)

f  (x)
g  (x)

(=1). l Hospital

1 + cos x
(x + sin x)
=
.

x
1

3( )
: f (n) = 1 n
n + 2 , f (n) = 0.


x0 ( ),
( lim

xx0

f (x)
) g(x)

g(n) =

h(n) =

f  (x)
.
g (x)

1
. 1
1 + g(n)

0.
h(n) . 0
,

,
;

1

k+1

k .

h(n) 0.
.
:
Goedel.

0 .

http://www.mathematica.gr/forum/viewtopic.php?p=89706

1 ( )

f (x) = 2x + sin(x)

g (x) = 3x + sin (x)


x +.

x x0 = 0 f x12 , g x12 .
:

.

60 ( ) f :
R R .
R ;

f (x) = 2x5 5x4 + 5 x [2, +)

http://www.mathematica.gr/forum/viewtopic.php?p=94877

( ) g1 : (, 0) R, g2 :
[0, +) R R (). -


[11, +).
. g
. lim g (u)

( ). ..

g1 (x) = ln(x) g2 .
*

u0

g1 (x)
f (x) g2 (x)
*
f (x) g1 (x)
G(x) =
g2 (x)
f
( ()).

f .
( )

. lim g (u) .

f (k). g(n)

(k, k + 2) k  n. -

f, g ( x0 )
xx0

n
!
k=1

de l Hospital,

lim

x + sin x

x

F (x) =

u0

45

, x<0
, x0
, x<0
, x0
= F + G F, G

61 ( ) [. , ,
( , 1976)]
P K
E Z
.
http://www.mathematica.gr/forum/viewtopic.php?p=94010

1 ( )

:
,
, L, C, T.
LC, CT.
G, R
: GC = CR, GCR = 2 .
:
F LG, T R
LT ,

CT 4 , GCRF
.


A L
EP 4 , B T KZ 4 ,
I P K
N .

(IA = IB, IAIB) (N G = N R, N GN R) ,
. [ (P IK) P IK = 2 IB =
2N R = 2N G = IA.]


: K, L 
A, A .
B, C K, L , A BC BC = 2h, h
. ,
, KLD KD = h.

2 ( )
L, T , EP ,
ZK 450 , *. A, B E  Z  ,
E  , Z  E , Z L,
T , . ( C , D ,
46

2BC + AD
MG
MG
(*)
=
= ,
GN
2AD + BC
GN
2B +
=
> 0 , B
2 + B

BZ , AP AE , BK , :
ABC = Z  BZ BAD = E  AE ,
ZBK = CBD EAP = DAC
450 , )
*
, 2 2 =
4 ,
,

.
, B , M, N G

62 ( ) .

2B +
.
2 + B

1
N M
+1

A = (1, 0), B = (4, 0), C = (4, 4),


D = (1, 1):
  M, N AD, BC

http://www.mathematica.gr/forum/viewtopic.php?p=94331

1 ( )
P , Q, R, S ABD , BCD , ABC , ACD
G ABCD ( AD//BC )
P Q, RS .
: G
|GP |
(ABD)
|AD|
|GR|
(ABC)
|BC|
|GQ| = (BCD) = |BC| |GS| = (ACD) = |AD| ,

.
A = (1, 0), B = (4, 0), C = (4, 4),
D = (1, 1):
P : 23 ( 52 , 0) + 13 (1, 1) = (2, 13 )
Q: 23 ( 52 , 52 ) + 13 (4, 0) = (3, 53 )
R: 23 ( 52 , 0) + 13 (4, 4) = (3, 43 )
S : 23 (1, 12 ) + 13 (4, 4) = (2, 53 )
7
P Q: y = 4x
3 3
RS : y = x3 + 73
7
G: ( 14
5 , 5)
2 (parmenides51) [, 15]
2 2
. (1) ABCD AD//BC
(AD ) M, N ,
E
BA, CD. AED, ABC
M N EM, M N .
(1) M N . (2) BD (z), (w)
(z) AD
DB . P, Q ABD, BCD
(z), BM (w), DN . (1)
P Q.
(3) (2), (3) G
ABCD P Q, M N
.
-

M G = GN N G =

1
, (4, 2) .

2
B = BC = 4, = AD = 1
24+1
9
3
2B +
=
= = .
=
2 + B
21+4
6
2
1,

NG =
(x 4, y 2) =
(x 4, y 2) =
(x, y) =
(x, y) =

1
NM
+1 

1
1
1 4, 2
3
2
2 +1


3
2
3,

5
2


3
6

4 ,2
5
5


14 7
,
5 5

3 (parmenides51) 1:

.
2:
( )


.
ABCD AD//BC (AD
) , E BA, CD V, G, W
EAD, ABCD, EBC .
V, W (
). E, V, G, W (1) EAD, EBC E . E ( ) EBC T
ET = EW -

KG
BC
=
AD
GL

47

 

 2
8 2
4
4 5
(EW ) =
+
=
3
3
3

V E .
(2) (EAD)(EV ) + (ABCD)(EG) = (EBC)(ET )
.
(EAD)(EV ) + (ABCD)(EG) = (EBC)(EW )

EG EG//EV ,
G, ABCD.

16
(EB)(BC)
=
= 8 EB BC .
2
2
ABCD G

(EBC) =

A = (1, 0), B = (4, 0), C = (4, 4),


D =*(1, 1):
*
A = (1, 0)
C = (4, 4)
AB : y = 0,

B = (4, 0)
D = (1, 1)
*
AB : y = 0,
E(0, 0)
CD : y = x ,
CD : y = x
EAD


V

0+1+1 0+1+0
,
3
3


=

(ABCD) =

EB BC ( ).

(EAD)(EV )
+ (ABCD)(EG) =
(ECD)(EW )
4 5
7 5
1 5 15
+ (EG) = 8
(EG) =

2
3
2
3
5
*
E = (0, 0)


EV : x = 2y V = 23 , 13
E, V, G  G(x, y) = (2y,
y)
2
2
2
2
(2y) + y = |y| 5
(EG) = x + y =

7 5
7
= |y| 5 y = y > 0
(EG) =
5
5


14 7
,
G(x, y) =
5 5


2 1
,
,
3 3

 
 2
1
5
2 2
(EV ) =
+
=
3
3
3

1
(EA)(AD)
= EA AD .
2
2
EBC

(EAD) =

W

0+4+4 0+4+0
,
3
3


=

15
(AD + BC)(AB)
=
2
2

(
) .


8 4
,
,
3 3

48

:
63 ( )
P (x) . P (1) = 6 P (7) = 3438,
x = 3.

A(0) .
B(7) = 70 A(0) = 1 (4)

B(1) = 4.

: C(x)

http://www.mathematica.gr/forum/viewtopic.php?f=60&t=15830

1 ( )
4 74 < 3438 < 75 .

B(x) = xC(x) + B(0)

P (x) = ax4 + bx3 + cx2 + dx + e

B(7) = 7C(7) + B(0) (5)

a, b, c, d, e ,

P (1) = 6 a + b + c + d + e = 6

B(1) = C(1) + B(0) (6)


B(0)
B(1), 7. (5)
B(7) : 7. C(7)
B(0) .
C(7) = 10 B(0) = 0 (6)
C(1) = 4. C(x)


, P (7) = 3438
: a = 1, b = 3, c = 0, d = 1, e = 1,
P (x) = x4 + 3x3 + x + 1 P (3) = 166.
2 ( )
[0, +)
. A(x)

4 = C(1) < C(2) < C(3) < C(4) < C(5) < C(6) < C(7) = 10
, C(3) = 6.

B(3) = 3C(3) + B(0) = 18

P (x) = xA(x) + P (0)

A(3) = 3B(3) + A(0) = 55

P (3) = 3A(3) + P (0) = 166

P (7) = 7A(7) + P (0) (1)

64 ( )

P (1) = A(1) + P (0) (2)


P (0)
P (1), 7. (1)
P (7) : 7. A(7)
P (0) .
A(7) = 491 P (0) = 1 (2)

P (x) = xn + a1 xn1 + + an

Q(x) = xn + b1 xn1 + + bn

C.
P x1 , x2 , ..., xn ,
Q x21 , x22 , ..., x2n . a1 + a3 + a5 + R
a2 + a4 + a6 + R,
,

A(1) = 5.
B(x)
A(x) = xB(x) + A(0).

b1 + b2 + bn R.

A(7) = 7B(7) + A(0) (3)

http://www.mathematica.gr/forum/viewtopic.php?f=60&t=18478

A(1) = B(1) + A(0) (4)

( ) :

A(0)
A(1), 7. (3)
A(7) : 7. B(7)

P (x) =

n
%
i=1

49

(x xi )

Q (x) =

n
%

x xi 2

n
%

i=1

P (1) = a1 + a2 + + an R

(1 xi ) =

n
%

i=1

i=1

n
%

n
%

(1 + xi ) =

i=1

(1 x
i )

(1 + x
i ).

i=1

P (1) = a1 a2 + + (1)n an R.

n
%

i=1

, :

n
%


1 x2i =
1x
2i ,
i=1

P (1) = P (1)

Q (1) = Q (1)

P (1) = P (1),

b1 + b2 + + bn R

50

http://www.mathematica.gr.
http://www.mathematica.gr.
LaTEX: , , , , , ,
, : , : ,
, , : . LaTEX.
.

Leonardo da Vinci
(32-) . 30
.
- - quasiregular ,
( - ).

(0, 0, ), 21 , 2 ,

1+ 5
2

1+
2

:http://en.wikipedia.org/wiki/Icosidodecahedron , :

http://www.mathematica.gr
.

http://www.mathematica.gr

12. (R BORIS)

13. ( )

1. (Mihalis_Lambrou)

14. ( )

2. (nsmavrogiannis)
3. ( )

15. (dement)

4. ( )

16. (swsto)

5.

(m.papagrigorakis)

17. (achilleas)

6. ( )

18. ( )

7. ()

19. (xr.tsif)


1. (grigkost)
2. (cretanman)

20. (Demetres)

1. (spyros)
2. (p_gianno)

1. (stranton)
2. ( )
3. (vittasko)
4. ()
5. (s.kap)

3. (kostas.zig)
4. (exdx)
5. ( )
6. (mathnder)

6. (nkatsipis)

7. (mathematica)

7. ( )

8. ( )

8. (chris_gatos)

9. (rek2)

9. (matha)

10. (hsiodos)

10. (mathxl)

11. ( )

11. (gbaloglou)

12. (bilstef)

7 ( )

a(b + c) c(a + b)
+
= a + c,
a+b
b+c
B.
a, b, c ABC

1 ( Atemlos) .
.
8 . 6 4740
3 4695.
;

8 ( )

:


1
5
+

6
18


2
2) B = 1 + 1 +
5

1) A =

2 ( ) :
,
.

1
5

6
18


2
2
2
1+ 1+
6
7
58

9 ( KARKAR) x, y,
:

x1+

y1= x+

10 ( ) x, y, z x + y + z = 0
:

3 ( )
1618 , 1816 .

x3 y3
z3
x2 + y2 y2 + z2 z2 + x2
+
+
=
+
+ .
x+y
y+z
z+x
yz zx xy

4 ( ) 1, 2, 3, 4, . . . , :
1
2
3 4
5
6
7
8
9
10 11 12 13 14 15 16

11 ( KARKAR )
 = 90 ), E
ABC ( A
 D AB

AD . D AC,
CE, BE S , T . AES T .

()
20 , ;
() 2011;

5 ( KARKAR)
a.

x a.
6 ( )
:
() A = 3183 B = 2307 2306 2305 .
() =

320
518
= 42 .
30
2
2
12 ( KARKAR ABC
. = 150 , A MC
 = 135 ,
AM . C

A BC .

13 ( )

17 ( KARKAR) BC ABC ,
: M BC , N
AM L BN . AL
BC S .

111 (x) + 111 (x) = 1 = (x) + (x) = 1.


( :
)

18 ( )
b ,

a
= 5

14 ( )

y1

, 2 x1
3
3
2
P(x)
=
3
x
+
ax
+
bx

6,

(x
1 , y1 )

y = 3 + 6
:
x + log y = 3 .

a, b

1
BC.
6

BS =

: 3

b
a


b =

5
b + 3
a


a.

a
b.

19 ( )

f (x) = xex+ ,

x R,

> 0.

()

15 ( )
 ABC (O, R) H .
K AB : AK = AH L AC : AL = AO : KL = R.




f
2

= 0.

()
C f M (0, f (0)) 45 xx .
() f .
() f
.
20 ( )

f (x) = x2 + (3 )x ( + 5),

x, R.

f
;
,
,

21 ( ) z1 , z2 , z3 , z4 C, z1 , z2 0

16 ( )  ABC
AD, BE, CZ . H A , HB , HC
 ZAE,  DBZ,  ECD ,
 HA HB HC = DEZ .

z1
C \ R.
z2

z1 + z3 + z4 z2 + z3 + z4
,
.
z2
z1
z1 + z2 + z3 + z4 = 0.

,
,

22 ( 77)

z, w 60 .

|z + w| 3 |z w|.
,
, ,

29 ( BAGGP93)
f : [0, +) R f 2 (0) = 1
f 2 (x)  6 x x [0, +).

23 ( )
f f ( f (x)) = x x R,
f () = .

i) f 2 (x) 6 x =

: f (2008) = 1 , f (2009) = 10
f ( f (x))
f ( f (x) + 3)
=
, x R :

f ( f (x) + 2)
f ( f (x) + 1)
) f (2) f (3) = f (4) f (5).

) f 1 1.

e1

x R.

31 ( )
f : [0, 1] R
 x 1  x
+ f
= 2 x x [0, 1].
f (x) f
2
4 4
f .


x R .

32 ( ) : f : R+ R g : R R ,
: f (0) = g(0) = 0 f  (x) + g( f (x)) = 0, x 0.
: f (x) = 0, x 0.

27 ( -
) f : R R

f (t)dt
0

f (x)
1
dx = (b a).
f (x) f  (x)
2

,
,

,
,

33 ( )

f (t)dt = 2 x2 + 2 x + 1 x R.

lim

x+

28 ( -
) f : [0, +) R


x

f (t)dt < f (x),

x2 ln x x
.
(ln x)x + x

34 ( )
f : R R
f (0) = 0 f (x + y) f (x) + f (y), x, y R.

x [0, +).

f (x) > 0,

3. f (b) f  (b) = f (a) f  (a).



f  (x) ln( f 2 (x) + x2 ) + x4 =

f 2 (x)

tdt
1

b

e1

2. f (x) > f  (x), x [a, b]

25 ( )
f : R R
f  (x) 0, x R. :


f  (x) ( f 2 (x) + x2 )



f 4 (x) 12 x f 2 (x) dx + 72

1. f  (x) = f (x), x [a, b]

,
,

30 ( )
f [a, b],
:

3 f (x0 ) = f (x1 ) + 2 f (x2 ).

Pla.pa.s)

x [0, +) .

) f [2008, 2009] x1 , x2
[2008, 2009], x0
[2008, 2009] :

x+1

ii)

) [2, 3] : f 2 () = f (2) f (3).

26 (

f : R R

2

x [0, +) .

24 ( Chris) f
R f (x)  0, x R.

f (x + f  (x)) f (x),

216 x3 2 2 f  (x) f (x) 6 = f 6 (x) 18 x f 4 (x) + 108 x f (x) ,

x [0, +).


Juniors, -
-

42 ( )  ABC
(I) .
AI, BI, CI I, (I)
S , M, N, P, .
AM, BN, CP T.

35 ( )

x(x + 1) = pq(x y),

p, q , . 2
(x, y) N , y
.

43 ( )
M

36 ( )
f : N N

(a + bc)(b + ac)(c + ab)

f (x)y
f (x) + y
2(x + y)
+
=
, x, y N
x + f (y) x f (y)
f (x + y)

45 ( ) f : [0, 1]

xn

= xn +

x1

46 ( )
xn 1 n

n Z[x].

47 ( )

G a, b a3 b = ba2 a2 b = ba3 .
a5 = e, e G.
48 ( ) H C 2 = x 2
x. : C2 Aut(H) (x)(h) = h1 h H . ( H
, ).
G := HC2 . G
h2 = 1 h H .

40 ( )
x1 , x2 , . . . , xn

= = xn1 +

Mabc

[0, ) .
 1
 1
1
x2 f (x) dx
f (x) dx.

39 ( )
2011 2011 .
:
2 2
, , .


.


Seniors, -
-

x3

() 2010 .
1005
1005 .

= x2 +

() 100 .
, 50 .

38 ( )
ABCD
: E A DE.
Z B AZ .
H C BH K D CK. (EZHK) = 5(ABCD).

44 ( )

37 ( )
 ABC AB = AC K, L
A, B, AK = KC CL ACK
BK = 2 KL.  ABC.

x2

a, b, c
a + b + c = 1.


Juniors,

x1 +

=2


Seniors,

49 ( ) xn
x0 , x1

41 ( )
 ABC . BD AB
BC K L .
K L X . BX
AC .

xn =

(n 1) c
1
xn1 +
xn2 , c > 0 .
1 + (n 1) c
1 + (n 1) c

lim xn .
n+

50 ( ) f : R2 R

58 ( KARKAR) S T

s, (s <

), AB,
2
OAB. , S D, T E,
OA, S Z, T H , OB.
S T ,
. : E1 + E2 + 2 E3 , .
( E1 , E2 , E3 )

| f (x) f (y)| d(x, y) , x, y R2 ,


d R2 .


51 ( ) X P() :

(B  C) X = (B X)  (C X)
B, C ( ).
52 ( ) X P() :

A (X B) = (A X) B
A, B ( ).

53 ( )
N 169 | 33n+3 26n 27.
54 ( ) a b
. a b a + b
2 q r
,

a b = q(a + b) + r 0 r < a + b.
(a, b)

q2 + r = 2011.

59 ( ) z5 = 1,

55 ( )
f : R R f (0) = 0
x R f (2 x) x + f (x) f (3 x) 2 x + f (x).
56 ( )
f : [0, 1] R | f (x) f (y)| |x y|,
x, y [0, 1].

 1 
0



1
1
f (t)dt dx +

12

A=

z
z2
z3
z4
+
+
,
+
2
4
1+z
1+z
1 + z 1 + z3

B=

z
z2
z3
z4
+
+
.
+
1 z2
1 z4 1 z 1 z3

2 , z  1.

60 ( ) x1 , x2 , ..., xn , ,

f (x)dx.
0

nx +

n

k=1

xk =

n

k=1

x2k
xk x

0 n 1

57 ( KARKAR) ABCD, K , CB, DA,


S.
M, N AB, CD : CD = 2 AB,
3
: MN = S K .
4


()

61 ( )
, (x, y)
|x| |y| 1 |y| 1. .
62 ( )

x4 3 x3 + 5 x2 + ax 2 = 0, a R,
.

:
(x 2)15 = 4740 x = 318. : 15318 = 4770

1 ( Atemlos) .

.
8
. 6 4740
3 4695.
;

2 ( )

:
, .

http://www.mathematica.gr/forum/viewtopic.php?p96480
http://www.mathematica.gr/forum/viewtopic.php?p93279

( ) abcd
1000a + 100b + 10c + d = k2
1000(a + 1) + 100(b + 1) + 10(c + 1) + (d + 1) = l2 .

k2 + 1111 = l2 (l k)(l + k) = 11 101


lk = 11 l+k = 101 k = 45
2025.

( (KARKAR) ) x, y .
:

(x 2)y = 4740 (x 5)y = 4695.


:

3y = 45 y = 15 :

:
3 ( )
1618 , 1816 .

4 ( )
1, 2, 3, 4, . . . ,
:
1
2 3 4
5 6 7 8 9
10 11 12 13 14 15 16

http://www.mathematica.gr/forum/viewtopic.php?f33&t21069

1 ( )
1618 = (24 )18 = 272
1816 = (32 2)16 = 332 216 .

()
20 ,
;

216
256 = (27 )8 (34 )8 ,

() 2011;

27 = 128 34 = 81.
.

http://www.mathematica.gr/forum/viewtopic.php?f33&t20237

2 ( )
,
( )
:

18

16

1618

182

162

8
9 =

1
324
= 8
256
2

3248
2569

( )
() 12 = 1, 22 = 4,
32 = 9, . . . , 19
192 = 361. , 20
361 + 1 = 362, 212 1 = 440.

1
3248

256 2568

324
=
512

() 442 = 1936 452 = 2025, 2011


45 (14
74 ).

<1

: 6
22 21.

:
5 ( KARKAR)
a.

x a.

( )
()

A = 3183 = (33 )61 = 2761

http://www.mathematica.gr/forum/viewtopic.php?f34&t20737

B = 2307 2306 2305

( ).
a 2a.

a2 + (2a)2 = 2

= 5a.
k
.

k
2

= 2305 (22 2 1)
= 2305
= (25 )61
= 3261

= a
2

>A

2 5a
.

5
2 5a

5a = 2a2 ,
5
a
. x2 + k2 = a2 , x =
5.

k=

()

9
> 1,
8

10

(32 )10 910


320
9
= 30 = 3 10 = 10 =
8
2
(2 )
8

6 ( ) :

0 <

() A = 3183 B = 2307 2306 2305 .

125
< 1,
128

(53 )6 1256
518
125
= 42 = 7 6 =
=
6
128
2
(2 )
128

320
518
() = 30 = 42 .
2
2

, > .
http://www.mathematica.gr/forum/viewtopic.php?f34&t20539

> 1,

6
< 1.

:
7 ( ) a, b, c ABC

1 (angvl () ) 1:
.

a(b + c) c(a + b)
+
= a + c, a+b
b+c
B.

1
5
+
=
6
18

http://www.mathematica.gr/forum/viewtopic.php?p76733

( (Eagle) ) (, , , ) :


B =

.
8 ( )

 :

1) A =
2) B =

2
1+
5

2
1+
6

1
5

6
18

2
1+
7


1+

18

5+1
.
6

2 ( () )
2:

(a c)2 = 0 a = c.

1
5
+

6
18

51
.
6

1
.
3

2abc = a b + c b a 2ac + c = 0
2

a2 b + abc + ab2 + cb2 + ca2 + ac2 + abc + c2 b


2

2

5+1

3+ 5
18

ab2 + 2abc + ac2 + ca2 + 2abc + cb2


2



a(b + c) c(a + b)
+
=a+c
a+b
b+c
a(b + c)2 + c(a + b)2 = (a + c)(a + b)(b + c)
=

3
5
+
=
18
18

2
58




2
1+
5
7

2
1+
6




2
1+
7

10
...
8

1+
59

57

2
58

60
58

7 8 9 10
59 60

...

5 6 7 8
57 58

 

59 60
59

=
10 = 118.
5
6
5

http://www.mathematica.gr/forum/viewtopic.php?p91685

:
9 ( KARKAR)

 x1+ y1 =
x, y, :

x+

a = 0
2

b = 2 0

x1 =
2

y 12 = 22

http://www.mathematica.gr/forum/viewtopic.php?f19&t20443

1 ( ) x 1
y 1

(x 1)

10 ( ) x, y, z

x + y + z = 0 :

x3 y3 z3
x2 + y2 y2 + z2 z2 + x2
+
+
=
+
+ .
x+y
y+z
z+x
yz zx xy

1
y1
( x 1 )2 + (
1)2 = 0,
2


, x =

y1 
1
x1+ +(
y 1 + 1 ) = 0,
4

x1=a =

y 1 = b = 2

1
5

1
=
x

x = 1

4
4

y 1 = 4
y = 5 1

x1=

5
y = 5.
4

1
y 1 = 2.
2

http://www.mathematica.gr/forum/viewtopic.php?f19&t12860

1 ( )

2 (parmenides51)

x 1 = a2
x1=a 0

y 1 = b2
y1=b0

x = a2 + 1 (1)
,

y = b2 + 1 (2)

x2 + y2 y2 + z2 z2 + x2
+
+
=
z
x
y

x3 y + xy3 + y3 z + yz3 + z3 x + zx3


=
xyz
x3 (y + z) + y3 (x + z) + z3 (x + y)
=
xyz

x 1 y 1 (1),(2)

a + b = a2 + 1 +

b2 + 1

x3 y3 z3
x4 y4 z4
=
+
+
xyz
yz xz xy
2 ( )

4
4a + 4b = 4a2 + 4 + b2 + 1

x2 + y2 y2 + z2 z2 + x2
+
+
=
x+y
y+z
z+x

2
x + y2 y2 + z2 z2 + x2
+
+
=

z
x
y

2
x2 y2 y2 z2 z2
x
+
+
+
+ +
=

z
y
z
x
x
y

2
x (y + z) y2 (z + x) z2 (x + y)
+
+
=

yz
xz
xy

4a2 4a + 1 + b2 4b 4 = 0

(2a 1)2 + (b 2)2 = 0,

2a 1 = 0

b 2 = 0
,
10

x3 y3 z3
+
+
yz xz xy

B=
3 (dr.tasos) :

(x2 + y2 + z2 )2 2(x2 y2 + z2 y2 + x2 z2 )
.
xyz

(x + y)2 2 xy (z + y)2 2zy


A=
+
x+y
z+y
(x + z)2 2zx

+
z+x

(x2 + y2 + z2 )2 = 4(x2 y2 + z2 y2 + x2 z2 ).


xy yz zx
+ +

A=2
z
x
y

(x2 + y2 + z2 )2 = 4(x2 y2 + z2 y2 + x2 z2 )

(x2 + y2 + z2 )2 = 4(x2 y2 + z2 y2 + x2 z2 ),


A=2

x y +z y +x z
xyz
2 2

2 2

2 2

+ 8 xyz(x + y + z)
.

11

:
 + C AD
 = 45 x + 2 x = 45 + x = C AT
 ,
.
DAT
 C AT
 .
ACE,
CE AT .
BE S A. E
AE S T .

11 ( KARKAR )
 = 90 ), E
ABC ( A
AD .
 D AB
 CE, BE
DAC,
S , T . AES T .

12 ( KARKAR ABC
 =
. = 150 , A MC
AM . C

135 , A
BC .

http://www.mathematica.gr/forum/viewtopic.php?f20&t18759


(KARKAR) C 
BA ACB

2 x 90 2 x .
E AS T .

http://www.mathematica.gr/forum/viewtopic.php?f20&t21008

1 ( ) BDAC
 = 30
C
MDC (120 , 30 , 30 ), MBD
BAD. 60 + 45 = 105 .

 = 45 x.
CE ACE
 = ACD
 , AD .
 =
DAB
D AT



45 x. C AD = D BA, AD
12

AKC AC = 2AK .
B B K .

BC = 2KB + 2BM = 2K M = 2AK = AC.
 = 75 . ABC , B
= 105o .
A BC

2 ( ) AK . AK M .
AK = K M .

13

:




2 x 109 x 1 = 2 x 1 109 x

13 ( )

(x) + (x) = 1 = (x) + (x) = 1.


111

111

|x| 1 109 x 1


109 x 1 0 2 x 109 x 1 0.

( :
)

http://www.mathematica.gr/forum/viewtopic.php?p59915

|x| 1 109 x 1


1 109 x 0 2 x 1 109 x 0.

1 ( ) a = x, b = x.

a111 + b111 = 1

(1).

(1) 0 a 1 0 b 1.
1 b < 0

(1) a111 = 1 b111 > 1 .

14 ( ) a, b

a  1 b  1 : a (2)
b111 b2 (3) (2), (3)
a = 0 b = 0 .
a = b = 0
(2), (3):
111




2 x 109 x 1 = 2 x 1 109 x = 0
x = 0 , x = 1

x + x = 1
x = 1 , x = 0

1 ( )
y > 0.
1 : y 6 = 3x
log3 (y 6) = x, y > 6,
2 :

(a + b)2 = 1
a2 + b2 + 2ab = 1

log3 (y 6) + log3 y = 3 log3 (y2 y) = 3

ab = 0

y2 6y = 27 y = 9

(a = 0 b = 1) (b = 0 a = 1).

( y = 3 < 6 ).
x = 1, x1 =
1, y1 = 9.
2 x1 =

(1).
2 ( )

2,

x + x = 1
111

x + x = x + x
111

, 2 x1

http://www.mathematica.gr/forum/viewtopic.php?p104757

a = 1 ( b = 0) b = 1 ( a = 0).
a + b = 1.
.
a + b = 1

111

2
P(x) = 3 x3 + ax
+ bx 6, (x1 , y1 )

y = 3x + 6
:
.

x + log3 y = 3

a111 + b111 < a2 + b2 1 < 1, .

111

y1

y1

3

= 3 ,

24 + 4a 2b 6 = 0
P(2) = 0

...
P(3) = 0
81 + 9a + 3b 6 = 0

a = 2
b = 19

111 x 2 x = 2 x 111 x
14

:
15 ( )  ABC (O, R)
H .
K AB : AK = AH
L AC : AL = AO : KL = R.

Z H AC ,
AOZ, AKL , AO = AL, AK =
AH = AZ OAB = LAH, LAZ = LAH OAZ =
KAL. , KL = OZ = R.

http://www.mathematica.gr/forum/viewtopic.php?f22&t21153

16 ( )  ABC AD, BE, CZ .


HA , HB , HC  ZAE,  DBZ,  ECD ,
 HA HB HC = DEZ .

1 ( ) T

AK F BC. OFC =90 , OF =


=
2
, FOC =, OC = AL  AT L =  OFC
LT AT . LT AK , AL = LK = R.

2 ( )
,

,


(OAC = BAH ):

http://www.mathematica.gr/forum/viewtopic.php?p107737

( )
.
15

Ha EDHb
. : Ha E = 2R A cos E (1)
= AHZ
.
: 2RA = AH E = AEZ
DHb = 2RB cos D (2) : 2RB = BH
= BDZ
= BHZ

D
Ha = Hb (3).
(1) (2) (3)
=
: 2RA cos E = 2RB cos D AH cos(AHZ)


BH cos(BHZ)
HZ = HZ. .
(3).

: Ha //Hb AB.
Ha EDHb
DE = Ha Hb .

.
.

16

:
17 ( KARKAR)
BC ABC ,
: M
BC , N AM L BN . AL
BC S .

BS =

BN M
S L A. :

BS MA 
NL

=1
S M AN 
LB
AN
BS
=

S M AM
BS
AN
=

BS + S M AN + AM
AM
1
BS
=
= AM 2
BM
3
+ AM
2
1
BS
= .
BM 3

1
BC.
6

http://www.mathematica.gr/forum/viewtopic.php?p98514

1 ( ) K S M
BS = S K. BNK LS  NK.
AS M K S M ,
BS = S K =
N AM KN  AS .

KM =

BM
3

BC
6

S BM.
B S
BC .

5 ( ) KN//BM .
: BLS = NLK KN = BS (1). :
1
KN = S M (K, N S AM) (2) :
2
(1), (2) BS = 21 S M BM = 3 BS BC = 6 BS .

2 ( )
B(0, 0), C(a,
0), A(p, q) a, p, q 
a 
2p + a q
2p + a q
0. M , 0 , N
, , L
, .
2
4
2
8
4
q 4q
6q
a  6 p AL =
=
,
2 p+a
6p a
p 8
6q
(x p). S (s, 0)
(AL) : y q =
6p a
0q=

6q
6p a

(s p) s =

6 pq
6 pa

6q
6 pa

aq
6q

a
6

6 ( ) ML
AB K .
BS MN AK

=1

. Ceva ABM :
SM NA KB
KB
KB
BS
BS
(1).

=
=
SM
AK
BM
AB
N K M
AB T .

1
BS = BC .
6
a = 6 p, AS ,
.

AN
AT
=

TK
NM
BL
BK
=
BK =
AT = T K KL//T N
KT
LN
KT .
1
BK = AB, (1)
. T N//K M

= xBC

= x(
b
a), (1)
3 ( ) BS
1

+ yAS

, y  1, (2)

= BL

+ LS

= BN
BS
2

a), BM

= 1 BC

=
a+ BS

= 1 (
b
a), AS

= 1 (BM

BN
2
2
2
1

b, (3)

= 8y 3
a +
(2) BS
8(1 y)
8(1 y)
1

= 1 BC

.
(1), (3) = x = BS
6
6

3
1
1
BS = BM BS = BC .
3
6

7 ( ) CB
|BD| = |BM|, AB
ADM , DN .
P AB DN ADM , MP AD

4 ( )
17

(3 p2 3 p = 5 p + 3 3 p2 3 p = 5 p 3) p = 3
1
p = . p > 0,
b = 3
a,
3

a
b = 15.

, Q. BN//DA, MP
BN , L.
AS , BN , MP ABM , Ceva
|BS |
1
1
|NA| |PB|
=

= 1 = ( N AM
|S M| |MN| |AP|
2
2
|BS |
1
= .
P ADM )
|BM| 3
18 (


)



a

a
= 5 -

2 (
) x =
a
b,

x 5
.

=
5

a,
b ,

x 5
x
5

a
b

= 5
5,

a
b

=
5
5


a,
b :

a
b

b
, :

b
a


b =

5
b + 3
a


a.
a
b.

a
b

5
b + 3
a

= 5(x2 + 6 x + 9),

5
b + 3
a

= 5|x + 3|.

a
b

= 5 |5 x| .

http://www.mathematica.gr/forum/viewtopic.php?p103950

a, 3 |5 x| x = 25 |x + 3|,
x = 15.

1 ( ) 3

b
a


b =

5
b + 3
a


a
a,
b ,

b = p
a, p > 0. :

b
a


b =

5
b + 3
a


a
3|p 1||
a|p
a = |5 p + 3||
a|
a
|3 p2 3 p| = |5 p + 3|,

18

:
() f  (0) = e . f x0 = 0
x x 45 .

19 ( )

f (x) = xex+ ,

x R,

> 0.

()

f 

f

f  (0) = 45 e = 1 = 0,

2 f

> 0.

= 0.

() f  (x) = ex+ + xex+ x R, > 0

() C f M (0, f (0))
45 xx .

f  (x) = 0 ex+ + xex+ = 0


ex+ (1 + x) = 0
1 + x = 0

()
f.

x=

() f
.

f  (x) > 0 ex+ + xex+ > 0


ex+ (1 + x) > 0

http://www.mathematica.gr/forum/viewtopic.php?f18&t20955&start20

1 + x > 0

( )

x> .

() f (x) = xex+ , x R, > 0,


f ( 1 ) = 1 e1+ > 0. f
R
f  (x) = ex+ + xex+ x R, > 0

f
(, 1 ] [ 1 , +).
f x0 = 1
f ( 1 ) = 1 e1 .

f  ( ) = 2e1+ > 0.

() g() = 1 e1 , > 0.
g (0, +)

f  R

g () =

f  (x) = ex+ + ex+ + 2 xex+


x+

= 2e

x+

+ xe
2

x R, > 0

e1 =

e1 (

), > 0.

3e1+ = 0
19

e1

e1 = 0

1
1
)=0
e1 (

1
=0

=1

1
3e1+ 2e1+ 2 e1+ =
3e

e1

g () = 0

f  ( ) f  ( ) 2 f ( ) =

1+

f  ( 1 ) = 3e1+ > 0.
1

g () > 0

e1 > 0

1
1
)>0
e1 (

1
>0

<1
2

e1

> 0 (3 )2 + 4( + 5) > 0

9 6 + 2 + 4 + 20 > 0
2 2 + 29 > 0.
 = 112 < 0 R f . x1 , x2 .
: S = 3 P = ( + 5)

g (0, 1]
[1, +).
= 1 g(1) = 1.
f
= 1.

x21 + x22 = (x1 + x2 )2 2 x1 x2


= S 2 2P = ( 3)2 + 2( + 5)
= 2 4 + 19.
g() = 2 4+ 19 R.
g R
g () = 2 4 R.

20 ( )

f (x) = x2 + (3 )x ( + 5),

x, R.

g () = 0 2 4 = 0 = 2


f ;

g () > 0 2 4 > 0 > 2.


http://www.mathematica.gr/forum/viewtopic.php?f18&t20955&start60

( )
,

g (, 2]
[2, +)
= 2 g() = 15.

20

:
21 ( ) z1 , z2 , z3 , z4

z1

z2
z1 + z3 + z4 z2 + z3 + z4
C \ R.
,
z2
z1
. z1 + z2 + z3 + z4 = 0.
C, z1 , z2 0

z2 + z3 + z4
z2 + z3 + z4
R
+1R
z1
z1
z1 + z2 + z3 + z4
R

z1
z1

R
z1 + z2 + z3 + z4

http://www.mathematica.gr/forum/viewtopic.php?f51&t19392

1 ( )

z1 + z3 + z4
z1 + z2 + z3 + z4  0.
R
z2
z1 + z2 + z3 + z4
z1 + z3 + z4
+1=
R.
z2
z2
z2 + z3 + z4

R
z1
z1 + z2 + z3 + z4
z2 + z3 + z4
+1=
R.
z1
z1
(1), (2)

z1 + z2 + z3 + z4 = 0.

z1 +z2 +z3 +z4


z2
z1 +z2 +z3 +z4
z1


z2
z1
+
R
z1 + z2 + z3 + z4 z1 + z2 + z3 + z4
z1 + z2
R

z1 + z2 + z3 + z4
(1) ( + i) z2 + z2

R
z1 + z2 + z3 + z4
(2) z2 (1 + + i)

R
z1 + z2 + z3 + z4
z2
(1 + + i) R

z1 + z2 + z3 + z4

(1 )

(2)

z1
R, .
z2

z2
= 0 ,
z1 + z2 + z3 + z4
, z1 + z2 + z3 +
z4 = 0.

2 ( )

w = z1 + z2 + z3 + z4 .
w z2
w
z1 + z3 + z4
w

=
=
1
1 R
z2
z2
z2
z2
w
w
R.
R. w  0.
z2
z1
z1
w/z2
=
R, , w = 0.
w/z1 z2

22 ( 77)
z, w 60 . |z + w|

3 |z w|.

3 ( )

http://www.mathematica.gr/forum/viewtopic.php?f51&t19645

z1
z1
C\R
= + i,  0
z2
z2
z1 = ( + i) z2 , , R,  0. (1)

1 ( ) OAB (A, B
 = 60
O ) AOB

z1 + z2 + z3 + z4  0.

z1 + z3 + z4
z1 + z3 + z4
R
+1R
z2
z2
z1 + z2 + z3 + z4
R

z2
z2

R
z1 + z2 + z3 + z4

|z w|2 = |z|2 + |w|2 |z||w|.

(1 )

, , , |z+w|2 +|zw|2 = 2(|z|2 +|w|2 ),


, (1)

(2)

|z + w|2 = |z|2 + |w|2 + |z||w|.


21

(2)

, (1),(2) (|z| |w|)2 0, .

(1)

|z + w|

2
2

AB

OA

OB

2 ( 77) A, B z1 , z2 ,
M AB :

3 |z w| 2

OM

AB

2
4

OM

AB

OA

+ 2

OB

AB

AB

(2)

, (2)

: 2

OA

OB

OA

OB

OA

OB

2

OA

OB

0, .

(1)

22

:
23 ( )
f f ( f (x)) = x x R,
f () = .

x f (a),
.
4 ( )
g(x) = f (x) x, x R.

http://www.mathematica.gr/forum/viewtopic.php?f52&t11134

f (0) = 0, .

1 ( )
R. g (x) = f (x) x
R , R. g (x) > 0 x R
(1) g (x) < 0 x R (2)
f 1 1 R,
f .
f ( f (x)) = x, x f 1 (x),
f (x) = f 1 (x) x.
(1) f (x) > x, x f 1 (x),
x > f 1 (x) x > f (x) (
(2) ). R
f () = .

f (0) =  0,
> 0.
: f () = f ( f (0)) = 0.
g [0, ] f ()
x (),
g () = f () = < 0, g (0) = f (0) 0 = ,
, g()  g(0) = 0 (g(), g(0)),

(0, ) g () = = 0 f () = .
24 ( Chris)
f R f (x)  0, x
R.

2 ( )
f (0) = 0 = 0.
f (0)  0 (


f (0) > 0) 0, f (0)
h (x) = f (x) x h (0) = f (0) > 0
h ( f (0)) = f ( f (0)) f (0) = f (0) < 0.
. Bolzano (0, f (0))
h () = 0 f () = .

: f (2008) = 1 , f (2009) = 10
f ( f (x) + 3)
f ( f (x))
=
, x R :

f ( f (x) + 2)
f ( f (x) + 1)
) f (2) f (3) = f (4) f (5).
) [2, 3]
: f 2 () = f (2) f (3).
) f 1 1.

3 ( ) f (x1 ) = f (x2 )
f ( f (x1 ) = f ( f (x2 ), x1 = x2 . ,
1 1 R,
.
.
xo f (xo ) = xo ,
f (xo ) > xo f (xo ) < xo .
. f ( f (xo )) > f (xo )
xo > f (xo ), .
. ,
R R,

) f [2008, 2009]
x1 , x2 [2008, 2009],
x0 [2008, 2009] :
3 f (x0 ) = f (x1 ) + 2 f (x2 ).

http://www.mathematica.gr/forum/viewtopic.php?f52&t10208

1 ( ) ,
f (x)  0, f (2008) > 0, f (x) > 0 .
1 : ,
q (2008, 2009) f (q) = 2.
23

x = q .

f (x2 ) f (2009) = 10,


f (x1 ) + 2 f (x2 )
10,
1

2 : f (2) = f (3) r = 2 r = 3.
,
 f (2)  f (3). .. f (2) > f (3).
f (2) >
f (2) f (3) > f (3) (). ,

 , (2, 3)
f () = f (2) f (3).

3
,
f. (
.)

3 : 1-1,
f (5)
f (2)
=
,
. 1)
f (4)
f (3)
. f , < 1,
> 1. .

2 (Chris) 3 )
: f 2 () = f (2) f (3), [2, 3].
)
: f 2 (k) = f (4) f (5), k [4, 5]



f 2 (k) = f 2 () f (k) = f () f (x) > 0 , x R.
k  [2, 3] k [4, 5] f
1 1.

4 :
x1 , x2 [2008, 2009], 1 = f (2008) f (x1 ) f (2009) = 10, 1 = f (2008)

24

:
f 
.

25 ( )
f : R R
f  (x) 0, x R. :

f (x + f  (x)) f (x),

1. A 0 f  (x) 0, x R f
x + f  (x) x f (x + f  (x)) f (x).

x R.

2. () A < 0 a : f  (a) = 0
f  0 (, a] f

x + f  (x) x, f (x + f  (x) f (x), x


(, a] x [a, +), f  (x) f  (a) = 0

http://www.mathematica.gr/forum/viewtopic.php?f53&t8226

1 ( ) x
f  (x) = 0 !

1 .
() A < 0 a1 : f  (a1 ) = 0
f  (x) < 0, x R, 2()
.
f (x+ f  (x))
f (x).

x ( )

f  (x) < 0 [x + f  (x), x]


f (x) f (x + f  (x)) = f  (c) f  (x) c
x + f  (x) < c < x f  (c) f  (x) < 0.
f (x) f (x + f  (x)) = f  (c) f  (x) < 0 .

26 ( Pla.pa.s)
f : R R

x ( ) f  (x) > 0 [x, x + f  (x)]


.



f  (x) ln( f 2 (x) + x2 ) + x4 =

2 ( ) :

. .
.
x0 R f (x0 + f  (x0 )) < f (x0 )
f  (x0 ) = 0 .
f  (x0 ) > 0 .
.. [x0 , x0 + f  (x0 )]
:

f (x) ( f (x) + x )

> x0

f  ()


x R .

f 2 (x)

http://www.mathematica.gr/forum/viewtopic.php?f53&t19535

1 (Pla.pa.s)

f (x0 + f  (x0 )) f (x0 )


< 0,
f  () =
f  (x
 0)


x0 , x0 + f (x0 )

f  (x)( ln( f 2 (x) + x2 ) + x4


( f 2 (x) + x2 ) +

f  (x

0 ) > 0 .

f (x0 ) < 0 [x0 + f  (x0 ), x0 ].

f 2 (x)

= 0),

x R .

f 2 (x), x2 > 0

f (x0 ) f (x0 + f  (x0 ))


> 0.
f () =
f  (x0 )
< x0 f  () f  (x0 ) < 0 .
f (x + f  (x)) f (x), x R.


ln( f 2 (x) + x2 ) + x4 ( f 2 (x) + x2 ) +


> ln f 2 (x) +

3 ( )
.
25

f 2 (x)

1 (1)

f 2 (x)

g(u) = ln u + u1 1 Dg = (0, +).


u1
g (u) = 2

ln f 2 (x) +

f 2 (x)

10

(2).

(1) (2)

u

(0, 1), (1, +)
u = 0 g g(1) = 0. u = f 2 (x)
g

ln( f 2 (x) + x2 ) + x4 ( f 2 (x) + x2 ) +

f 2 (x)

> 0, x R .

f  (x) =
0, x R . f  (x) = 0.

26

:
27 ( )
f : R R


3
0


f (t)dt

28 (

- )
f : [0, +) R


f (t)dt = 2 x + 2 x + 1 x R.
2

f (t)dt < f (x),

x [0, +).

f (x) > 0,

x [0, +).

http://www.mathematica.gr/forum/viewtopic.php?f54&t20929
http://www.mathematica.gr/forum/viewtopic.php?f54&t15152

( )
3 f (x) + f (x) = 4 x + 2 x R. x x

( )
x

g (x) = e

x [0, +)

f (t)dt
0

3 f (x) + f (x) = 4 x + 2,


g (x) = e

f (x) =
1
2x +

2

.
.


f (x)


f (t)dt > 0

x [0, +), g
[0, +). , x > 0 g (x) > g(0) = 0,
x

f (t)dt > 0, f (x) > 0. ,

f (0) > 0,
f (x) > 0 x [0, +).

27

:
29 ( BAGGP93) f : [0, +) R
f 2 (0) = 1 f 2 (x)  6 x x [0, +).



ii)
e1
e1
2
( f (x) 12 x)dx =
(g2 (x) 36 x2 )dx
1

2

216 x3 2 2 f  (x) f (x) 6 = f 6 (x) 18 x f 4 (x) + 108 x f (x) ,

e1
e1
=
g2 (x)dx 36 x2 dx

x [0, +) .

i) f 2 (x) 6 x =

x+1

e1
e1
1
2
1

g (x)dx =
dx = [ln(x + 1)]e
1
x+1

x [0, +) .

ii)


e1 


f 4 (x) 12 x f 2 (x) dx + 72

e1 

e1x
e1  x

1 2
tdt dx =
t dx

2 0

tdt

1
2

e1
x2 dx
1

http://www.mathematica.gr/forum/viewtopic.php?p100301

e1

1 ( )

i) g(x) = f 2 (x) 6 x
[0, +)
g(0) = f 2 (0) = 1. g(x)  0, x
[0, +) 2 g (x) =
g (x)
= 1
g3 (x) g(x)  0 2 3
g (x)
(

g2 (x)
1

g2 (x)

x+1

= g2 (x) |g(x)| =

x+1

e1

e1

g2 (x)dx =

=
1

tdt dx

x+1

1
dx = [ln(x + 1)]e
1 .

30 ( )
f
[a, b], :

= x + c, x [0, +).

g2 (x)

e1
e1
e1
2
2
g (x)dx 36 x dx + 36 x2 dx
=

) = (x) , x [0, +)

x = 0 c = 1

e1
x



4
2
f (x) 12 x f (x) dx + 72

= x+1

1. f  (x) = f (x), x [a, b]

, x [0, +).

2. f (x) > f  (x), x [a, b]


3. f (b) f  (b) = f (a) f  (a).

g(x)  0, x [0, +) ,
[0, +) g(0) = 1 > 0 g(x) > 0 1
, x [0, +) f 2 (x) 6 x =
g(x) =
x+1
1
, x [0, +).

x+1

b
a

28

f (x)
1
dx = (b a).

f (x) f (x)
2

I J = b a
http://www.mathematica.gr/forum/viewtopic.php?p99309

b
I+J=

( )

b
I=
a

b
J=
a

b

f (x)
dx
f (x) f  (x)

f (x) + f  (x)
dx =
f (x) f  (x)

f  (x) f  (x)
dx =
f (x) f  (x)



ln( f (x)) f  (x) ba = 0.

f  (x)
dx
f (x) f  (x)

2I = b a .

29

:
1
M M = 0. h(x) = 0
2
x [0, 1].  
1 x

g(x) = g
g(x) = 0
2 2
32
x [0, 1]. f (x) =
x x [0, 1],
9
.

31 ( )
f : [0, 1] R  x 1  x
f (x) f
+ f
= 2 x
2
4 4
x [0, 1]. f .

http://www.mathematica.gr/forum/viewtopic.php?p100097

1 ( ) g(x) = f (x)

 

32 ( )
: f : R+ R g : R R
, : f (0) = g(0) = 0
f  (x) + g( f (x)) = 0, x 0. : f (x) =
0, x 0.

x
1
f
2 2

 

1 x
= 2x
2 2


x
1
1
1
g(x) n g
= 2 x n1
n
1
2
2
2
2
 x  1 x
1
1
g n1 n g n = 2 x n1
n
1
2
2
2
2
4

g(x) g

http://www.mathematica.gr/forum/viewtopic.php?p78058


1 ( ) f (x) + g( f (x)) = 0
x 0 (1).
> 0. f
[0,], .

n = 1, 2, ..., n n, ,
n +
1
8x

g(x) 0 = 2 x
=
1 1/4
3
 x  1 x
1
1
f
n f n = (8 x/3) n1 .
n
1
n
1
2
2
2
2
4

f [0,]
:

f (x) 0 = (8 x/3)(4/3) =

i) f
k (0, a).

Fermat f (k) =

32
x,
9

(1 )

0 g( f (k)) = 0 (2)

x [0, a] :

2 ( )

g(x) = f (x)
g(x) = g

 x
2

32
x.
9

 x
1
g
.
4

(1)

g( f (x)) 0 (3) f (x) 0




f (x) 0
f [0,]
x [0, a] f (0)

 

1 x
h(x) = g(x) g
. h(x) =
2 2
 

f (x) g(0) g( f (x)) 0 g( f (x)) (4).


(3), (4)
x [0, a] g( f (x)) = 0 (1)

f [0,]
f (x) = 0 .
f (0) = 0 f () = 0 .

1 x
h
. x [0, 1].
2 2
M |h| [0, 1]

|h(x)|

(2)

f (x) f (k) g( f (x)) g( f (k))

1
M.
2
30

ii) f
k (0, a).

(ii) f
0


f () = 0 f (x) = 0 x 0.

f

:

2 ( ) g
R : f (x)g( f (x)) 0, x 0 ().
x 0 :

(i) f 0
.
g( f (a)) 0.
x [0, a] :

( 1)


f  (x) + g( f (x)) f (x) = 0

f  (x) f (x) + f (x)g( f (x)) = 0

2 f  (x) f (x) = 2 f (x)g( f (x)) 0




f 2 (x) 0

f (0) f (x) f (a)


g(0) g( f (x)) g( f (a))
0 g( f (x)) g( f (a)) 0
(1)

f 2 [0, +)
x 0 : f 2 (x) f 2 (0) f 2 (x) 0 f (x) = 0
: f (x) = 0, x 0,
.

g( f (x)) = 0 f (x) = 0
f () = 0.

31

:
33 ( )

( , ..
2(ln x)2 x
1 .
l Hospital

x2 ln x x
.
x+ (ln x)x + x

lim

34 ( )
f : R R

f (0) = 0 f (x + y) f (x) + f (y), x, y R.

http://www.mathematica.gr/forum/viewtopic.php?p78536

( ) x > e

2 ln x > 2 > 1, x2 ln x x > 0.

http://www.mathematica.gr/forum/viewtopic.php?p72699

( ) y g(x) = f (x + y) f (x). g
R g(x) f (y) = g(0),
x. , g 0 .
, Fermat, g (0) = 0.
g (x) = f  (x + y)(x + y) f  (x) = f  (x + y) f  (x),
f  (y) = f  (0).
y. , f (y) = y f  (0),
f (0 ) = 0 .

x2 ln x
x2 ln x
x2 ln x x

()
0<
(ln x)x + x (ln x)x + x (ln x)x
.
x ee ln x e
() <

x2 ln x
.
ex

0 x +. ,
2(ln x)2 x . -

32

,
,

35 ( )

(1)
.
2 :
(ap, a (p b)) p > b > 0 bq ap = 1.
q > a. (mq, m (q k)) q > k > 0
bq ap = kp mq = 1
kp mq = 1.
(b + m) q = p (a + k).
q/ (a + k). a + k < 2q.
a(p b) = m(q k).

a + k = q b + m = p.

x(x + 1) = pq(x y),


p, q , .
2
(x, y) N , y .

36 ( )
f : N N

http://www.mathematica.gr/forum/viewtopic.php?p88955

( ) (x, y)
x(x + 1) = pq(x y), p, q
, .
p, q x
x + 1. x = apq a
a (x + 1) = x y x + 1 a (x + 1) = x y < x,
. x + 1 = apq

f (x) + y f (x)y 2(x + y)


+
=
, x, y N
x + f (y) x f (y)
f (x + y)

http://www.mathematica.gr/forum/viewtopic.php?p102816

1 ( ) : f (x) = x, x
N .
f (n) + n f (n)n
4n
+
=
,
x = y = n
n + f (n) n f (n)
f (2n)
f (2n) = 2n ().
f (1) = a N . x = 2, y = 1
a(3a + 6)
2
6
2+1

+
=
, f (3) =
=
2 + a 2a
f (3)
2a + 1
a(a + 5)
.
a+
2a + 1
, a 2a + 1 ,
2a + 1|a + 5. 2a + 1 a + 1, a 4.
a = 1 a = 4.
a = 4.
x = 4, y = 1 ()
f (4) = 4,
4
10
5a(a + 4)
4+1
+
=
.
f (5) =
, 4 + a 4a
f (5)
3a + 2
a = 4 .
a = 1. , f (1) = a = 1.
x = 2n, y = 1, (),
2n + 1 2n
2(2n + 1)
=
+
, f (2n + 1) = 2n + 1,
2n + 1 2n
f (2n + 1)
() f (m) = m, m,
.

x ax = x y < x.

.
1: x = ap x + 1 = bq a, b
. ab = x y.
y = a (p b) .

p>b>0

(1)

bq ap = 1

(2).

,
,
a, b (1),(2). p, q , . (a0 , b0 )
(2). (a, b) (2)
a = a0 + kq
b = b0 + kp k .
(1) 1

b0
b0
> k > .
p
p

.
2: x + 1 = kp x = mq k, m
. km = x y.
y = m (q k) .

q>k>0

2 ( ) y = x
f (2 x) = 2 x.

(1).
33

x = 1, y = 2 f (3) N f (1) = 1
f (3) = 3 f (1) = 4 f (3) = 1. f (1) > 4
f (3)  N.
f (1) = 4 x = 2, y = 1
f (3) = 8 .

f (1) = 1 x = 1, y = 2 x f (2 x + 1) = 2 x + 1.

f (x) = x .

34

37 ( )
 ABC AB = AC
K, L A, B, AK = KC
CL ACK BK = 2KL.
 ABC.

K D CK.
(EZHK) = 5(ABCD).

http://www.mathematica.gr/forum/viewtopic.php?f110&t17457

( )  AEZ,  ABD
EAZ + BAD = 180 o

http://www.mathematica.gr/forum/viewtopic.php?f110&t14381

(KARKAR ) KL = x, AL = y,
KC = x + y AB = AC = 3 x + y.



,

(AZ)(AE) 2(AB)(AD)
(AEZ)
=
=
= 2 =
(ABD) (AB)(AD)
(AB)(AD)
KAC ,

x+y
CK LK
=
=
=
CA
LA
3 x + y
= 3 x2 + xy = xy + y2 = y = x 3

, 
AC,

3 x+x 3
AD
3+ 3
2
=
A =
=
AK
x + x 3 2( 3 + 1
B = C = 75o

x
y

(AEZ) = 2(ABD)
,  CHK,  CBD
HCK + BCD = 180o

DAK, D

(1)

3
= A = 30o
2

(CHK) (CK)(CH) 2(CD)(CB)


=
=
= 2 =
(CBD)
(CB)(CD)
(CB)(CD)
(2)
(CHK) = 2(CBD)

38 ( ) ABCD : E
A DE.
Z B AZ .
H C BH

(1), (2) =

(AEZ) + (CKH) = 2(ABD) + 2(CBD) = 2(ABCD)



35

(3)

(BZH) + (DEK) = 2(BAC) + 2(DAC) = 2(ABCD)

(EZHK) = (AEZ) + (CHK) + (BZH) + (DEK) + (ABCD) =


5(ABCD).

(4)

(3), (4) =

36

, ,

39 ( )
2011 2011
.
:

2 2 , ,
.


.

40 ( )
x1 , x2 , . . . , xn

x1 +

x2

= x2 +

x3

= = xn1 +

xn

= xn +

x1

=2

http://www.mathematica.gr/forum/viewtopic.php?f111&t14802

( ) , ,
xi . .. x1 < 0. ,
1
1
= 2, 0 < x2 < ,
x1 +
x2
2
1
2
1
= 2 < x3 <
x2 +
x3
2
3

n1
xn <
.

http://www.mathematica.gr/forum/viewtopic.php?f111&t16299

( ) 2 2

. 20102 2 2,

20112 20102 = 2010 + 2011 = 4021

xn > 2,
n1
, .
2 <

,
20102 .

2010 ,
.
:

xi > 0 i = 1, 2, . . . , n. ,

n


xi +

i=1

n

1
= 2n,
x
i=1 i

,
n


http://www.mathematica.gr/forum/viewtopic.php?f...85039

i=1

xi

xi

xi = 1 i = 1, 2, . . . , n.

37

= 0,

41 ( )
 ABC .
BD AB BC K L
. K L X . BX
AC .

http://www.mathematica.gr/forum/viewtopic.php?f112&t19853

1 ( ) E
(O) AD, BX.
XK, XL (O)
BE X,
KBLE
D.KBLE , D (O)
, .

(O) BD  XMN (I)


AC ,
F.
D, F,
MN  XMN
FN = MD , (1)
KD, KB,


F , F, F F (I).
 KAD,  LCD,
M, N
AD, CD, MA = MK = MD , (2)
NC = NL = ND , (3)
(1), (2), (3) = FC = NF+NC = MD+ND , (4)
(4) = FC =
.

B BT
AC BK DK BT DB BL
DL BE DE,

D.KBLE, B.AT LE , B.AT LE
.

AD CD AC
+
=

2

42 ( )
 ABC (I) .
AI, BI, CI I, (I)
S , M, N, P, .
AM, BN, CP T.

AC  BT,
MA = MC, M AC BX .
2 ( ) M ACXK
N AC XL.

http://www.mathematica.gr/forum/viewtopic.php?f112&t20361

( ) D, E, F,
38

(I)  ABC,
BC, AC, AB, .

(I) EF
M , S
MI, EF  MI
Y.
, Y S EF,
A, M
(I), AM
.
, X, Z S DF, DE
, BN, CP , (I).
X, Y, Z ,
Simson S ,  DEF.
,
AM, BN, CP
(I),
T .

39

:
43 ( )

M


(a + bc)(b + ac)(c + ab)

http://www.mathematica.gr/forum/viewtopic.php?p98985

( )

()
.


.
x
.
y1 < y2 < < y100 .
y = (y50 + y51 )/2
.

Mabc


a, b, c a + b + c = 1.

http://www.mathematica.gr/forum/viewtopic.php?p98985

) 1
64
M
.
a = b = c =
3
3
,
64
.
M =
3
a + b + c = 1, a + bc = (a + b)(a + c),
b + ca = (b + a)(b + c), c + ab = (c + a)(c + b)

() P .

( ) . ()
r1 < r2 < < r1000 P
P (r500 + r501 )/2
.


2 1 1 1 
+ +

(a + b)(b + c)(c + a)
a b c
64
(a + b + c)2 abc.
3
(..
,
8
(a + b)(b + c)(c + a) (a + b + c)(ab + bc + ca).
9
, ,
(ab + bc + ca)3 27(abc)2 ,
-.

()
.

.

44 ( )
() 100 .
,
50 .

AlexandrosG: .
.
2 .


2 .

.

() 2010 .
1005
1005
.

40

:
45 ( )
f : [0, 1] [0, ) .

1
x f (x) dx
2

http://www.mathematica.gr/forum/viewtopic.php?f59&t20779

( ) . ,
m n d1 , . . . , dk
"
n (di ) = m. ( f - .) k
2 k .
3 .
p2 pn . m p2 pn+1 ,
m = (pn+1 1)t + r,
0 < r pn+1 2 p2 pn . ( p2 pn + 2 p2 , . . . , pn .)
t < p2 pn a = t, b = r.

f (x) dx.
0

http://www.mathematica.gr/forum/viewtopic.php?p106562

( ) , a, x, y [0, 1]

f (ax + (1 a)y) a f (x) + (1 a) f (y) a f (x)


y = 0, a = x x f (x) f (x2 ).
x [0, 1]

1
x f (x) dx =
2

2
1
=
2
1
=
2

r + (t p2 pn )(pn+1 1) = m p2 pn (pn+1 1)

p2 pn .

x f (x)2 xdx


f (x2 )2 xdx

m = (pn+1 1)p2 pn + r + (t p2 pn )(pn+1 1)

f (x2 )d(x2 )

= a(pn+1 1) + b,

f (x) dx.

a = p2 pn , b = r + (t p2 pn )(pn+1 1). m = (pn+1 1)a + b a, b p2 pn .


a, b f - p2 pn

pn+1 a.

46 ( )

xn 1 n
n Z[x].

41

:
47 ( )
G a, b a3 b =
ba2 a2 b = ba3 . a5 = e, e
G.

48 ( )
H C 2 = x
2 x. : C2 Aut(H)
(x)(h) = h1 h H . ( H
, ). G := HC2 . G
h2 = 1
h H.

http://www.mathematica.gr/forum/viewtopic.php?f10&t20719

1 ( )
a3 = ba2 b1
a2 = ba3 b1 .
a6 = ba4 b1
a6 = ba9 b1 . a4 = a9 ,
a5 = e .

http://www.mathematica.gr/forum/viewtopic.php?f10&t20713

( )
,

2 ( )

h2 = 1 h H = h1 = h h H

a3 b = a(a2 b) = a(ba3 ) = aba3

= (x) = 1H

a3 b = aba3

= H  C2 = H 1H C2 = H C2 .

ba2 = aba3
b = aba.

G .
, G ,

a3 b = ba2 a3 aba = ba2

(h, 1)(1, x) = (1, x)(h, 1) h H

a4 b = ba

= (h (1)(1), 1 x) = (1 (x)(h), x 1) h H

a(a4 b) = a(ba)

= (h, x) = (h1 , x) h H

a5 b = aba

= h = h1 h H

a5 b = b

= h2 = 1 h H.

a5 = e.

42




lim xn = x1 1 +
ak x0
ak .
n

49 ( ) xn
x0 , x1

k=2

(n 1) c
1
xn =
xn1 +
xn2 , c > 0 .
1 + (n 1) c
1 + (n 1) c

lim xn .

n+

S =
http://www.mathematica.gr/forum/viewtopic.php?f9&t12954

( ) x1 = x0
c
1
x2 =
x0 +
x0 = x0 ,
1+c
1+c
2c
1
x0 +
x0 = x0 ... xn = x0
x3 =
1 + 2c
1 + 2c
lim xn = x0 ( ).

1
1 + (n 1)c

S (x) =

1 + (n 1)c

S = S (1)

c
2 xc

+ . . .
1 + c (1 + c) (1 + 2c)
x
S (x)
1
=

+ . . . ,
x
1 + c (1 + c) (1 + 2c)
x
S (x)
1
+ c S  (x) =

+ ... =
x
1 + c (1 + c) (1 + 2c)
x2
x
+
...
1
(1 + c) (1 + c) (1 + 2c)
c S  (x) =

(xn1 xn2 ) .

yn1 y1 = x1 x0 = A  0.

{yn }n=1,2,3..
1
1
1
A , y3 =
y2 =
A
y2 =
(1 + c) (1 + 2c)
1+c
1 + 2c
yn =

S (x)
+ c S  (x) = 1 S (x) .
x

n1

(1)
A .
(1 + c) (1 + 2c) . . . (1 + (n 1) c)

(1)n1
A=
(1 + c) (1 + 2c) .. (1 + (n 1) c)
(1)n1
.
xn1 + an A , an =
(1 + c) (1 + 2c) .. (1 + (n 1) c)
xn = xn1 + an A
n
n
n
n




xk =
xk1 + A
ak xn = x1 + A
ak

xn = xn1 +

k=2

k=2

k=2

n
n



xn = x1 1 +
ak x0
ak
k=2

x2
x

1 + c (1 + c) (1 + 2c)
x3
...
+
(1 + c) (1 + 2c) (1 + 3c)

x [0, 1] .

yn = xn xn1 ,
1

1
1 + c (1 + c) (1 + 2c)
1
+
...
(1 + c) (1 + 2c) (1 + 3c)

xn xn1 =

(-).

x1  x0 .
1 + (n 1)c 1
1
xn1 +
xn2 =
xn =
1 + (n 1)c
1 + (n 1)c
1
(xn1 xn2 )
xn1
1 + (n 1)c

yn =

k=2

S (x)
+ c S  (x) = 1 S (x)
x

1+

S (x) + S  (x) =

e x/c x1/c ,


1
1 x/c 1/c
x/c 1/c 
e x S (x) + 1 + e x S (x) =

k=2

k=2

43

 1
ex/c x1/c ex/c x1/c S (x) = ex/c x1/c
c
c
1
1


1
ex/c x1/c dx
ex/c x1/c S (x) dx =
c
1

1/c

e S (1) =

1

|F(x) F(y)| x y2 .

ex/c x1/c dx

, int(E m )  E m ,
T = [a, b] [a, b]. N N.
[m, m] N 2 2m/N 2 .
, T N 2 ,
(N + 1)2 ( ).
x, y - |F(x) F(y)| 2Nm2 .

S =

1/c

1

ex/c x1/c dx

lim xn = x1 (1 + S ) x0 S .

x y2

50 ( )
f : R2 R

ba
N .

N N.

d R2 .

2 ( )
, : E =
f (R2 ) g : E R2 f ,
1-1, 1-Lipschitz.

http://www.mathematica.gr/forum/viewtopic.php?f9&t20799

sn = 1 + 1/2 + . . . + 1/n
En = E (0, sn ).

| f (x) f (y)| d(x, y) , x, y R2 ,

, A1 = E1 An =
En \ En1 n 2 En = E (sn , 0)
. , A , A
An = En \ En
n
n
1
#
n=1 (An An ) = E .

1 ( )

Em = {x R2 : | f (x)| m} m = 1, 2, . . . .
, Baire m N
int(E m )  . f E m F
:

, diam(g(Ak )), diam(g(Ak )) k1 , g(Ak ), g(Ak ) Bk , Bk , 1/k. m2 Lebesgue R2 , :

|F(x) F(y)| x y2 x, y E m .


(.. x E m \ Em (znx ) Em
znx x. , | f (znx )| m BolzanoWeierstrass
( f (znx )) zx
[m, m]. F(x) = zx = limn f (znx ).)

"

m2 (g(E))
k=1 m2 (g(Ak )) + m2 (g(Ak ))
" 2
"

2
k=1 m2 (Bk ) = k=1 k2 < + ,
, m2 (g(E)) = m2 (R2 ) = .

F : E m [m, m]

44

:
A, B ( ).

51 ( )
X P() :

(B  C) X = (B X)  (C X)
http://www.mathematica.gr/forum/viewtopic.php?p105616

B, C ( ).

( ) x A B,
http://www.mathematica.gr/forum/viewtopic.php?p105616

x  (A \ X) \ B

( ) x X

x (B  C) X

x  A \ (X \ B)

x  (B X)  (C X)

x A, x X \ B.

x B.
A B = . ,
X

X=
.
52 ( )
X P() :

A \ (X \ B) = A \ X = (A \ X) \ B

A (X B) = (A X) B

45

:
f  (x) = (n + 1)xn (n + 1).

53 ( )
N 169 | 33n+3 26n27.

f (0) = 0 f  (0) = 0.
, 0 f (x).

, g(x)

http://www.mathematica.gr/forum/viewtopic.php?f63&t19640

1 ( )

f (x) = x2 g(x)

33n+3 26n 27 = 27(27n 1) 26n

= 27(27 1)(27

n1

+ 27

n2

, f (26) = 27n+1 26(n + 1) 1 (3).

+ 27 + 1) 26n

4 ( ) .
n = 1, n = 2 .
n N ,

= 26(27n + 27n1 + + 27 n).


,
26.
,

33n+3 26n 27 = 169.


,

27n + 27n1 + + 27 n

= (27n 1) + (27n1 1)+


+ (27 1) 0

(3)

(mod 26).

33n+6 26(n + 1) 27 = 169.

, 262 | 33n+3 26n 27.

2 ( )
33n+6 26(n + 1) 27 = 27 33n+3 26n 53
3n+3

26n 27 = 27(27 1) 26n


n

= 27(169 + 26n + 27) 26n 53

= 2 13(27 1) + 27 1 26n.
n

= 169.

27n 1 26n 169.

54 ( ) a b . a b a + b 2
q r

27n 1 26n = 26(27n1 + 27n2 + ... + 27 + 1 n)

= 2 13 [(27n1 1) + (27n2 1)+


+ (27 1)].

a b = q(a + b) + r 0 r < a + b.


26, 13.

(a, b)

3 ( )

q2 + r = 2011.

f (x) = (x + 1)n+1 (n + 1)x 1


46

a q = 1 b q = 2011 a q = 2011 b q = 1.
http://www.mathematica.gr/forum/viewtopic.php?f63&t13925

(4),
(a, b, q, r)

1 ( )

q  2011 = 44,
ab
r
=

q < q +
a+b a+b

(4)

q < a q < b.

(5)

(q + 1, q + 2011, q, 2011 q2 )
(q + 2011, q + 1, q, 2011 q2 )
q = 0, 1, . . . , 44, 90

(1, 2011, 0, 2011), (2011, 1, 0, 2011)


(2, 2012, 1, 2010), (2012, 2, 1, 2010)
(3, 2013, 2, 2007), (2013, 3, 2, 2007)
...
(45, 2055, 44, 75), (2055, 45, 44, 75).

(a q)(b q) = ab q(a + b) + q2 = r + q2 = 2011


(5) ( a q > 0 b q > 0),
2011 ,

47

:
55 ( )
f : R R
f (0) = 0 x R f (2 x)
x + f (x) f (3 x) 2 x + f (x).

( ) ,
.
,
 1
 1
 1
1
1
f (x)dx
x f (x)dx +

f (x)dx,
12
2 0
0
 01
1
1
x f (x)dx .
,
2
12
0
, f (x) = x + g(x). ,

http://www.mathematica.gr/forum/viewtopic.php?p113239

(air) x R .
:
x
2n 1
f (x)
x + f ( n )n N
n
2
2
x
3n 1
f (x)
x + f ( n )n N .
3n
3
n f
:
f (x) x + f (0) = x f (x) x + f (0) = x
f (x) = x. x R
.

(2 x 1)g(x) 0 ( -

). ,
|g(x) g(y) + x y| |x y| x, y [0, 1] ().
, g [0, 1].
, x, y [0, 1] x > y. , g(x) > g(y), ()
g(x) g(y) + x y x y g(x) g(y), .
g(x) g(y) g .
, 2 x 1 g .
Chebyshev

56 ( )
f : [0 , 1 ] R

| f (x) f (y)| |x y|, x, y [0, 1].


 1  x
 1
1
1
f (t)dt dx +

f (x)dx.
12
2 0
0
0


0

 1
(2 x 1)dx
g(x)dx
0
0
 1
g(x)dx = 0
= 0

(2 x 1)g(x)dx

0
http://www.mathematica.gr/forum/viewtopic.php?p99634

48

:
57 ( KARKAR)
ABCD, K , CB, DA, S .
M, N AB, CD :
3
CD = 2AB, : MN = S K .
4

2 ( )
:

L MN KS L KS .

( ) ABCD ( )
:( S AB  S DC) , ( KAB  KDC)
AB DC=2AB 1
=
,
( )
DC
2

http://www.mathematica.gr/forum/posting.php?modeedit&f62&p99515

1 ( )
CB = a, AD = b, CS D = x.

 S AB  S DC
AB
1
SB SA
=
=
=

S D S C DC 2

S D = 2S B : (1)

():

DS = 2BS , BS =

a
a + 2b KB
=
,
3
KD 2b

S C = 2S A : (2)

DC = 2AB : (3)

+ DA

2N
M = CB
4 MN 2 = CB2 + DA2 + 2CBDAcosx =

KD = 2KA : (4)
:

KC = 2KB : (5)

= a2 + b2 + 2abcosx
:

a
aS
D + 2bS
B
KB
=
S
K =
KD 2b
a + 2b
2
2
2
a
S
D
+
4
b
S
B2 + 4abS DS Bcosx
S K2 =
(a + 2b)2
4a2 S B2 + 4b2 S B2 + 8abS B2 cosx
S K2 =
(a + 2b)2
2
2
a + b + 2abcosx
S K 2 = 4S B2
=
(a + 2b)2
(a + 2b)2 4 MN 2
16
= MN 2 .
=4
9
9
(a + 2b)2

( S AB  S DC) , ( KAB  KDC)


1
,
2

S M S ACS DC 1 KACKDC K M
=
=
S N
2
KN

S N = 2S M : (6)

KN = 2K M : (7)
 K MB  KNC

 KAB  KDC
49

 = NPL
 : (15)
KAS

 = K
Mathematica : KNC
MB : (8)

(11) , (13) , (15)

(10):MN= 34 LN

 NPL = S AK

MN =

3
KS
4

 S MB  S ND ( -

LN = KS

) : S
ND = S
MB : (9).

58 ( KARKAR) S T -

 + S
(8) , (9) KNC
ND = K
MB + S
MB

s, (s <

),
2
AB, OAB.
, S D, T E , OA,
S Z, T H , OB.
S T , . :
E1 + E2 + 2E3 , . (E1 , E2 , E3
)

 S
KNC+
ND=1800 S
NK,K
MB+S
MB=K
MS


MS
180 S NK = K


S NK + S NK = 1800
0

K MS ,KNS

(K MS )
(KNS )

(MK)(MS )
(NK)(NS )

NK=2K M,NS =2 MS

(K MS )
1
=
...
(KNS )
4
MM

(K MS )

(KS ) (KNS ) = MM
NN 

1 LNN LMM NN  = LN LM
1
MM 
=
=

NN 
4
LN
4
LN MN
MN
1
1
3
= 1
= . . . MN = LN :

LN
4
LN
4
4
(10)


LM

http://www.mathematica.gr/forum/viewtopic.php?f62&t19080

1 (
) R = 1.
D(x1 , 0), E(x2 , 0), Z(0, y1 ), H(0, y2 ).
:

S D :

N DC

.
 DS C :

P DS

S C S C=2AS

NP = AS : (11)

NP = 2

NP//S C NPD
 = DS
B : (12)

L DK
.
:
 DS K
:

P DS

KD KD=2KA

LP = KA : (13)

LP = 2

LP//KD LPS
 = KDS
 : (14)
 BDS

Eo = (OE)(OZ) (OD)(OH) =
= x2 y1 x1 y2 =
= cos(T OA) sin(S OA) cos(S OA) sin(T OA) =
= sin(S OA T OA) = sin(S OT ).

 )
CBD(

 = DS
B + KDS

CBD

 CAD(
 )
(12),(14),CBD=

 , NPD+
 

 1800 KAS
LPS =1800 NPL
CAD=

 = NPD
 +
CAD
LPS
0


180 KAS = 1800 NPL

50

2 (
)

ZO  S P (S ZP) = (S OP) , OD  PT (PDT ) =


(POT ) (S ZP) + (PDT ) + (S PT ) = (S OT ) E1 + E2 +
2E2 = 2 (S OT ).

51

:
59 ( ) z5 = 1,

60 ( )
x1 , x2 , ..., xn ,
,

z4
z
z2
z3
+
+
+
,
1 + z2 1 + z4 1 + z 1 + z3

A=

nx +

n


n


xk =

k=1

z4
z2
z3
z
+
+
+
.
1 z2 1 z4 1 z 1 z3

B=

k=1

x2k
xk x

0 n 1

2 , z  1.
http://www.mathematica.gr/forum/viewtopic.php?f60&t15151

1 ( ) x = 0

http://www.mathematica.gr/forum/viewtopic.php?f60&t10440

n


( )

xi =

n


i=1

A=
=

z3

2z

x1 < x2 < ..... < xn

( x  0) :

nx +

z4
z3
z3
z4
+
+
+
=
z3 + z5 z + z5 1 + z 1 + z3

z3

. ( )

z
z
zz
zz
+
+
+
=
3
2
4
1
+
z
+z z
1z+z z
1 + z3
3

=
=

z4
z
z2
z3
+
+
+
=
1 + z2 1 + z4 1 + z 1 + z3

+1

2z

z+1

2z

(z +

2z4

(z + 1)(z2 z + 1)

1)(z2

z + 1)

2z

z+1

nx +

B=

nx +

2z3 (z2 z + 1)
=
(z + 1)(z2 z + 1)

n


n


xi =

nx +

x2i x2 + x2

xi x

n

(xi x)(xi + x)

xi x

n


xi = nx +

i=1

n


xi +

n


i=1

x2

n


x
i=1 i
n

i=1

x2
xi x

i=1

x2

xi x

x0

x
1

xi x

=0
=0

f
1

f (x) =

+ ... +

x1 x x2 x
xn x
x (xi , xi+1 ), i = 1, 2...n 1

z4
z3
z3
z4
+
+
+
=
z3 z5 z z5 1 z 1 z3
3

xi =

x2i

xi x

z4
z
z2
z3
+
+
+
=
1 z2 1 z4 1 z 1 z3

i=1
n


i=1

2z3 (z2 + 1)
2(z5 + z3 )
=
=2
z3 + 1
z3 + 1

n


i=1

i=1

z3
z4
z z2
z z3
+
+
+
=
= 3
z z3 z2 1 z z4 z 1 z 1 z3
=

xi =

i=1

2z4 + 2z3 (z2 z + 1)


2z3 (z + z2 z + 1)
=
=
=
(z + 1)(z2 z + 1)
z3 + 1

n

i=1

xi

i=1

, ,

z
z
z
z
+
+
+
=0
1 z 1 1 z 1 z3

z3

f  (x) =
52

x1 x

x2 x

+ ... +

xn x

x (xi , xi+1 ), i = 1, 2...n 1

.
.

(xi , xi+1 ), i = 1, 2...n 1

n


k=1

xk x

= 0 (1)

lim f (x) =

g(x) = (x x1 )(x x2 )...(x xn )

xxi

lim f (x) = +

(1)

xxi+1


R.
0 R i (xi , xi+1 )(i = 1, 2...n 1)
( ) :
f (i ) = 0 (i = 1, 2...n 1)
2 ( ) -

g (x)
= 0 g (x) = 0
g(x)

n 1
k
(xk , xk+1 ), k = 1, 2, ..., k 1, (Rolle) k .

53

:
61 ( )
,
(x, y) |x| |y| 1 |y| 1.
.

g(x) = a,
2

g(x) = x3 3 x2 + 5 x , x  0.
x


g (x) = 3 x2 6 x + 5 +

1 ( )
:
1

x2

> 0, g


(, 0), (0, )

http://www.mathematica.gr/forum/viewtopic.php?f27&t21060

 1

lim g(x) = , lim g(x) = +

x+

1+|y|

1|y|

lim g(x) = +, lim+ g(x) = ,

1dxdy,

x0

x0

g
(, 0), (0, ) R.

g(x) = a
R , ,

y-
x y 6.
2 ( )
y = 1, y = 1, x = |y| + 1 x = |y| 1
:

x4 3 x3 + 5 x2 + ax 2 = 0.
2 ( ) Vieta:
xi , i = 1, 2, 3, 4 4 .
:
4


E =422

xi = 3

(6)

xi x j = 5

(7)

i=1
4


1
2 1 = 6 ..
2

i j=1
4
'

xi = 2

(8)

i=1

62 ( )

(8) 4 .
:

x4 3 x3 + 5 x2 + ax 2 = 0, a R,
.

4


i=1

4 2
4


2

xi =
xi 2
xi x j
i=1

i j=1

(6)(7)

= 9 10

http://www.mathematica.gr/forum/viewtopic.php?f27&t20681

= 1

1 ( ) 0 .

4 .
54

2 2
, (8)
.

(i)
p(x) = x4 3 x3 + 5 x2 + ax 2 (
a > 0 a < 0
a = 0) 3 1 .

:
.

(ii) p(x) =
x4 + 3 x3 + 5 x2 ax 2 ( a > 0 a < 0 a = 0) 1
.

3 ( ) ,
Descartes, .
,
, ,
.
, .

(1 1 ) (3
1 ).
4 ,
4


k=1

55

2k = ... = 1. .

http://www.mathematica.gr.
http://www.mathematica.gr.
LaTEX: , , : , : , , : . LaTEX.
.

Leonardo da Vinci

(32-) . 30
. -
- - quasiregular ,
( - ). (0, 0, ),



1+ 5
1
,
, 1+
,
.
2
2
2
2

:http://en.wikipedia.org/wiki/Icosidodecahedron
:

mathematica.gr
(http://www.mathematica.gr) .

mathematica.gr


1. ()
2. (Mihalis_Lambrou)
3. (nsmavrogiannis)

10. ( )
11. ( )
12. (dement)
13. ( )
14. (swsto)

4. ( )

15. (achilleas)

5. (chris_gatos)

16. ( )

6. (m.papagrigorakis)

17. ()

7. ( )


1. (grigkost)
2. (cretanman)

18. (xr.tsif)
19. (Demetres)

1. (spyros)
2. (p_gianno)
3. (kostas.zig)

1. (stranton)

4. (exdx)

2. ( )

5. ( )

3. (vittasko)

6. (mathnder)

4. (nkatsipis)

7. (mathematica)

5. ( )

8. ( )

6. (mathxl)

9. (rek2)

7. (matha)

10. (hsiodos)

8. (gbaloglou)

11. ( )

9. (R BORIS)

12. (bilstef)


1 ( )
( ) !

25
10 (
).
6 ( KARKAR) , 60% 70% ,
. ,
,
.

2 ( )
7 : 38.


7 ( )
1
.


3 ( )
1
2

A=(

ZE.
8 ( KARKAR)

.

3
5
7
97
99
) + ( ) + ... + (

)
4
6 8
98 100

1. , .

B=

2. .

1
1
1
1
1
1
1
1
+ +
+ ... +
)( + +
+ ... +
),
2 6 10
98
4 8 12
100

A + B.
,
4 ( parmenides51)
492
23%. 13%,
;

9 ( )

, 1 2 :

x2 x + = 0
:
3 1 2 2 =

10 ( irakleios)
m

5 ( )
. 6m
1m ,
.
;

x2 2(m + 1)x + m2 = 0
(0, 2);

14 ( )

x3 x2 + 27x 3

(x + x)2 = 6

11 ( KARKAR) ABC 
B = 70


C = 60 . S , B
SC =
100 S B = S C . CS , BS AB, AC D, E
 =
. S DE
.

,
15 ( KARKAR) BC
BC ,

BC . BC.
BC !

12 ( ) AB
B = E.
A = E.

16 ( KARKAR) D AB
ABC, (AB = AC), ,
AC E, BC Z .
AD = CZ , E BC ,
AB.
:

, D ;

,
13 ( )
1 , 2 , ..., 2+1 ,
:

1 3 21
...

2 4 2

,
17 ( KARKAR) S
1
1
y = x, A y = x
2
2
: (S A) = 6. M
S A.

1
.
2+1

18 ( )

x2 y2
C1 : 2 2 = 1 C2 : x2 + y2 = a2 a, b > 0.
a
b
K(x0 , y0 ), x0 > 0, C2
b
( ) : y = x C1 , C2 K
a
E .

, , ,
23 ( )
f : R R :

lim
x2

f (x)
= 3.
x2

:
,

lim
x2

19 ( ) X
t1 , t2 , ..., tv , x  0 , R


g (x) =

x(t1 +t2 +...+tv )2v x


,
x2 4
v x
2

x=2

0<x2

f (x)
.
sin(x)

24 ( ) f
R 1 1 :


.

f (x) f (1 x) = f (ax + b),

x R

1. g xo = 2 , = 21 .

1. a = 0

2. g

A(3, 20) , vi=1 ti = 100.

2. f (b)  0

3.
i=1 ti fi = 1, fi
, v .

4. f R.

v

3. f (1 b) = 1

20 ( )
s, s  0.

R 0 < 2 s v.

, ,

25 (

)
R, f

, ,

f (x + f (x)) = f (x)

(1)

1. f

21 ( ) z, w
4|z|2 2zw
= 1,

x R

2. (1)

|w|2 2zw = 3.

3. f a, b a < b.
u (a, b) f (u)  0.

1. |2z w| = 2.
2. z w
,
.

26 ( )
f [4, 10]
f (A) = [1, 5] f (4) = 2, f (10) = 4.
1 , 2 (4, 10) : f (1 ) = f (2 ) = 0.

3. |6z + w|.
4.
z w.

, ,

22 ( ) f
R, f (x)  0 x R

|z|

f (x)dx = 0

27 ( ) f : [0, 1] [0, +)
,

f (1 ) = 1 .

f 2 (t) 1 + 2

1. f (x) > 0.

f (u)du, t [0, 1].


0

f (t) t + 1, t [0, 1].

2. z.

(|z + z| 3) x + x
.
(|z z| 3) x2 + x
4. f x x x = 0 x = 1
|z + 2z|,
 x
f (t)dt = 3 x2 + 6 x 6
3

28 ( ) f
[0, 1]

3. lim

f (t)dt
x

1 x2
x [0, 1].
2

( f (x))2 dx

(0, 1).

x f (x)dx.
0

33 ( )
, ,

lim

x+

29 ( ) :

sin(cos x) = x
.
;

x2 ln x x
(ln x)x + x

34 ( )
ABC I . BI, CI AC, AB
B , C . AB AC , AI 2 A
, .

cos(sin x) = x

30 ( ) z w  2
2z 1
.
z+1

|z 1| = 1 w =

f (x) = ln x

Juniors,
- -

|w| x.
35 ( ) a > 1,
f : R (0, +)

1. f ,
2. f ,
3. f ,

1. f (x) ax , x R

4. lim x2 f 1 (x) .

2. f (x + y) f (x) f (y), x, y R.

x0

36 ( )

, ,

2 x+1 = 2[x] + 2{x} .

31 ( )
f : R R,

f (0) = 1
ex f (x) = f 2 (x) x R.

Juniors,

32 ( ) f : R

R f (x) =

37 ( )
ABCD BCD, DAB
80o , 40o , . BC = CD = DA,
ABC = x.

t2

g : R R

dt

ex (1+t )
dt.
1 + t2
2

g(x) =
0

1. x0 R,

lim

xx0

ex

2 (1+t 2 )

dt =

ex0 (1+t ) dt.


2

2. x0 R,

g (x0 ) = 2ex0
2

x0

et dt
2

f (x) + g(x) =

x R.

3. lim g(x) = 0
x+

lim

x+

et dt =

38 ( KARKAR) 
ABC, M AC. (B, BM)
BC N. N M BA
S .
,

 AMS .

44 ( ) a, b, c
a + b + c = 1,

1+b
1+c
1
1+a

b+c

c+a

a+b

abc


45 ( vzf)

,
.
46 ( Vojtech
Jarnk 1997)
a . d|(a2 + 2)
d 1 mod 8 d 3 mod 8.

Seniors,
- -

47 ( ) :

39 ( ) n 17
.
n .


ac
1

b

40 ( ) P,
,


P(0) = 0 P (x + 1)3 = (P(x) + 1)3 .

ab
1


bc

1 = (a b)(b c)(c a).

a

48 ( ) G
a, b G a2 b = ba2 =

ab = ba.

() G 2n , .
() .

Seniors,
41 ( )
ABCD AB = AD B = D = 90o . DC, BC,
E, Z , AE, DZ .
BE AZ.


49 ( )

42 ( ) I
 ABC IA, IB, IC
D, E, Z ,

+

(2n)
n=1

IA IB IC
+
+
3
ID IE IZ

2n

(s) Riemann.

 1+ 5 

50 ( ) a = ln
:

1.

43 ( - 1995) , :

+

(1)n1
4a
1
2n
= +
n=1

2.

+

n=1

(.
), ,

.

n1

(1)
2a
2n
= .
5
n n


,
.

5 5

51 ( )
9.999
.

56 ( KARKAR) D
ABCD DA ,
S . CT S D S A, S B
Q, P . CP = PQ.

52 ( )
n

a=n+3

b = n2 + 3

()
53 ( ) :

x5 40 x4 + Px3 + Qx2 + Rx + S = 0
.
10.
|S |.
54 ( )

ee

e .

,
55 ( KARKAR) (O) (K)
A, B .
A, B T . S (O)
S T C . S A
(K) P, AC Q. S T
PQ.

57 ( )
f, g C[x], .
f 2 (x) + g2 (x) r C,

( f (r))2 + (g (r))2 = 0.

58 ( )
Cauchy. a1 , a2 , ..., an

a1 + a2 + ... + an
n a1 a2 ...an
n
.

, ,
.

:
1 ( )
(
) !

25 10 (
).

http://www.mathematica.gr/forum/viewtopic.php?fF44&tF18414

( ) A
, B 10 A B ,

http://www.mathematica.gr/forum/viewtopic.php?fF44&tF19577

(KARKAR)
12 , 30o ,
.
360o 60 ,
6 .
7.38 , 19o ,
210 + 19 = 229 ,
6 38 = 228 .
!

A + 3 B + 5(10 A B) = 25

50 4A 2B = 25,
( F ).
2 ( )
7 : 38.

:
[ + ]

3 ( )
1
2

3
4

5
6

7
8

A = ( ) + ( ) + ... + (

97
99

)
98 100

B=

1
1
5 1
9
1
+
+
+ ...
+
+
+
2 2
6 6
10 10

97
1
3
1
7
1
11
1
+
+
+
+
+

...
98 98
4 4
8 8
12 12

1
1
1
1
1
1
1
1
+ +
+ ... + ) ( + +
+ ... +
),
2 6 10
98
4 8 12
100

99
1
+
=

100 100

A + B.

= 1 + 1 + 1 +...+ 1 1 1 ... 1 = 0
25 25 .

http://www.mathematica.gr/forum/posting.php?modeFquote&fF33&pF113111

4 ( parmenides51)
492
23%.
13%,
;

( ) :

1 3
5 7
A+B=

+
+ ...
2 4
6 8

1 1
97
99
1

+ +
+ ...
+
+
98 100
2 6 10
1
1
1
1
1

+ +
+ ... +

=
+
98
4 8 12
100

http://www.mathematica.gr/forum/posting.php?modeFquote&fF33&pF105629

( ) 492
,
( ), 23 % .
23
492
.
100
, 492
1 + 0, 23 = 1, 23. ,
492 : 1, 23 = 400 .
, 13 % , (
) 400 1, 13 = 452 .

[ ]
1 3 5 7
+ + ...
2 4 6 8
97
99
1
1
1

+ + +
+ ...
+
98 100 2 6 10
1
1
1
1
1

...
=
+
98 4 8 12
100

:
5 ( )
.
6m
1m ,
.
;

(R 1)2 = (R 1)(R 1) = R2 R R + 1 = R2 2R + 1.
6 ( KARKAR)
, 60% 70%
, .
,
,
.

http://www.mathematica.gr/forum/viewtopic.php?fF34&tF22414

( ) a, k
.
0, 6a
0, 7k. , :

http://www.mathematica.gr/forum/viewtopic.php?fF34&tF22032

( ) R
, K PQ AB,

0, 6a = 0, 7k

KP = R, KQ = KB QB = R 1.

k=

KPQ :

KP = KQ + PQ
2

6a = 7k

6a
.
7

(2)

, :

: 0, 6a+0, 7k = 0, 6a+0, 6a = 1, 2a .

R2 = (R 1)2 + 62
R2 = R2 2R + 1 + 36

2R = 37

(2)

a+k = a+

AB = 37.

6a
7a + 6a
13a
=
=
7
7
7

, :

, 37m.
: (ab)2 =
a2 2ab + b2 ,

p=

1, 2a
13a
7

8, 4a
8, 4
=
13a
13

840
% 64, 6%.
13

:
7 ( )
1 .

AK EZ//K Z
AK (5).
(3),(4),(6)

ZH = ZE + EH =
K M K
=
+ K
2

2
3K
=

KM
2

K =

ZE =

ZE.

1 ( )
A K M .
ZE K.
1
K = .
K A .
3
1
ZE = .
6

1
.
6

k2 k
.
k2 1
=
2
,
10 99

(4)

k2 1
2

99

20 k2 1 198

(5)

AK M ZH//K M H
AM .

ZE =

( + 1)2 2 = k2 2 + 1 = k2 .

(3)

10

KM

( ) , + 1
, .

Z AK

http://www.mathematica.gr/forum/viewtopic.php?fF35&tF17146

A , B , H K, E .

2.
.

180
AB AB

KM

1. ,
.

2 (Parmenides 51)
A ,K M H
AB, ZE .

ZH =

KM

8 ( KARKAR)
.

http://www.mathematica.gr/forum/viewtopic.php?fF35&tF19335

EH = K

KM

21 k2 199
21 k

199

4, . . . k 14, . . . .

(6)

k 5 7 9 11 13.
10

132 1
= 84 + 1 = 85
2
2
85 842 = 169 = 132 .

85
84.
k

.
(61, 60), (41, 40), (25, 24)
(13, 12).

.

k = 13 =

11

:
9 ( )

,
1 2 :


,
, :

(31 22 = 32 21 = )

(7)
0

(31 22 ) (32 21 ) = 0

2 4 0

131 2 6 2 + 2 (1 + 2 ) + 2 = 0
1
2

2 4 0

25 62 = 0

62

= 25

2 4 62 0
25

62 = 25
62 = 25

2 0

x2 x + = 0
:
31 22 =

http://www.mathematica.gr/forum/viewtopic.php?fF19&tF22435

1 ( ) .
Vieta:
1 + 2 = , 31 22 =
:

1 =

3
3
2
, 2 =
= .
5
5
5
3

, 1 2 = , 5 5 =
():
62
= ,

25
1
= 2 4 1 =
2
25
:

1 =
51

+ 51
2

3
5

62
25

62 = 25
.
10 ( irakleios)
m

x2 2(m + 1)x + m2 = 0
(0, 2);

http://www.mathematica.gr/forum/viewtopic.php?fF19&tF23137

( ) m = 0
: x2 2 x = 0 x1 = 0, x2 = 2
.
m  0. :

2 ( ) :

= 2 4,

(9)

()( 21 + 22 = (1 + 2 )2 21 2 )

2
31 22 = .
2
5

62
= .
,
25

1 =

(8)

()

D = b2 4ac = 4(2m + 1)
:

(7)

1 + 2 = ,

(8)

1 2 = .

(9)

1
2

D>0m> .
12


1
: : D = 0 m =
2
1
b
: x =
= (0, 2)
2a
2
1
m = .
2
D > 0
x1 , x2 x1 < x2 .
0 2
. :

2
:

a f (2) < 0 m(m 4) < 0 m (0, 4) .


m (0, 4) 2
0 .
(0, 2)
.
m
:

f (0) = m2 > 0

 
1
A=
(0, 4).

0 .
:
f (2) = m2 4m = m(m 4).

13

:
11 ( KARKAR) ABC
 = 60 .

B = 70 C
S C = 100 S B = S C . CS , BS
S , B
AB, AC D, E .
 =
.
S DE

= ZDE ZDC
DZ=ZC

ZDE DCZ

= 30 .
0

http://www.mathematica.gr/forum/viewtopic.php?fF20&tF22214

2 ( )

1 ( )
BS C
,

DBC
B = D = 700 , BEC = 800
ECS ,

S C = CE.

(10)

ECZ . (10)
S CZ ,

S ZC = 700 = ABC
S Z//BD (BZS D )
DZ = BS = ZE

DZE
DZE = DZS + S ZE = DBS + S ZE = 40 0 .
14



S BC 100 , 40 , 40 ,



CBD 40 , 70 , 70



CS E 20 , 80 , 80 .
BE EZ = EC ,
ECZ, S BC (
 = 60 ,
CZ = CB = CD) DCZ
CDZ . DE CZ ,
= 30 .

K//AB AKB ,
AK = B = E K
( ).
K = 

AK = E = 1200 AK, E
( ), A = E.

12 ( )
AB
B = E. A = E.

2 ( )

AB 60
AZ = 60 ,
A ( AZ). A = AZ 
AZ
E, Z ( ) E = Z = A.
http://www.mathematica.gr/forum/viewtopic.php?fF20&tF21523

: KARKAR
.

1 ( )

15

:
13 ( )
1 , 2 , ..., 2+1
, :

1 3 21
...

2 4 2

2 ( ) a, b, c ..,

ac a+c
= b.

2

1
.
2+1

a1 a3  a2 ,

a3 a5  a4 , ...,

a2n1 a2n+1  a2n

a1 (a3 a5 ...a2n1 ) a2n+1 a2 a4 ...a2n

1 ( )

a1 a3 a22

a (a + 2) (a + 2)

a2 + 2a a2 + 2a + 2 0 2

a1
a2n+1 a2 a4 ...a2n

3 ( ) ,

.

a3 a5 a24
(a + 2) (a + 4) (a + 3)2

i1 i+1 2i

a2 + 6a + 82 a2 + 6a + 92 0 2

i = 2, 3, ..., 2.

a21 a2+1  a22

x =

(a + (2 2) ) (a + 2)  (a + (2 1) )2


a2 + 2 (2 1) a + 42 4 2 

a2 + 2 (2 1) a + 42 4 + 1 2

1 3 ...21
2 4 ...2

x2 =

0  2

1 (1 3 )...(23 21 )21

22 24 ...22

1 22 24 ...222 21
22 24 ...22

1
x 
2+1

a1

a1
a1 a3  a22 a1 a3  a2

a2
a3

a3

a3
2
a3 a5  a4 a3 a5  a4

a4
a5

....................................

a21 a2+1 a22

http://www.mathematica.gr/forum/viewtopic.php?fF21&tF23056

a21
a21 a2+1 a2
a2

=


1 21

22
1
2+1

14 (

x3 x2 + 27x 3

a21

a2+1

a21
a1 a3

...
a2 a4
a2

(x + x)2 = 6

a1
a3
a21
a1
 ...
=
a3
a5
a2+1
a2+1

.
16

27a = r1 r2 + r2 r3 + r3 r1 3


3

r12 r22 r32



3
3
= 3 92 a2 = 3 9(r1 + r2 + r3 )2 a2
(1) 
3
3 9 34 a3 = 27a

http://www.mathematica.gr/forum/viewtopic.php?fF21&tF2114

( )
Vieta r1 , r2 , r3 ,

.

r1 r2 =


r2 r3 = r3 r1 r1 = r2 = r3 = .
3

r1 + r2 + r3 = r1 r2 + r2 r3 + r3 r1 = 27a r1 r2 r3 = 3a
-

r1 + r2 + r3 3 3 r1 r2 r3 = 3 3a = 3 3 3(r1 + r2 + r3 )a

a =
x + x =


(r1 + r2 + r3 )2 34 a (1)




2 x +

17

34

2 x +


4

> 1 .

:
15 ( KARKAR)
BC BC ,
BC .

2 ( )

BC. BC !

AB = BC = CA = BD = DC

BS = S T = T C, K AS BD

L AT DC.

BK =

BD
2

BC 
2

DBC =

BK = K L = L C, CK BD,

3
BL CD.

. BC

K , L K K L L.

http://www.mathematica.gr/forum/viewtopic.php?fF22&pF116634#p116634

1 ( ) O BC .
BOC AB : 2 = a : 2
BOK CS A

BK : AC = 0, 5

BS = 0, 5S C =

1
1
BC = a
3
3

TC = 31 a S T = 31 a

16 ( KARKAR) D
AB ABC, (AB = AC),
, AC E ,
BC Z .
AD = CZ , E
BC , AB.
:
, D ;
18

http://www.mathematica.gr/forum/viewtopic.php?fF22&pF117040#p117040

1 ( ) .
BD
ABC EC
AE = BZ = cosB (1)
sinC = sinB =

DE
(3)
sinA =
AE

ES
(2)
EC

M
MA m
=
M Od
MB
n

sinA
=
(3) : (2)
sinB

DE
AE
ES
EC


.
2) .

sinA DE EC (1)
=

sinB ES AE
DE
sinA
= cosB

sinB
ES
sinA
DE
=
ES
sinB cosB
.

sinA BC
=
sinB AC

cosB

AC
BC

=2

AC
BC

DE
=2
ES



: OA1 = m, OB1 = n


M , ,
.
3) :
.

2 ( )
D
E
, E
B 1 : 2.
:
1) .
19

4 ( ) AM
 ABC (AB = AC)
BM =


.
: .



.
3 (KARKAR) (. )
DA = ZC , :

 ) :
( CBA
BD
BD
ZB
=
= BC
AB BM
2
BD
ZB
=2
:
AB
BC

(2)

 ADE,  ABC
 :
BAC
(ADE) AD AE
=
(ABC)
AB AC

DB
=
ZB

(*):

EC = EA

: (1).

 ZDB  AMB

DE
ES
, EA =
EC =

EC DB
=
()
EA
ZB
AM , ,

BC

AC=AB

(ADE) AD AE
=
: (3)
(ABC)
AB2

DE
ES
=

DE
DE
ES =
ES =
2
2

 ECZ,  ABC
 + ACB
 = 1800 () :
ECZ
(ECZ) CE CZ
=
(ABC) CB AC

AC=AB,CZ=AD

(ECZ) CE AD
: (4)
=
(ABC) CB AB

(3) : (4)

(ADE)
(ABC)
(ECZ)
(ABC)

ADAE
AB2 AC=AB

CEAD
CBAB

20

(ADE) CB AE
: (5)
=
(ECZ) AB CE

 ABC
ZED :
1=

ZB CE AD ZC=AD

ZC AE DB
ZB CE

: (6)
1=
DB AE

(ADE) BC AE ZB CE

=
(ECZ) AB CE BC AE

(5) (6)

(ADE)
(ECZ)

ED
ES (

ED
=
ES
BD
ED
=2
ES
BC

() ,
E AC.  ABC
DEZ,

AD, CZ)

BD
ZB BC (2): ZB
AB =2 BC

AB BD
BC

= 2 ED = 2ES
BD

DA ZB EC

= 1 =
DB ZC EA

.
4 ( )

ZB
EA
=
, (1 )
EC DB
F , C AB
CF  ZD, ,
BC
BC
EA
ZB
=
, (2) (1), (2) =
=
, (3)
DB
BF
EC
BF

E :
P ABCP
Q
AP, PQ = BF. P
CQ,
AC E,
(3). , D
E AB, AD = CZ ,
Z BC DE () .
5 ( ) CT , K
D AM , N AB
EN  BC , I EN AM L
AC T L  BC .
ADK, BDZ CT  DZ , :

 DBZ
AEC, ,

ED CZ AB

= 1 =
EZ CB AD
ED EZ
=
, (1 )
BC
AB

BD T D
DK
=
=
DK = T D
AD
BZ
ZC

( AD = CZ ).
 MAB,  S EZ
ES
EZ
2ES
=
=
, (2) ( BC = 2BM ).

AB

BM

(1), (2)


T K AT
L
ADK, AIE , :

BC

AT
AK AD AT
=
=
=
AI
AE
AC
AB

ED 2ES
=
=
=
BC
BC
ED = 2ES , (3)

, I
ABC , K AT L

() .
21

NE
AT
. EI = EC =
,
AB
2
ES C, EDN
ES
1

= . , E
ED
2
A

,
ABC .

22

:
17 (

KARKAR) S
1
y = x, A
2
1
y = x : (S A) = 6.
2
M S A.




MA = 3

2
x1 + x2 2
x1 x2
1
+ x2
=3
x2
2

 x x 2
2
1
2

 x + x 2
2
1
4

http://www.mathematica.gr/forum/viewtopic.php?fF23&tF21532

x2M
S (2, ) ,

4y2M

1 ( )

A (2, ) ,

 x 2

(2yM )2 +

62

M (x, y) .

x2M
36

=9
=9
=1

y2M
+
2 = 1,
3
2

2a = 12, 2b = 3.
:

18 ( )

2 2

x=

= x

+ = 2y

y=

x2 y2

= 1 C2 : x2 + y2 = a2
a2 b2
a, b > 0. K(x0 , y0 ), x0 > 0,
b
C2 ( ) : y = x C1 ,
a
C2 K
E .
C1 :

:
http://www.mathematica.gr/forum/viewtopic.php?fF23&tF21164

1 ( )
K(x0 , y0 )

(S A)2 = 62
( )2 + (2 + 2)2 = 36

x0 =

x + (4y) = 36
2

x2
36

x2
62

y2
36
16
2

a2 + b2

a2
ba
ba
y0 =
=
2
2

a +b

(
E(, 0)
). K
xx0 + yy0 = a2

=1

y
+
2 = 1.
3
2

a2

. ( E(, 0)
x0 > 0).
E(, 0) x0 = a2 x0 =

2 ( ) S x1 ,

x + x x x 
1
2
1
2
,
.
M

a2

x1 
2


x2 
, A x2 , .
2

2 ( ) C2

K xx0 + yy0 = a2 . y0 = x0
a

2
4
S A = 6 :

23

E(c, 0), :
x20 + y20 = a2
b2
+ 2 x20 = a2
a
a2 x20 + b2 x20 = a4 .

cx0 = a2

x20

c2 x20 = a4
(a2 + b2 )x20 = a4 ,

(11)

(11).

24

:
19 ( )
X t1 , t2 , ..., tv ,
x  0 , R


g (x) =

x(t1 +t2 +...+tv )2v x


,
x2 4
v x

x=2

0<x2

2.

g(3) = 20
3v x 2v x
= 20
94


.

(12)

v x = 100

1. g xo = 2 ,
= 21 .

t1 + t2 + ... + tv = 100
v

ti = 100.

2. g

A(3, 20) , vi=1 ti = 100.

i=1

3. vi=1 ti fi = 1, fi
, v
.

3.
:

v


ti fi = 1

i=1

http://www.mathematica.gr/forum/viewtopic.php?fF18&tF21166&startF20

t1 f1 + t2 f2 + ... + tv fv = 1
v1
v2
vv
t1 + t2 + ... + tv = 1
v
v
v
t1 v1 + t2 v2 + ... + tv vv = v

1 ( )
1.

t1 + t2 + ... + tv

v
t1 + t2 + ... + tv = v x

t1 + t2 + ... + tv = v

x =

v = 100.
(12)

x(t +t +...+t )2v x

1 2 x2 4 v
g (x) =

v x

20 ( )
s, s  0.

R 0 < 2 s v.

,0 < x  2
.
,x=2

x(t1 + t2 + ... + tv ) 2v x
x2
x2 4
v x (x 2)
= lim
x2 (x 2)(x + 2)
v x
v x
=
= lim
x2 (x + 2)
4

lim g(x) = lim

x2

g(2) =

v x
2

http://www.mathematica.gr/forum/viewtopic.php?fF18&tF5875

1 ( ) :

R = tmax tmin

= (tmax x) (tmin x)

xo = 2
v x
4

R = (tmax x)2 + (tmin x)2 2 (tmax x) (tmin x) .


2

v x
2

1
2

= .

:
25

, . (
a < x < < x < a ). ti , i = 1, 2, ...,

[(tmax x) + (tmin x)]2 0

, :

(tmax x)2 + (tmin x)2 +




2 (tmax x ) (tmin x) > 0

s2 =

2 (tmax x)2 + (tmin x )2 (tmax x)2

(ti x )2 .

i=1

+ (tmin x)2

ti

2 (tmax x) (tmin x)


(ti x )2 <

2 (tmax x)2 + (tmin x )2 R2

 

2 (tmax x) + (tmin x )
2

i=1

(tmax x)2 + (tmin x)2

(ti x)2

(ti x )2 < s2

| ti x | < s .

R.

(ti x)2

i=1


 2 
(ti x)2
2 (tmax x)2 + (tmin x)2
i=1


2 (tmax x)2 + (tmin x)2 2s2
 


2 (tmax x)2 + (tmin x)2 s 2

:
R s 2

| tmax x | < s

tmax x < s

tmax < s + x

(13)

| tmin x | < s

x tmin < s

tmin > x s .

(14)

0

, 3
.

(13),(14) :

tmax tmin < 2 s R < 2 s .

2 ( ) s  0
2

26

:
21 ( )
z, w
4|z|2 2zw = 1,

2.

|w|2 2zw = 3
ww 2zw = 3

|w|2 2zw = 3.

w(w 2z) = 3

1. |2z w| = 2.

|w| |w 2z| = 3
2 |w| = 3

2. z w
,
.

|w| =

3
2

3. |6z + w|.

4 |z|2 2zw = 1

4.
z w.

zw=wz

4zz 2zw = 1
4zz 2zw = 1
2z(2z w) = 1
1

http://www.mathematica.gr/forum/viewtopic.php?fF51&tF21713&startF60

|2z| |2z w| = 1
4 |z| = 1
1
|z| =
4

( )
1.

z
1
1 = ,
4
w
3
2 =
2

4 |z|2 2zw = 1
2zw = 4 |z|2 1

4 |z|2 1
R.
zw =
2

3.

zw = zw =

zw = zw

1
4 16
1

1
4

1
2

43
2

|6z + w|2 = (6z + w)(6z + w)

|2z w| = (2z w)(2z w)


2

= 36zz + 6zw + 6wz + ww

= 4zz 2zw 2wz + ww

zw=zw

==== 36 |z|2 + 12zw + |w|2



36
3
9
=
+ 12 +

= 4 |z| 2zw + |w| 2zw


2

= 4 |z|2 2zw + |w|2 2zw

16
8
9 18 9
=
+
4
4
4
=0

= 4.

|2z w| = 2
27

3
8

|6z + w| = 0. A
6z B w, A B.

http://www.mathematica.gr/forum/viewtopic.php?fF51&tF21713&startF100

4.

( )
1. f x 
.
f (1) = 1 > 0, f (x) > 0 x 

|z w|2 = (z w)(z w)
= zz zw wz + ww
zw=wz

==== |z|2 2zw + |w|2



1
3
9
=
2 +
16
8
1
3 9
=
+ +
16 4 4
1
=3+
16
49
=
16

|z|
1

f (x) > 0 x  |z| = 1.


z
.
3. z = a + bi, a, b  |z + z| = 2 |a|
|z z| = 2 |b|.

7
.
4

|z w| =

(|z + z| 3) x3 + x
=
x (|z z| 3) x2 + x
(2 |a| 3) x3 + x
=
lim
x (2 |b| 3) x2 + x
(2 |a| 3) x3
=
lim
x (2 |b| 3) x2


2 |a| 3
x = .
lim
x 2 |b| 3
lim

7
= .
4
: 3
A 6z B w
, z,
, O, B

|z w|min = |z w|max =

OB O .

: z
3
|a| 1 < , 2 |a| 3 < 0.
2
2 |b| 3 < 0.

3 1
7
|z w| = B = BO + O = + = .
2

4. C f
x = 0, x = 1

22 ( )
f R, f (x)  0 x R

|z|

f (x) dx = 0 x 

2.

f (x)dx = 0

1

f (x) dx 1 f (x) > 0

f (1 ) = 1 .

x .

1. f (x) > 0.

1
f (x) dx < |z + 2z|

2. z.

(|z + z| 3) x3 + x
.
x (|z z| 3) x2 + x

|z| + 2 |z|

3. lim

= |z| + 2 |z|

4. f x x x = 0
x = 1 |z + 2z|,

= 3 |z|
= 3.

f (t)dt = 3 x2 + 6 x 6

1

(0, 1).

f (x) dx 3 < 0.
0

28

(15)

x
h (x) =

1

f (t) dt 3 x 6 x + 6
2

h (0) = 6 > 0 h (1) =

x [0, 1]. h (x) [0, 1]

x

x

(15).

f (t) dt ( f (x)

Bolzano,
xo (0, 1)

xo

f (t) dt ) 3 x2 6 x + 6

f (x) dx 3 < 0

h (xo ) = 0

( ).

f (t) dt 3t2 6t + 6 = 0.
0

29

:
1. a = 0

23 ( )
f : R R :

2. f (b)  0

f (x)
lim
= 3.
x2 x 2
:

3. f (1 b) = 1
4. f R.

f (x)
.
x2 sin(x)
lim

http://www.mathematica.gr/forum/viewtopic.php?fF52&tF11884
http://www.mathematica.gr/forum/viewtopic.php?fF52&tF10921

1 ( )

1 ( )

1. x = 0, x = 1 :

f (x)
=
x2 sin(x)
lim

f (x)
x2
lim
x2 sin(x)
x2

f (1) f (0) = f (a + b) = f (b),


f 1 1,

sin(x)
x2 x 2
sin(u)
= lim
u0
u
sin(u)

= lim
u0
u
= .
=

lim

a + b = b a = 0.
2. : f (x) f (1 x) = f (b). f (b) = 0,
f (x) f (1 x) = 0. x = 0, x = 2, :
f (0) = 0 f (1) = 0

2 ( )

f (2) = 0 f (1) = 0.

f (x)
f (x) x 2
= lim

x2 sin 2x
x2 x 2 sin x
x2
f (x)

= lim
x2 x 2 sin (2 x)
f (x)
x2
= lim
x2 x 2 sin (x 2)


(x 2) 1
f (x)
lim
= lim
x2 x 2 x2 sin (x 2)

lim

f (0) = f (2)
f (0) = f (1)
f (1) = f (2)
f (1) = f (1)

f 1 1,
0=2

24 ( )
f R 1 1
:

f (x) f (1 x) = f (ax + b),

2 = 1 1 = 2

1 = 1 ,

, f (b)  0.
3. x = b, :

x R

f (b) f (1 b) = f (b) f (1 b) = 1.

:
30

4. x0 R, f (x0 ) = 0.
x = x0 , :

2 ( )
2. f (b) = 0 x = 0

f (x) f (1 x) = f (x0 ) f (x) f (1 x) = 0.

f (0) = 0 f (1) = 0 b = 0 b = 1

x = 2, x = 3

x = 3

f (2) = 0 f (1) = 0
f (3) = 0 f (2) = 0,

f (3) = 0 f (2) = 0 b = 3 b = 2

f (2) = f (x0 )

f (1) = f (x0 )

f (3) = f (x0 )

f (2) = f (x0 )

4. f
k
0 x = k 0 = f (b) .

R.

f 1 1,

2 = x0

1 = x0

3 = x0

2 = x0 ,

3 (KARKAR)
2. f (b) = 0.

f (1/2) f (1/2) = 0
f (1) f (0) = 0,
f (1/2) = 0 f (1), f (0)
0, .

2 = x0 = 3

2 = x0 = 2

1 = x0 = 3

1 = x0 = 2,

3. x = b

, x0
R f (x0 ) = 0, 0
, f (A)  R

4. f (x) = 0 x f (b) = 0 .

31

:
25 ( ) f
R,
f
f (x + f (x)) = f (x)
x R
(1)

x.

. ,
...
.
f (x)

1. f

(
) x + f (x)
A x B (1)
f (A) = f (B) Rolle

2.
(1)
3. f a, b a < b.
u (a, b)
f (u)  0.


f (x)  0, x R
... f (x)  0 :
[A, B] [B, A]

x R
Rolle f [A, B] :
f () = 0 f (x)  0 x R.

http://www.mathematica.gr/forum/viewtopic.php?fF53&tF22417

1 ( )
2.
.... ;
.

1.
1
f (x) . (
)

f (x) = c
(1)

. . .
(
) :
f (x)

f
(1) f .

( )
2 . f (x)  x R
x R : f (x) = 0.

deg( f ) = n > 2, deg( f ) = n 1


deg(x + f (x)) = n 1, n > 2 0, 1, 2.


.. f (x)
. f (x) f (x) .
. 1 1

n(n 1) = n > 2
n = 0 f (x) = c

f (1)
x + f (x)
A x B f (A) = f (B), f :
1 1

n = 1 f (x) = ax + b, a  0.
n = 2 f (x) = ax2 + bx + c...
a = 1, b = 0, c = 0 f (x) = x2 ...

A = B x + f (x) = x f (x) = 0
32

3. f (a) = f (b) = 0

(2)

f (x) = 0, x (a, b).

: u (a, b) f (u)  0

f (x) < 0
b.

1 f
1 1 (a, b)
R.
y (a, b) y + f (y) (a, b)
f 1 1.

26 ( )
f
[4, 10] f (A) = [1, 5]
f (4) = 2, f (10) = 4.
1 , 2 (4, 10) : f (1 ) = f (2 ) = 0.

f (x)

(a, b)

f (x) > 0

(2)

(
f ,
) a < x + f (x) <?
b.
x
, x
x + f (x).

( )
x1 , x2
[4, 10]


f (x)

a x + f (x).
; . (


).

1 < f (4) < 5 1 < f (10) < 5


x1 , x2 (4, 10), , Fermat
f (x1 ) = f (x2 ) = 0
f (x1 ) = 1, f (x2 ) = 5 x1 < x2 .
. ()

http://www.mathematica.gr/forum/viewtopic.php?fF53&tF22505

1 = f (x1 ) f (x) f (x2 ) = 5, x [4, 10]

a, b (x1 , x2 ) : f (a) = 2, f (b) = 4.

lim (x + f (x)) = a + f (a) = a + 0 = a.

1. a < b 2 Rolle [4, a] [b, 10]

xa+

x+ f (x) a
b.

2. b < a () [a, x2 ], [x1 , b]

x0 (a, b) : x0 + f (x0 ) < b

c > a : f (c) = 4 d < b : f (d) = 2

a < x0 + f (x0 )
x0 , x0 + f (x0 ) (a, b) f 1 1
(1)

2 Rolle [4, d] [c, 10]


.

x0 + f (x0 ) = x0 f (x0 ) = 0

( a = b 2 = 4).

33

:
27 ( )
f : [0, 1] [0, +) ,

!
!
!
"
g (t) =

t

1+2

f (u)du t, t [0, 1] g (0) = 1.


0

f (t) 1 + 2

f (u)du, t [0, 1].

g
[0, x] x (0, 1] g (t) 0, t [0, 1].

(0, x)

g (x) g (0)
g () =
x0

f (t) t + 1, t [0, 1].

http://www.mathematica.gr/forum/viewtopic.php?fF54&tF23072

( )
f , f (t) 0 t [0, 1]

1+2

x
x(0,1]

f (u)du 0

!
!
!
"

x

1+2

t
1+2

f (u)du x 1

t
2

#x

!
!
!
"

f (u)du > 0

f (u)du x 1 0
0

x

1+2

f (u)du x + 1
0

t
f (t) 1 + 2

!
!
!
"

f (u)du

f (t)

t

1+2

!
!
!
"

f (u)du

t = 0

t

f 2 (t) 1 + 2

t

1+2

f (t)
1+2

#t

f (u)du .
0

f (u)du

.
28 ( )
f [0, 1]

f (u)du

f (t) t + 1 t (0, 1] .

f (t)0

f (t)

1+2
0

!
!
!
"
| f (t)|

t

f (u)du

f (t)dt

1 x2
x [0, 1].
2


!
!

!
t
"

1 + 2
f (u)du t 0

( f (x)) dx
2

34

x f (x)dx.
0

:
http://www.mathematica.gr/forum/viewtopic.php?fF54&tF2583

1

( )
:

1 1

1

f (t)dt
0

( f (x) x f (x))dx

1 x2
dx
2

(x)

1
+

f (t)dt

x [0, 1]

f 2 (x) 2 x f (x) + x2 0,

x [0, 1]

f (x) x f (x) x f (x) x2 ,

1

1
f (x)dx
2

(2)
0

( f (x) x)2 0,

1
3

( f 2 (x) x f (x))dx 0

x f (x)dx

1

1
x f (x)dx
3
1
x f (x)dx
3

1

x2 dx

(2) 1 1
= 0.

+1
f (t)dt

1

1

1

( f 2 (x) x f (x))dx

1
x f (x)dx

1

1

1
2

x [0, 1]

35

x f (x)dx.
0

:
29 ( )
:

sin(cos x) = x

g [1, 1]
cos(sin x) (
) x ()

cos(sin x) = x

f (x) = cos x sin(sin x) 1,

.
;

g (x) < 0, x [1, 1] cos x, sin(sin x)


(1, 1), g ,
.
,
g 0,

( - )
sin(cos x) = x ,
[1, 1] sin(cos x) [1, 1].

g(0) = 1 > 0



2
2
<
g
< 0,
= cos

x [1, 1].

f [1, 1]
sin(cos x) (
) x ()

Bolzano




g(x) = 0 0,

,
4
(1, 1),
.
x1 x2
.

f (x) = sin x cos(cos x) 1, x [1, 1].


f (x) < 0, x [1, 1] sin x, cos(cos x)
(1, 1), f ,
.
,
f 0,

[1, 1] 4
cos(sin x) ( ) x
()

http://www.mathematica.gr/forum/viewtopic.php?fF55&pF116855#p116855

f (x) = sin(cos x) x,

x [1, 1].

x1 = sin(cos x1 ) = |sin(cos x1 )| < |cos x1 | = cos x1 ,

[1, 1] 4
sin(cos x) ( ) x
()

x1 < cos x1

(3)

f (0) = sin 1 > 0



2
2
<
f
< 0,
= sin
4

Bolzano

0 < sin x2 < x2 <

cos(sin x2 ) > cos x2




f (x) = 0 0,

x2 = cos(sin x2 ) > cos x2 ,

,
4
(1, 1),
.
cos(sin x) = x ,
[1, 1] cos(sin x) [1, 1].

g(x) = cos(sin x) x,

x2 > cos x2 .

x1 x2 ,
(3)

(4)

x1 < cos x1 cos x2 < x2 x1 < x2 ,


.

x1 < x2 .

x [1, 1].
36

(4)

30 ( )
z w  2

|z 1| = 1 w =

2z 1
.
z+1

f (x) = ln x

( ) f
.
3.
f , ,


|w| x.

lim+ f (x) = lim+ ln x 1 x =

1. f ,

x0

2. f ,

3. f ,

2 1

4. lim x f
x0

x0

f (1) = ln 1

(x) .

11=0

f .
,
http://www.mathematica.gr/forum/viewtopic.php?fF55&pF113735#p113735

f () = (, 0] .

( )
1. w :

w=

4. ( ). f
( )
(, 0] (0, 1],

w+1
2z 1
,
z=
z+1
2w

f (1) = 0 f 1 (0) = 1



1
= 1 |2w 1| = |w 2|
|z 1| = 1 2w+

1
w
(2w 1) (2w 1) = (w 2) (w 2)
ww = 1
|w| = 1
D f = (0, 1]

2. : f (x) = ln x 1 x,
x (0, 1)

f (x) =

2 1x

lim f 1 (x) = 1.

x0


lim x2 f 1 (x) = 0 1 = 0.

x0

> 0,

37

:
31 ( )
f : R R,

ex f (x) = f 2 (x)
f (x) ex f (x) f
#x

#x
et f (t)dt
et f (t)dt

f (x) e0
e0
f

f (x)
x
# t

f (0) = 1
ex f (x) = f 2 (x) x R.

(x) = 0
(x) = 0

= 0

e f (t)dt

e0
f (x)
#x

= c.

et f (t)dt

e0

http://www.mathematica.gr/forum/viewtopic.php?fF56&pF113616

x = 0 c = 1.
1 ( ) ,

. x0 R f (x0 )  0.

#x

f (x)
#x

= 1 f (x) = e0

I
.

ex f (x) = f 2 (x)
f (x)
= ex
f 2 (x)


 
1
= ex

f (x)

f (x)
= ex , x I
f 2 (x)


1
= ex , x I

f (x)
1

ex + c

I
, a, f (a) = 0.
0 = f (a) = lim
xa

ex + c

= ex + m.
f (x)
x = 0 m = 0

, x I

<0

et f (t)dt

e0
x R.

f J
x0 f (x) f (x0 )
x J .

f (x) =

et f (t)dt

f (x)

= ex f (x) = ex

xR

Zaris


.

.
I = R
c = 0.

32 ( )

f : R R f (x) =

f (x) = ex , x R,
.


g(x) =
0

38

dt

2 ( )

t2

ex (1+t )
dt.
1 + t2
2

g : R R

1. x0 R,

lim

x2 (1+t2 )

xx0

2.

dt =

g(x) g(x0 )
=
x x0

ex0 (1+t ) dt.


2

1 + t2

x20

x0

g (x0 ) = 2e

g(x) g(x0 )
2
lim
= 2 x0 ex0
x x0
xx0

x R.

x+

lim

x+

t2

dt =

1. t [0, 1] x0 R.

e
ex

B x = max{x2 (1 + t2 ), x20 (1 + t2 )}.



ex
[A x , B x ]
2

ex0 (1+t
x x0

(1+t2 )

= e (x + x0 )(1 + t2 )

= (t, x) x20 (1 + t2 ), x2 (1 + t2 ) .

ex

(x + x0 ) > 0,

ex (1+t ) ex0 (1+t )

= 2
x (1 + t2 ) x20 (1 + t2 )
2

ex (1+t ) ex0 (1+t )

x2 (1 + t2 ) x20 (1 + t2 )
2

ex (1+t ) ex0 (1+t )

= 2
x (1 + t2 ) x20 (1 + t2 )
2

A x = min{x2 (1 + t2 ), x20 (1 + t2 )}

x20 (1+t2 )

e(x0 t) dt.


t2 )

( )

x2 (1+t2 )

x0 0, x x0 x < 0.
,
t [0, 1],

ex x20 (1 +t2 ), x2 (1 +

http://www.mathematica.gr/forum/viewtopic.php?fF56&pF113616

e =

3. lim g(x) = 0

= (t, x) (A x , B x ).

ex

 1

 1

x2 (1+t2 )
x20 (1+t2 )
e
dt
e
dt
0
0

0
 1

x2 (1+t2 )
x20 (1+t2 )

e
dt
e



0

= |x
2

x20 |

e (1 + t2 )dt

ex

xx0

ex

(1+t2 )

dt =

(x + x0 )(1 + t2 ) <
ex0 (1+t
x x0

(1+t2 )

ex0 (1+t ) (x + x0 )(1 + t2 )


2

(1+t2 )

ex

lim

(x + x0 )(1 + t2 ) <

ex0 (1+t ) (x + x0 )(1 + t2 ),

xx0

(1+t2 )

e (x + x0 )(1 + t2 ) <

 2|x2 x20 | 0,


f (x) + g(x) =

ex0 (1+t )
dt.
x x0

(1+t2 )

et dt

ex

2. x0 R,

ex0 (1+t ) dt.


2

39

<

(x + x0 )

ex

(1+t2 )

ex

(1+t2 )

g(x) g(x0 )
x x0

dt <


0

1 + t2

c = f (0) + g(0)
 1
1
dt
=
1
+
t2
0
 /4
1
1
t=tan x

dx
=
2
1 + tan x cos2 x
0
 /4
=
1 dx

ex0 (1+t )
dt <
x x0

(x + x0 )

ex0 (1+t ) dt
2

(5)

x x0 ,
1,

lim

xx0

g(x) g(x0 )
2
= 2 x0 ex0
x x0

e(x0 t) dt.
2

f (x) + g(x) =

x0 > 0,
, (5)
.

g (x) = 2 xex

x2

xt=u

0

= 2e

ex , f
R, 2

x2

( f (x) + g(x)) = 2e

ex (1+0)
dt
1 + t2
0
 1
1
x2
dt
=e
2
0 1+t
2 x+
0.
= ex

du.

ex (1+t )
dt
1 + t2

u2

x R.

0 g(x)

e(xt) dt
x

3.

x R

t2

e
0

2ex

dt


2 , lim f (x) + g(x) =

eu du
2

lim f (x) = lim

= 0,

x+

x+

lim

x+

f (x) + g(x) = c x R.

40

x2

e
0

x+

ex

:
33 (

x
x
lim
x
x+ (ln x) + x
2 ln x

cos

A
2


=

=

http://www.mathematica.gr/forum/viewtopic.php?pF78536

cos

( ) x > e

2 ln x > 2 > 1, x2 ln x x > 0.
0<

A
2


=

1 + cos A
=
2

1+

b2 +c2 a2
2bc

(b + c)2 a2

4bc
s (s a)
.
bc

(7)

(6) (7) :

x2 ln x
x2 ln x
x2 ln x x

().
(ln x)x + x (ln x)x + x (ln x)x

AI 2 =

x ee ln x e
() <

bc (s a)
.
s

, :

2 ln x

ex

AB AC < AI 2

0
x +.
, 2(ln x)2 x .
( , ..
2(ln x)2 x
1 .
l Hospital

b2 c2
(a+b)(a+c)
bc(sa)
s

AB AC
<1
AI 2

<1

bcs
<1
(a + b) (a + c) (s a)
bc (a + b + c) < (a + b) (a + c) (b + c a)

34 ( )

ABC I .


BI, CI AC, AB B , C .
AB AC ,
AI 2 A
, .


a < ab2 + ac2
3

a2 < b2 + c2
A < 90 .
.

http://www.mathematica.gr/forum/viewtopic.php?fF69&tF19524

2 ( )
, .
AD I
ABC ,
ABD

1 ( )

AB =

bc
a+c

AC =

bc
.
a+b

, IKAB,

AK
A sa
cos =
cos =
2
AI
2
AI
A

AI =
(6)

cAD
.
BD + c

s ABC.
, :

BD =
41

ac
.
b+c

AI =

,
.

AD (b + c) AD (b + c)
=
.
a+b+c
2s

AI 2 =

(b + c)2
AD2 .
2
(2 s)

AI 2 =

AD2 =

4bcs (s a)

(b + c)2

bc (s a)
.
s

42

,
,

:
x0 R f (x0 ) > ax0 , 2.

35 ( )
a > 1, f : R (0, +)

1 = f (0) = f (x0 x0 ) f (x0 ) f (x0 ) > ax0 ax0 = 1,

1. f (x) ax , x R

2. f (x) = ax , x R.

2. f (x + y) f (x) f (y), x, y R.

36 ( )

http://www.mathematica.gr/forum/viewtopic.php?fF109&tF21529

2x+1 = 2[x] + 2{x} .

1 ( ) g(x) = loga f (x),


g(x) x

(8)

g(x + y) g(x) + g(y)

(9)

http://www.mathematica.gr/forum/viewtopic.php?fF109&tF20461

1 ( ) x =

[x] + {x}

(8),(9) x = y = 0

0 g(0) 0

(10)

(9) y x (10)
g(x) + g(x) 0 g(x) g(x),
(1)

2[x] =

f (y) =

x x g(x) x
, (8) g(x) = x
x.
,
f (x) = ax . 2 ( ) -

2 2{x} 1

(14)

y
y [1, 2) .
2y 1

f [1, 2)


f ([1, 2)) =

: 1. x = 0

2
,1 .
3

[x]
(14) [x] 1 2[x] 2 [x] 1
2[x] 0
f .

[x] = 0 2{x} = 1
{x} = 0
x = 0.

(11)

2. x = y = 0

f (0) f 2 (0)
f 2 (0) f (0) 0
f (0) ( f (0) 1) 0
f (0) 1

2{x}

2{x} = y 1 y < 2

g(x) x g(x) x.

f (0) 1

(13)

0 {x} < 1
(13) :

g(0) = 0

2[x] 2 2{x} 1 = 2{x}

2 ( )

(12)

x  0 2x 2[x] = 2{x} 2x

(11) (12) f (0) = 1.


43

x < 0 2x+1  2 2[x]+{x} 2x  2x 2x  1

2x 2[x]  0  2{x} 2x

x = {x} = [x]

.
x = 0.

x = 0.

44

:
KB = KD = BC = CD = DA , (3)

37 ( )
ABCD
BCD, DAB 80o , 40o , .
BC = CD = DA,
ABC = x.

(2) DAB = 40o ,


(K, KB) A, D
KB = KD.
 KAD KA =
KD = CD = AD AKD = 60 o =
ABD = 30o , (4)

, ABC = ABD + DBC = 30 o + 50o = 80o =


x = 80o .
2 ( )
DCB AB
G BD (
 CDD ), E.

http://www.mathematica.gr/forum/viewtopic.php?fF110&tF19142

1 ( ) K,
C M BD KBCD BC = CD
,
.

GAD, GCD 40o


CD = DA, GD
AC AGC.
AGD = DGC = CGB = 60 o , (1)

DBC = 50o

BCK

BCD
2

40o

(1)  BGC ABC =


180o BCGCGB = 80o = x = 80o
.

, (1 )
KBCD DKB = BCD =
DKB = 80o , (2)

3 ( ) BD.

45

http://www.mathematica.gr/forum/viewtopic.php?fF110&tF20616

1 ( ) :

 BDC DCB = 80o BC = DC = a


= DBC = CDB = 50o
. :

BC
BD
=
=
80o
50o

BD
a

80o
50o

240o 40o
(90o 50o )

240o

= AM =

 ABD . :
BD
40o

AD
ABD

BD
a

2 3
a
2

40o
ABD

= 240o = ABD =

: BM = BN =

, ABD < 140o , :


40o
ABD

a 2
2

NC = a a

3
=
2


 ABC N MS

1
= ABD = 30o
2

BN CM AS

=1
NC
MA S B

a 3
AS
31
2

= AS = a
=
, (1 )

4
a( 2 3 ) AS + a
:

, x = 50o + 30o = 80o


38 ( KARKAR)
 ABC, M AC.
(B, BM) BC N.
N M BA S .
,
 AMS .

S M

 S BM S M =

6
, (2)
a
4
:

x = 180o 60o = 120o , (3)


(
)
:

cos y =

= y = 45o , (4)
: S MA = 180o 120o 45o = 15o , (5)
2
2


 BMA  S MA
:

(AMB)
SA
(AS M)
(AMB) = ... =
=
= (AS M) =
S
A
AB
AB

3 3 2
a ( (1)
32
)

46

2) MDS B. AMD (60o , 30o , 90o )

2 ( ) 1) BM
- -  ABC
a, (
- ) N MC = S MA =

NBM

MAS = 120o MS A = 45o

AM =
(

= 15
o

, AD =

30o

, AS =

a 3

) MD = DS =

31

a 3
4

4
4
4

 S DM

a 6
S M =
4

2
AS MD a 3 3
3) (AMS ) =
=
2
32

47

, ,

:
n 17 .

39 ( )
n 17 .
n .

40 ( )
P,

, P(0) = 0
3
P (x + 1) = (P(x) + 1)3 .

http://www.mathematica.gr/forum/viewtopic.php?fF111&tF14393

( ) n = 1, 4, 9
n .

m n ,
m
m n ,

m+(n 1) .
1 + 3k + 8l ,
k, l . l = 0, 1, 2 1 + 3k + 8l
n n 17 (

(mod 3)).

http://www.mathematica.gr/forum/viewtopic.php?fF111&tF15622

( ) x = 0 P(1) = 1.
x1 = 1 xn+1 = (xn + 1)3
n 1. P(xn ) = xn
xn > 0 n 1.
xn+1 > xn n 1,
{xn }n1 P,
P(x) x. ,
, , P(x) = x.

48

41 ( )
ABCD AB = AD B = D = 90 o .
DC, BC, E, Z ,
AE, DZ .

BE AZ.

http://www.mathematica.gr/forum/viewtopic.php?fF112&tF22060

BZ AZ ,
: (AZ )(AZ) =

1 ( )
(
) (AE)2 (AB)2 = (ZE)2 (ZB)2 , (1)
AE DZ = (AZ)2 (AD)2 = (ZE)2 (DE)2 , (2)

(AE )(AE) = (AD)2 = (AD)2 = (AB)2

4 ( )
(A, AD = AB), ED, ZB
DZ AE .
, E
DZ , Z
, BE .

, BE AZ
.

(2) = (AB)2 + (ZB)2 (AD)2 = (ZE)2 (AE)2 +


(AD)2 , (3)
(3) AB = AD = (1)
.

42 ( )
I  ABC
IA, IB, IC
D, E, Z ,

2 ( )
A w = (AB)2 = (AD)2 .

IA IB IC
+
+
3
ID IE IZ

http://www.mathematica.gr/forum/viewtopic.php?fF112&tF22976

1 ( )  BID

BC DC
C1 C2 , AD AB ,
E E ( C1 AE )
E , D, E Z .
Z Z , Z
C2 , BC .
ABZE
(), B, Z E
( ),
BE AZ .

, BID = IBD =

ID = BD.

3 ( ) E ,
AE, DZ EZ , AZ (
2 ).

A + B
2

 BKD,  ABD :

49

IA
b+c
AI + ID
c
AD AB
=
=
= ac =
+1 =
BD BK
ID
ID
a
b+c
IA b c
= + 1.
=
ID a a
IB c a
IC a b
= + 1
= + 1.
:
IE b b
IZ
c c
IA IB IC
+
+
=

:
ID IE IZ


a
b
b
c
c
a
b + a + c + b + a + c 3

IA IB IC
+
+
=
ID IE IZ
(IA)2
(IB)2
(IC)2
+
+
=
(IA)(ID) (IB)(IE) (IC)(IZ)
(IA)2 + (IB)2 + (IC)2
.
2Rr

(IA)2 + (IB)2 + (IC)2 6Rr.


2 + 2 + 2 3 = 3.

(IA)2 + (IB)2 + (IC)2 = s2 + r2 8Rr ( ..


(IA)2 = r2 + (s a)2 ).

2 ( )
Euler

, s2 + r2 8Rr 6Rr,
s2 14Rr r2 .

(IA)(ID) = R2 (IO)2 = 2Rr


(IB)(IE) = (IC)(IZ) = 2Rr

,
s 16Rr 5r2 R 2r. ( Gerretsen Euler,
).
2

50

:
43 ( 1995)
, :



.
a + 3b + c 6.
(a, b, c):


(.

), ,

() (1, 3, 0)

() (2, 1, 1)
() (2, 2, 0)


,
.

d, e, f
4,5,6. (),
, d 3.

e + 3 f 6. d + e + f = 5
d = 3, e = 0, f = 2.

. ()
d 4 e + 3 f
6. d + e + f 6, .
(), a + 3b + c = 6,

.



.

.
.


.

. d 3
().

http://www.mathematica.gr/forum/viewtopic.php?pF116050

( )
18.
18 (
)
.
18.


.

10.
.
X

X
X
X
X

.
, ,
4 .
a, b, c
,
3a + b 6
( )

44 ( )
a, b, c
a + b + c = 1,
1+c
1
1+a 1+b
+
+

.
b + c c + a a + b abc

51


3 -
ab + ac + bc 3 a2 b2 c2
3
: 3 a2 b2 c2 3abc
a2 b2 c2 a3 b3 c3 a, b, c (0, 1)
2 ( )

http://www.mathematica.gr/forum/viewtopic.php?pF118939

1 ( ) :
1+a
1+b
1+c
1
+
+


1a
1b
1c
abc

3abc (ab + bc + ac) (a + b + c) + 3


1

(1 a)(1 b)(1 c)
abc
3abc (ab + bc + ac) + 2

abc

1+a

b+c

(b + c)(a + c)(a + b)
abc

=
=

-
(-)

(b + c)(a + c)(a + b) 8abc


(b + c)(a + c)(a + b)
8
abc

b+c
 bc
(a2 + a)
(a2 + a)
b+c
4


2ab + (b + c)a2


2ab +


(1 a)a2
4

(a + b + c)2
4

a3

()

1
4

1
9

()

3
3
3
3 a + b + c
a+b+c
1

a3 + b3 + c3

: 8 3abc (ab + bc + ac) + 2


6 + ab + ac + bc 3abc

52

2
9

:
a1 = b1 = 1 . a3 = b3
(
) a3 = 0 b3 = 2

45 ( vzf)

,
.

S (x) = x + x2 + x2 + x3 + x3 + x4

T (x) = x + x3 + x4 + x5 + x6 + x8 .

http://www.mathematica.gr/forum/viewtopic.php?fF59&tF21407

: (1, 2, 2, 3, 3, 4)
(1, 3, 4, 5, 6, 8).

( )
61 (x + x2 + x3 + x4 + x5 + x6 ).
r1 , . . . , r6
s1 , . . . , s6

46 ( Vojtech
Jarnk

1997) a .
d|(a2 + 2) d 1 mod 8 d 3 mod 8.



(
xri )(
xsi ) = (x + x2 + x3 + x4 + x5 + x6 )2


.


T (x) = xri S (x) = x si ,

http://www.mathematica.gr/forum/viewtopic.php?fF59&tF23187

( ) p
d. a2 2 mod p, , p
, Legendre ( p2 ) 1.
,

p1
p2 1
1 2
2
=
= (1) 2 (1) 8 .

S T = (x + x2 + x3 + x4 + x5 + x6 )2 .
Z[x]

(x + x2 + x3 + x4 + x5 + x6 ) = x(1 + x)(1 x + x2 )(1 + x + x2 ).

p 1
2

p1

p 5 mod 8 8 2
, .

T (x) = xa1 (1 + x)a2 (1 x + x2 )a3 (1 + x + x2 )a4

p 7 mod 8

p2 1

p1

2 ,
.
d 1 mod 8
3 mod 8. , 32 1 mod 8
d.

S (x) = xb1 (1 + x)b2 (1 x + x2 )b3 (1 + x + x2 )b4


ai + bi = 2 i. T (1) = S (1) = 6
a2 = a4 = b2 = b4 = 1.

53

:
47 ( )
:

() G 2n ,
.



ac ab bc
1

1
1 = (a b)(b c)(c a).

b c a

() .

http://www.mathematica.gr/forum/viewtopic.php?fF10&tF21186
http://www.mathematica.gr/forum/viewtopic.php?fF10&tF22610

( - )
1 ( )



xc xb bc

1
1 .
f (x) = 1
b c x

()
. , a G
|G|

,
. f (b) = f (c) = 0

.
, f (x) = k(x b)(x c).



0 0 bc
f (0) = 1 1 1 = bc(c b),
b c 0

2n

22

..

.2

= a = ./01
a
= e.
n

b G eb = b = be
b G
22

..

.2

22

..

.2

a
b
b a./01 = ./01
n

k = c b.
, f (x) = (c b)(x
b)(x c) .


.
22

2 ( )

..

.
22

..

b ./01
a
= ./01
a
b.
n1




ac ab bc ac ab ac bc ac


1
1
1 = 1
0
0



b c a b
cb
a b


a(b c) c(b a)
=
cb
a b

n1

, n
ab = ba a, b G G
.
()
x3 = 1 x.
a2 b = ba2 a(a2 b)a = a(ba2 )a

= a(b c)(a b) + c(c b)(b a)

ba = ab.

= a(b c)(a b) + c(b c)(a b)

1 a b

0 1 c a, b, c {0, 1, 2}

0 0 1
mod 3. .

= (c a)(b c)(a b).


48 ( )
G a, b G
a2 b = ba2 = ab = ba.

54

:
+


49 ( )

+


(2n)
2n

n=1

(2m) x2m =

m=1

2 2 cot 2/2
4

(s) Riemann.

 1+

5

, :

http://www.mathematica.gr/forum/viewtopic.php?fF9&tF13167

an =
1
2

+


n=1

cos(xt) cos(nt) dt =

1. ( ) (
):
+

(n!)2 4n x2n+2
= arcsin2 (x) .
(n
+
1) (2n + 1)!
n=0

+


sin x 
2 x sin x cos(nt)
+
(1)n+1
.
x
(n2 x2 )
n=1

n=1

x
=
2
n x2

+
2

n=1

+ 
+  2m

x
n=1 m=1

n2
=

+ n 2n

d2
4 x
1
2n
= 2 (arcsin2 (x)) ==
dx
n=0

2
1 x2

4 x2 x

2 x arcsin(x)

====

(1 x2 ) 1 x2

 x 
+

4 x + x arcsin 2
xn
2n
= 4
,

(
4

x)
4

x
n=0 n
x = 1

 
+

5 + i arcsin 2i 2
(1)n
2n
= 4
==

x
.
x2

+


2n
= arcsin2 (x)

t = sin x

cot x =

4n x2n+2

(2n + 2) (2n + 1)

n=0

Fourier f x (t) t R
, f x () = fx ()
an = O(1/n).

(1)n1
2a
2n
= .
5
n n

http://www.mathematica.gr/forum/viewtopic.php?fF9&tF15033

n=1

+


5 5

an cos(nt) ,

sin((n x))
=

n+x
nx


2 x sin x
(1)n sin x sin x

= (1)n+1
.

n+x
nx
(n2 x2 )

cos(xt) =

n=1
+


2.

3
cos((x + n) t) + cos((n x) t) dt =

1 sin((n + x))

+

(1)n1
4a
1
2n
= +

1.

( ) x R |x| < 1.
Fourier f x (t) = cos(xt)
t [, ]

a0

50 ( ) a =

ln

fx (t) =

1 x cot(x)
2

n=0

(2m) x .
2m

5 5

5 arcsinh

1

5 5

m=1

|x| < 1
.

3

==

4
4a
.
5 5 5

+
+


(1)n1
(1)n
2n
=
2n
=
n=1

55

n=1

 (1)n

4a
1

2n
1 = + .
5 5 5
n=0

2. ( )

+
+
+


(1)n1
(1)n  (1)n (n!)2
=
2n
=
2n
=
n (2n)!
n=1 n n
n=1 n n
n=1

n=1

n (2n + 1)

(1)n xn (1 x)n1 dx =

 1
+

1 
n
n
(1) (x (1 x)) dx =

0 1 x n=1

 1
+

1 
(x (x 1))n dx =

0 1 x n=1
 1
 1
x2 x
1
x
dx =
dx =
2
2
0 1 x x x1
0 x x1
 
 
2 ln 1+2 5
ln 3+2 5
2a
=
= . 

1 : ,
2 : arcsin(i x) = i arcsinh (x)
,

1
1+ 5
3 : arcsinh 2 = ln 2 .

+

(1)n n (n + 1) (n)

n=1

+ 


56

:
51 ( )
9.999 .

http://www.mathematica.gr/forum/viewtopic.php?fF63&tF23284

( ) n
n+ 3, n2 + 3
,

http://www.mathematica.gr/forum/viewtopic.php?fF63&tF23220

( )
6 mod 9.
( )
0, 1, . . . 8 mod 9
0, 1, 4, 0, 7, 7, 0, 4, 1 mod 9.
9999
6666 6 mod 9.
3 9
.

(n + 3) n2 + 3 = n3 + 3n2 + 3n + 9
= (n + 1)3 + 8.

(n + 1)3 < (n + 1)3 + 8 < (n + 3)3


n < 3 n > 1.

52 ( )
n

a=n+3

(n + 1)3 (n + 3)3
(n + 2)3 . (n + 1)3 + 8 = (n + 2)3


3n2 + 9n = 1.
n = 3, 2, 1.
n + 3, n2 + 3 .

b = n2 + 3
.

57

:
53 ( )
:

http://www.mathematica.gr/forum/viewtopic.php?fF27&tF22605

x5 40 x4 + Px3 + Qx2 + Rx + S = 0

1 (

.

10. |S |.

f (x) =

a a
, , a, ak, ak2 .
k2 k

k2

a k2

ee > e .

+ 1 + k + k2 = 40.

2 ( )
f (x) = x ln x 2 x

x > e.


+ 1 + k + k2 = 10.

a2 =
4 |a| = 2.
a5
|S | = |a|5 = 25 = 32.

f () > f (e)
ln 2 > e
ln + e > 2
ln( ee ) > ln e2

54 ( )

ee

x>0

f () > f (e) = 0 f () > 0

,
10
1

1
(ln x 1).
2

f x > e

Vieta
40 S .

e
ln x x + ,

f (x) =

http://www.mathematica.gr/forum/viewtopic.php?fF27&tF22408

e .

58

ee > e2
ee > e .

:
55 ( KARKAR) (O)
(K) A, B .
A, B T .
S (O) S T
C . S A
(K) P, AC Q. S T
PQ.

2 ( )
OK (O), (K)
AB
T A = T B
( (O)) T
AB O, T , K .
 = BAQ
 = BS

M S BMP
BPM
,
 = S
T K MB T

KB = APB
MB.
S BMP T K MB
 = BS

 K
B = AT

A = AOT
MB = OT
O,
BMQ
MQ = 900 :
, K
K M PQ M
PQ.

http://www.mathematica.gr/forum/viewtopic.php?pF115044#p115044

1 ( )
PCAMS Q B
Miquel.

 = BS


S PMB BPQ
C
 = BCS
.
BCMQ BQM
S BC
PBQ.

S ACB BA S BC .
M CS

BC = ABS
 (1).

M

BCMQ

56 ( KARKAR)
D ABCD DA
, S . CT
S D S A, S B Q, P .
CP = PQ.

(1)

BC .

 = QCM
 = S
=M
QBM
CA = ABS

S BC PBQ
BC M M
 = M

QBM
.
59

2 ( ) S X (
X A ) (D)
D AD E AC S B
F (D) S B.
AC B
(D) S , E, F, B,
C.S EFB
.
, XS Q = S CE QS P =
ECF PS C = FCB
S .XQPC .

, CQ  S X CP = PQ
.

http://www.mathematica.gr/forum/viewtopic.php?fF62&tF22133

1 ( ) F (D) S B
CK = KA BC 2 = BK BD = BF BS ,
S , F , K , D .
KFS = KDT = KCP K , F , P, C .
KPF = KCF = AS F KP  AQ,
CK = KA CP = PQ.

60

:
57 ( )
f, g C[x],
. f 2 (x) + g2 (x)
r C,

58 ( )
Cauchy.
a1 , a2 , ..., an

a1 + a2 + ... + an n
a1 a2 ...an
n

( f (r))2 + (g (r))2 = 0.


.

,
,
.

http://www.mathematica.gr/forum/viewtopic.php?fF60&tF23507

( ) :

f 2 (x) + g2 (x) = (x r)2 P(x)

http://www.mathematica.gr/forum/viewtopic.php?fF60&tF17875

( f 2 (r) + g2 (r) = 0 )
x :

1 ( )

.
,
. , (
) George Polya.
,
.
:

f (x) f (x) + g(x)g (x) = (x r)P(x) + (x r)2 P (x)


:

f (r) f (r) + g(r)g (r) = 0

a1 + a2 + ... + an
 n a1 a2 ...an
./01
n
./01

f (r) = g(r) = 0

ex1 x

f (r)  0

x = 1

g(r)
g (r)
f (r) =
f (r)

xi
,
A
x1

x1
A

x2

x2
A

xn

xn
A

e A 1

+2
g(r)

2

2
g (r) + (g (r))2
( f (r)) + (g (r)) =
f (r)
f 2 (r) + g2 (r)
= [g (r)]2
f 2 (r)
= 0.

e A 1
...

e A 1


g(r)  0

e0
61

x1 x2 ...xn
Gn

A G.
An
An



Cauchy
.

Jensen (
).
2 ( )
Jensen

2.

x1 + x2 + ... + xn n
x1 x2 ...xn
n


.

f (x) = lnx, x > 0


f (x) =

x2

a=

x1 + x2 + ... + xn
n

, x>0

min xi a max x j
i, j xi < a < x j
, xi xi = a
x j xj = xi + x j a .

. :

x1 + x2 + + xn
f (x1 ) + f (x2 ) + + f (xn )
f(
)
n
n
x1 + x2 + + xn
ln(x1 x2 . . . xn )
ln(
)
n
n
ln(x1 x2 . . . xn )  ln (

n

(x1 x2 . . . xn ) 

xi xj = a(xi + x j a) = xi x j + (x j a)(a xi ) > xi x j

x1 + x2 + . . . + xn n
)
n

.
, ,
xi a .
, a
,
, .
4 ( )

Cauchy
3 .
1 n = 2

x1 + x2 + . . . + xn
n


3 ( )
,
.
1. xi > 0 ,

c=

x1 x2 ...xn

x1 + x2

x1
x1 x2
x1 x2 x3
x1 ...xn
a1 = , a2 = 2 , a3 =
.....an =
=1
3
c
cn
c
c

b1 =

, b2 =

, bn =

x1 x2


(x1 x2 )2 0

2 n = 2k , k N

. k = 1 1
k 2
l
1 l k 1.

=1
a1
a2
an
ai , bi
:

a1 b1 + a2 b2 ... + an bn a1 bn + a2 b1 + a3 b2 + ... + an bn 1

x1 + x2 + + x2k

2k

xn
x1 x2
1 + 1 + 1... + 1
+ ... +
c
c
c

x1 +x2 ++x2k1
2k1

x1 + ...xn
x1 + ... + xn
c
n
c
n

2k1

x2k1 +1 +x2k1 +2 ++x2k


2k1

x1 x2 x2k1 +

2k1

2
62

=


x2k1 +1 x2k1 +2 x2k

2k1

k
1
x1 x2 x2k1 2 x2k1 +1 x2k1 +2 x2k =

2k
x1 x2 x2k

x1 xn
.
(*)
.
xi
x1 xn .
x1 xn

2
3 n  2k
n 2 ,
m 2 n . x x1 , x2 , . . . , xn , mn xn+1 , xn+2 , . . . , xn xn+1 = xn+2 = . . . = xm = x.
:
: x
x1 , x2 , . . . , xn

x=

:
;
(x1 , . . . , xn ) Rn : x1 , . . . , xn 0 x1 + + xn = 1
( )
Rn .
6 ( )
:
: a1 , a2 , ..., an a1 a2 ...an = 1
a1 + a2 + ... + an n
n.
( ):
n. n = 1
. n = k.
n = k + 1.

1 2 k k+1 = 1

x1 + x2 + + xn + (m n)x
m

x=

x1 + x2 + + xn + (m n)x
=
m

x1 + x2 + + xn + xn+1 + xn+2 + + xm ()



m

m
x1 x2 xn xn+1 xn+2 xm =

m
x1 x2 xn xmn

1 + 2 + + k + k+1 k + 1

() :

2
m 2.

m
x x1 x2 xn xmn
n
m x
x1 x2 xn
.
5 ( )
n = 1 n 
2. x1 , x2 , . . . , xn
x1 + + xn = 1 , x1 xn  (1/n)n
x1 = = xn = 1/n .
x1 = a1 /(a1 + + an ), . . . , xn =
an /(a1 + + an ) .
, x1 , . . . , xn
x1 + + xn = 1 x1 xn
. (*)
xi . x1 = =
xn .
x1  x2 .

x 1 = x 2 =

1 2 (k k+1 ) = 1
k 1 , 2 , , (k k+1 )
1
k :

1 + 2 + + (k k+1 ) k

x1 + x2

1 + 2 + + k + k+1 k + 1

2
:

x 1 + x 2 + x3 + + xn = 1

1 , 2 , , k , k+1 1
1
1

1
1.
k 1, k+1 1.

x 1 x 2 x3 xn (x1 x2 x3 xn ) =

 x x 2
1
2
2

1 + 2 + + k + k+1 1 + 2 + + (k k+1 ) + 1
x3 ...xn

>0

k + k+1 k k+1 1 0
63

. Newman, D. J. Classroom Notes: Arithmetic, Geometric Inequality. Amer. Math. Monthly 67 (1960), no. 9, 886
n . n = 1 .
n = k .
n = k + 1
x1 xk+1 = 1 . xk+1  1 .

k + k+1 k k+1 1 = (k 1) (1 k+1 ) 0


n
1
, ...., n
1 ...n
1 ...n

x1 xk =

7 ( ) Proofs
from THE BOOK M.Aigner G.Ziegler
H. Alzer (1996)

p

x1 + + xk  k

a1 , a2 , . . . , an , p1 , p2 , . . . , pn

k
xk+1

a1 1 a2 2 an n p1 a1 + p2 a2 + + pn an ,
n


xk+1

x1 + + xk + xk+1  k

pi = 1.

i=1

k
+ xk+1
xk+1

x  1 ,

A := p1 a1 + p2 a2 + + pn an

k
k + x  k + 1
x

G := a1 1 a2 2 an n .
,

,
,

a1 a2 an .

k
k + x
x

a1 G an ,
k {1, 2, . . . , n 1} , ak G ak+1 .

k

i=1

 G
pi

ai

dt +

 ai

n


pi

i=k+1

.
Bernoulli


dt 0 (1)

(1 + y)r  1 + ry


.
(1)
n

i=1
n


ai
pi

dt 

n

i=1

ai
pi

y  0 r . (
.)

y = k x r = k + 1

x k x  1 + (k + 1)( k x 1) = (k + 1) k x k

dt

k x .
9 ( )
P.S.Bullen Handbook of Means and their Inequalities (2003), 74
- . , ...
A.Hurwitz (1891) , .

ai G 

pi
pi (ln ai ln G)
G
i=1
i=1
n

< pi
A
1  ln
ai ln G = 0
G
i=1
n

AG
!
,
(1) ,
a1 = a2 = = an = G.
8 ( ) .

f (x1 , x2 , . . . , xn )
n x1 , x2 , . . . , xn ,
64

P ( f (x1 , x2 , . . . , xn )) :=

1
(1 + 2 + + n1 ) ,
2n!



f x(1) , x(2) , . . . , x(n) ,

S n

(
n! )

xn1 + xn2 + + xnn


x1 x2 xn 0,
n


- .
10 (liolios19)

:

x1 , x2 , . . . , xn .
,

f (x1 , x2 , . . . , xn ) = x1 x2 xn ,


n
f (x1 ) = x1 + x2 + + xn n x1 x2 xn )

P ( f (x1 , x2 , . . . , xn )) = n!x1 x2 xn ,

x1 , x2 , , xn > 0 n N :

f (x1 , x2 , . . . , xn ) = xn1 ,

f (x1 ) = 1 (x1 )

P ( f (x1 , x2 , . . . , xn )) = (n 1)! x1 + x2 + +
n

xnn

1n

(x2 x3 xn ) n

f (x1 ) 0 x1 (x2 x3 xn )1/(n1)

k , k {1, 2, . . . , n 1} ,
:

( n1 0 )
f (x1 )
1
x1 = (x2 x3 xn ) n1 :

(x1 x2 ) x3 x4 xk+1 .
k := P xnk
xnk
1
2

(x2 x3 xn ) n1 + x2 + xn n((x2 x3 xn ) n1 x2 x3 xn ) n
n

= x2 + xn (n 1)(x2 x3 xn ) n1

g(x2 ) = x2 + xn (n 1)(x2 x3 xn )1/(n1)

x1 = x2 = = xn .
,

1
1
(x1 x2 ) =
1 := P xn
xn
1
2

1
n1
x
x

P
x

P xn1 + P xn2 P xn
2
1
1
2

1
1 = 2P xn1 2P xn
1 x2

x2 = (x3 x4 xn )1/(n2)

xn = xn1 ,

xn2 =

2 = 2P x1 x2 2P x1 x2 x3 ,

2
n2
3 = 2P xn
1 x2 x3 2P x1 x2 x3 x4 ,
n1

n2

= (x2 x3 xn ) n1 + x2 + xn n((x2 x3 xn ) n1 ) n

k {1, 2, . . . , n 1} xi 0
i {1, 2, . . . , n} , k (x1 , x2 , . . . , xn ) 0. ,

k (x1 , x2 , . . . , xn ) =

2
nk+1
P (x1 x2 ) x1
+ + x2nk+1 x3 x4 xk+1 0

xn xn1 = xn

x1 = x2 = = xn

( ),
0.


.
11 ( )

..
.

n1 = 2P x21 x2 x3 xn1 2P (x1 x2 xn ) .



1 + 2 + + n1 = 2P xn1 2P (x1 x2 xn ) ,

a1 + a2 + ... + an n n a1 a2 ...an (1)

xn1 + xn2 + + xnn


x1 x2 xn =
n

a1 , a2 , ..., an
65

b1 + b2 + ... + bn > n

(1) ln x
: a1 = a2 = ... = an = 0
a1 , a2 , ..., an > 0
a1 = a2 = ... = an (1) .
a1 , a2 , ..., an
. :

M=

an
a1 a2
+
+ ... +
>n
M M
M
a1 + a2 + ... + an > n.M

a1 + a2 + ... + an > n n a1 a2 ...an

a1 a2 ...an
:

ak
bk = , k = 1, 2, ..., n
M
: b1 b2 ...bn = 1
b1 , b2 , ..., bn 1 .

a1 + a2 + ... + an n
a1 a2 ...an
n
:

a1 = a2 = ... = an

f (x) = b1x + b2x + ... + bnx , x R


:

f (x) = b1x ln b1 + b2x ln b2 + ... + bnx ln bn


f (x) = b1x (ln b1 )2 + b2x (ln b2 )2 + ... + bnx (ln bn )2 > 0, x R

1. G.Hardy, E. Littlewood, G. Polya, Inequalities,


Cambridge Mathematical Library

f R ,
x > 0 :

2. E.F. Beckenbach,
Springer Verlag

f (x) > f (0) = ln b1 + ln b2 + ... + ln bn

R.

Bellman,

Inequalities,

3. P.S. Bullen, Handbook of Means and their Inequalities, Kluwer Academic Publishers.

= ln(b1 b2 ...bn ) = ln 1 = 0
f [0, + ) f
:
[0, + ).

4. M.Aigner, G.Ziegler, Proofs from THE BOOK,


Springer Verlag

f (1) > f (0)

66

http://www.mathematica.gr.
http://www.mathematica.gr.

To {Eikosidwdekedron} parousizei jmata pou qoun suzhthje ston isttopo


H epilog kai h frontda tou perieqomnou gnetai ap tou
Metatrop se
dopohsh:
Mpore

Epimelht tou

LATEX: Fwtein Kald, Anastsh Kotrnh , Leutrh Prwtopap , Aqilla Sunefakpouo

Anastsh Kotrnh , Nko Mauroginnh ,

Grhgrh Kwstko .

Exfullo:

Stoiqeiojetetai

Eikosidwekedro filoteqnhmno ap

ton

kurt

kai

do

poledro

quasiregular

oktedro).

twn

pentgwna

pou

do

poledra

To

(0, 0, ),

Phg:

to

, Sqmata:

Miqlh Nnno , Qrsto Tsifkh

Seli-

LATEX.

Leonardo da Vinci

To eikosidwdekedro enai na poledro (32-edro) me ekosi trigwnik dre kai ddeka pentagwnik .

trgwna

me

na anaparaqje kai na dianemhje elejera.

pou

kai

exnta

akm

perissteroi tpoi

uprqoun,

eikosidwdekedro

se

1
,
, 1+
2
2
2

pou

kje

polugnwn

dhlad

stere

pou

, pou

o qrus lgo

tou

sunantntai

mpore

qei eikosiedrik summetra

ma

kai

1+ 5
2

na

oi

qwrzei

me

qei

ton

do

na

dio

trgwno

trpo

tpou

suntetagmne

sti

edrn

twn

ap

na

'Eqei 30 panomoitupe koruf st opoe

pentgwno.

koruf

oi

korufn

opoe

en

en to duadik tou poledro enai to

tou

kai

Enai

arqimdeio

eidiktera enai

enallssontai

eikosadrou

me

sthn

to

koin

monadiae

stere

na

sunantntai do

dhlad

ap

koruf

akm

ta

(To

enai

oi

na

hmikanonik

do

oiwne

llo

enai

kanonik

to

kubo

kuklik metajsei

rombik triakontedro.

http://en.wikipedia.org/wiki/Icosidodecahedron

Apdosh:

Pno Giannpoulo

O diktuak tpo

mathematica.gr

ankei kai dieujnetai smfwna me ton kanonism tou pou uprqei sthn arqik

tou selda (http://www.mathematica.gr ) ap omda Dieujunntwn Meln.

Dieujnonta Mlh tou mathematica.gr


Suntoniste

7. Girgo Mpalglou (gbaloglou)

Airet Mlh

8. Rodlfo Mprh (R

1. Fwtein Kald (Fwtein) Genik Suntonstria


2. Miqlh Lmprou (Mihalis

Lambrou) Genik Sun-

tonist

9. Miqlh Nnno (Miqlh Nnno )


10. Leutrh Prwtotopap (Prwtopap Leutrh )
11. Dhmtrh Skoutrh (dement)

3. Nko Mauroginnh (nsmavrogiannis) Genik Suntonist


4. Spro Kardamtsh (Kardamtsh Spro )

12. Mpmph Stergou (Mpmph Stergou)


13. Swtrh Stgia (swsto)
14. Aqilla Sunefakpoulo (achilleas)

Upejuno Enhmrwsh
5. Qrsto Kuriaz (chris

BORIS)

gatos)

Upejuno Programmatismo
6. Mlto Papagrhgorkh (m.papagrigorakis)
Upejuno Oikonomikn
7. Girgo Rzo (Girgo Rzo )
Upejuno Ekdsewn

Mnima Mlh
1. Grhgrh Kwstko (grigkost) Diaqeirist
2. Alxandro Sugkelkh (cretanman) Diaqeirist

Epimelhte
1. Strth Antwna (stranton)

15. Kwnstantno Thlgrafo (Thlgrafo Ksta )


16. Serafem Tsiplh (Serafem)
17. Qrsto Tsifkh (xr.tsif )
18. Dhmtrh Qristofdh (Demetres)

Melh
1. Spro Basilpoulo (spyros)
2. Ksta Zugorh (kostas.zig)
3. Girgh Kalajkh (exdx)
4. Qrsto Kardsh (QRHSTOS KARDASHS)
5. Jansh Mpelhginnh (mathfinder)

2. Andra Barberkh (ANDREAS BARBERAKHS)

6. Jwm Rakftsalh (Jwm Rakftsalh )

3. Kwnstantno Btta (vittasko)

7. Kwnstantno Rekomh (rek2)

4. Nko Katsph (nkatsipis)

8. Girgo Rodpoulo (hsiodos)

5. Anastsio Kotrnh (Kotrnh Anastsio )

9. Staro Staurpoulo (Staro Staurpoulo )

6. Jno Mgko (matha)

10. Baslh Stefandh (bilstef )


1 ( ) 100
, 99% .

98% .

2 ( )
() ( )
,
...
6210001000.
.

()
.
() .

6 ( KARKAR) A , B ,
,
.
A :
! B :
! . :
!
( ) B

3 ( )
B,
B, A.


;


7 ( )
:

P(x) = x15 2012 x14 + 2012 x13 ... 2012 x2 + 2012 x.


P(2011).
4 ( )

8 ( )
ABC
A = 900

A = 62006 + 32003 + 182001 + 92005

ab, bc, (a + c)(a c)

30.


,

5 ( ) .
AB 6m
4m.
9m .

9 ( )
m (m 1)x4 5 x2 + 3m 2 = 0
.

( )

10 ( ) :

x = |{z}
111...1 y = |{z}
111...1
2

+1

z = 666
6,
| {z...}

N
.

: x + y + z + 8
.


( ).
- ( ).
14 ( KARKAR) : 2 x = 3a = 6b ,

: x =
11 ( )
AB O .
b
A O E, Z EO
24, Z.

ab
.
ab

,
15 ( )
(O, R) , (K, r) E .
AB, , OK
AB .

.
16 ( )
ABCD AC, BD
CAB, BCA,CDB, BDA 70, 30, (50 a), a ,
, . BD
CBA a.

12 ( KARKAR) S
CD ABCD. BS AD
T . AM, T M, CN, S N
b =
. M DN
b = .
A

17 ( KARKAR)
,
A(1, 3) B(4, 2),
.

18 ( ) n,
n 2, :

1
1
1
1 + + + + > n.
n
2
3

,
13 ( )
( ) 1 = 4

+1 =

,
19
( )

f (x) =
x x + 1 x 0
= {1, 2, . . . , n}, P(k) = 125 f (k)
k .

8 9
+ 2

N .

(6) k, m (2) (5) ,


|z u|.

() f .
() n.
()
x = n. 16%
33, .

, , ,

20 ( Parmenides51)
lit
< , R. 1, 6 lit
20%
1, 4 lit 90%
.
[, ] :

23 ( )

f (x) = ex ln x + 2

1 1,

2 1

ex

ln(x2 + 1) = e1 ,

e x + 2 ln x > ex .

() , .
() .

24 ( ) f

()
,

1000 2 lit.

h(x) = e f (x) f 3 (x) + 2 .


f (x) :

  f (x2 x)
1
2

, ,
21 ( )
f : R R z z , 1/2

f 2 (x) + sin2 (x) = 2 x f (x)


x R
m =

  f (4x)
1
2

>0

, ,

25 ( )
f [a, b], (a, b)
f (a) , f (b). 1 , 2 (a, b) 1 , 2

f (x)
lim
= m,
x0 x

|z2|
.
|2z1|

: f (1 ) f (2 ) =

() |z 2| = |2z 1|.
() z

!2
f (b) f (a)
.
ba

26 ( )
f : [1, e] R f (1) = 0, f (e) = 1
1

f (x) + e f (x) = x + , x [1, e] . :

f (sin x)
() lim 2
.
x0 x x

x
f (x) = ln x x [1, e] .

() g(x) = f (x) x
(, 0) (0, +).
() f .
() (|z + 3 4i| + 5)x = x3 + 10
[1, 2].

, ,

22
t R k > 1.

27 ( ) :

z = (k t) + (k t)i

Z2

x10 + 210
181
dx <
.
x15 + 215
2816

(1) z.
(2) w
y = x (k 1) , k |z w|min = 5 2 2 1.

28 ( )
f : R R :

(3) k (2)
z |z z|.

( f (x))2 + ( f (x))2 x2 + 1,

x R.

(4) k (2)
|w 3 + 4i|.

1. ( f (1))2 ( f (0))2

(5) u

2. | f (1)|

u = (1 + mt) + (1 + mt)i,

4
,
3

3. |F (1) F (0)|

m
u .

4
,
3

10
, F f .
9

34 ( )
! f : (0, +) R

, ,

f (x) x f (y)
x
=


y
y
y2

1, f (1) = 1


29 ( )
f (x), g(x) : [a, b] R
g (x) > 0, x [a, b].

1)

35 ( )
n,
sin2n x + cos2n x + n sin2 x cos2 x = 1,
x R.

g (x) > 0 f (x) < 0, x (a, b)


Rb
a

f (x)dx =

Rb
a

x, y (0, +)

Juniors,

- -

(x a) f (x) = (x b)g (x), x [a, b]

2)

g(x)dx

36 ( ) a, b, c
a + b + c = 3,

a2
b2
c2
3
+
+
.
(b + c)3 (c + a)3 (a + b)3
8


) f (b) = g(a)
) (a, b) : f () = g()
30 ( parmenides51)

Juniors,

z1 = a + ea i, z2 = b + bi, z3 = c + i ln c a, b R, c > 0.

37 ( )
ABC BC, I .
BI AC D E,
D CI . EI AB
Z. DZ CI.

1.
z1 , z2 , z3 ,
2. |z1 z2 | z1 , z2 ,
3. |z1 z3 | z1 , z3 .

38 ( )
ABC E M, .

, ,

31 ( )
f : [0, +) R

f (x) + x f (x) =

1
f
2

x

x [0, +) .

32 ( )

f (x) =

ln(x2 )
,
x2+1

N ,

M, ABC
,
E1 , E2 , E3 .
1
1
1
18
+
+
.

N ,

E1

f (0) = 0

ef

(x)

E3

Seniors,

- -

33 ( )
f : [0, 1] R

E2

39 ( )
f :

f (x) dx = f (1).

(x y) f (x + y) (x + y) f (x y) = 4 xy(x2 y2 )

f (x) = 0, x [0, 1]

40 ( )

f : {1, 2, . . . , 10} {1, 2, . . . , 100}

45 ( IMC 1996) n
.

x + y|x f (x) + y f (y), x, y {1, 2, . . . , 10}.

sin nx
dx.
(1 + 2 x ) sin x

Seniors,
46 ( ) (an )nN
an > n1

41 ( )
ABCD (O)

n.

P AC BD.

+
X

an .

n=1


47 ( ) M = R \ {3}
x y = 3(xy 3 x 3y) + m, m R.
m (M, ) .
48 (vzf) m n n
.

, n , .


49 ( )

ABCD Q,

QAB + QCB = QBC + QDC = 90o .

ln(1 aix)
dx ,
x2 + m

a > 0, m > 0

50 ( ) an = 1 +

lim( n)

P, Q, O , O
(O).
42 ( )
P ABCD,
AB, BC, CD, DA,
.
ABCD .

an

Pn

j=2 ln j


51 ( ) An = {n, 2n, 3n, ...} n N
J N ,

Ai

iJ

52 ( )
A, B
AB = (A \ B) (B \ A), .
A1 A2 ...An
Ai


43 ( - 1
1995) 6-
5. ,
6

44 ( )
ABC AB AC
D E , DE

53 ( ) p P
p > 5 (p 1)! + 1 p
( pk k N ).

AD AE
. :
+
=1
DB EC

54 ( dimtsig)
.


55 ( ) S
,
s1 , s2 , ..., sn s1 +s2 +...+sn <
1. S ;
56 ( )
(entire) f, g

PQ S ,
S T . AT
M BC .

f (z)2 + g(z)2 = 1 z C.

60 ( [
] ) ABC A
, BAC
BC .
ABC
.

()
57 ( )
BE B

3
BO
= . O
OE
2
.
58 ( )

sin arctan
+ arctan

1
1
1
+ arctan
+ arctan
3
5
7

1
1
111
+ arctan
+ arctan
11
13
121

,

61 ( ) C
:



z + 1 = z2 + 1 = 2


2

!!
.

62 ( ) P(x)
, n
x1 , x2 , ..., xn .

,
59 ( KARKAR)
A ABC BC .
(O) A
AB, AC P, Q .

n
X
P (xi )
=0
P (xi )
i=1

1 ( )
100 ,

99% .

98% .

()
(
)
,
...
6210001000.

http://www.mathematica.gr/forum/viewtopic.php?f=44&t=18473

1 ( ) x .
0, 98 x 0, 02 x .
()
, 0, 02 x = 1, x = 50 .

http://www.mathematica.gr/forum/viewtopic.php?f=44&t=21037

( ) 1210
.
2
.

4-
1210.
4 ,
. 1
0
1.
1 2 ,
2 .

1 2

1 .
2
. 0
1 .

2 (Parmenides51)
. !
100 ,
99 .
x . 100 x
, 99 x
.
99 x
= 0, 98 99 x = 98 0, 98 x
100 x

x 0, 98 x = 99 98 0, 02 x = 1 x = 50 .

100 50 = 50 .
2 ( )

3 ( )
B, B,
A.


;

A = 62006 + 32003 + 182001 + 92005


30.

http://www.mathematica.gr/forum/viewtopic.php?f=33&p=110973

(Gauss 1988) 62006


6
6.
32003 7
(34 )500 33 34
1 33 7.
182001 8
(184 )500 18 184
6 18 8.
92005 9
(92 )1002 9 92
1.
,

http://www.mathematica.gr/forum/viewtopic.php?f=33&p=117780

( ) K ,
KA + KB + K.
, K B,
KB + K B.
, AK .
A B
.

62006 + 32003 + 182001 + 92005


0, 6 + 7 + 8 + 9 = 30 .

10, 3
3,
62006 3, 62006 = 3 32005 22006 .

3 10,
30, gcd(3, 10) = 1

4 ( )

5 ( )
. AB
6m 4m.
9m
.

6 ( KARKAR) A , B
, ,
.
A :
! B :
! .
:
!
( ) B
() .
() .

http://www.mathematica.gr/forum/viewtopic.php?f=34&p=123216

( ) A, B,
x .

A + x = 2 (B + )
B + x = 3 (A + )
+ x = 5 (A + B)
:
7A + 6B + 4
A + B + + 3 x = 8A + 7B + 5 x =
3
2A + 5
3B + 7A + 6B + 4 = 9A + 9 B =
9
:
4A +
B A = 3A + 2B B =
3
4A +
2A + 5

=
5A =
3
9
7A + 18A + 20A

B = 3A x =
= 15A
3
B
3
1


=
= = 12, 5%
A + B + + x 1 + 3 + 5 + 15 8

http://www.mathematica.gr/forum/viewtopic.php?f=34&p=120912

( ).

E 9m ,
6m A B
2m .

E=

92
2

+2

62
4

+2

22
4

121
..
2

:
2011 20112 + 2011) + (201114 + 2011 201113 ...

7 ( )
:
15

14

13

20112 + 2011 2011) = 2011 + 0 = 2011

P(x) = x 2012 x + 2012 x ... 2012 x + 2012 x.

8 ( )

ABC A = 900

P(2011).

ab, bc, (a + c)(a c)

http://www.mathematica.gr/forum/viewtopic.php?f=35&t=18552

( ) :
P(x) = x15 2012 x14 + 2012 x13 ...

http://www.mathematica.gr/forum/viewtopic.php?f=35&t=10214

2012 x2 + 2012 x =

(kanenas) ABC a2 = b2 + c2

x15 (2011 + 1)x14 + (2011 + 1)x13 ...

ab, bc, a2 c2 = (b2 + c2 ) c2 = b2

(2011 + 1)x + (2011 + 1)x =


15

14

13

a > b ab > b2 , a > c ab > bc,


ab.

= (x 2011 x + x ... 2011 x + x)+


(x14 + 2011 x13 ... x2 + 2011 x) (1)


a2 b2 , b2 c2 , b4

P(2011) :
15

14

a2 b2 = (b2 + c2 )b2 = b4 + c2 b2 ,
,
.

13

(1) 2011 2011 2011 + 2011 ...

10

9 ( ) m
(m 1)x4 5 x2 + 3m 2 = 0 .

z1 z2 =

3m 2
=
0
a
m1
3m 2 0
3m 2
2
m
3

http://www.mathematica.gr/forum/viewtopic.php?f=19&t=24722

( ) m = 1
1
: 5 x2 + 1 = 0 x = .
5

.
m , 1 x2 = z


m > 1

5 + 19
.
1 < m
6
10 ( ) :

x = |{z}
111...1

(m 1)z 5z + 3m 2 = 0

y = |{z}
111...1

+1

z = 666
6,
| {z...}

: x + y + z + 8
.

(z) = (5)2 4(m 1)(3m 2)


= 25 4(3m2 5m + 2)

http://www.mathematica.gr/forum/viewtopic.php?f=19&t=23823

= 25 12m + 20m 8

= 12m2 + 20m + 17 0

12m2 20m 17 0

( ) :
(1)

x = 111
}1
| {z
2

= 1 100 + 1 101 + 1 102 + + 1 1021

(m) = (20)2 4(17)12 = 400 + 816 = 1216

2
a1 = 1 = 10.
:

20 + 1216
5 + 2 19
m1 =
=
,
24
6

20 1216
5 2 19
m2 =
=
.
24
6

102 1
9

y = 111
}1
| {z
+1

5 2 19
5 + 2 19

(1)
m
.
6
6
, ,
.
Vieta :

= 1 100 + 1 101 + 1 102 + + 1 10

10+1 1
9
10 10 1
=
9

b
5
z1 + z2 = =
>0m>1
a m1

11

z = 666
}
6
| {z

102 1 10 10 1 6 10 6
+
+
+8
9
9
9
102 + 16 10 + 64

x+y+z+8=
9
!
2
10 + 8
.
x+y+z+8=
3

x+y+z+8=

= 6 100 + 6 101 + 6 102 + + 6 101


10 1
=6

9
6 10 6
.
=
9

12

11 ( )
AB O . b
A O
E, Z EO 24,
Z.

2 ( )
.

x
A
2
A
=
= 2, x =
a.
ax
A
1+ 2
,

1
2

AO OE =
a, x = 2OE = 48.
2 1+ 2

http://www.mathematica.gr/forum/viewtopic.php?f=20&t=120

( ) ON//Z.
AZ b
A = 45 ,

12 ( KARKAR) S
CD
ABCD. BS AD T .
AM, T M, CN, S N
b
.
M DN
=
b = .
A

Zb
A = = 22, 5 .
Ab
EO = 67, 5 (1) ,


AOE.
Eb
NO = Eb
AB = 22, 5 + 45 = 67, 5 (2) ,

- . (1) , (2)
ONE (OE = ON = 24). O
A ZA
Z .
Z = x = 2ON = 48.
13

DCN = MAD = x + y.
MABE , ABM =
AE M = y.

http://www.mathematica.gr/forum/viewtopic.php?f=20&t=23657

(Antonis-Z) BM I
BM, DC . BN
AD E . N, M ,
BN

BEA = EBC = x
. IBCN
, CBN = CIN = x,
DIN = NED
DIEN .
DIB = ABI = y,
DIB = DE M , MIDE
.
DIEN, MIDE MDNE ,

S BC = S BN + NBC = 2 x
BM ABS = ABM +

MBT = 2y.

MDN = = 180 x y = 180

14

b
B
2

= 90 + .
2

13 ( )
( ) 1 = 4

+1

av 3 =

N .
( )

v =

8 9
=
+ 2

av 3

v+4

v+4

av = 3 +

av 3

av =

v+4

3v + 17
, v N .
v+4

14 ( KARKAR) : 2x =

3a = 6b , : x =

N
.

ab
.
ab

http://www.mathematica.gr/forum/viewtopic.php?f=21&t=24475


( ).

1 ( f reyia)

x ln 2 = a ln 3 = b ln 6 = b ln(2.3) = b ln 2 + b ln 3

- ( ).

x ln 2 = a ln 3
http://www.mathematica.gr/forum/viewtopic.php?f=21&t=24924

(a b) ln 3 = b ln 2 ln 2 =

( )

v+1 =

av+1 3

x ln 2 = a ln 3 x

8av 9
av +2

8av 93(av +2)


av +2

av + 2
=
5(av 3)

3
av + 2
=
5av 15

2x = 3a = 6b = m

x=

av + 2
1
av + 2 5

5(av 3) av 3
5(av 3)
av 3
1
v+1 v =
= :
5(av 3)
5

v+1 v =

S v = (1 + v ) =
2

1+

v+4
5

vv+9
2

v2 + 9v
10

m
m
m
, a=
, b=
,a , b
ln 2
ln 3
ln 6

ab =

. ,
1
1
v+4
1 =
= 1 v = 1 + (v 1) =
, v N .
a1 3
5
5

ab
ab
ln 3 = a ln 3 x =
b
ab

2 ( )

, v N , :

ab
ln 3.
b

m2
ln 2
, ab=m
ln 3 ln 6
ln 3 ln 6
ab
m
=
= x.
a b ln 2

3 ( )
2x = 3a = 6b x ln 2 = a ln 3 = b ln 6

ab
=
ab

15

x ln 2 x ln 2
ln 3 ln 6
x ln 2
x ln 2
ln 3 ln 6

x2 ln2 2
=
x ln 2 ln 6 x ln 2 ln 3

ln2 2
ln2 2
= x
=x
ln 2 (ln 6 ln 2)
ln 2 ln 2

5 ( ) x = a = b = 0 .

a, b, x , 0 a , b,
(

4 ( )
x
a

3=2 ,

x
b

6=2 3=2

xb
b

a=b
3a = 6a .
3a = 6b

x xb
ab
=
x=
.
a
b
ab

16

15 ( )
(O, R) , (K, r)
E . AB,
, OK AB .

.

http://www.mathematica.gr/forum/viewtopic.php?f=22&t=24890

1 ( ) :

M AC BD,
= A BD
= x,
C BD
= 70 + x
C MB

http://www.mathematica.gr/forum/viewtopic.php?f=22&p=127551#p127551

x = 40o

1 (KARKAR)

R2 + r2 AL = CL .
1
1
E = E E E E E
2
2
1
= (R2 + r2 ) E E .
2
E !

ABC, BCD, CDA, DAB


sin 70 sin(50 a) sin(70 a)


=

sin 30
sin a sin(60 + a)
sin 70 cos 20 cos(120 2a)
=

sin 30
cos 60 cos(60 + 2a)
cos 20 + cos(60 + 2a)
2 cos 20 =

1
cos(60 + 2a)
2

cos(60 + 2a) = 0

a = 15o
2 ( ) ABC (70 , 80 , 30 ).
2 ( ) LA = LC =
, (1) ,

L LA = LB.
, (1)
(
R2 + (LO)2 = (LA)2 = (LC)2 = r2 + (LK)2
R + r = OK = LO + LK ).

R2 + r 2

16 ( )
ABCD
AC, BD CAB, BCA, CDB, BDA
70, 30, (50 a), a , , .
BD
CBA a.
17

C BD,
BA E . BCE (80 , 50 , 50 )
( - ),
BCDE AEDC
b = C DA
b = 50 ).
(C EA
b = ACE
b = 20 , DCE (70 , 55 , 55 )
ADE
BCD : a = 15 .
3 ( ) A
BD CB
E ADCE
( 50 ),
DEB, DAB a = 15
.

18

a ()
r > 0 .
(AC) : y 3 = a(x 1) ax y a + 3 = 0
(BD) : y 2 = a(x 4) ax y 4a + 2 = 0.

17 ( KARKAR)
,

A(1, 3) B(4, 2), .

, :

d(O, AC) = r
|3 a|

=r
a2 + 1
|3 a| = r a2 + 1
d(O, BD) = r
|2 4a|
=r

a2 + 1
|2 4a| = r a2 + 1.

|3 a| = |2 4a|,
1
a = 1 a = .
3

a = 1 r = 2,
1
C : x2 + y2 = 2 ( ) a =
3

r = 10 C : x2 + y2 = 10 ( ).

,
.
3 ( ) y = x + , , R
1 A
x x. 3 = 1 + = 3 .
y y K (0, 3 ).

http://www.mathematica.gr/forum/viewtopic.php?f=23&t=21244

1 ( ) > 0
C(, ), D(, ).
AC, BD :
x + y = 2 x y = 2 , .
A B :
+ 3 = 2 4 2 = 2 .
=

,
2
2
:
2 + 2 = 2 . . . 2 = 2, c
x2 + y2 = 2.

y = x + , , R 2
B 1 .
2 = 4 + = 2 4.
y y L (0, 2 4).


O R C, D , KON, LOD ,
[ = DOL
[ ( ),
CO = DO = R, CON
OK = OL |3 | = |2 ! 4|
1
... = 1 = .
3

2 ( )

R = d (O, 1 ) = ... 2 R =
4 ( parmenides51)

10.


AC, BD .

19

AB.

1+4 3+2
5 5
M
,
= , .
2
2
2 2
OM AC, BD
, AC, BD,
AC k BD.
5
x M = , 0 = xO
2

5
0
yM yO
=1,
OM =
= 52
xM xO
0
2
OM y 0 = 1(x 0) y = x.
AC, OM,

http://www.mathematica.gr/forum/viewtopic.php?f=23&t=22805

1 (

k + k 1 > k k > 1 k = 1
.

>
k

k1

>

k,

k 1.

,
1
1
1
1 + + + + >
n
2 3

1 + 2 1 + 3 2 + +

n 1 n 2 + n n 1 = n.
2 ( )

= d(AC, OM) = d(A, OM) = |yA x2A | 2 = |31| =


2
(1) +1

2
=
2.

2
2
x2 + y2 = 2 = 2 = 2.


AC, BD
, AB ( C D).
xA = 1 , 4 = xB

yB yA
23
1
AB =
=
= ,
xB xA 4 1
3
AB
1
y 3 = (x 1) 3y 9 = x + 1 x + 3y = 10.
3

AB,

|0 + 0 10|
10
= d(O, AB) =
= = 10.
10
12 + 32



x2 + y2 = 2 = 10.

n
X
1
>n
k
k=1

v
t n
Y 1
n
n
= 2n .
k
n!
k=1

n
> n,
n!

n
n > n!.
2n

-
n(n + 1)
n2 >
= 1 + 2 + 3 + + n >
2
n
n
n 1 2 3 n = n n!,
.
3 ( )

1
1
1
1
1
1 + + + + +
> n+
,
n
3
2
n+1
n+1
> n + 1.
(
)

1
1
.
n+1 n=
<
n+1+ n
n+1

18 ( )
n, n 2, :

1
1
1
1 + + + + > n.
n
3
2

20

() 16% 33
x s = 33 s = 2.

19 (
)

f (x) =
x x + 1 x 0
=
{1, 2, . . . , n}, P(k) = 12
f (k) k .
5

s
100% = 5, 71%
x
CV < 10% CV =

() f
.

() n.
20 (

Parmenides51)


lit < , R.
1, 6 lit
20%
1, 4 lit
90% .

[, ]
:

()
x = n. 16%
33,
.

http://www.mathematica.gr/forum/viewtopic.php?f=18&t=24101

( )

() , .

()

f (x) =

() .
1

()

,
1000
2 lit.

>0

2 x 2 x+1

.
()

http://www.mathematica.gr/forum/viewtopic.php?f=18&t=15918

P(1) + P(2) + + P(n) = 1

1 ( )

f (1) + f (2) + + f (n) =

() X .
A = {X 1, 6} B =
{X 1, 4}.
X , [, ]. :
1, 6 a
P(A) =
= 0, 2

5
12

f (x) x = 1, . . . , x =
n

5
1
1
1
1
= + +
12
2 2 2
2 2 2 3
!
1
1

2 n
1

2 n+1
1
=
2 2 n+1

1, 4
= 0, 9 .
a

= 1, 2 = 3, 2
P(B) =

n = 35.
21

()

a+
2

, [1.6, ]
80% . [1.4, ]
90%
[1.4, 1.6] 10% .
1, 6 1, 4 = 0, 2
10% .

= 2, 2.

()

= {X 2}

2 0, 2 = 0, 4
2 10% = 20%
1, 6 20%

N()
2 1, 2
= P() =
= 0, 4
N()
3, 2 1, 2
N() = 1000 N() = 2500.

1, 6 = 0, 4 = 1, 6 0, 4 = 1, 2.

2 (parmenides51)
[, ],
,

.

10 0, 2 = 2
10 10 = 100%
[, ] 100%

= 2 = + 2 = 1, 2 + 2 = 3, 2.

() :
100%
.
1, 6
20% .
100

=
.
1, 6
20

()

,

(1)

x =

100%
.
1, 4
90% .

100
=
.
1, 4
90

+
2

1, 2 + 3, 2
4, 4
=
= 2, 2.
2
2

() 3, 2 1, 2 = 2
( ).
2 1, 2 = 0, 8 1000 .

=
0, 8
1000

(2)

(1),(2)
= 1, 2 = 2, 2.

:
[, ] 100%
[, 1.6] 20%

22

2000
= 2500
0, 8

()

21 ( )
f : R R z z , 1/2

2

|2z 1| = |z 2|

|2z 1|2 = |z 2|2

f (x) + sin (x) = 2 x f (x)

(2z 1)(2z 1) = (z 2)(z 2)

4zz 2z 2z + 1 = zz 2z 2z + 4

x R

3zz = 3

f (x)
lim
= m,
x0 x
m =

|z| = 1.
()

|z2|
|2z1| .

lim

() |z 2| = |2z 1|.

x0

() z
() lim

x0

f (sin x)
.
x2 x

f (sin x)
f (sin x) sin x 1
= lim
2
x
0
sin x
x x1
x x
= 1 1 (1) = 1.

f (sin x)
lim
x0 sin x

() g(x) = f (x)x

(, 0) (0, +).

sin x=u
x0
u0

lim

u0

f (u)
= 1.
u

()

f 2 (x) + sin2 x = 2 x f (x)

() f .

f 2 (x) 2 x f (x) + x2 = x2 sin2 x

() (|z + 3 4i| + 5)x = x3 + 10


[1, 2].

( f (x) x)2 = x2 sin2 x


g2 (x) = x2 sin2 x.

| sin x| 6 |x|, x R
x = 0 x , 0
x2 sin2 x , 0, g2 (x) , 0.
g (, 0) (0, +),

.

http://www.mathematica.gr/forum/viewtopic.php?p=111574#p111574

( )
() x , 0

() 4 f .

f 2 (x) sin2 (x) 2 f (x)


+
=
,
x
x2
x2

(1) f (x) = x +

(2) f (x) = x

p
p

x +
(3) f (x) =

x +
(4) f (x) =

!
!2
f (x) 2

sin
x
= lim 2 f (x) ,
lim
+
x0
x0
x
x
x

m2 + 12 = 2m m = 1.
2|
= 1 |2z 1| = |z 2|.
m = 1 ||z
2z1|
23

x2 sin2 x.

x2 sin2 x.
p
x2 sin2 x
p
x2 sin2 x
p
x2 sin2 x
p
x2 sin2 x

x > 0,
x < 0.
x 6 0,
x > 0.

()

(6) k, m (2) (5) ,


|z u|.

h(x) = (|z + 3 4i| + 5)x x3 10


x [1, 2]. h
.

http://www.mathematica.gr/forum/viewtopic.php?p=110457#p110457

h(1) = |z + 3 4i| + 5 11

( )

= |z + 3 4i| 6 6 0

(1) z = x + yi, x, y R

|z + 3 4i| 6 |z| + |3 4i| = 6.

x = k t, y = k t.

h(2) = 2 |z + 3 4i| + 10 8 10
= 2(|z + 3 4i| 4) > 0

|z + 3 4i| > ||z| |3 4i|| = 4.


h(1)h(2) 6 0.

2 (t) + 2 (t) = 1 (x k)2 + (y k)2 = 1.

z
A(k, k) 1 = 1 k > 1.

h(1)h(2) < 0 Bolzano


x0 (1, 2)
h(x0 ) = 0.

(|z + 3 4i| + 5)x = x3 + 10


(1, 2). h(1)h(2) =
0, x = 1 x = 2 .

(|z + 3 4i| + 5)x = x3 + 10


[1, 2].
22

z = (k t) + (k t)i

k1|
(2) d(A, ) = |3
> 1 k > 1,
2
.

t R k > 1.
(1) z.

|z w|min = d(A, ) 1 =

(2) w
y = x
(k 1) , k

|z w|min =

5 2
2

|3k 1|
1

1.

|z w|min

(3) k (2)
z
|z z|.

5 2
|3k 1|
5 2
=
1
1=
1
2
2
2
|3k 1| = 5

k = 2,

k > 1.

(4) k (2)
|w 3 + 4i|.

(3) k = 2

(5) u

z = (2 t) + (t 2)i.

u = (1 + mt) + (1 + mt)i,

z = + i, , R = 2 (t)
= (t) 2

m
u
.

2 (t) + 2 (t) = 1 ( 2)2 + ( + 2)2 = 1.


24

z
B(2, 2) 2 = 1 = 1.

2 t + 2 t = 1
2

m2

: (1) k = 2

= 1 m = 2 m = 2.

 2
1

 2
1

=1

(6)

|z (2 + 2i)| = 1 |z (2 2i)| = 1

c =z u

z
B(2, 2) 2 = 1 = 1.

=(k t) + (k t)i

(1 + mt) (1 + mt)i

=(k t) + (k t)i + (1 mt) + (1 mt)i

|z z| =

=(k t + 1 mt) + (k t + 1 mt)i




= k + 1 (1 + m)t + [(k + 1 (1 + m)t)] i

= |(2 t) + (2 + t)i (2 t) + (2 t)i|


= |2(2 t)i|

|z z|min = |2 1| = 2 |z z|max = |2 3| =
6.

c = x + yi, x, y R

x = k + 1 (1 + m)t x (k + 1) = (1 + m)t,

y = k + 1 (1 + m)t y (k + 1) = (1 + m)t.

(x (k + 1))2 + (y (k + 1))2

=(1 + m)2 2 t + (1 + m)2 2 t

(4) |w 3 + 4i| = |w (3 4i)|


w K(3, 4).
|w 3 + 4i|min = d(K, ) = |34+1| = 0 (
2
K ).

(x (k + 1))2 + (y (k + 1))2 = (1 + m)2


2
k=2
(x 3)2 + (y 3)2 = (1 2)

m= 2

(5) u = + i, , R.

(t) =

1+

, (t) =

1+

M(x, y)


2
(x 3)2 + (y 3)2 = 1 + 2 .

( m , 0 m = 0, u
(1, 1)).

( + 1)2 ( + 1)2
+
= 1 ( + 1)2 + ( + 1)2 = m2
m2
m2
u
(1, 1) 3 = |m|.
O(0, 0) (0 + 1)2 + (0 + 1)2 =
m2 m2 = 2 m = 2.
:
w
m R

1 + mt = 0
t = m
u = 0 + 0i

t = 1
1 + mt = 0
(

25

OK ,

2
2

A, B. (OK) = d(K, O) = 3 + 3 =
3 2. OB
(OM) (OA).

OK ,

2
2

A, B. (OK) = d(K, O) = 3 + 3 =
3 2. OB
(OM) (OA).

|C|max = (OA) = (OK) + = =

=3 2+1+ 2 =4 2+1

|C|max = (OA) = (OK) + = =

=3 2+ 21=4 21

|C|min = (OB) = (OK) + = =

= 3 2 1 2 = 2 2 1.

|C|min = (OB) = (OK) + = =

= 3 2 2 + 1 = 2 2 + 1.

M(x, y)

(x 3)2 + (y 3)2 =

2

21 .

26

23 (

) e x + 2 ln x > ex

e x + ln x + 2 > ex ln x + 2

f (x) = ex ln x + 2

1 1,
2

ex

ln(x2 + 1) = e1 ,

 
1

x>0

> f (x) x1 < x x2 > 1 x > 1

24 ( )
f

e x + 2 ln x > ex .

h(x) = e f (x) f 3 (x) + 2 .


f (x)
:

http://www.mathematica.gr/forum/viewtopic.php?f=52&t=17341

  f (x2 x)

( ).

1
2

) x > 0.

  f (4x)
1
2

>0

http://www.mathematica.gr/forum/viewtopic.php?f=52&t=12656

0 < x1 < x2 x1 > x2

ex1

>

ex2 (1)

( ) h
R, x1 , x2 R x1 > x2

(y = e x . )

0 < x1 < x2

h(x1 ) > h(x2 )


e f (x1 ) f 3 (x1 ) + 2 > e f (x2 ) f 3 (x2 ) + 2

ln x1 < ln x2 ln x1 + 2 > ln x2 + 2(2)


(y = ln x . )

( f (x1 ) f (x2 ))( f 2 (x1 ) + f (x1 ) f (x2 ) + f 2 (x2 ))

(1) , (2) f (x1 ) > f (x2 ) f


1 1
2

) ex

e(x




ln x2 + 1 =

+1)



ln x2 + 1 + 2 =


<

e f (x2 ) e f (x1 )
(I)
e f (x1 ) e f (x2 )

* f (x1 ) = f (x2 ) () 0 < 0,


.
1

+2

* f (x1 ) > f (x2 ) ()


()
, .

f :11

f x2 + 1 = f (1)

x2 + 1 = 1 x = 0 x > 0,

: f (x1 ) < f (x2 ), f


R.

27

g(x) = x , 0 < < 1


R,
:

  f (x2 x)
1
2

>

  f (4x)
1
2

- x > y h ,
h(x) < h(y), .

f (x2 x) < f (4 x)

x < y.

x2 x > 4 x x2 > 4 x < 2 x > 2.

2 ( ) ( )
f

. .

f .
. x1 , x2 R
x1 < x2 f (x1 ) 6 f (x2 ).

h R
: x < y h(x) > h(y),
x, y R.

e f (x1 ) > e f (x2 )


f 3 (x1 ) > f 3 (x2 ) h(x1 ) 2 > h(x2 ) 2
h . .

* .
* .

x1 , x2 R x1 < x2

- x = y, h(x) = h(y), .

f (x1 ) > f (x2 ), f . .

28

25 ( )
f [a, b],
(a, b) f (a) , f (b).
1 , 2 (a, b) 1 , 2

f [a, b],
(a, b) f (a) , f (b).
1 , 2 , . . . , n (a, b)
(n N )

!2
f (b) f (a)
.
: f (1 ) f (2 ) =
ba

!n
f (b) f (a)
f (1 ) f (2 ) f (n ) =
.
ba

http://www.mathematica.gr/forum/viewtopic.php?f=53&t=23144

1 ( )
a < x < b

2 ( )
f

f (b) f (a)
= f () > 0.
ba
U
y : f (x) > 0, f (x) U .
ln f ()

f (x) f (a)
f (b) f (a)
=
:= ,
xa
ba

f (b) f (x)
=
bx

[a, x] [x, b].

1 , 2 U : ln f () =

,
> 0

(1 , 2 ) = V .

ln f (1 ) + ln f (2 )
2

, f (1 ) > 0, f (2 ) > 0

f (x) f (a)
f (b) f (a)
f (b) f (x)
>
>
,
xa
ba
bx
a < x < b.

,
1 < < 2 .

3 ( )

g(x) :=

f (x) f (a)
x a)

1 (a, k), 2 (k, b),

(a, b]

[, c) (
) ,

f (1 ) =

f (k) f (a)
ka

(1)

f (2 ) =

f (b) f (k)
bk

(2)

f (x) f (b)
h(x) :=
[a, b)
xb
(d, ] (
).
k > 1 g() = k
h( ) = /k , (a, b).
[a, ] 1
(a, ) f (1 ) = g().
[ , b] 2
( , b) f (2 ) = h( ).

!2
f (b) f (a)
f (1 ) f (2 ) =
,
ba

(1) (2),
:

f (k) f (a) f (b) f (k)


f (b) f (a)

=
ka
bk
ba



k .

!2

(3)

f (b) f (a)
,
ba
(3) k,
- =

29

(4)
(5).

f (k) f (a) f (b) f (k)

= 2
ka
bk

,
[a, k], [k, b],
,
.

( f (k) f (a)) ( f (b) f (k)) = (k a) (b k)

()

(atemlos)

..

k
(a, b),
:

f (k) f (a) = (b k)

(4)

f (b) f (k) = (k a)

(5).

26 ( )
f : [1, e] R f (1) = 0,

1
f (e) = 1 f (x) + e f (x) = x + , x [1, e] .

: f (x) = ln x x [1, e] .

http://www.mathematica.gr/forum/viewtopic.php?f=53&t=24617

1 ( )

k (4).
(5) ...
(4):

h(x) = f (x) lnx, x [1, e],


( ) .
x0 (1, e) h(x0 ) , 0,
, , f (x0 ) > 0,
(
) x1
[1, e] (
), Fermat

1
h (x1 ) = 0 f (x1 ) = ,


f (b) f (k) = f (b) f (a) + (b k)
= f (b) f (a) (b k)
= [b a b + k] =

= (k a)

(2)

(5)

k. (4),
( )

x1

x = x1

h(x) = f (x) f (a) (b x), x [a, b]

e f (x1 ) = x1 f (x1 ) = lnx1 h(x1 ) = 0,

Bolzano

.
2 ( )
,

h(a) = (b a) = f (a) f (b)


h(b) = f (b) f (a) 0
= f (b) f (a).

g(x) = f (x) lnx

[1, e].

g [1, e]
.
: g(1) = g(e) = 0 :

h(a) h(b) = ( f (a) f (b))2 < 0,

g (x) = x(eg(x) 1), x [1, e]

f (a) , f (b).
.Bolzano h
[a, b]

(1)

g
.
.

k (a, b) : h(k) = 0,
30

e f (x) = (g(x) f (x)) e f (x) (g(x) f (x)) = 1.

, x1 ,
g(x1 ) > 0 .Fermat

x2

eg(x) = x e 2 :
eg(x) f (x)




2
2
x

x
(g(x) f (x)) = xe 2 , eg(x) f (x) = e 2 ,

g (x1 ) = 0.

x2

eg(x) f (x) = e 2 + c.
x = 1 c = 0
f (x) = lnx, x > 0.

, (1) .
.
; ...

x2 , (1) Fermat
g(x2 ) = 0.
.
g .
g [1, e]...

5 ( ) h : (0, +) , h (x) =

e f (x)
.
x

h (1) = h (e) = 1

h (x) + xh (x) 1 h (x)


f (x) =
= +
,
xh (x)
x h (x)

1 h (x)
1
+
+ xh (x) = x +
x h (x)
x

h (x) = xh (x) (1 h (x)) .

g(x) = 0, x [1, e] f (x) = lnxx [1, e].


3 ( )
. :

,
x = x0 ,

h (x0 ) = 1 h (x0 ) = 0 (.Fermat) .

g(x) = f (x) lnx g (x) = x xe f (x)

xeg(x) + (eg(x) ) = x

h (1) = h (e) = 1, . h (x0 ) = h (1) = h (e) = 1.


g(x)

h(x) = e

, xh(x) + h (x) = x,

h (x) = 1, x [1, e] e f (x) = x


f (x) = ln x, x [1, e].

x2

e 2

x2

x2

6 ( )
x : lnx , f (x), lnx > f (x).

1
f (x) = x e f (x) = elnx e f (x) = (lnx f (x))e

(e 2 h(x)) = xe 2
x2

x2

e 2 h(x) = e 2 + c elnx f (x) = 1 + ce

lnx, f (x) e x .

x2
2

lnx, f (x) [1, e]


{1, M}
e max{e, eM } = k ( ).

x = 1 , c = 0.

( f (x) lnx) = (lnx f (x))e (lnx f (x))k

(( f (x) lnx)ekx) ) < 0


1 x e,

elnx f (x) = 1 f (x) = lnx.

0 = (( f (e) lne)eke) ) (( f (x) lnx)ekx) )

(( f (1) lnx)ek) ) = 0, f (x) = lnx .


lnx < f (x), lnx = f (x), x [1, e].

4 ( ) x (0, +)
1
: f (x) + e f (x) = x + (1).

f
f + f e = 1 2

x+

g(x) =

+lnx, x > 0.

f = f x + x f + 1 2 =
x
x

(1) :
31

x2

= ( f )2 (x + 1/x) f + 1 1/x2 (

!
1
1
y + 1 2 , y = f (
e f ) y = y2 x +

/2

f (1) = 1 y(1) = 1


Ricatti).
!
1
1
1

x = 1,
y = + , ... u + u

(xe 2 y) = (ex
1 a x2
y= + e2.

a = 0 f (x) = lnx + b
f (1) = 0 b = 0 f (x) = lnx, x [1, e] (
f (e) = 1)?

32

1.

27 ( )
:

Z2

181
x10 + 210
dx <
.
15
15
2816
x +2

x2 + 1 f 2 (x) + f (x)
Z1 

http://www.mathematica.gr/forum/viewtopic.php?f=54&t=8997

1
x10 + 210
dx =
15
15
32
x +2

1
16

0
Z 1
0

4
f 2 (1) f 2 (0)
3
1 + (x/2)10
dx
1 + (x/2)15

2.



 
(1 + x2 )2 > 1 + x2 ( f (x))2 + f (x) 2

> f (x) + x f (x) 2

2
2
1 + x2 > f (x) + x f (x)


f (x) + x f (x) 6 1 + x2


1 + x2 6 (x f (x)) 6 1 + x2

1 + y10
dy
1 + y15

!
y10 y15
dy
1 + y15
0
!
Z 1
1
10
15
<
1+
(y y ) dy
16
0
!
1
1
1
=
1+

1
=
1+
16

16
11
181
.
=
2816

Z1

16


( f (x))2 + f (x) 2 x2 + 1,

1 + x dx 6

Z1

(x f (x)) dx 6

Z1 
0

1 + x2 dx

4
| f (1)| 6
3

28 ( )
f : R R
:

3. t > 0

x R.

Rt

4
1. ( f (1))2 ( f (0))2 ,
3




Rt
Rt 
1 + x2 dx 6 (x f (x)) dx 6 1 + x2 dx
0

4
2. | f (1)| ,
3
3. |F (1) F (0)|

2 f (x) f (x)


x2 + 1 dx f 2 (1) f 2 (0)

( )

2

10
, F f .
9

t3
3

t f (t) t +

t3
3

t2
3

f (t) 1 +

t2
3

f
t = 0

R1

http://www.mathematica.gr/forum/viewtopic.php?f=54&t=23705




R1
R1 
2
2
1 + t3 dt 6 f (t)dt 6 1 + t3 dx.
0

( - Atemlos -
)

33

29 (

)
f (x), g(x) : [a, b] R
g (x) > 0, x [a, b].

[(x a) f (x)]ba
[(x b)g(x)]ba

(x a) f (x) = (x b)g (x), x [a, b]

1)

Rb
a

f (x)dx =

f (x)dx =

f (x)dx =

g(x)dx
Rb

g(x)dx

[(x a) f (x)]ba = [(x b)g(x)]ba

g (x) > 0 f (x) < 0, x (a, b)


2)

Rb

Rb

Rb

Rb
a

(b a) f (a) = (b a)g(b) f (a) = g(b)


g(x)dx

2.)

h(x) = f (x)g(x), x [a, b]

[a, b]

) f (b) = g(a)
) (a, b) : f () = g()

h(a) = f (a) g(a) = f (a) f (b) > 0


f ( (1)) [a, b]
f (a) > f (b) h(b) = f (b) g(b) = g(a) g(b) < 0
(1) g [a, b]
g(a) < g(b) h(a)h(b) < 0
Bolzano x0 (a, b)

http://www.mathematica.gr/forum/viewtopic.php?f=55&t=24860

( )
1) g (x) > 0, x [a, b] g
[a, b] x a

h(x0 ) = 0 f (x0 ) = g(x0 )

(x a) f (x) = (x b)g (x)


0 = (a b)g (a) g (a) = 0 x > a
g (x) > g (a) = 0

h (x) = f (x) g (x) < 0, x (a, b) h


[a, b] x0 .

(xa) f (x) = (xb)g (x) a < x < b


g (x) > 0 x a > 0, x b < 0
f (x) < 0

30 ( parmenides51)
z1 = a + ea i, z2 = b + bi, z3 = c + i ln c
a, b R, c > 0.
1.
z1 , z2 , z3 ,

:
g (a) = 0, f (b) = 0 [a, b]
g (a) 0, f (b) 0

2. |z1 z2 |
z1 , z2
,

2) )

(x a) f (x) = (x b)g (x)


Rb
a

(x a) f (x)dx =

Rb
a

3. |z1 z3 |
z1 , z3
.

(x b)g (x)dx
34

* f (, 0]
* f x = 0 f (0) = 1.
d(L, )
a=
1
2
0 ( L(0, e0 ) L(0, 1)) =
.
2
2

1.
L : y 1 = 1(x 0) y = x + 1.

y = x, y = x + 1
!
1 1
(x, y) = , ,
2 2

.
2
|z1 z2 |
1
1
z1 = i z2 = + i.
2 2
)
y = ex , y = lnx y =
x,
2

|z1 z2 | ,|z2 z3 |
2
z3
L(0, 1) y = x,
N(1, 0),

!
1 1
( ()
z2 M ,
2 2
).
|z1 z3 |
:

2
z1 = i z3 = 1 |z1 z3 | = 2
= 2.
2

http://www.mathematica.gr/forum/viewtopic.php?f=55&t=24493

( ) ) z1
z = x + yi, x, y R, x = a, y = ea , a R
y = e x .
z1
y = e x .
z2 z = x+yi, x, y R,
x = b, y = b, b R y = x.
z2
y = x.
z3 z = x+yi, x, y R,
x = c, y = lnc, c > 0 y = lnx.
z3
y = lnx, x > 0.
) z1 = k + ek i, k R
|z1 z2 | ,

L(k, ek ) y x = 0,
: d(L, ) = p

|ea a|

12 + (1)2

f (x) = e x x
R
e x () x ()
f (x) = ex 1. :
* f (x) = 0 x = 0
* f (x) > 0 x > 0,
* f [0, +)

35

:
(3)

31 ( )
f :
[0, +) R

f (x) + x f (x) =

1
f
2

x

= 0 0 = 0 g (x) = 0 g(x) = c
x f (x) = c 0 f (0) = c ,
f (x) = 0 , x [0, +) .

x [0, +) .

2 ( ) f
x1 , x2 [0, ] , > 0 m
M , , [0, ] .

http://www.mathematica.gr/forum/viewtopic.php?f=56&t=21507

m f (x) M , x [0, ] .

( ) g(x) = x f (x) ,
x [0, +) [0, +)

1 g(x/2) g(x/2)

=
, x>0

2 x/2
x
g (x) =

0 ,
x=0

m f

(1) .

(x f (x)) =

..... g [0, x/2]


0 < 1 < x/2 ,

g () 0

x f (x)

M x 
2

0.

Mt

g(x) g(0)
M
= f (x)
.
x0
2
M
f (x)
.

2
x = x1 , x1 ( x1 = 0 ,
M = 0)

.....

n , n N , 0 < n+1 < n /2 .

f (x1 )

, (x f (x)) =

(2) .

g(x/2) 00
1 g (x/2)
= lim
=
lim g (x) = lim
x0
x0
x0 2
x
x

x f (x)

mx 
2

M 0.

1
f
2

x

m
2

h(t) = t f (t)

1
1
lim g (x/2) = lim g (x) lim g (x) = 0 = g (0) .
x0 2
x0
2 x0

g [0, +) .

, f (x)

m
2

mt
2

t [0, ]

x = x2 , x2 ( x2 = 0 ,
m = 0)

n + n 0 ,
0 < n+1 < n /2 0 < n < 1 /2n1 .

m 0.
2
2
0 m f (x) M 0
f (x) = 0 ,
x [0, ] .
, [0, +).
, .
f (x2 )

lim g (n ) = 0 (3) .
, x [0, +),

1
1
g (n ) = lim n lim g (n )
n
n+ 2
n+ 2 n+

lim g (x) = lim

n+

g () =

1
g(1 /2) g(0) g(1 /2) (1)
g (2 ) =
=
= g (1 ) .
2
1
1

(2)

, t [0, ] .
2
x > 0 [0, x]
(0, x)

..... g [0, 1 /2]


0 < 2 < 1 /2 ,

n+

x

M , x [0, ] .

g(t) = t f (t)

1
g(x/2) g(0) g(x/2) (1)
g (1 ) =
=
= g (x) .
2
x
x

1
1
1
g (x) = g (1 ) = 2 g (2 ) = . . . = n g (n )
2
2
2

1
f
2

x

36

. ,
f
.
2. f ()
A(x1 , f (x1 )) , B(x2 , f (x2 ))

32 ( )

f (x) =

ln(x2 )
, N , N ,
x2+1

y f (x1 ) = f (x1 )(x x1 ) (1) .

http://www.mathematica.gr/forum/viewtopic.php?f=56&t=5181

:
) f (x1 ) = f (x2 )
1. f
x2 = x1 .
B (x1 , f (x1 )) = (x1 , f (x1 )) .
( - - x1 > 0)
) B (1)
f (x1 ) f (x1 ) = f (x1 )(x1 x1 )

ln(x2 )
( ) f (x) = 2+1 ,
x
2
2

(
2

+
1
)
ln(x
)

f (x) =
x2+2
2(4 + 3) + (2 + 1)(2 + 2) ln(x2 )
f (x) =
,
x2k+3
x R , N , N .

f (x1 ) = x1 f (x1 ) (2)

f f .
1.
f
(
) .

ln(x21 )
x21 +1

= x1

2 (2 + 1) ln(x1 )

x21 +2

ln(x1 ) = 2 (2 + 1) ln(x1 )
2

+1


f A (x1 , f (x1 )) , B (x2 , f (x2 )) ,
0 < x1 < x2 . f (x1 ) = f (x2 ) = .

(2 + 2) ln(x1 ) = 2

f [x1 , x2 ]


(1) :

x1 = e +1

f (x2 ) f (x1 )
= .
(x1 , x2 ) , f () =
x2 x1
0 < x1 < < x2

ln(x1 ) =

x1 = e 2+2 .

f (x1 ) = f () = f (x2 ) .

y = f (x1 ) x f (x1 ) =

e( + 1)

: y=
x.
e( + 1)

. Rolle f
[x1 , ], [, x2 ] f

(1)

37

(2)

33 ( )
f : [0, 1] R

f (0) = 0

ef

(x)

( ) y x
f (1) = 0.

f (x) dx = f (1).

f (x) = 0, x [0, 1]

f (1) = lim
x1

f (x) f (1)
=1
x1

http://www.mathematica.gr/forum/viewtopic.php?f=69&t=24696

lim
x1

( )

ef

(x)

f (x)dx = f (1) = f (1) f (0) =

(e f

(x)

f (x)(e

x x0 , h =

, f (x), e
.
(x)

f (x)

L = lim

(x)

f (x0 )
h

h1

x0 f (h)
h2

f (x0 ) x0 (1 h)
h

,
(1) :

f (x)(e f

1) 0

(. f (x0 ) < 0),

x 

f (x0 )
h
.
x0
x0
h
( x = x0 y = h)
f (x) f (x0 )
L = lim
= lim
xx0
h1
x x0

x0 [0, 1] f (x0 ) , 0

R1

x0
1.
x

1)dx = 0

x (0, 1), f (x)(e f

(3)

x0 > 0.

f (x)dx

f (x) f (x))dx = 0

f (x)

f (x)
=1
x1

1)dx > 0, .

L=

, f (x) = 0, x [0, 1]

f (x0 )
+ 1.
x0

x0 , f
/ .. :

f (x) = c, x [0, 1].

!
f (x)
f (x) =
+ 1, x > 0
x

!
f (x)x (x) f (x)

f
(x)
1


=
=
,
x
>
0
=
(lnx)
x
x
x2
!
f (x)
= lnx + c, x > 0 .
x

f (0) = 0, f (x) = 0, x [0, 1]


34 ( )

f : (0, +) R
!

x
f (x) x f (y)
=


y
y
y2
x, y (0, +) 1, f (1) = 1


http://www.mathematica.gr/forum/viewtopic.php?f=69&t=22009

38

f (x) = xlnx + xc, x > 0 .

f (x) = xlnx, x > 0


.

x 1 : c = 0.

39

,
,

35 ( )
n,
sin2n x + cos2n x + n sin2 x cos2 x = 1,
x R.

http://www.mathematica.gr/forum/viewtopic.php?f=109&t=24754

1 ( BillK ) f (x) =

f (x) > 0 .
Jensen
f (a) + f (b) + f (c) 3 f ( a+b+c
)
3

http://www.mathematica.gr/forum/viewtopic.php?f=109&t=24910

( ) x =

x2
, x (0, 3)
(3 x)3

f (a) + f (b) + f (c) 38


a2
b2
c2
3
+
+

(b + c)3 (c + a)3 (a + b)3 8

1
n
1
+ n + =1
n
2
2
4
2n (n 4) + 8 = 0. (*)
n 4 2n (n 4) 0 2n (n 4) + 8 > 0
0 n < 4,
n = 2 n = 3 (*).

2 ( )

 CS Nesbitt : 
a2
(b+c)3

b2
(c+a)3

a
(b+c)

c2
(a+b)3

b
(c+a)

[(b + c) + (c + a) + (a + b)]

2
c
(a+b)

49 .

3 ( )

sin4 x + cos4 x + 2sin2 xcos2 x = (sin2 x + cos2 x)2 = 1


x R
Euler
sin2 x + cos2 x + (1) = 0
(sin2 x)3 + (cos2 x)3 + (1)3 = 3 sin2 x cos2 x(1),
sin6 x + cos6 x + 3 sin2 x cos2 x = 1, x R.
, n 2 3.

a2 (a + b + c) b2 (a + b + c) c2 (a + b + c) 9
+
+

8
(b + c)3
(c + a)3
(a + b)3
X a3
X a2
9

+
.
(b + c)3
(b + c)2 8

:

Chebyshev 9(x3 + y3 + z3 ) (x + y + z)3 .

Nesbitt

36 ( )
a, b, c
a + b + c = 3,

a2
b2
c2
3
+
+
.
3
3
3
8
(b + c)
(c + a)
(a + b)

40

X a 3 27
a3

b+c
8
(b + c)3

X a 2 9
a2

b+c
4
(b + c)2

37 ( )
ABC BC,
I . BI AC
D E, D
CI . EI AB
Z. DZ

CI.

http://www.mathematica.gr/forum/viewtopic.php?f=110&t=13272

(KARKAR)
C, E BC

B
CEI = CDI = 90o +
= DIE = 90o , (1) (1)
2
A = 90o DIZA
IDZ = IAZ = 45o , (2)

(2) DIC = 180o IDC


90o

B
2

2
, (3)

C
2

= 180o

= 45o , DIE = 90o =

BD EZ
(3) = IE = IZ , (4) BD
B.
(1), (4) DEZ,
DZ k FI CI .
38 ( )

ABC E M,
.

(2) ADB = 90o

C
2

= ACI

B
2

, ADZ =

, (3)

(3) DZ k CI
.
2 ( ) E , D CI BC,
FD = FE , (1) F CI DE.
DIC = EIC , (2)

M,
ABC ,
E1 , E2 , E3 .
1
1
18
1
+
+
.

E1

41

E2

E3

E2
(MC1 )(MC2 )
=
, (2)
(MA1 B1)
(MA1 )(MB1)
(MB1)(MB2)
E3
=
, (3)
(MA2C1 ) (MA2 )(MC1 )
(1), (2), (3) = E1 E2 E3
=
(MA1 A2 )(MC1C2 )(MA2C1 ) , (4) (4) ,
,

http://www.mathematica.gr/forum/viewtopic.php?f=110&t=17211

( )

MA1 A2 , MB2C2

.
- :

A1 MA2 = B2 MC2

E1
(MA1 )(MA2 )
=

(MB2C2 ) (MB2)(MC2)

, (1)

E1
6

E2

E3

E1 E2 E3

E1 E2 E3 (MB2C2 )(MA1 B1 )(MA2C1 )

18

E1 + E2 + E3 + (MB2C2 ) + (MA1 B1 ) + (MA2C1 )

42

18

, ,

f (1) = c + 1

39 ( )
f :

f (2y + 1) = c(2y + 1) + (2y + 1)(4y2 + 4y + 1)

(x y) f (x + y) (x + y) f (x y) = 4 xy(x2 y2 )

= c(2y + 1) + (2y + 1)3 ,


f (x) = x3 + cx x R.
40 ( )

http://www.mathematica.gr/forum/viewtopic.php?f=111&t=24585

1 ( ) x + y = a

x y = b x =

a+b
2

y =

ab
2

f : {1, 2, . . . , 10} {1, 2, . . . , 100}

x + y|x f (x) + y f (y), x, y


{1, 2, . . . , 10}.

b f (a) a f (b) = ab(a2 b2 )


a, b R, a, b R

http://www.mathematica.gr/forum/viewtopic.php?f=111&t=18963

f (a) f (b)

= a2 b2 ,
a
b

( ) , x 1, 2, . . . , 9,
2 x + 1 x f (x) + f (x + 1)(x + 1)
2 x f (x) + 2(x + 1) f (x + 1).
, 2 x + 1
f (x + 1) f (x). ,

f (a)
f (b)
a2 =
b2 .
a
b
f (x)
h(x) =
x2
x
x R , f (x) = x3 + cx,
x R . f (0) = 0,
f (x) = x3 + cx, x R, .
2 ( ) x = y + 1

99 f (10) f (1) =

9
X

f (x + 1) f (x)

9
X

(2 x + 1)

x=1

x=1

= 99.

, f (x+ 1) f (x) = 2 x+ 1 f (1) = 1. f (x) = x2


x 1, 2, . . . , 10.

f (2y + 1) = (2y + 1) f (1) + 4y(y + 1)(2y + 1).

43

41 ( )
ABCD (O)
P AC BD.

(1) QBD + QDB = 180o BQD DBC +


BDC = A,
BQD = 90o + A (2)
, AQC = 90o + D , (3)
(2), (3), Q
(K), (M),
BD, AC
BQD, AQC R,
Q, (K), (M.)
(K) ,
ABCD Q,

BKD
BQD +
= 180o , (4)
2
K (K).
(2), (4) BOD = 2A,
BOD + BKD = 180o , KBOD

QAB + QCB = QBC + QDC = 90o .


OBK + ODK = 180o = OBK = ODK = 90o ,
KB = KD OB = OD.
,
(O)
ABCD,
(K) , (M)
(
MAOC , M
(M) ).
, O (O)
(K), (M) ,
QR,
,
P AC BD,

P, Q, O ,
O (O).

http://www.mathematica.gr/forum/viewtopic.php?f=112&t=17404

( .) QBC + QDC = 90o


= QBD + DBC + QDB + DBC = 90o , (1)
44

(O), (K), (M) .

P
A, (a + d)
,
, (b + c) ,
P
C.

, a + b + c + d ,
A, C
.
,
B, D .

ABCD
,
.
,
,
,
.

42 (

)
P
ABCD,
AB, BC, CD, DA, . ABCD .
http://www.mathematica.gr/forum/viewtopic.php?f=112&t=24878

( .)
, .
. P BC
ABC,
. P,
ABCD a, b, c, d,
AB, BC, CD, DA,
.

45

43 ( 1 1995)
6- 5. ,

6

1 ( ) H BO
W CO AH k OD AW k OE .
W AH = DOE = DAE = 60 , WOH = 120 .

http://www.mathematica.gr/forum/viewtopic.php?p=120445

1 ( )
. .

6
b 599.997 10 b2 <
599.998 106 .
b > 599.997 103 > 774 103
2b > 1500 103 .

(b + 2)2 = b2 + 4b + 4 > 599.997 106 + 3 106 =


600.000 106 .
b2 (b + 2)2

A, W, O, H

AW H = AOH = 60 = AOH = AHW.

.
b2 < 599.998 106 < 600.000 106 < (b + 2)2 .

(599.998 106 , 600.000 106 )
,
599.998 599.999.
.
2 ( )
100000
5 .
.
,
(700000, 790000)
(70000)2 < 5 1011 (79000)2 > 6 1011 .
89999
.



(
) : OW + OH = OA(= OB = OC).

AW H .

AD OH OH

=
=

OA

DB OB

AE
OW
OW

=
=
EC
OC
OA
AD AE OH + OW OA
+
=
=
= 1.
DB EC
OA
OA
: 1) D S E A
AD AE
AB/2
0
+
=
+
= 1.
DB EC AB/2 EC

2) 1)
E
AC , D
A.
2 ( )
: BD = x
CE = y, : AD = x AE = y. ,
, : ED + BC = BD + CE ,
: ED + = x + y : ED = x + y .

44 ( )
ABC
AB AC D E ,
DE
. :

AD AE
+
=1
DB EC

http://www.mathematica.gr/forum/viewtopic.php?p=111876

46

:
AF = BF = AG = CG =

c
2

(1)

FD = DH , GE = HE (2) ( )

c = 2 3 (3) ( . . OGC )
+ = 60 , = DOH = HOE
( FOG = 120 FOD = DOH , HOE =
EOG)
+ = 60 ( + ) =

3 3( + ) = 3(1 ) (4)


ADE : ED2 = AE 2 + AD2
2AD AF (1) F E AD.
FEA = 30 , AFE
1
1
( y). , (1),
: AF = AE =
2
2
2
: (x + y ) = ( y)2 + ( x)2 ( x) ( y)
3 xy
=
. -

AD AE
+
BD EC

=
(1)

=
=

x+y
x (2y x)
: AD = x =

x+y
y (2 x y)
AD
AE
AE = y =
. :
+
=
x+y
DB EC
x (2y x) y (2 x y)
+
= 1.
x (x + y)
y (x + y)
3 (Math Rider ) c
. H
DE
F, G
AB, AC :

(2)

(3)

=
=
(4)

=
=

47

AF FD AG GE
+
BF + FD CG + GE
c
FD 2c GE
2
+
c
+ FD 2c + GE
2
c 2FD c 2GE
+
c + 2FD c + 2GE
c 2DH c 2EH
+
c
+ 2DH c + 2EH

2 3 2DH 2 3 2EH
+

2 3 + 2DH 2 3 + 2EH

3 DH
3 EH

DH
EH
3+
3+

3
3
+

3 +
3 +
6 2

3 + 3( + ) +
6 2
3 + 3(1 ) +
6 2
=1
6 2

45 ( IMC 1996)
n .

46 ( )
(an )nN
an > n1 n.

sin nx
dx.
(1 + 2x ) sin x

+
X

an .

n=1

http://www.mathematica.gr/forum/viewtopic.php?f=59&t=25166
http://www.mathematica.gr/forum/viewtopic.php?f=59&t=24073

( )

In =

=
=
=
=

Z0
Z0

Z0
0

sin nx
dx
(1 + 2x ) sin x
Z 0
sin nx
dx
+
(1 + 2x ) sin x
Z
sin nx
dx +
(1 + 2x ) sin x
Z0
sin nx
dx +
(1 + 2x ) sin x
0
sin nx
dx.
sin x

sin a sin b = 2 sin

In In2 =
=

Z0
0

=2

!
+
+
X
X
kn1
an
(kn kn1 )akn >
1
.
kn
n=1
n=1
n=1

+
X

sin nx
dx
(1 + 2x ) sin x
sin nu
du
(1 + 2u ) sin u
2u sin nu
du
(1 + 2u ) sin u

1 kknn 1 0 ln
x1 1 x 1,

ln(kn ) = ln(k1 ) +

n
X
i=1

1 ( ) kn
.

a
2

b
2

cos

a
2

b
2

.
2 ( )
, kn
kn > 2kn1 n.
1 kknn 1
0 .

sin nx sin (n 2)x


dx
sin x
2 sin x cos (n 1)x
dx
sin x

3 ( )

!
ki
ln
,
ki1

cos (n 1)x dx = 0.

n
P

k=1

n
P

k=1

ak

(ak an ) (

),
nan . , an > nc
c > 0 n,
.
lim nan = 0,

I0 = 0 I1 =

0 n = 2k,

In =

n = 2k + 1.

48

n+

.
a, b, c M

47 ( )
M = R \ {3} x y = 3(xy 3 x
3y) + m, m R. m
(M, ) .

a (b c) = 9abc 27(ab + bc + ca) + 81(a + b + c) 240


= (a b) c

http://www.mathematica.gr/forum/viewtopic.php?f=10&t=23549


(M, ) .

( )
,
e. :

48 (vzf) m nn .
, n
, .

1 e = 1 3e 9 9e + m = 1 m = 6e + 10

http://www.mathematica.gr/forum/viewtopic.php?f=10&t=23825

( )
k > n A1 , A2 , . . . , Ak n n .
:

2 e = 2 6e 18 9e + m = 2 m = 3e + 20.
e = 10

3
m = 30. m = 30
.
a, b M

,I{1,2,...,k}

= 3(ab 3a 3b + 27) + 3

det (A1 + A2 + A3 ) det (A1 + A2 ) det (A2 + A3 )

= 3(a 3)(b 3) + 3 , 3

det (A3 + A1 ) + det (A1 ) + det (A2 ) + det (A3 ) = 0.

a , 3 b , 3.
. a M

(1) :
m > n A1 , A2 , . . . , Am nn .
r
, r 6 n, .
, (1) k = n + 1
r
, r 6 n + 1, .
, (1) k = n +
2, . . . , m A1 + A2 + + Am
.
(1).
[k] := {1, 2, . . . , k} :

10
10
=
a
3
3
10
= 3a 9a 9
3
= 10a 9a 30 + 30

=a

10
.
3
a M

ax = xa =

x =

X
(1)|I| det
A j
jI
,I{1,2,...,k}

n
X

X
Y
X

(1)|I|
(i, (i)) .
=
sgn ()
A
j

10
3

3ax 9a 9 x + 30 =

(1)

jI

, k = 3, (1)
2 2 A1 , A2 , A3

a b = 3ab 9a 9b + 30

X
(1)|I| det
A j = 0

10
3

27a 80
,3
9(a 3)

,I[k]

49

S n

i=1

jI

, :

, J

j1 , j2 , . . . , jn .)
, I {1, 2, . . . , k} J I ,
aJ (2)
, (1)|I| .
|J| 6 n < k, r = |I| |J|,
0 6 r 6 k |J|, a J
(2)

X
(1)|I| det
A j
jI
,I{1,2,...,k}

n
X

Y
X
X

.
(1)|I|
(i,
(i))
=
sgn ()
A

S n

i=1

,I[k]

jI

, , S n .
ai j = A j (i, (i)) . :

,I{1,2,...,k}

n
Y
X

ai j = 0.
(1)|I|

i=1

k|J|
X

|J|+r

(1)

r=0

(2)

jI

|J|

=(1)

(2)
a J := a1, j 1 a2, j2 an, jn , J :=


j1 , j2 , . . . , jn ( j1 , j2 , . . . , jn
,

k|J|
X

k |J|
r

k |J|
(1)
r
r=0
r

=(1)|J| (1 + 1)k|J| = 0.
(2), (1), .

50

,
branch cut

49 ( )

ln(1 aix)
dx ,
x2 + m

i
, .
a

ln(1 aiz) = ln(az + i)ln i = ln |az + i|+i arg(az + i)


i
i
ln i = ln |i| + i arg(i) =
.
2
2
C : [0, ] C
A (R, 0) , B (R, 0).
z1 =

a > 0, m > 0

http://www.mathematica.gr/forum/viewtopic.php?f=9&t=7842&start=160

1 ( )

I1 =

x2

b2

I2 =

x


x2

c2

 dx =

branch cut,
f ( ),
Z

, b, c > 0 .
2 (b + c)

f (z) dz = 2i Res( f, i m ) ,
C

i m.
Z
Z
Z R
it
it


f (z) dz =
f (Re ) iR e dt +
f (z) dz .
C
0
R
Z
Z
ln(1 aiR eit )
it
it

f (R e ) iR e dt =
iR eit dt .
2 e2it + m
R
0
0
R
Cauchy:

ln(1 + x2 )

dx = ln(m + 1) ,
2
2
m
x +m

ln(1 + x2 )
dx =
x2 + m2
0
!
Z
Z x
1
2y
y = xt
dy dx ===
2
2
2
x +m
0
0 1+y
!
Z
Z 1
1
2 x2 t
dt dx =
x2 + m2 0 1 + x2 t2
0
!
Z 1 Z
x2
2
t

 dx dt =
x2 + m2 1 + x2 t2
0
0
Z 1 Z

1
x2
I1


2
dx
dt ==
 1
2
2
2
t
0
0
x + m t2 + x
I2 =

Z1

Z
it
it
iR e f (R e ) dt 6



ln(1 ai R eit )
dt .
R
R2 e2it + m

R2 e2it + m > R2 |e2it | m = R2 m , R > m,




ln(1 aiR eit ) = |ln(1 + aR sin t aiR cos t)| =


ln 2 + 2aR sin t + i arg(1 + aR sin t aiR cos t) 6

dt =
ln(m + 1) .
t m + 1t
m
0
Z
ln(1 axi)
ax = y

dx ====
2
x +m

Z
Z
a ln(1 + y2 ) + i arctan y
ln(1 + yi)
a
dy =
dy
2
2
2
y2 + a2 m
y + a m
Z
Z
ln(1 + y2 )
ln(1 + y2 )
I2
=a
dy
+
0
=
a
 2 dy ==
2 + a2 m
y
0
0 y2 + a m


ln 1 + a m . 
m

3 1
3 + ln(1 + 2aR)
+ ln(1 + 2aR sin t) 6
.
2
2
2



ln(1 aiR eit )

3 + ln(1 + 2aR)
dt =
2 e2it + m|
|R
2R2 2m
0
0
3 + ln(1 + 2aR)
0 , R .
R
2R2 2m
Z

lim
f (Reit ) iR eit dt
=
0
R 0
Z R
Z
lim
f (z) dz =
f (z) dz .

R+

2 ( ) ln(1 aiz)
f (z) =
, z C, a > 0, m > 0
2

z +m

51

dt 6 R

f (z) dz = 2i Res( f, i m ).



ln ai m + i
Res( f, i m ) =
=

2i m


 

ln i a m + 1 + i arg i a m + 1 i/2
=

2i m


ln 1 + a m
.

2i m


ln(1 aix)

dx = ln 1 + a m .
2
x +m
m

n
X

 ln( j/n) 
1 
1 + O ln n
=

ln(x/n) dx + ln(2/n)

n
X
j=2

n an
an
n
= e n ln n .

j=2

j=2

n

n
ln n
. :

an+1 an
=
n+1
n
ln(n+1) ln n

ln(n+1)
(n+1) ln nn ln(n+1)
ln(n+1) ln n

ln n
 =
ln n+n 1 n n


ln n
1
=
1
ln(1+ n1 )n
n + ln n
ln
1
ln n + 1
1+ n
an
Cesaro-Stolz
: lim n = 1 .

ln(x/n) [2, n].


n

n 
ln2 n

an
n
n
X
X
1
1
, 1 +
>1+
.
ln j
j1

n
X


P
1
n1
=
+ O ln12 n nj=2 ln( j/n) .
ln j
ln n
j=2

n
X
1
n1
=
+O
ln j
ln n
j=2

an
an
ln n = n .
n
ln n

ln(x/n) dx + O(ln n) = n + O(ln n) .

ln n 1 + ln( j/n)
ln n
ln n


1
ln( j/n)
, 1 <
+ O ln(ln2j/n)
0.
n
ln n
ln n
ln j

2 ( )

1 ( ) 2 j n
1

!
n an
ln n 
n1
2 
n
= exp
1+
+ O n ln n =
n
ln n


exp 1 + O(ln1 n) e . 

http://www.mathematica.gr/forum/viewtopic.php?f=9&t=23985

ln( j/n) =

j=2

50 ( ) an =

P
1 + nj=2 ln1 j lim( n n)an

ln(x/n) dx ,

n+1

ln( j/n)

ln n

e . 

52

51 ( ) An = {n, 2n, 3n, ...}


n N

A1 A2 ...An

Ai

J N ,
\

Ai
iJ

http://www.mathematica.gr/forum/viewtopic.php?f=64&t=23440

http://www.mathematica.gr/forum/viewtopic.php?f=64&t=22887

( ) A
x
A(x) = 0 x < A
A(x) = 1 x A.

(AB)(x) A(x) + B(x) mod 2.

x A1 An (A1 An )(x) = 1
A1 (x) + + An (x) 1 mod 2
x Ai .


modulo2.

( ) : .
A j ,
N . ,
m N . J , j J j > m
( J {1, 2, ..., m} = ).
m < A j , .
.
52 ( )

A, B AB = (A \ B) (B \ A),
.

53

, p 1 k.
,
k p 1, pk p p1 > (p 1) p1 . ,
pk (p 1) p1 + 1 > (p 1)! + 1 = pk ,
. .

53 (
) p P p > 5 (p 1)! + 1
p (
pk k N ).

54 ( dimtsig)

.

http://www.mathematica.gr/forum/viewtopic.php?f=63&t=20371

( )
p > 5 ,
(p 1)! + 1 = pk k. ,

(p 1)! = pk 1.


: (p 1)! 0 mod(p 1)2 .
,
p1
(p 1)2 = 2
(p 1) ,
2
, p > 5,
p1
2<
< p 1.
2
p1

, 2,
, p 1
2
1 2 3 (p 1) = (p 1)!,
.
,  (p 1)2
pk 1 = (p 1) pk1 + + p + 1 .
, p 1
pk1 + + p + 1.
i {1, 2, . . . , k 1}
i
p 1 (mod (p 1)) ,

http://www.mathematica.gr/forum/viewtopic.php?f=63&t=22945

( ) 1 .
m
111...1 = m2 , n > 1.
|{z}
n

, m
1 9.
m m = 10a 1,

a N .

,
111...1 = 100a2 20a + 1,
|{z}
n

111...1 0 = 100a2 20a,


|{z}
n1

111...1 = 10a2 2a.


|{z}

n1

, ,
.

pk1 + + p + 1 k (mod (p 1)).

54

:
( ) f 2 (z) + g2 (z) = 1

55 ( )
S ,

s1 , s2 , ..., sn s1 + s2 + ... + sn < 1. S ;

( f (z) + i g (z)) ( f (z) i g (z)) = 1


f (z) + i g (z) , 0 z C
(z)

http://www.mathematica.gr/forum/viewtopic.php?f=13&t=18203

( C ) f (z) + i g (z) = e(z) .

( ) n N
An = {sk : sk n1 }.
( n
). S = An , S .

f (z) + i g (z)
(

56 (

) (entire)
f, g

f (z) =

e(z) + e(z)

f (z)2 + g(z)2 = 1 z C.

= f (z) i g (z) = e(z) .


)
f (z) + i g (z) = e(z)

f (z) i g (z) = e(z)

g (z) =

e(z) e(z)
2i

(z) = i (z)

f (z) = cos ( (z)) g (z) = sin ( (z))


http://www.mathematica.gr/forum/viewtopic.php?f=13&t=13878

(z) .

55

57 ( )
BE B

O
BO
3

= .
OE
2
.

2
AC

1 + = 1 +
. AB + BC
3

( )
ABE, BEC :
3

=
2

2 3
2+
3 = AB + BC + CA .

AB + BC
2

http://www.mathematica.gr/forum/posting.php?mode=edit&f=27&p=119266

1
1
AB BC = (AB + BC + CA)r (1)
2
2

AB
AB2 4AB BC + BC 2 = 0
= 2 3.
BC

AB
o

= 2 3 = tan 15
BC
sin C

= tan 15o .
sin A

C = 15o A = 75o .

AB

= 2 + 3 = tan 75o
BC

BO AB BC AB + BC
=
=
=
.
OE AE CE
AC

58 ( )

sin arctan

sin C + cos C =
1 + sin 2C

2
3
=
2
= 150

+ arctan

1
111
1
+ arctan
+ arctan
11
13
121

http://www.mathematica.gr/forum/viewtopic.php?f=27&t=23387

( )

1
1
2 1
2
AB BC = AB BE
+ BC BE
.
2
2
2
2
2

AB BC
BE
=
AB
+ BC
2

A = 750

2 ( )
(ABC) = (ABE) + (BEC).

1
1
1
+ arctan
+ arctan
3
5
7

arctan (x) + arctan (y) = z


x+y
tan z =

1 xy
!
x+y
z = arctan
.
1 xy

(1).

r
ABC BO = 2r
2 3
EO =
r.
3

56

!!

1
1
1
1
arctan
+ arctan
+ arctan
+ arctan
+
3
5
7
11

arctan
tan (x) = z


1
= tan
x
z
2
!
1

arctan
= x = arctan (z)
z
2
2
!
1

arctan
+ arctan (z) = .
z
2

1
111
arctan
+ arctan
=
13
121

1
1
1
4
+ arctan
+ arctan
+ arctan
+
7
7
11
13

111
arctan
=
121

7
1
1
111
arctan
+ arctan
+ arctan
+ arctan
=
9
11
13
121

43
1
111
arctan
+ arctan
+ arctan
=
46
13
121

121
111
arctan
+ arctan
=
111
121

1
1
1
+ arctan
+ arctan
sin arctan
3
5
7

+ arctan

57

1
1
111
+ arctan
+ arctan
11
13
121

!!

= 1.

3 ( )
A = AK AN = AP AB = AT AM =
AQ AC . (o) BC .
S T
A, M, S ,(S S ,
AS ) BC
M . S A M
M M AS , = PBCQ,
= QCS S , = ()//BC ,
ABCS , M
AS , M BC .

59 ( KARKAR)
A ABC
BC . (O)
A AB, AC
P, Q .

PQ S ,
S T . AT
M BC .
http://www.mathematica.gr/forum/viewtopic.php?f=62&t=22297

1 ( ) AQT P ,
AT AQP.
BPQC ( )
BC PQ.
AT
BC .
2 ( ) AM
S . PQ ( S )
S .
- A.PRQS .
BC// BC
AT .
4 ( ) N
[ = MAC
[ ,
PQ, NAP
ABC, APQ . S , A, O, N, T
S O,

[ = S[
[ = MAC
[
S[
NA = S[
T A = S[
AT , NPA
AQ NAP
( ).

58

QH
2Rr + r2
=
QH QA = KQ R = 2Rr + r ,
Q
r2

H
2R

=
.
Q
r

Q KL AH
=
=

r
R
2R

H
AH 2R

H = AH D = AD,

r
2R
r
QZ = A, ct, Q

.
, r2 = Q QA.

60 (
[ ] ) ABC
A , BAC
BC
.
ABC
.

http://www.mathematica.gr/forum/viewtopic.php?f=62&t=22802

1 ( )
,
p, c,
Q = (0, p), KQ KA = c(1).
BKC = 2A,
KBC
,
> 1,
KA = y0 x20 = 2 y20 y20 2 + 2y0 (2). (1)

x20 + (y0 p)2 = y0 + c (2)

2y0 ( p c) = c2 + 2 p2 .
c = p, c2 + 2 = p2 
2 + 1
2
c = 2
> 0, p =
< 0.
1
1! 2
2
, Q, 2
.
1
2
( )

AW, AS , BAC = AEZ = AZE (
), c ,
.
: BAC = AEZ = AZE
KBFC, QZFE.
L, F, .
Q QA = r2

3 ( ) AD
A = AD2
() (c) AD B , C
B, C .
59

(o) (ABC) BC .
A , BC
(n)
(c).

( B C (n)) ,
(ABC)
(n ) (n).

60

61 ( )
C :


z +

|w| = |w2 2| = 2



1
1 2
= z + 2 = 2
z
z

2=

|w2 2| = 2 |w2

|z +

+ 2| = 4. w = z + 2 .
z2
z
|w + 2| = 4 |w| = 2.



= 2 z +

z2 +




1

2
= 4 z + 2 + 2 = 4 : (1)
z
z

i=1



z2 + 1 = 2 : (2)

z2
z2

(2)

= x + yi, (x, y R)

Q(x) = a1n P(x).


P(x) = (x x1 )(x x2 )(x x3 )...(x xn1 )(x xn )
P (x) = (x x2 )(x x3 )...(x xn1 )(x xn )
+(x x1 )(x x3 )...(x xn1 )(x xn )
...
+(x x1 )(x x2 )(x x3 )...(x xn2 )(x xn )
+(x x1 )(x x2 )(x x3 )...(x xn1 )
P (x) = (x x3 )...(x xn1 )(x xn )+(x x2 )(x x4 )...(x
xn1 )(x xn ) +... + (x x2)(x x3 )...(x xn2 )(x xn ) + (x
x2 )(x x3 )...(x xn1 )
+(x x3 )...(x xn1 )(x xn ) + (x x1 )(x x4 )...(x
xn1 )(x xn )+ (x x1 )(x x3 )...(x xn2 )(x xn ) + ... + (x
x1 )(x x3 )...(x xn1 )
...
+(x x2 )...(x xn2 )(x xn ) + (x x1 )(x x3 )...(x
xn2 )(x xn ) +.. + (x x1 )(x x2 )...(x xn3 )(x xn ) + (x
x1 )(x x2 )...(x xn2 )
+(x x2 )...(x xn1 ) + (x x1 )(x x3 )...(x xn1 )
+... + (x x1 )(x x2 )...(x xn2 )
: P (xi ) = (xi x1 )...(xi xi1 )(xi xi+1 )...(xi xn )

x2 + y2 = 2 x2 + y2 = 4 : (3)

(1) |(x + 2) + yi| = 4


(x + 2)2 + y2 = 4 (x + 2)2 + y2 = 16 : (4)

(3) (4)
(x, y) = (2, 0).

z2 +

z2

= 2 z4 2z2 + 1 = 0


2
z2 1 = 0 (z = 1 z = 1)
3 ( )

z+

=0

1 ( ) an = 1,

P (xi )

http://www.mathematica.gr/forum/viewtopic.php?f=60&t=22338

| = 2 |2w w|
=2

n
X
P (xi )

1 2

62 ( )
P(x) , n
x1 , x2 , ..., xn .


w = 2 z2 = 1.
z = 1 z = 1.
2 ( )


z +

ww

w = x + yi |w| = 2
|2w w|
= 2 w = 2 w = 2 ( x = 2
y = 0). .

1
1 ( ) |z + | = 2
1

http://www.mathematica.gr/forum/viewtopic.php?f=60&t=24667

ww

=w
61

p
1
) = , x , u1 , ..., un1 u1 , ..., un1

p
S
p ()

P (xi ) = 2(xi x2 )...(xi xi1 )(xi xi+1 )...(xi xn ) +


...+ 2(xi x1 )...(xi xi2 )(xi xi+1 )...(xi xn )+
2(xi x1 )...(xi xi1 )(xi xi+2 )...(xi xn ) + ...+ 2(xi

x1 )...(xi xi1 )(xi xi+1 )...(xi xn1 )


:

P (xi )
=
P (xi )

xi x1

+ ... +

n
X
P (xi )
i=1

P (xi )
2

+2

=2
1

x2 x1
1

xn x1

xi xi1
1

x1 x2
+
+

x2 x3
1

xn x2

xi xi+1
1

x1 x3
+ ... +
+ ... +

+ ... +

+ ... +
1

x2 xn

xi xn
1

x1 xn
!

xn xn1

(p )2 p p S
p
S
=

=
1
+
, x , u1 , ..., un1
p
S2
S2
(p )2

1
1
1
p ( xr1 + ... + xrn ) ( (xr1 )2 + ... +
=
2
p
( 1 + ... + 1 )

xr1

+
2

+ ...
!

x r1

i, j

xrn

xi x j

1
( xr
+ ... +
1

xrn )

=0

P (ri )
=
P (ri )

2 ( )


1
1
p
( ) =
+ ... +
= S , x , r1 , ..., rn

n
P

(xrn )2

2
...

x rn

62

ri r1

+ ... +

ri ri1

ri ri+1

+ ... +

ri rn

  

     
    
   http://www.mathematica.gr

      

  
  ! 
!  http://www.mathematica.gr
"
 ! 
LATEX#  
   $ %# 
    %
#  
   
  $ &' #    %


 
 LATEX
"
    
   



(




  
     Leonardo da Vinci

 

 
    (
  )*+,
 - 

   ! 
!  '
 
 ! .
 */  
!   ! ! 
!   '  (
    ( 
   
&  
! !    
 !  
         
      
 

 ,      
  (
   (  

 ( '   '  
    !   !  , 


       ( 
   
, quasiregular (
     $  

  
  
 ( (!
 '  
!
         )  
   0 ,

 - 
 

  


  
     

!    '
!
  
 
! !
   ! 

!



 (0, 0, )$ 12 , 2 , 1+
$    ! ! 1+2 5
'    (
 
   0   
 
2
1#http://en.wikipedia.org/wiki/Icosidodecahedron
2# 1 ! 3 !

 
mathematica.gr             
 
  
   http://www.mathematica.gr 
     


  mathematica.gr
*
 ( < @: 2)<5 

 

 @ & matha


      
$ /(  &
 /(  &

 
 !" Mihalis Lambrou #  ' 8&&
 ( emouroukos
$ % &
 nsmavrogiannis #  , 3& " #&  gbaloglou
' "( !
 !
 "( )"(* 1 5# A "#
 R BORIS
+
!

, - . chris gatos )"(* /&!!0 4 
 % 
 %
!(
= 
 /"" /"" 

1   /"&
&
 m.papagrigorakis )"(* > 6
!
 
 dement
2!3
? "!"
 & "!"
 &
4 3& 5. 3& 5. )"(* +#
 
 #& swsto
#! 

   A#"  achilleas



&#
  grigkost 6
$  <
&A <
&A 3
 7 & 
 cretanman 6
' A! <"
 A!

  
, - <A
 xr.tsif 

  stranton
1 
 -"
 polysot
  89
  %65+  8 58+5 :
4 6
!
 -A
 Demetres
$ "( 8 #"  spyros
 
'  8 vittasko
- 
 -5:<2  56 :
, 6
!
 ; 6::<5:
 @
 "
&
 mathfinder
1 8
 9 KAKABASBASILEIOS
$ @! 5B#A
 @! 5B#A

4 3&
  *
 exdx
'  5(!
 rek2
= "( " 
 s.kap
, 3& 5#"  hsiodos
> % "
 nkatsipis
?  3
 3
 
1 8
 A
 bilstef 

, ,

10 22.


1 ( KARKAR) :
11, s s , 11,
s s
.


.
4 ( )
:

.. 917356 s = 9 + 7 + 5 = 21
sa = 1 + 3 + 6 = 10, s s = 21 10 = 11 = o11.

N=

: 917356 = 1183396.

2012
1
1
+
+
+ ... +
12 23 34
2012 2013
1

10 9876543210. 10
9.
8, 11 ( ; )

, , 8, 9, 10, 11

5 ( KARKAR) A ,
AB , AC . S , T B .
,
, ;

2 ( )
. !

6 ( )
5
2
a = 22 55


7 ( ) 1 .
ZE


3 ( )
.

8 ( ) ABC ,
a, b, c
:
a2 + c < 2a + b

b2 + a < 2b + c
c2 + b < 2c + a
ABC .

,
9 ( ) x2 + x + = 0 :

|| > 0

|| + || < ||

(1 )
(2).
,

13( )
: 2(sin t + cos t) = tan3 t + cot3 t.

10 ( )
f (x) = ax2 + bx + c ,
a < b f (x) 0 x R.
P =

14 ( )
P(x), Q(x)

a+b+c
.
ba

Q(x2 ) = (x + 1)4 x(P(x))2 .

11 ( )
K AB. = 150 . AC , C ,
C , B AC

CD = AC . D
AB .

15 ( ) ABC
(C) .
BC T AB
AC S .
T Z, S E (C).
BE, CZ A .

12 ( )
xCy = 600 . B Cy
BA C x. ABD
C, D AB.
1) P CD AE
ABD, AC = PE .
16 ( KARKAR) (K) (O, R) . S T .

2) P M AB
C x S Cy T , T M = MP = PS .

4. |z w|

h
i z C
22 ( )

z2 + z + 1 = |z + 1| , : Re (z) 1, 31 .
, , ,
23 ( Mulder) f : R R , c R
f (x) = c .
24 ( )
,

1.

f (0, +)
f (x) > 0 x > 0,
 
1
g g (x) = f (x)
+ f x1
(0, +).

17 ( ) AB = 2,
.
18 ( ) A(a, b) a, b , 0 a , b
~ 2 , O
~ = OA
~ A
K(k, 0) (0, ) AK
.
1) K
(), OA.
2) () (1 ) : x y = 2 P,
(OP) = 2, A
() : y = x .

2. h (x) =
1

ex

+ e x ln x

3. e2x + e 2x

> ln 2 x + e

, ,

25 ( ) :ex +
ex + 2 cos x 4, x R

,
19 ( )
( xi ): 0, 1, 2, ..., n ,n N
2k + 1
d=
,k N.
2
1. n

26 ( ) f : R R
f (x) = f (x + 1) 1, x R . f .

2. n = 2k + 1
3. k = 2 .

, ,

4. yi = axi + b,ab < 0


x = y sy = sx a, b.

27 ( )
f : [0, 2] R .

20 ( ) A .

Z2 




3

2
x + x f (x) + 2 x + 2 f (x) dx > 8 f (2) .

1
[P(A)] + [P(A )] .
2
2

28 ( ) 0 < a ,
1 f R,
:

, ,
21 ( ) z w

|(1 + i)z 2 4i| =

18

Z1

w = 2z 11 + 5i.

f (x) ln(1 + a

f (x)

)dx = ln a

Z1

f 2 (x)dx.

:
1. z
2. |z|

, ,

3. w

29 ( ghan) f : R R
2e x
x R : f (x) =
.
1 + f 2 (x)

36 ( )

x2010 2006 = 4y2009 + 4y2008 + 2007y.

) f (0)
) f

Juniors,

) ln f (x) > 0.

37 ( KARKAR) ABCD (O) S


AB CD T AC BD.
A, T, D B, T, C, P.
A, O, P, B O, P, S
.

30 ( )

(x2 )x , x , 0,
f (x) =

1, x = 0

,
.

38 ( erxmer) ABC ,
BAC = 30o ABC = 70o . M
MAB = MCA = 20o .
MBA.

, ,

31 ( )

Seniors,

- -

 Rx 3 1 2 
lim x et sin( t )x dt .

x+

39 ( ) a, b, c
,
a + b + c = 3. :
ab2 + bc2 + ca2 + abc 4.
;

32 ( )

f, g :

[0, 1] (0, )
Z 1
Z 1
f (x) dx =
g(x) dx = 1 .
0

40 ( ) f : R
N

, [a, b] [0, 1] :

f (x) dx =
a

b
a

f (x +

1
g(x) dx = .
2

f (y)

) = f (y +

f (x)

x, y R. n N
f .


Seniors,
33 ( )
f : [0, +) [0, +)

x [0, +) f (x) = x

Z1 

x+

41 ( )

2

ABCD P, AD
(O), P , A , D. PR k AD
R AB PT k AC T CD. S PB AD,
R, S , T .

f (t) dt.

34 ( parmenides51)
f

f (x)

42 ( .)
ABCD AB = a, BC = b a = b 2.
CD , . M, MA, MB,
CD K, L, . AD
D DZ = KC BC C
CE = DL. KLEZ
a.

(x + y) f (y)dy = x x R.

Juniors,
- -
35 ( )
n

x4 2011 x2 + n = 0 4 .

43 ( - 1995) A B
3 3, ,

50 ( ) f : R (0, +),
:

x Q, y R Q

.

. ,
.

.

f (x) f (y) |x y| .


51 ( ) f : R R
. h, g : R R

g( f (x)) = h( f (x)) = x,
44 ( - 3
1995)
P(x) = an xn + an1 xn1 + + a1 x + a0

an , an1 , . . . ,a1 , a0 R. a0 > 0, an > 0 a21 + a22 + + a2n1


4 min a20 , a2n
(1), P(x) 0 x R.
n1

x R,

g = h.
52 ( )
, .


53 ( ) n   ,
22n 22n+1 1 .


45 ( IMC 2012)
na2n
1
a0 , a1 , . . . a0 = 1, a1 =
an+1 =

2
1 + (n + 1)an

54 ( )
12 22 + 32 42 + 52 62 +

X
ak+1

n 1.
ak
k=0

+ . . . + 20012 20022 + 20032 ?

(mod 2005).

46 ( IMC 2012) n n! + 1
(2012n)!,


55 ( )
R R.

47 ( )

56 ( )
S = {a1 , a2 , a3 , ..., a} (an ) 1 1
lim an = a.

G n
a, b G \ {e}, m N ab = bm a.
n
b (m,n) = e.

S 2
( ).

n+

48 ( ) G 2(2n + 1), ( n N),


2n + 1.

()
57 ( )
a, x1 , x2 , ..., xn R a 2.

A = cosx1 +

cosx2 cosx3
cosxn
+
+ ... + n1
a
a2
a

B = sinx1 +

sinx2 sinx3
sinxn
+ 2 + ... + n1 ,
a
a
a

49 ( )

X
n=1

arctan

n2 + 1

A2 + B2 > 0.

q
q

2+1
 q  22+1

2
e
+
e

21
arctan tan
q
q
2

2+1
2+1
e 2 e 2

58 ( )
n,


.
8

sin(na) cos(na)

= n 1,
sin a
cos a
a ,

k
2

, k Z.

61 ( ) z
,
,

A = |z2 + z + 1| + |z2 z + 1|

59 ( erxmer) ABC (C 1 , R)
(C2 , r). d
, (Euler) R2 2Rr = d2 .
;

62 ( ) :

a1 ,
a2 , .....,
an

60 ( ) a,
.

c =

an
a2 .....
a1

:


c 2

11. 9 + m k n 11.

mkn = 2, k, m, n {1, 4, 3}

1 ( KARKAR) : 11, s s ,
11, s
s
.

2 :
98765kmn40, k, m, n {1, 2, 3}
11 9 + 7 + 5 + m + 4 8 6 k n
/ 11, 11 + m k n /
11 mkn = 0
m = 3, k = 2, n = 1 m = 3, k = 1, n = 2.

9876523140 9876513240.
10, 9, 11
8 9876513240

.. 917356 s = 9 + 7 + 5 = 21
sa = 1+3+6 = 10, s s = 2110 = 11 = o11.
: 917356 = 1183396.
10
9876543210.
10 9.

2 ( )

. !

8, 11 ( ; )
,
,
8, 9, 10, 11

http://www.mathematica.gr/forum/viewtopic.php?f=44&t=20063

( ) .
8, 4. ,
2 4
, 4.
,
:
1 :
98765kmn, k, m, n {1, 4, 3}.
9 + 7 + 5 + m + 2 8 6 k n
7

F : 14.
http://www.mathematica.gr/forum/viewtopic.php?f=44&t=22167

G : , J : 24.
( )

H : .

A: .
B: 5.

I : 9012345678.

C : .

K:
13.

D : 7.
E : .

L:
.

3 ( )

.
,
,
10
22.



4 .
4 ( ) :

2012
1
1
+
+
+ ... +
12 23 34
2012 2013
.

N=

http://www.mathematica.gr/forum/viewtopic.php?f=33&t=31086

( )
10 : 0, 1, 2, 3, 4, 5, 6, 7, 8, 9

http://www.mathematica.gr/forum/viewtopic.php?f=33&t=30872

( )

N=

22 , :
4, 5, 6, 7 ,

2012
=
1
1
1
1
1
1
1 1
+ + + ... +

1 2 2 3 3 4
2012 2013

2012
2012
=
= 2013, .
1
1
2012

1 2013
2013

5 ( KARKAR) A ,
AB , AC .
S , T B .
,
, ;

AB (O, R)
1
(AB) = 2R = R, .
2

[ BOS AOS
[
[
OBS , BS O,
[.
OBS

http://www.mathematica.gr/forum/viewtopic.php?f=34&t=30934

(AS ) =
( ).

(AT) =

R 2
180

2R
180

R
90

90


.
, , ,
.
6 ( )
2
5
a = 22 55
http://www.mathematica.gr/forum/viewtopic.php?f=34&t=27337
5

( ) : a = 22 55 = 232 525 =
225 27 525 = (2 5)25 27 = 1025 27 = 128 1025
(B, AC) 2R,
1
(AC) = 2 2R = R
4

128 25
, 28 .

10

7 ( ) 1 .
ZE

A ,K M
H AB ,ZE .

EH = K (1) 180 AB
AB A , B , H K, E .
KM
(2) Z AK (3) ZH =
2
AK M

http://www.mathematica.gr/forum/viewtopic.php?f=35&t=19335

1 ( )
A K M.

ZH//K M H AM.
K
ZE =
(4) AK
2
EZ//K Z AK (3).

ZE K.

ZH = ZE + EH
K A .

K =

KM
2

1
.
3

KM
2

K
2

+ K

3K
KM
K =
2
3

ZE =

1
ZE = .
6

(1),(2),(4)

K
2

KM

KM
6

1
6

8 ( )
ABC , a, b, c :

2(Parmenides51)

a2 + c < 2a + b
b2 + a < 2b + c
11

c2 + b < 2c + a

(c 1)2 = c2 2c + 1 a b.

ABC .

(a 1)2 + (b 1)2 + (c 1)2

http://www.mathematica.gr/forum/viewtopic.php?f=35&p=103474#p103474

(b c) + (c a) + (a b) = 0.
, a = b = c = 1.

( )

(a 1)2 = a2 2a + 1 b c,
(b 1)2 = b2 2b + 1 c a,

, , 1.

12

9 ( )
x2 + x + = 0 :

http://www.mathematica.gr/forum/viewtopic.php?f=19&t=30621

1 ( )

|| > 0

ax2 + bx + c 0, x R

(1)

|| + || < ||



(a = b = 0 , c 0) a > 0, b2 4ac 0 .

(2).

a < b
0 < a < b b2 4ac 0 , c

b2
.
4a


http://www.mathematica.gr/forum/viewtopic.php?f=19&t=27655

( ) D 0 b2 4ac
2

P=

a +c +2|ac| < 4ac.

ba

b2
4a =

5
9a
b
b2 + 4ab + 4a2
+ +
=
=
4a (b a)
4a 4 4 (b a)
ba
3
9a
=
+
+
4a
4 (b a) 2
s
!
!
9a
3
ba
+ =3,
2
4a
4 (b a)
2

a, c 2
. a, c :

a2 + c2 + 2ac < 4ac (a c)2 < 0, .

2 .

. :

|a| + |c| < |b| 1 + |x1 x2 | < |x1 + x2 | < |x1 | + |x2 |
(1 |x2 |)(|x1 | 1) > 0

)
ba
9a
b2
=
a>0
c =
4a
4 (b a)
4a

b = c = 4a > 0.


1.

Pmin = 3.
2 ( )
:

,
c = 0
|ac| > 0.

0 < a < b c

10 ( )
f (x) = ax2 + bx + c ,

P=

b+c+a
=
ba

b2
.
4a

!2
!2
c b 2 b
b
b
b c

+1
+ + 1 a 2a a + 2a + 1
2a
a a

=
b
b
b
1
1
1
a
a
a

a < b f (x) 0 x R.

P=

a+b+c

ba

a+b+

a+b+c
.
ba
13

b
= x > 1 :
a
2
2
x
x
+1
+1
2
2
=

x
x1
2 1

(y + 1)2
2(y + 1) 3

3
w=y+1,w>
2

w=3y+1=3y =2 x =4
1
y= ,y>
2
2 2 (y + 1)
=
2y 1

4a

c 16a2
c = 4a.
=
a
4a2

b
=4b=
a

3 ( ) :

b > a > 0 4ac b2 . :

w2
2w 3

(b c)2 0 b2 2bc + c2 0 b2 2bc c2

3
: (w 3)2 0, w >
w2 6w 9 =
2
3
w2
3
3(2w 3), w >
3, w >
2
(2w 3)
2

4ac 2bc c2 4a 2b c

2a + c 2(b a)

2a + c

2
ba
a+b+c
2a + c
+13
3
ba
ba

P3

b c = 0 b2 = 4ac b = c = 4a.

14

11 ( ) K AB. = 150 . C , BAC


AC , C ,
CD = AC .
D AB .

12 ( )
xCy = 600 . B Cy BA C x.
ABD C, D AB.
1) P CD AE
ABD, AC = PE .
2) P M AB C x S Cy T ,
T M = MP = PS .
http://www.mathematica.gr/forum/viewtopic.php?f=20&t=30565

( ) DD AB (D AB).
o

[
ACB
=
900

AC=CD

ABD

( BC ),
BD = AB = 2R
[

oo

15
BD =
BD =
[ BAC=
D[
2BAC
= D[

300

DD B ooo

o D

DD =

BD BD=AB=2R

DD = R
http://www.mathematica.gr/forum/viewtopic.php?f=20&t=30530

D AB .

b 60o
( ) 1) ACB
: AC =

15

BC
2

(1).

AE, CB
E BD P CD BC
: EP =
(2). (1) (2)
2
AC = EP.

T M = MP = PS .

2) DM AC
P CD, P MS ,
: MP = PS (3).
BMT AMP
AM = MB
, T M = MP (4).
(3) (4)

16

:
2 sin7 (2t) + 2 sin6 (2t) 36 sin4 (2t) + 96 sin3 (2t) 64 = 0

13 ( )
: 2(sin t + cos t) = tan3 t + cot3 t.

P(t) = 2t7 + 2t6 36t4 + 96t3 64, t [1, 1]

P(1) = 0

http://www.mathematica.gr/forum/viewtopic.php?f=21&t=28422

1 ( )
t .

sin t + cos t [ 2,

P(t) = (t 1)(2t6 + 4t5 + 4t4 32t3 32t2 + 64), t [1, 1]

2t6 + 4t5 + 4t4 32t3 32t2 + 64 =

2]

2t4 (t2 + 2t + 2) + 32(1 t3 ) + 32(1 t2 ) > 0 t [1, 1]

2(sin t + cos t) [2, 2]


3

, sin(2t) = 1 2t = 2k +

t = k + , k Z
4

2 tan t + cot t 2.

,
2, , ,
2, .

14 ( )
P(x), Q(x)

Q(x2 ) = (x + 1)4 x(P(x))2 .

tan3 t + cot3 t = 2 = tan t = 1 = t = k + , k Z,


4

http://www.mathematica.gr/forum/viewtopic.php?f=21&t=28418
3

tan t + cot t = 2 = tan t = 1 =

( ) deg Q = n
deg P = m.

t = k 4 , k Z,

2 , ,
k .
2 ( BAGGP93)

m 2
2n = 2m + 1, .

m = 0 m = 1
m = 0 P(x) = c, x R
:

t , k + , t , k , k Z
2

Q(x2 ) = x4 + 4 x3 + 6 x2 + (4 c2 )x + 1,


x.

tan3 t + cot3 t = (tant + cott)((tant + cott)2 3) = (tant +


cott)3 3(tant + cott)

m = 1 P(x) = ax + b, x R
:

tant + cott =

sint cost sin2 t + cos2 t


2
+
=
=
cost sint
sintcost
sin(2t)

Q(x2 ) = x4 + (4 a2 )x3 + (6 2ab)x2 + (4 b2 )x + 1


x
a2 = b2 = 4 4 (a, b).

tan t + cot t =
sin3 (2t) sin(2t)

96
36
64

+
2 sint + 2cos2 t + 2 sin(2t) =
6
4
sin (2t) sin (2t) sin2 (2t)

3

2 sin (2t) + 2 sin (2t) = 64 96 sin (2t) + 36 sin (2t)

P(x) = 2 x + 2 P(x) = 2 x 2

Q(x2 ) = x4 2 x2 + 1 Q(x) = x2 2 x + 1.

P(x) = 2 x 2 P(x) = 2 x + 2

Q(x2 ) = x4 + 14 x2 + 1 Q(x) = x2 + 14 x + 1.

17

. . Ceva 2 ( )

15 ( )
ABC (C) .
BC T AB AC S .
T Z, S E (C).
BE, CZ
A .

F, (O)
ABC,
AD A AZBFCE,
AF, BE, CZ, ,
.


(AZ)(BF)(CE) = (ZB)(FC)(EA) , (1)

(1) BF = FC
(AZ)(CE) = (ZB)(EA) , (2)

AZT, ZBT

http://www.mathematica.gr/forum/viewtopic.php?f=22&t=31113

1 ( )

AT
(AZ)2 AT
AZ ZT
=
=
=
=
BZ BT
ZT
(BZ)2 BT

[
[
[
sinCBE
sinCES
sinCES
CS
=
=
=

[
[
[
ES
sinABE
sinACE
sinECS

, AES , ECS

[
[ ZT
[
sinZBA
sinZBT
sinACZ
=
=
=
.
[
[
[
sinBCZ
sinT
ZB sinT
ZB BT

AS
EA ES
AS
(EA)2
=
=
=
=
2
CE CS
ES
CS
(CE)

, (3)

ES
S C S A BCkT S ES
SC

=
=

2
2
TB TA
ZT
ZT
T B2
CS ZT

= 1.
ES BT

(3), (4)

, (4)

AS
AT
=
, BC k T S , BT
CS

sin 2

sin A2

EA
AZ
(AZ)2 (EA)2
=
= (2)
=
=
2
2
ZB CE
(BZ)
(CE)

[
[ sinACZ
sinCBE

=1
[ sinBCZ
[
sinABE

, (2), (1)
.

18

http://www.mathematica.gr/forum/viewtopic.php?f=22&t=30051

1 ( ) P
(O), (K) Q AB
(K).
P Q
o.AS B, P, Q, S .
C S .

[ = 90o OQPC S[
OQ = QPC
.
p

S T = S Q S P = S O S C = 2R.
2 ( )
.
3 ( ) T Z 2 = T B T A,

S E2 = S C S A

S = S B2
(O) AB ,
AB (O) .

TZ S E TZ S E
=
,
=
,
TA S A TB S B

T Z ZB
=
,
T A ZA
ZB EC
S E EC
=

=
.
S A AE
ZA AC

(K)
,

S T 2 = = S B2 S T = R

2.

ZB BL
=
,
AC LC
ZB AC BL
BC CL
=

=
,
ZA LA
ZA BC AL
:

EC AB C M
LB MA CD
=

=1
EA BC AM
LA MC DB
(L AB ZC, M AC BE) .

.
3 ( .) H, P, S
.

16 ( KARKAR) (K) (O, R)


.
S T .

, :

S HP

= .
OS .. KPH

= 90
HPB


OPS

OS P = (1)

KHP

19

KH=KP=

(2)

90
OPS O=

o (1), (2) OHS :

T S 2 = 2 R2 T S =

OS =R

OH(= R) = OS S S
S (K, )

T S 2 = S P S H (3)
SH
PH

OS H KPH
= R S H = R PH (4)
(3), (4) T S 2 = S P HP

(5)

OPS LPH

= LPH
= 90 LHP
= OS P = )
(S OP
OS
HP

SP
LH

S P HP = 2 R (6)

(5), (6) T S 2 = 2 R

20

2 R o..

:
2

BC + 2CA BC = 0,

17 ( )
AB =
2, .


BC (BC + CA + CA) = 0

BC (BA + CA) = 0

http://www.mathematica.gr/forum/viewtopic.php?f=23&t=25883

2BC MA = 0, M BC.

1 ( ) :

, BC
, . h

4 ( ) cos C > 0 C 0, 2

a
2

BC = 2CD, AD A.

2 + a2 2
.
2a

.
5 ( ) = 2

, :

2 + a2 2
a
=

2
2a

~ = 2|A|
~
| B|

~ B
~
A
~ 2 = 2A
~ B
~
B
~ B|
~
|A||

~ AB)
~ (AB
~ ... |AB|
~ .
~ 2 = 2A
~ A)
~ = |A|
(A

a2 = 2 + a2 2

18 ( ) A(a, b) a, b , 0 a , b
K(k, 0) (0, )
~ 2 , O .
~ = OA
~ A
AK
1) K
(), OA.
2) ()
(1 ) : x y = 2
P, (OP) = 2,
A () : y = x .

2 = 2 = ,
.
2 ( ) a = b cos C + c cos B
( ), :

b cos C = c cos B

b cos B
=
c cos C
http://www.mathematica.gr/forum/viewtopic.php?f=23&t=22924

b < c, c < b
.
b = c.
3 ( )

( ) AK = (k , b),

A = (, b) OA = (, b)
2
AK A = OA k + 2 b + b2 = 2 + b2
k = b(1).
!
k
1) K : M , .

a2 = 2ab cos C,

2 2

2

BC = 2CA CB

x =

21

y =

x =

k
2

(1)

x =

b
2

x = by x + by = 0.

( + b) x = 2b x =

2b

+b

(2) y =

!
2
P
,

+b +b

(OP) = 2

2
.
+b

2b

() : x+by = 0
b

= . OA OA = .
b

OA = 1 OA.

42 + 4b2

( 2) P (
x + by = 0
x + by = 0
.

x y = 2 (2) b bx by = 2b

= 2 22 + 4b + 2b2 = 42 + 4b2
( + b)2
2 + 2b + b2 = 22 + 2b2 ( b)2 = 0 = b.

A A (, ),
() : y = x.

22

:
n+1

19 ( )
( xi ): 0, 1, 2, ..., n ,n N
2k + 1
d =
,k N.
2
1. n

d=

1+ n+2 1
2

2k+1
2

n
2

n = 2k + 1.

3. k = 2, n = 2 2 + 1 = 5, 0, 1, 2, 3, 4, 5

2. n = 2k + 1

x =

3. k = 2 .

15
6

5
2

y= x

y = x + b (1 ) x = b(1)
yi :

http://www.mathematica.gr/forum/viewtopic.php?f=18&t=26302

sy =s x

1 ( )

sy = || sx || = 1 = 1 = 1

1. n + 1.

= 1 (1) b = 0
ab < 0

n , n + 1 ,
d ,
d N .
2k+1
2

4. yi :

4. yi = axi +b,ab < 0


x = y sy = s x a, b.

d =

0+1+2+3+4+5
6

= 1 (1) b = 2 x b = 5

< N , 2k + 1 .

= 1 b = 5.

n .
( ) 2. .
2. n + 1,
.

n+1


2
n+1
+ 1.
2

m
1)
1
m + 1, d = 2k+
= m+(m+
k=m
2
2

m m + 1 = k + 1
n + 1.
m + 1 = k + 1,
(n + 1) (k + 1) = n k.


1 = 0 = 1
= 1
n+1 =

n+1

1 n+1 +1 =
2

n+1
2

( )
23

2 xy


0, .
3 ( )

k + 1 = n k n = 2k + 1.
20 ( )
A
.
[P(A)]2 + [P(A )]2

, f (x) = 2(2 x 1) xo = 21

1
.
2

f (x) f ( 21 ) f (x) 21 , x [0, 1]


f (P(A)) = (P(A))2 + (P(A ))2 21 .

4 ( ) :

1 ( ) [P(A)]2 + [P(A )]2 =

P2 (A) + (1 P(A))2 = 2P2 (A) 2P(A) + 1 =


2P (A)

2
2

2

1
2

2(x2 + y2 ) (x + y)2 ...


5 ( ) P(A) = x , P(A ) = y
0 x, y 1

x+y =1

1
2

P (A) =

1
.
2


1
1
1
1
x = a y = + a, a
2
2
2
2

2 ( ) x = P(A), y = P(A ),
[P(A)]2 + [P(A )]2

(x + y)2 2 xy

1
2

x+y=1

1
2

1 2 xy

f (x) = x2 + (1 x)2 , x [0, 1]

http://www.mathematica.gr/forum/viewtopic.php?f=18&t=26233


1 2
2

1
1 y=1x
1
xy
x x2
2
4
4

:
1
2

1
2

1
2

x2 + y2 = ( a)2 + ( + a)2 =

24

1
1
+ 2a2 .
2
2

21 ( ) z w

|(1 + i)z 2 4i| =

18

w = 2z 11 + 5i.

:
1. z
2. |z|

2.

3. w

|z|
OK

4. |z w|

(OK) =

32 + 12 =

10

|z|min = (OA) = (OK) 1 =

http://www.mathematica.gr/forum/viewtopic.php?f=51&t=8746

( )

|z|max = (OB) = (OK) + 1 =

10 3.

10 + 3.

1. z = x + yi x, y R.

|(1 + i)z 2 4i| =


|1 + i||z

2|z

|z

2+4i
|
1+i

2(1+2i)(1i)
(1+i)(1i) |

2(1i+2i+2)
|
2

18 |(1 + i)(z

2+4i
)|
1+i

3. w = x + yi x, y R.

18

w = 2z 11 + 5i 2z 6 2i = w + 5 7i
2(z 3 i) = w + 5 7i

18

=3 2

|2(z 3 i)| = |w + 5 7i|

= 3 |z (3 + i)| = 3

|w + 5 7i| = 2 |z 3 i| |w (5 + 7i)| = 6.


z K(3, 1)
1 = 3.

w
(5, 7) 2 = 6.

25

4.
(K) =

p
(5 3)2 + (7 1)2 = 10.
p

|z w|min =

(K M) =

() = (K) 1 2 = 10 3 6 = 1

|z w|min = (EZ) = (K) + 1 + 2 = 10 + 3 + 6 = 19

(11 3)2 + (5 1)2 = 10

|z w|min = (MN) = (MK) 1 = 10 3 = 7

z w
,

|z w|max = (M) = (MK) + 1 = 10 + 3 = 13

|zw|
,
z w

i
h
1, 31 .

) z C
22 (

z2 + z + 1 = |z + 1| , : Re (z)

http://www.mathematica.gr/forum/viewtopic.php?f=51&t=28297

|z w|min = () = (K)1 2 = 10 3 6 = 1



(S T OPJOHN ) z2 + z + 1 = |z + 1|

|z w|min = (EZ) = (K) + 1 + 2 = 10 + 3 + 6 = 19


2
z2 + z + 1 = |z + 1|2

(zz)2 + zz2 + z2 z + z2 + z2 = 0

|z w| = |w z| = |2z 11 + 5i z| = |z 11 + 5i| =
|z (11 5i)|, (1)

|z|4 + 2|z|2 (Re(z) 1) + (Re(z) 1)2


(Re(z) 1)2 + 4(Re(z))2 = 0

z K(3, 1) 1 = 3
(1) M(11, 5) K(3, 1)
1 = 3.

3(Re(z) + 1)(Re(z) 31 ) = (|z|2 + Re(z) 1)


3(Re(z) + 1)(Re(z) 31 ) 0 1 Re(z)

26

1
3

23 ( Mulder)
f : R R ,
c R f (x) = c
.

24 ( )
1.

f (0, +)
f (x) > 0 x > 0,

http://www.mathematica.gr/forum/viewtopic.php?f=52&t=15491

g g (x) =
( )

f (x)

+f

 
1

(0, +).

2.

< f () = f ()

h (x) =

x (, ) f (x) < f ()

+ e x ln x
1

3. e2x + e 2x

f (x) > f ().

ex

> ln 2 x + e

http://mathematica.gr/forum/viewtopic.php?f=52&t=17360

m f , ,

( )
1. 0 < x1 < x2 f (x1 ) < f (x2 )

) m
c1 (, )
(, ), c2 (, ).

f (x1 )

>

f (x2 )

(1)

( f . f (x) > 0)

) c1 , c2 (, ) c1 < c2

0 < x1 < x2

) y (m, f ())


x1 (c2 , ) , x2 (, c1 ) , x3 (c1 , )

 
1

x1

> f

 
1

x2

x1

>

x2

(2)

(1) (2)

f (x1 ) = f (x2 ) = f (x3 ) = y,

g (x1 ) > g (x2 ) g .


(0, +).

c1 , c2 (, ) c1 < c2 f (c1 ) = f (c2 ) = m

2. x > 0 0 < x1 < x2 ln x1 < ln x2

c (c1 , c2 ) y = f (c) ,

ln x1 > ln x2 (3)

x1 (, c1 ) , x2 (c1 , )

f (x1 ) = f (x2 ) = y

t (x) = ln x . (0, +).

( ) ,
.
27

3. e2x + e 2x

f (x) = e x . ,
1.
1
g (x) = f (x)
+f
(0, +).

 
1

e2x + e 2x ln 2 x >

e1x + e x .

h (2 x) > h (1)

h (x) = g (x) + t (x) .


h (x) =

ex +e

ln x . (0, +).

28

2x < 1 x <

1
2

0 < x <

1
2

> ln 2 x + e
1

x>0

+e

f R,
x > 0 f (x) > f (0) = 0

25 ( ) :e x + ex + 2 cos x 4, x R

x < 0 f (x) < f (0) = 0


http://www.mathematica.gr/forum/viewtopic.php?f=53&t=30908

(0, + ) f
( , 0) f .

1 ( ) 2

x = 0 f (0) = 0,

cosh(x) + cos(x) 2

f (x) f (0) = 0 x R .
4 ( ) :

x
: 2 + 12
+R
R ,
.
2 ( )
()

f (x) = e x ex 2 sin x, x R
.

g(x) = ex + ex + 2 cos x, x R.

A = [0, +).

g (x) = e x + ex 2 cos x 2 2 cos x 0,

f (x) 0, x A.

g R.
,


g(x) + g(x) 2g(0)

, g(x) = e x ex 2 x g (x) = e x + ex 2 0
x = 0,

Jensen

.
3 ( )

g(x) g(0) = 0, x 0.

f (x) = ex + ex + 2cosx 4, x R.

ex ex 2 x 2 sin x = f (x) 0 x 0.

f (x) = ex ex 2 sinx, x R

f (x) = ex + ex 2cosx, x R.

ex ex 2 sin x 0, x 0, , -

x R

ex

ex

ex ex 2 sin x 0, x 0.

2 2cosx 2

(I), (II)
[0, x], [x, 0], , .

e x + ex 2cosx 0, x R

f (x)

5 ( ) ,
:

0 x R.
29

x x,
x 0.

http://www.mathematica.gr/forum/viewtopic.php?f=53&t=31186

( ) f ,

f (x) 0, x R.

p + 1/p 2, p > 0

ey + ey 2 cos y 0.

f (x + 1) 1 x R,

0 t, t 0.

f (x) 1 x R.

et et 2 sin t 0.

a, f (a) > 1.
f (a) > f (a + 1) f (a).

0 x,
x 0.

,
b (a, a + 1), f (b) = f (a + 1) f (a),

26 ( )
f : R R

f (a) > f (b), , f . -

f (x) = f (x + 1) 1, x R .
f .

f (x) = f (x + 1) 0.

30

27 ( )
f : [0, 2] R
.

Z1

( ) :

f (x) ln(1 + a f (x) )dx.


1

x = t dx = dt
t1 = 1, t2 = 1.

(1)
(2)

x R.

f 2 (x)dx.

I=

|x| x,

Z1

http://www.mathematica.gr/forum/viewtopic.php?f=54&t=31050

http://www.mathematica.gr/forum/viewtopic.php?f=54&t=26274

x R

)dx = ln a

Z2 




3
2

dx > 8 f (2) .
(x)
(x)
+
2
x
+
2
f
x
+
x
f

x2 + 1 2|x|,

f (x) ln(1 + a

f (x)

I=

f (t) ln(1 + a f (t) )(dt)

2



(x + 1)x f (x) + 2 f (x) = (x2 + 1) x f (x) + 2 f (x)


(1)
2|x| x f (x) + 2 f (x)


= 2 x2 f (x) + 4 x f (x)


= 2 x2 f (x)

(2) 
2 x2 f (x) .

=
=

f (t) ln(1 + a f (t) )dt


!
1
f (t) ln 1 + f (t) dt
a
f (t) ln(1 + a

f (t)

)dt + ln a

f 2 (t)dt.

(3)

g(x) = f 2 (x).

g : (1, 1) R

[0, 2],:

g(x) = f 2 (x) = ( f (x))2 = f 2 (x) = g(x)

Z2
Z2 

(x3 + x) f (x) + (2 x2 + 2) f (x) dx
2 x2 f (x) dx
0

, g ,

h
i2
= 2 x2 f (x)

= 8 f (2).

f (t)dt = 2
1

f 2 (t)dt.

(4)

(3) (4)
28 ( )
0 < a , 1 f R, :

2I = 2 ln a

31

f 2 (t)dt I = ln a

f 2 (t)dt.

< 0 (*)

29 ( ghan) f : R R
2e x
x R : f (x) =
.
1 + f 2 (x)

f (x1 ) f (x2 ) < 0 f (x1 ) < f (x2 ) f


R.

) f (0)
) f

) ln f (x) > 0 ln f (x) > ln 1

) ln f (x) > 0.

ln f (x) > ln f (0) f (x) > f (0) x > 0

ln x

(*) x2 + x + 2 + 1



x R = 2 4 2 + 1 =

http://www.mathematica.gr/forum/viewtopic.php?f=55&t=22746

( ) ) x = 0

f (x) =
f ( 0) =

2e x
1 + f 2 (x)
2
1+ f 2 (0)

(1) :

x = f (x1 ) = f (x2 )
f (x1 ) + f (x1 ) f (x2 ) + f 2 (x2 ) + 1
2

f 3 (0) + f (0) 2 = 0


2 42 4 = 32 4 < 0

30 ( )

(x2 )x , x , 0,
f (x) =

1, x = 0

( f (0) 1) f (0) + f (0) + 2 = 0


f (0) = 1 f 2 (0) + f (0) + 2 = 0
= 7 < 0

,
.

f (0) = 1
http://www.mathematica.gr/forum/viewtopic.php?f=55&t=30568

) (1) :

( )

f 3 (x) + f (x) = 2ex (2)


x1 , x2 R
ex

lim x ln(x2 ) = lim

x0

(2)

x1 < x2 2ex1 < 2ex2

ln(x2 )

x0

lim (2 x) = 0

x0

f 3 (x1 ) + f (x1 ) < f 3 (x2 ) + f (x2 )



( f (x1 ) f (x2 )) f 2 (x1 ) + f (x1 ) f (x2 ) + f 2 (x2 ) +

lim ex ln(x ) = e0 = 1

x0

f (x1 ) f (x2 ) < 0

f 0



( f (x1 ) f (x2 )) f 2 (x1 ) + f (x1 ) f (x2 ) + f 2 (x2 ) + 1

32

= lim

x0

x2

2x

x12

f (x) f (0)
x
x0

lim

= lim

x0

lim

ex ln(x ) (ln(x2 )+x


1

x0

( x2 )x 1

1
x2

2 x)

ex ln(x ) 1
x
x0

= lim

= lim ex ln(x ) (ln(x2 ) + 2) =


x0

33

lim x0+

31 ( )

sin(1/t)

> e2t

t3 sin(1/t)

dt >

et

sin(1/t)

x t3 sin(1/t)
e
dt
2

ex /x

ex

g(x) dx =

1
.
2

lim x0+

=e

i
h
f (t) dt , 0, 21 ,
h

1
>0=F
2

1
.
2

10
Z

f (t) dt =

1
.
2

 [0 , 1 0 ] [0, 1] 0

0, 21

lim e

1
F (0 ) =
2

sin(1/x)

x2

x3 sin(1/x)x2


, 

0 0, 21

x+ e (2 x2 1)
x2

x+

f (x) dx =

F(0) = 1 >

(2 x2 1)
lim
= 2 ,
x+
x2
3

g(x) dx = 1 .

0, 21

e x /x + .

lim

x + .

x+

( )

(2t2 /) + 1dt +

http://www.mathematica.gr/forum/viewtopic.php?f=56&t=3264

> 2t2 / + 1 .

dt + ,

F() =

 Rx 3 1 2 
lim x et sin( t )x dt = lim

x+

f (x) dx =

, [a, b] [0, 1] :

0 < 1/t < /2 .

t3 sin(1/t) > 2t2 /


et

( ) :
2y
1) sin y >
, 0 < y < /2 ( Jordan)

e1/6

32 ( ) f, g : [0, 1] (0, )

http://www.mathematica.gr/forum/viewtopic.php?f=56&t=27029

2) 2/ < 1 t x

DLH 1/6 .

 Rx 3 1 2 
lim x et sin( t )x dt .

x+

sin ww
w3

sin ww
w3

f (t) dt =

1
, a = 0 , b = 1 0 .
2

g .

34

34 ( parmenides51)
f

33 ( )

f : [0, +) [0, +) x [0, +) f (x) = x

Z1 

x+

2
f (t) dt .

f (x)

f (x) = x

x+

R1

2
f (t) dt =

f (x)

R1 p
0 f (x)dx 2 x 0 f (x)dx.
Z 1p
f (x)dx 0.
,
0
Z 1p

f (x)dx > 0 f

f (x) =

a=

a=

2 ( )

a=

f (x)dx b =

Z1 p

f (x) =

f (x)dx

Z1

f (x)dx 2 x

a=

f (x)dx 0 b =

a=

Z1 p

f (x)dx = 2b x a

Z1 p

b=
f (x)dx 0

b=

f (y)dy + 1 x +

R1
0

y f (y)dy

R1

f (y)dy b =

R1

f (y)dy

R1
0

y f (y)dy

R1
0

f (x)dx a =

(a+1)x2
2

R1
0

"

R1
0

((a + 1) x + b)dx

1
+ bx a =

a+1

R1
0

+ b a = 2b + 1 (1)

y f (y)dy

x f (x)dx b =

(a + 1) x3
3

bx2
2

#1


R1
(a + 1) x2 + bx dx
0
b=

a+1
3

b
2

6b = 2a + 2 + 3b (2)

Z1 


f (x)dx a =
2b x a dx

(1) (2)

3 1

2
4b
2b
2 x
a 2a =
a = 2b
a= 0
3
3
3

b =

Z1

a=

Z1

a=

R

(x + y) f (y)dy = x

f (x) = (a + 1) x + b

f (x)dx = 0 f .

c = c = c = 0 = f 0.
Z1

x.
.

R1

(x + y) f (y)dy = x x R.

(S tavroulitsa)

1 ( )

http://www.mathematica.gr/forum/viewtopic.php?f=69&t=30094

http://www.mathematica.gr/forum/viewtopic.php?f=69&t=27833

Z1 

3b 2a 2 = 0

a 2b 1 = 0

3b 2a 2 = 0

4b + 2a 2 = 0

b = 4 a = 7 f (x) = 6 x 4

a = b = 0.

35

,
,

35 ( )
n

36 ( )

x2010 2006 = 4y2009 + 4y2008 + 2007y.

x4 2011 x2 + n = 0 4 .

http://www.mathematica.gr/forum/viewtopic.php?f=109&t=15584&start=960#p127711
http://www.mathematica.gr/forum/viewtopic.php?f=109&t=15584&start=760#p106547

1 ( )
:

( )
m

x2010 = 4y2009 + 4y2008 + 2007y + 2006

m ,

, x
:

m4 2011m2 + n = 0 n = m4 + 2011m2

x2010 0(mod9) , x2010 1(mod9)


, x = 3k, k Z

x4 2011 x2 m4 + 2011m2 = 0

(x2 m2 )(x2 + m2 ) 2011(x2 m2 ) = 0


(x2 m2 )(x2 + m2 2011) = 0

x2010 = 32010 k2010 = o9

x2010 0(mod9)

x2 + m2 2011 = 0 x2 = m2

x = m x2 + m2 = 2011

x = 3k + 1

x2010 = (1 + 3k)2010 = [(1 + 3k)3 ]670 =

, x2 + m2 = 2011

(1 + 9k + 27k2 + 27k3 )670 =


= (1 + 9n)670 = 1 + 9n1 .

x2010 1(mod9)

:
1 :

m = 2k, kZ .
x2 + 4k2 = 2011 x2 x
x = 2t + 1 t.

4t2 + 4t + 4k2 = 2010

x = 3k + 2

x2010 = (2 + 3k)2010 = [(2 + 3k)3 ]670 =


(8 + 36k + 54k2 + 27k3 )670 =
= (1 + 9 + 9n)670 = (1 + 9n1 )670 =
(1)670 + 9n2 = 1 + 9n2 .

4 2010
2 :

x2010 1(mod9)

m = 2k + 1.
x2 + (2k + 1)2 = 2011 x2 = 2011 (2k + 1)2

x2 x .

x = 2t. :
2

A = 4y2009 + 4y2008 + 2007y + 2006,


(2t) = 2011 (2k + 1) 4t + 4k + 4k + 1 = 2011


4(t2 + k2 + k) = 2010

A 0(mod9), A 1(mod9)

, 2010 4.

A = 4y2 y2007 + 4yy2007 + 2007y + 2006 =


= 4yy2007 (y + 1) + 2007y + 2007 1 =
4yy2007 (y + 1) + 2007(y + 1) 1 = 4yy2007 (y + 1) + 9n 1

y = 3k + m, m 0, 1, 2. :
36

A = 4(3k + m)(3k + m)2007 (3k + m + 1)+ 9n 1. :

A 7(mod9)

m = 0 , :

m = 2 ,

2007

A = 4(3k + 2)[(3k + 2)3 ]669 (3k + 3) + 9n 1 =

A = 4(3k)(3k)
(3k + 1) + 9n 1 = 9n1 1 =
9n1 9 + 8 = 9n2 + 8

4(3k + 2)(8 + 9k1 )669 (3k + 3) + 9n 1 =

A 8(mod9)

= 4(3k + 2)(1 + 9k2 )669 (3k + 3) + 9n 1 =

m = 1, :

4(3k + 2)(1 + 9k3 )(3k + 3) + 9n 1 = ... = 9a 6k + 2

2007

A = 4(3k + 1)(3k + 1) (3k + 2) + 9n 1 =


4(3k + 1)[(1 + 3k)3 ]669 (3k + 2) + 9n 1 =

, 6k + 2 / 9
9 +1, () ,

= 4(3k + 1)(1 + 9k1 )669 (3k + 2) + 9n 1 =


4(3k + 1)(1 + 9k2 )(3k + 2) + 9n 1 =

A
0(mod9) 1(mod9).

= 4(3k+ 9k3 + 1)(3k+ 2)+ 9n 1 = 4(9k4 + 2)+ 9n 1 =


9k5 + 7

37

37 ( KARKAR)
ABCD (O) S AB CD T AC BD.
A, T, D B, T, C,
P.
A, O, P, B
O, P, S .

T AD + AT D = 180o BDA

BPA = BCA + BDA = 2BCA.

(2)

(2) BOA = 2BCA

http://www.mathematica.gr/forum/viewtopic.php?f=110&t=26246

BPA = BOA.

(3)

(3)
BPOA
.

CPD = CPT + T PD
[]

= CBD + CAD
= COD
CPOD (L).

, OP, (K), (L), S ,


(O), (K), (L)
.

( )
38 ( erxmer) ABC , BAC = 30o
ABC = 70o . M MAB = MCA = 20o .
MBA.

APT D, BPTC

BPA =
o

360 BPC CPT T PD APD =

360o BTC CBT T AD AT D

(1)
www.mathematica.gr/forum/viewtopic.php?f=110&t=27314

(1)

BTC + CBT = 180o BCA

1 ( )
38

QBM = QOM
= OAC + OCA
= 40o .

MBA = ABC QBM = 70o 40o = 30o


.
2 ( )

MAB = MAC = 20o ACB = 80o

MCB = 60o

(1)

BCO ABC, (1)


O, M, C OB = OC,
O BC.
,

BOC = 60o = 2BAC,


O ABC

OAC = OCA = 20o

ABC = 70o BAC = 30o = ACB = 80o


AMD,
AB C .
C MA, C MD
C M MA = MD
C MA = C MD = 150o CA = CD
MDC = MAC = 10o MCD = MCA = 20o .
E AB CD AED =
180o ADE EAD = 70o EBC = 70o = EDA,
ADE, CBE AD = AE CE = CB.
ACE, DCB
CA = CD CE = CB ACE = 40o = DCB
DB = AE = AD = DM.
DBM DBM = DMB = 70o .

(2)

MAC + ACB = BAC BAM + ACB


= 30o 20o + 80o

= 90o =

ANB = 90o N BC AM.

Q BC AO BAM = 20o
= QAN = NAC = 10o , (2) BAC = 30o .
, AQC AN
.
MQC , M AN MCQ =
o
60 , QMC = 60o = BOC

OB k MQ.

(3)

MBA = DBM DBA = 70o 40o = 30o ,

60o

(3) BOC =
= OBC,
BQMO
(K).

DBA = DAB = 40o .


39

, ,

() :
: P(a, b, c)
3.

39 ( ) a, b, c ,
a + b + c = 3. :
ab2 + bc2 + ca2 + abc 4.
;

P(1, 1, 1) 0 P(a, b, 0) 0
a, b 0, P(a, b, c) 0 a, b, c 0.

http://www.mathematica.gr/forum/viewtopic.php?f=111&t=30086.
, Secrets in Inequalities - Vol. 2
. , 3
. 310-311 1.10.2 . 312.

http://www.mathematica.gr/forum/viewtopic.php?f=111&t=30064

( )

ab2 + bc2 + ca2 + abc

4(a + b + c)3
27

: a b, c.
bc2 + ca2 abc + c2 a
c(a b)(a c) 0
.

40 ( ) f : R N

ab2 + bc2 + ca2 + abc ab2 + 2abc + c2 a = a(b + c)2 .

f (x +

f (y)

) = f (y +

f (x)

x, y R. n N
f .

4(a + b + c)3
,
a(b + c)2
27

http://www.mathematica.gr/forum/viewtopic.php?f=111&t=27499
3

(2a)(b + c)(b + c)

8(a + b + c)
,
27

( ) f .
n N An = {x R : f (x) = n}.
A R r R A + r
A + r = {a + r : a A}.
1: m, n N
Am + n1 Ak k N .
x, y Am z f (z) = n.
f (x + 1/n) = f (x + 1/ f (z)) = f (z + 1/ f (x)) = f (z + 1/m)
f (y + 1/n) = f (z + 1/m).
2: m N n N
x Am
Am + n1 Am .
f (x + 1/ f (x 1/m)) = f (x 1/m + 1/ f (x)) = f (x) Am .
f (x 1/m) = n, f (x + 1/n) Am
1 Am + n1 Am .
3: m N Am + 1
Am . 2 n N

- :

p
2a + (b + c) + (b + c)
3
2a(b + c)2 .
3

(a, b, c) (1, 1, 1), (a, b, c)

(2, 1, 0).

( ):

Algebraic Inequalities Vasile Cirtoaje, GIL, . 25, Inequalities with beautiful solutions, . 87, . GIL,
V.Cirtoaje, et. al. Secrets in Inequalities Vol. 2, Pham Kim Hung, . GIL, . 102-103,
( ) ()
40

Am + n1 Am . Am + n2 Am + n1 Am
Am + 1 Am .

6: m N A1 +1/m =
Am . x A1 y Am f (x + 1/m) =
f (x + 1/ f (y)) = f (y + 1/ f (x)) = f (y + 1) Am .
5 A1 + 1/m = Am .
5 n N
A1 + 2/3 = An . q = 2/3 + (n 1)/n. q q = s/t s, t t > 1.
r rq = k + 1/t k.
n1
1 n1
= An +
= A1 + q A1 = A1 + 1 = A1 + +

4: m N Am + 1 =
Am . x Am k x 1 Ak .
3 x Ak
k = m. Am + 1 Am
3 .

5: m, n N
Am + n1 = Ak k N . k (
1) Am + n1 Ak .
4 Am = Am + 1 Ak + nn 1 .
1 N
Ak + nn 1 A . = m.
4 Ak = Ak + 1 Am + n1
.

A1 = A1 +rq = A1 +k+ 1/t = At ,


.
: shortlist 6
2008.

41

(3), , R, S , T
.

41 ( ) ABCD P,

AD
(O), P , A , D. PR k AD R AB
PT k AC T CD. S PB AD,
R, S , T .

42 ( .)

ABCD AB = a, BC = b

a = b 2. CD , . M,
MA, MB, CD
K, L, . AD
D DZ = KC BC
C CE = DL. KLEZ
a.

http://www.mathematica.gr/forum/viewtopic.php?f=112&t=31032

1 ( ) R
CD RP T PD = DBS , -

, ( )
R PT, ABD,
PD, BS .

SD
DT
SD
DT
=
=
=
DR
SA
AR
SA

(1)

(1), , R, S , T
.

http://www.mathematica.gr/forum/viewtopic.php?f=112&t=31281

1 ( )

2 ( .)
Q AB DP L CD QS .
DA BQ BP DQ,
S DA BP
DBQ

QS QL BD = QL k PT

(2)

(2) PR k AD

LT
QR QP
=
=
RA PD T D

(3)

DK = x KL = y LC = z :
42

S AB MD T AB MC
P, AB
MT, D.
S A = x BT = y
AP = y.
DS P DA S P

(KLEZ) = (DCEZ) (DKZ) (CLE)


DZ + EC
=
(CD)
2

(KLEZ) =

x+y+y+z
2

(DZ)(DK)

(CE)(CL)
2

(x + y + z)

x(y + z)
2

z(x + y)
(DA)2 = (AS )(AP) =

(x + y + z)2 + y2 2 xz

b2 = xy =

(1)

2b2 = 2 xy =

, MC, MD KP k AD, LQ k
BC, :

a2 = 2 xy

DC k S T , , :

KP MK
ML QL KL
=
=
=
=
= PK = QL.
AD
MA
MB BC AB
, KLQP

DL KC DC
=
=
=l
SB
AT
ST

(KL)2 = 2(PK)2

(2)

y = 2 xz

KC = l(y + a)

(7)

(DL)2 + (KC)2 = l2 [(x + a)2 + (y + a)2 ] =


(4)

(DL)2 + (KC)2 = l2 [(x + y)2 2 xy + 2a2 + 2a(x + y)] =

(1),(4)

(KLEZ) =

DL = l(x + a)
DC = a = l(x + y + a)

(3)

(2)
(2),(3) (KL)2 = 2(DK)(CL) =
2

(6)

(6)

KDP, LQC (
KPD = LCQ,
) :

(KL)
DK PK
=
= (DK)(CL) = (PK)2 =
QL
CL
2

(5)

(DL)2 + (KC)2 = l2 (x + y + a)2 = a2 (8)

(x + y + z)2

(CD)2

a2

= ct

(7)
DLZ, CEK

2 ( )

KE = LZ KE LZ

(9)

ZDL = 90o =
KCE, .
, (8)
(10)

KE = LZ = a
(9),(10),

(KLEZ) =

(KE)(LZ)

43

a2
2

43 ( 1995)
A B 3 3,

http://www.mathematica.gr/forum/viewtopic.php?p=128218

( )
:

, .

.

,
.

.



1 an1 xn1 + + a2 x2 + a1 x

q


a2n1 + + a22 + a21 x2(n1) + + x4 + x2 (2) -

CauchySchwartz.
2 xk

kxn + n k
(3) n, k n

k n.
-:
k

kxn

http://www.mathematica.gr/forum/viewtopic.php?p=105278

( )
1 9
1 2 3
4 5 6
7 8 9

. 5. .
:

+nk
n

nk
z
}|
{ z }|
{
n
n
n
x + x + + x +1 + 1 + + 1
n
p
n

xkn = xk

3 xm

(xm + 1)2

(4)
4

-.
4 (xn + 1)2 (n 1) 4

n1
X

x2k (5) n .

k=1

(A) 1. 9. ,
.

2X
m1

k=1

(B) 2. 8.

4. 6
.

xk


2m 1 m
x + 1 2 (6)
4

2X
m1

xm + xm + 1

k=1

44 ( 3 1995) P(x) = an xn + an1 xn1 + + a1 x + a0 n


an , an1 , . . . , a1 , a0  R.
 a0 > 0, an > 0
2 2
4 min a0 , an
(1),
a21 + a22 + + a2n1
n1
P(x) 0 x R.

xk

k=1

(3)

xm + xm + 1

xm +

(4)

44

X
m1

xk

m1
 X kxm + m k

k=1

m1

xm + 1

2

2m 1 m
x +1 2
4

2

(6) x x2 2m n
(5).


P(x) = (an xn + a0 ) + an1 xn1 + + a2 x2 + a1 x


(an xn + a0 ) an1 xn1 + + a2 x2 + a1 x
q
(2)

min (a20 , a2n ) |xn + 1|


q


a2n1 + + a22 + a21 x2(n1) + + x4 + x2

q
(1)
(xn + 1) n 1 2 x2(n1) + + x2
min (a20 , a2n )

n1
(5)

45

:
(n!)2012 .
((n!)2012 , n! + 1) = 1 (n! + 1)|(2012n)!

45 ( IMC 2012) 1
a0 , a1 , . . . a0 = 1, a1 =
2

na2n
n 1.
1 + (n + 1)an

an+1 =

X
a

k+1

k=0

ak

(n!)2012 (n! + 1)|(2012n)!

n
Strirling :

http://www.mathematica.gr/forum/viewtopic.php?f=59&t=28869

n 2012n
22012n( 2012
(2012n)!
(2012n)!
e )

(n!)2012 (n! + 1)
(n!)2013
2n2013 ( ne )2013n

n 2012n
22012n( 2012
e )
=
2n2013 ( ne )2012n ( ne )n

22012n(2012)2012n
=

2n2013 ( ne )n

22012n
=
2013
2n
( 2012n2012 e )n

( )

nan (n + 1)an+1 =

an+1
an

(1)

n 1. n
an > 0. nan .
(1) = 0.
(1) n 1

Xa

n+1

n1

an

(2012n)!
1
(n!)2012 (n! + 1)

= a1 lim(nan ) = a1 .

0.


a1
a0 + a1 = 1.

2 ( ) (n! + 1)|(2012n!) n! + 1
(n + 1, 2012n).
p n! + 1 2011.
, n , 2013n/ log (2012n)
2012n

46 ( IMC 2012) n
n! + 1 (2012n)!,
http://www.mathematica.gr/forum/viewtopic.php?f=59&t=28870

2013n

(2012n)2011 log (2012n) = e20112013n > n! + 1 n 1 ( )


. :

Stirling.

(2n)!
(2012n)!

n!
(2011n)!
!
!
!
3n
2012n
2012 2n
= (n!)


n
n
n

(2012n)! = n!

46

47 ( )
G n a, b G \ {e}, m N
n
ab = bm a. b (m,n) = e.

1 : H, K
G, 2. , HK (2n + 1)2
,
G
|H K|
Lagrange 2(2n + 1) = |G|. ,
2n + 1

2, ,
|H K|
|H K| = 2n + 1 = |H| = |K|. , H = K .

http://www.mathematica.gr/forum/viewtopic.php?f=10&t=31024

( )
Lagrange xn = 1 x
G (a1 ba)m = a1 bm a = b.

n
.
(m, n)

48 ( ) G
2(2n + 1), ( n N), 2n + 1.

2 : HK G,

(2n + 1)2
2(2n + 1),
|H K|

http://www.mathematica.gr/forum/viewtopic.php?f=10&t=31056

1
, |H K| n + 1.
2
, Lagrange |H K|
|H| = 2n + 1. 2n + 1
n, 2n + 1,
|H K| = 2n + 1 = |H| = |K|. , H = K .

( )
H, K G |H| = |K| =
2n + 1.

|HK| =

|H K| n +

|H| |K| (2n + 1)2


=
.
|H K|
|H K|

47

:
+
X

49 ( )

tan1

k=1

= lim

n+

arctan

n=1

n2 + 1

(1)


.
8

http://www.mathematica.gr/forum/viewtopic.php?f=9&t=23581

( ) :
tan1 z =
tan1 x + tan1
lim

n+

i
2

ln

i+z 
iz

(1 )

x > 0,

(2 )

z C r N0 ,

(3 )

z C r N0 ,

(4)

z C r Z0

(5)

n!nz
= (z)
z(z + 1) (z + n)
(z + 1) = z(z)
(1 z)(z) =

z C r {i},

sin(z)

: z := x + iy x , k + 21 ,
y , 0.

kZ

50 ( )
f : R
(0, +), :

eix ey eix ey
eix ey eix ey

x Q, y R Q

(eix + eix )(ey ey ) + (eix eix )(ey + ey )


=
(eix + eix )(ey ey ) + (eix eix )(ey + ey )
1 + i tan(x) coth(y)
.
1 + i tan(x) coth(y)

sin(z)
sin(z)

z :=

k=1

k2 + 1

tan1

= tan1 (tan(x) coth(y))


8
!!
!!!

3
3

4
4
= tan1 tan 2 cos
coth 2 sin
.

sin(z) i tan(x) coth(y) 1


=
.
sin(z) i tan(x) coth(y) + 1

n
X

n
Y

i + (k2 + 1)1

= ln lim

n+
2
i (k2 + 1)1
k=1

n
Y
k2 (1 + i)
i
= ln lim

n+
2
k2 (1 i)
k=1

n
Y
k2 z2
i
= ln lim

2
2
n+
2
k=1 k z

n
Y
i
(k + 1)2 z2
= ln lim

2
2
n+
2
k=0 (k + 1) z

Qn
Qn
1
1
i

1z
k=0 (k + 1 z) n!n1+z
k=0 (k + 1 + z)
= ln lim n!n1 Qn

Qn
1
n+
2
(k
+
1

(k
+
1
+
z)
z)
k=0
k=0
n!n1z
n!n1+z
!
(1 z)(1 + z)
( 3) i
= ln
2
(1 z)(1 + z)
!
z sin(z)
(4),(5) i
ln
=
2
z sin(z)
!
!
3i sin(z)
i
= ln exp
2
4 sin(z)
!
i
sin(z)
3
+ ln
=
8
2
sin(z)
!
i
1 + i tan(x) coth(y)
3

+ ln
===
8
2
1 + i tan(x) coth(y)
!
i
i A1
A:=tan(x) coth(y),0 3
==============
+ ln
8
2
i + A1
!
1
(1) 3
+ tan1
=
8
A


A>0,(2) 3

tan1 A
=

q
q

2+1
 q  22+1

2
+e

21 e
arctan tan
q
q

2+1
2+1
e 2 e 2

k2 + 1

2 exp

3i
8

f (x) f (y) |x y| .

http://www.mathematica.gr/forum/viewtopic.php?f=9&t=5563

1 ( ) fi

= x + iy
48

f fr f
.
, , q Q
lim fi (x) = 0. , r R Q lim fr (x) = 0.
xq

xn ,
y, f (xn ) 0. yn , x
f (yn ) 0.
h : R R
h(x) = f (x), x R Q h(x) = 0, x Q.

Q, , G ,
Q G .


xr

n N.
R In f (In ) (0, 1/n). An f 1 (0, 1/n)
.
, Baire, A =
T

nN An = , .

2 ( ) x Q y R Q,

49

:
x=g(y)

51 ( )
f : R R .
h, g : R R

g( f (x)) = h( f (x)) = x,

(2) f (g(y)) > y


(1)

h ( f (g(y))) h(y)

(4)

g(y) h(y)

x R,

(3) (4) g(y) = h(y).

g = h.
2. f (x) < y

3. x R , h > 0

http://www.mathematica.gr/forum/viewtopic.php?f=64&t=15097

(hsiodos) f
.

g( f (x)) = h( f (x)) = x

xR

x R ()

f (x) < y < f (x + h)

(5)

( 2 )

(1)

g, h .
g ( h)
(1)

(5) g( f (x)) g(y) g( f (x + h))

x g(y) x + h

x1 , x2 R x1 < x2 =

lim x lim+ g(y) lim+ (x + h)

h0+

f (x1 ) < f (x2 ) =

h0

h0

x g(y) x

(1)

g(y) = x

g ( f (x1 )) g ( f (x2 )) = x1 x2 ,
.

h(y) = x g(y) = h(y).

f (R) = R (
)

g(x) = h(x) x R.

f (R) , R x f (R)
()

52 ( ) , .


g(y) = h(y) y R f (R).
y R f (R).

:
http://www.mathematica.gr/forum/posting.php?mode=edit&f=64&p=50975

1.

f (x) > y x R.

(2)
( )
x > y

 ({x} > {y}) ({x} = {y}) (x > y)
x x {x} = x x.
,
x x + 1 x
x 1.

x=h(y)

(2) f (h(y)) > y


(1)

g ( f (h(y))) g(y)
h(y) g(y)

(3)

50

53 ( )
n ,


 22n 22n+1 1 .

54 ( )
12 22 + 32 42 + 52 62 +

+ . . . + 20012 20022 + 20032 ?

http://www.mathematica.gr/forum/viewtopic.php?f=63&t=30730

(mod 2005).

A = 22n 22n+1 1 , n = 1, 3, 5, ..



A = 22(2k+1) 22(2k+1)+1 1 ,

http://www.mathematica.gr/forum/viewtopic.php?f=63&t=30853

k = 0, 1, 2, 3, ...


2(2k+1)

2( 2k+1)+1

1 ( )

1 = 100 m + 28, m N.

k = 0 : A = 22 22+1 1 = 4 7 = 28.

A (k) = 2

2003
X

k=1



A (k) = 22(2k+1) 22(2k+1)+1 1 = 100 m + 28,

1+

1002
X

k=2

m N,

(1)k+1 k2

[(1)k+1 k2 + (1)2005k+1 (2005 k)2 ]


1 mod 2005



A (k + 1) = 16 22(2k+1) 16 22(2k+1)+1 1


= 16 22(2k+1) 22(2k+1)+1 1

(1)2005k+1 (2005 k)2 = (1)k+1 (2005 k)2

+ 16 15 22(2k+1) 22(2k+1)+1

(1)k+1 k2 mod 2005.

= 16 (100 m + 28)

+ 16 15 22(2k+1) 22(2k+1)+1


= 100 (16m) + 28 + 60 7 + 4 32 28k


2 ( )

2
2
2
2
2
2
2
12
|2 + 3 4 + . . . 2000
{z + 2001 2002 + 2003} =

60 7 + 4 32 28k = 100 x

42 + 24 32 28k = 10 x
8k

21 + 12 32 2

8k

21 + 384 2

2002

1 + 20032 20022 + 20012 . . . 42 + 32 22 =

=5x

1 + 20032 22 20022 32 . . . + 10032 10022 =

=5x

1 + 2005(2003 2) 2005(2002 3)+

1 + 28k+2 = 5
.




A = 22n 22n+1 1 , n = 1, 3, 5, .. 28.

51

2005(2001 4) . . . + 2005(1003 1002)

(mod 2005).

S = {a1 , a2 , a3 , ..., a} (an )

55 ( )
R R.

1 1 lim an = a.
n+

S 2
(
).

http://www.mathematica.gr/forum/viewtopic.php?f=13&t=15569

( ) M R R

http://www.mathematica.gr/forum/viewtopic.php?f=13&t=14065

( )
S 1 , S 2 .

f M S f , card(S f ) 0

f : S 1 S 2
1 1, ,

X R card(X) 0 MX

f 1 1

S f X , card(S f ) 0 .

card(S 1 ) = card( f (S 1 )) = card(S 2 )


X Q

card(S 1 S 2 ) = cardS = 0


c.

card(S 1 ) = card(S 2 ) = 0
a S 1

card(MX ) c

S 1 {a}

bn

MX c.

S 2 { f (a)} f (bn )

M =

MX card(M) c

bn a f (bn ) f (a)
a , f (a) S 1 S 2 = ,

f (x) = x + a, a R
, card(M) c

an
,

card(M) = c

56 ( )

52

j {1, 2, . . . , n}

57 ( )
a, x1 , x2 , ..., xn R a 2.

z j := cos x j + i sin x j ,

z j = 1

zn
z2

2
2

A + B = |A + iB| = z1 + + + n1
a
a

1 n1
n
2
= n1 a z1 + a z2 + + azn1 + zn
a

cosx2 cosx3
cosxn
+
A = cosx1 +
+ ... + n1
2
a
a
a

B = sinx1 +

sinxn
sinx2 sinx3
+ 2 + ... + n1 ,
a
a
a

A2 + B2 > 0.

an1

> 0.

2 ( )
http://www.mathematica.gr/forum/viewtopic.php?f=27&t=31000

A = B = 0.

1 ( )

:
n N n 2,
a R a 2 z1 , z2 , . . . , zn C
|z1 | = |z2 | = = |zn | = 1. ,

A + iB = z1 + bz2 + + bn1 zn = 0,
1  1i
z j = cos x j + i sin x j ,
b = 0,
a
2


n1
a z1 + an2 z2 + + azn1 + zn 1.

bz2 + + bn1 zn = z1 =

: n.

|bz2 + + bn1 zn | = 1 =

1 b + b2 + bn1 =

n = 2,
1

|az1 + z2 | ||az1 | |z2 || = a 1 1

1
1
1
1
+ 2 + + n1 = 1 1 n1 ,
2 2
2
2

58 ( )
n,


n 2.

sin(na) cos(na)

= n 1,
sin a
cos a
k
a ,
, k Z.

, :

n

a z1 + an1 z2 + + a2 zn1 + azn + zn+1
n


a z1 + an1 z2 + + a2 zn1 + azn |zn+1 | =




a an1 z + an2 z + + az + z 1
1
2
n1
n
|
{z
}

1

http://www.mathematica.gr/forum/viewtopic.php?f=27&t=30159

1 ( )

a,

a 1 1,

k
2

, k Z
sin (na) cos (na)

=n1
sin a
cos a


n + 1 .

(1) a ,
53

(1)

k
2

, k Z

sin ((n 1) a) =

n1
2

sin (2a).

(2)

(1) a ,
.

(2)
a a ,

k
2

, k Z

, k Z ,

2 ( ) a = /4

sin(n/4) cos(n/4) = (n 1) 2/2

(n 1) cos ((n 1) a) = 4 (n 1) cos (2a),


sin((n 1)/4) = (n 1)/2.

h
i
lim (n 1)3 cos ((n 1) a) = lim [4 (n 1) cos (2a)]

a0

1 n {1, 0, 1, 2, 3}.
0, 2 1, 1, 3
.

a0

(n 1)3 = 4 (n 1)

n = 1 n = 1 n = 3

54

I ABC
( DB = DI =
DC , ),
BI

,

(
D).
ABC
I , r1 Euler :
OI 2 = R2 2Rr1 R2 2Rr = R2 2Rr1 r = r1 .

59 ( erxmer) ABC (C1 , R)


(C2 , r). d ,
(Euler) R2 2Rr = d2 .
;

http://www.mathematica.gr/forum/viewtopic.php?f=62&t=27484

OI 2 = R2 2Rr, :
1) R > OI,

2) R > 2r.

60 ( ) a,

.

C2 C1 .
B C1
BI C1
D.
(D, DB = DI) C1
C.

http://www.mathematica.gr/forum/viewtopic.php?f=62&t=30805

1 ( ) EB, AK .
EB

AKC ( 2)

AKC .


2
3

1
1
6
GH = CH =
=
.
3
3
2
6

A C1 DI ,
ABC .

55

.
2 ( ) O
AH BD
AH AC BD.

OM BD, AH .
CN AH
ACH

HN =

AH

: AM = MN = NH
OM
CN .

56

:
61 ( ) z
,

62 ( )
:

a1 ,
a2 , .....,
an

A = |z2 + z + 1| + |z2 z + 1|

c =

a1
a2 .....
an

:


c 2

http://www.mathematica.gr/forum/posting.php?mode=edit&f=60&p=101658

( ) z = x + yi,
x, y R,

http://www.mathematica.gr/forum/posting.php?mode=edit&f=60&p=135540

A = (x2 y2 + x + 1) + (2 xy + y)2
q
+ (x2 y2 x + 1)2 + (2 xy y)2 .

( ) n = 1. n = 2
:

Minkowski

4(x2

y2

1)2

4y2

=2

(x2

y2

1)2


2
2
2
2
~a1 + ~a2 + ~a1 ~a2 = 2 ~a1 + 2 ~a2 4,

y2 .

3
(x2 y2 + 1)2 + y2 ,
4

Amin =



~a1 ~a2 2.

n 3.
n.
n 1. ~a1 , ~a2 , . . . , ~an ,
1.

3.

, y2 > 1,

(x2 y2 + 1)2 + y2 > 1 >



~a1 + ~a2 2

3
.
4

~a1 , ~a2 , ~a3 , ~a1 , ~a2 , ~a3

y 1 1 y 0.

, ~a1 , ~a2 ,


60 . , ~a1 ~a2 1,

~a1 ~a2 , ~a3 , . . . , ~an i {1, 1},
i = 1, 2, . . . , n 1 ,

(x2 y2 + 1)2 + y2 (1 y2 )2 + y2 =
y4 y2 + 1 = (y2 21 )2 + 43 .




1 ~a1 ~a2 + 2~a3 + + n1~an =
1~a1 1~a2 + 2~a3 + + n1~an 2


3
min((x2 y2 + 1)2 + y2 ) =
4
x = 0 y =

2
.
2

57

http://www.mathematica.gr.
http://www.mathematica.gr.
LaTEX--: , : . LTEX.
.

Leonardo da Vinci

(32-) . 30
. -
- - quasiregular ,
( - ).

1+

(0, 0, ), 21 , 2 , 2

, 1+2 5 .

:http://en.wikipedia.org/wiki/Icosidodecahedron
:

mathematica.gr (http://www.mathematica.gr) .

mathematica.gr

9. (emouroukos)
10. ( )

1. ()
11. (m.papagrigorakis)
2. (Mihalis_Lambrou)
12. ( )
3. (nsmavrogiannis)
13. (cretanman)
4. ( )

14. (achilleas)

5. (chris_gatos)

15. (xr.tsif)

6. (m.papagrigorakis)

16. (polysot)

7. ( )

17. (Demetres)

1. (stranton)

1. (grigkost)

2. ( )

2. (cretanman)

3. (spyros)

4. (vittasko)

1. (stranton)

5. ()

2. (vittasko)

6. (KAKABASBASILEIOS)

3. ()

7. (exdx)

4. (s.kap)

8. (s.kap)

5. ( )

9. (nkatsipis)

6. ( )

10. ( )

7. (chris_gatos)

11. ( )

8. (grigkost)

12. (matha)

13. ( )

24. ()

14. (emouroukos)

25. (xr.tsif)

15. (gbaloglou)

26. (polysot)

16. (R BORIS)

27. (Demetres)

17. ( )

18. ( )

1. ( )

19. (dement)

2. (mathnder)

20. ( )

3. ( )

21. (swsto)

4. (rek2)

22. (achilleas)

5. (hsiodos)

23. ( )

6. (bilstef)

6 ( ) x, a

1 ( ) 5. 50. 20. 1012. .


.
!
! .

1
2

8 5x
1
=
4
2

7x + 1
2 x 1,
2

x . x, a.


7 ( : KARKAR)
- .

2 ( )
, , ( ). , ,
. ,
.

8 ( : )
:
2015 (20162 9) 2017
2019 (20162 1)

9 (
)




(m + 1) x3 + m2 + m + 2 x2 m2 + m + 2 x (m + 1) = 0 m R
m , 1.
. , m , 1, .
. x1 , x2 , x3 x1 < x2 < x3
x2 m, , ,
3
m R x1 , x2 , x3
2
.

3 ( ) ,
: A = 999999 777777.
4 ( )


. 2 Km/h ,
3km/h 6km/h. 6 ,

10 ( ) (a, b) x2 + ax + b [2, 2]
x [0, 4].

11 ( ) M
AD ABCD P
B MC ,
AP = CD.

5 ( ) AB 6
ABE AE = 5. E.

12 ( )
K
T.

,
17 ( ) ~
, ~0.

,
T P Z .





~x
~ ~x = ~x + 8
~
~.

P = TZ.

18 ( ) E , E
:

x2

9
13 ( )
2
2

x xy + y = 21
:

y2 2 xy = 15

P , :

y2

=1

PE
= 2,
PE

PEE .

14 ( ) x, y
R , : x2 + 2 x sin(xy) + 1 = 0.

,
19 ( erxmer)

f (x) = 4 sx2 2 x x + 2012,

x s
x > 0. C f (1, 2012) :

15 ( KARKAR) AKLN ABC , (AB = AC)


AA . :

i)
.

A LKN

ii) (
) f .
iii) 2 sx + s > x x > 21 .
iv) lim f (x) = 2012
x x

.
v) ,
(1, 2) (0.5, 1)
.
20 ( )
( xi ): 0, 1, 2, ..., n , n N
2k + 1
, k N.
d=
2
) n .
) n = 2k + 1.

16 ( KARKAR)
ABCD M AB, N DC , : DC = 3 DN .

k = 2,
) .
) yi = axi + b , ab < 0
x = y sy = sx . a, b.

AN, DM S .
AS C

28 ( )
f :
, ,

21 ( ) m+2
m
z = m + (m + 2)i, w =
+
i, m , 1.
m+1 m+1
1) z, w.
2) m (OAB) = 2, A
z B w.

3) m , |z + w| = 2 2.
4) m , |z w| .

f (x)dx = 2
0

3 /4

f (x)dx.
1/4

xo (0, 1) : f (xo ) = 0.

, ,

22 ( ) 2 + 3ti

z =
1 + ti
t R.

29 (
)
2
f (x) = x + x + 1
() f .

p
x2 + 1)(y + y2 + 1) = 1, x + y = 0.
Z 1
1
()
dx

0
x2 + 1

() (x +
, , ,

23 ( KARKAR) : 8 x3
3
4 8 x + 15 15 = 0.

30 ( )
g : [0, a] R, a > 0 .
f : [0, a] R k f (x)+m f (ax) = g(x) x [0, a] k, m > 0 k , m
.

24 ( Ardid) f ,

f (x) f (x) = f 2 (x) (1)


x. f .

.
. f [0, a]

, ,

. (k + m)

f (0) f

1
2
f
2
3

f (x)dx =

25 ( ) f :
[0, 1] (0, +) [0, 1] (0, 1)

s

f (1) =

g(x)dx

1
1+e

1x x

1
2

dx =

x1 , x2 , x3 (0, 1) x1 < x2 < x3


6

f (x1 )
f (x2 )
f (x3 )
+3
+2
=0
f (x1 )
f (x2 )
f (x3 )

, ,

26 ( ) f : [a, ] R
[a, ] f (a) = f () = 0. (a, )
2 f ()
(a, ) f () =

31 ( )
f : [0, 1] [0, +) :

( a)( )

, ,

f (x) > 0, x [a, b].

f 2 (x)

dx +

f (x3 )dx 3

1
0

!2
x f (x)dx .

Rb

f (t)dt =

Rb

g (t)dt

x1 , x2 (a, b) x1 , x2

f (1 )
=e
f (0)
Z1

[a, b]

f : [0, 1] (0, +)

Z1

f (x)dx 1

32 ( ) f, g

27 ( )

, f (x1 ) f (x2 ) = g (x1 ) g (x2 ).


f (x) 2 dx 2

33 ( ) a, b, c
R, :

(a + b + c)2 9bc, (a + b + c)2 9ca

(a + b + c)2 9ab .

34 ( ) x + y = y + 2z = 5,
:

A = 2 x + 3y + 2 z

Juniors,
- -

35 ( ) Z2 x3 y3 = 2 xy + 8.

Seniors,

- -
39 ( )
x, y, z > 0 xy + yz + zx = 1,

36
q ( )
x +

[x] +

1 + {x} = 1

( x + y + z) (x + y)(y + z)(z + x) 2 2.
40 ( )
100.000.000 Fibonacci
( ).

Juniors,

37 ( )
ABCD a. BC E ,

BE =

Seniors,

DC F ,

CF =

a
2

41 ( KARKAR.) AA , BB, ABA B O M, S ,


. AA
BMB, BB AS A ,
O AA , BB S , M , AA BMB BB
AS A , A, B, A , B , .

G AE BF , G .

42 ( .)
ABC AD, BE, CF G. O1 , O2 , O3 , O4 , O5 , O6 ,
AFG, AGE, BGF, BGD, CGD, CGE ,
.


43 ( )

|a 3 b 5| >

4
4a + 5b

a, b.

38 ( ) O
(C) ABC(AB ,
AC) . H ABC .
AM (C) N . AM (C) P .
AP, BC OH AH = HN .

44 ( parmenides51) x A
A2 3 A + 2 I = O I x
O x ,
1 A2 (2 + 1)A + 2 I = O.

51 ( ) ~r(t) = (t, t2 , t3 )
x + z = 1.

52 ( Jeronymo Simonstone) ,

,
.

45 ( Putnam 2012) S
: S S S , :
x, y S , z S x z = y.
:

a c = b c = a = b a, b, c S .


53 ( dimtsig)
.

46 ( IMC 2004)
P(x) = x2 1.
P(P( P(x)) )) = 0.

| {z }

54 ( )
, , .
20.141,
.

2004


47 ( ) K
2. f : K K
1-1


55 ( )


.

sin = 0, 09 20m
. ,
44m .

, ,
. ,
100Kgr, 20Kgr, 60Kgr.
9 s.


.
80m
.
250N
.
. ( g = 10m/s2 )

() f (x + y) = f (x) + f (y) x, y K
() f (x) f (x1 ) = 1 x K {0}.

f f K .
48 ( ) A, An , n N,
m m . An A,

(An ) (A)
: > 0, N N,

n > N

(An )
(A) = {1 , . . . , m }.

m
[


z C : |z i | < ,
i=1

(B)
B, .

56 ( ) u : R R R, :


49 ( china university)
:

a) ux = 0,

b) ux = x,

c) uy = x,

d) uy = x + y

x ln sin x ln cos x dx =
0

( ln 2)2
8

4
192

e) ux = e + y.
.

50 ( .. ) f : C , . f
n n , f
.


57 ( )
x, y, z ,

||x + y|| + ||y + z|| + ||z + x|| ||x|| + ||y|| + ||z|| + ||x + y + z||.

58 ( ) (n )nN . (nk )kN

63 ( .-. ) ,
, ,
.

ni < n j i < j.

64 ( ) , , A ,
()

3 A = 3A 43 A.

59 ( ) R a, , .
.
60 ( ) f, g : [a, b] R. f
f (x) + f (x) g (x) + x = f (x) + g (x) , x [a, b]
f (a) = a, f (b) = b, f .

,

65 ( .. )
()
1
1 1
1, , , . . . , , . . . ,
2 3
n


() .

61 ( )
1
(an ) |am+n am an | <
, -

m+n

66 ( apotin) a, b, c d

m, n. .

62 ( ) f : R R
f (x + y) = f (x + f (y)), x, y R.

P (x) = ax3 + bx2 + cx + d


:

P (1) + P (2) + P (3) + + P (n) = n4 , n N

1 ( ) 5.
50. 20. 1012.
. .
!
!
.

http://www.mathematica.gr/forum/viewtopic.php?p=115911

http://www.mathematica.gr/forum/viewtopic.php?p=125401

( )
ab () x,
:

( ) A
B. B B1
B2 .

20[5(10a + b) + 50] + 1012 x =

= 100(10a + b) + 1000 + 1012 x =


= 1000a + 100b + (2012 x)


a, b (
1000a + 100b).
(
2012 x). ,
1912.

A
B3 B4. B

A. A B3 B4
. B



.
:
...

2 ( )

, ,
( ). , ,
.
,
.

3 ( ) ,
: A =
999999 777777.



. 2Km/h ,
3km/h 6km/h.
6 ,

http://www.mathematica.gr/forum/viewtopic.php?f=33& p=151422

( ) .

999999 = 1000000 1.
(1000000 1) 777777 = 777777 (1000000 1) =
777777 1000000 1 777777 =
777777000000 777777
= 777776222223.
7 + 7 + 7 + 7 + 7 + 6 + 2 + 2 + 2 + 2 + 2 + 3 = 54

http://www.mathematica.gr/forum/viewtopic.php?f=33& t=33726

( ) , ( ), a , b
c .
( )
 


a2 + b3 + 6c + 2c + 3b + 6a 6

.
46 (a + b + c) = 6 a + b + c = 9.
9 + 9 = 18Km.

4 ( )

10

5 ( ) AB 6 ABE AE = 5.
E.

http://www.mathematica.gr/forum/viewtopic.php?f=34&t=33497&start=80

1 (r) :
r
r
8 5x 1
8 5x 2
6 5x
=
=

2
4
4 r
r 4
r4
7x + 1
7x + 1 4x + 2
3x 1
2x 1 =

=
.
2
2
2
2
:
6
6 5 x 0 5 x 6 x
5
1
3 x 1 0 3 x 1 x .
3
x , x = 1. :

http://www.mathematica.gr/forum/viewtopic.php?f=34&t=33497&start=40

1 ( ) K M AB
E . AEB
E M , AM = MB = 3.
AE M , :
E M 2 = AE 2
AM 2 = 52 32 = 25 9 = 16,
E M = 16 = 4.
, KE = K M E M = 6 4 = 2.
K MA , K = AM = 3.
EK :
2
E = K 2+ KE 2 = 32 + 22 = 9 + 14 = 13,
E = 13.

6 ( )
x, a

1
2

8 5x 1
=
4
2

a1

a1

1
=1
2

a1

1
2

2
2
2

a1

1
2
4
2a 2 = 1
3
a=
2

7x + 1
2 x 1,
2

x . x, a.

11

1
=
4

2
2

a + b + c = 300 b + c = 300 a,
(b + c)2 = (300 a)2
b2 + c2 + 2bc = 90000 600a + a2
:
a2 + 4E = 90000 20E + a2
24E = 90000, E = 3750.

7 ( : KARKAR)
- .

8 ( : ) :
2015 (20162 9) 2017
2019 (20162 1)
http://www.mathematica.gr/forum/viewtopic.php?f=35&t=30871
http://www.mathematica.gr/forum/viewtopic.php?f=35&t=32794

)
1 ( )
2015 (20162 32 ) 2017
2015 (20162 9) 2017
=
=
2019 (20162 1)
2019 (20162 12 )
2015 (2016 + 3) (2016 3) 2017
=
=
2019 (2016 + 1) (2016 1)
2015 2019 2013 2017
= 2013.
=
2019 2017 2015
2 ( ) , x = 2016,
(x 1)(x2 9)(x + 1)
A=
=
(x + 3)(x2 1)
(x 1)(x + 1)(x + 3)(x 3)
=
= x 3 = 2013.
(x + 3)(x 1)(x + 1)

a, b, c , :
cb 60a
=
= 30a
E=
2
2
cb = 2E 30a = E

12

:
f (x) = x2 [2 + h, 2 + h], h 0

9 (

  )


(m + 1) x3 + m2 + m + 2 x2 m2 + m + 2 x
(m + 1) = 0 m R m , 1.
. , m , 1,

.
. x1 , x2 , x3
x1 < x2 < x3 x2
m, , , m R
3
x1 , x2 , x3
2
.

4. 0 , f
f (2 + h) f (0) = (2 + h)2 4
h = 0. 0 ,
h > 2 f (2 + h) f (2 h) = 8h > 16. ,
4
4, .
, g(x) = x2 + ax + b
y = x2 .
4 4
A(2, 2).

2a = 2 a = 4 g(2) = 2, b = 2
: h > 0,
h < 0.
2 ( ) x = 0
2 b 2.
x = 4 2 16 + 4a + b 2.

4 16 + 4a 4 4 a 3
a
3
2.
2
2
a
[0, 4].
2
a
x = ,
2

http://www.mathematica.gr/forum/viewtopic.php?f=19&p=155336

(m + 1) x3 +

 (

 )
2
2
2
m + m + 2 x m + m + 2 x (m + 1) = 0





(m + 1) x3 1 + m2 + m + 2 x2 x
=
0





(m + 1) (x 1) x2 + x + 1 + m2 + m + 2 x (x 1)
=
h

 
 i
2
2
=
0 (x 1) (m + 1) x + x + 1 + m + m + 2 x


i
h
2
2
0 (x 1) (m + 1) x + m + 2m + 3 x + m + 1
=


2
2
0 x = 1 (m + 1) x + m + 2m + 3 x + m +

2
1 = 0.
= m2 + 2m + 3 4 (m + 1)2 =



m2 + 2m + 3 2m 2 m2 + 2m + 3 + 2m + 2
=



2
2
2
m + 1 m + 4m + 5 > 0. m + 1 > 0
m2 + 4m + 5 > 0 < 0 m , 1 . x1 , x2 , x3

1
2
x1 , x2 = 1, x3 . x1 x3 = = m+
m+1 = 1 = x2 . -


a2
a 2
+b
x2 + ax + b = x +
2

, b

a2

.
4
a, b

b2
16 + 4a + b 2


.
)
2

2m+3
3 x2 = x1 + x3 = S = = m +m+
.
1
2
2
m + 2m + 3 = 3 (m + 1) m + 5m + 6 = 0
m = 2 m = 3, .

a2

2
4
: 2a + b + 6 0.

a 2 b + 2.
b

10 ( )
(a, b) x2 +
ax + b [2, 2] x [0, 4].

0 6 + 2a + b b + 2 4 b + 2 + 4 =

http://www.mathematica.gr/forum/viewtopic.php?f=19&p=159601

2
b + 2 2 0.

b = 2 a = 4.

1 ( )
13

11 ( ) M AD ABCD
P B
MC , AP = CD.

12 ( )
K
T.
,
T P Z .
P = TZ.

http://www.mathematica.gr/forum/viewtopic.php?f=20&t=34663

( )

KT
.

http://www.mathematica.gr/forum/viewtopic.php?f=20&t=34268

K o

bT = 90
K

b
K = b
ZK = Kb
= Kb
P = KZ, K
KP.

1 ( ) AS BP
T AS BC. ATC M
BT = TC . BS = S P,
AS BP AP = AB = CD.

, KPZ K
P = TZ .
2 ( ) PC M
BA E .
MAE, MCD AE = CD =
AB. PEB PA ,
PA = AB = CD.

14

y = 5 or y = 3 (I)

y2 + 15

(II)
x=
2y
* y = 5 (II) x = 4,
(x, y) = (4, 5).
* y = 5 (II) q = 4,
(x, y) =

(4, 5).
(II) q = 3 3,
* y = 3

(x, y) =(3 3, 3).

* y = 3 (II)
q = 3 3,
(x, y) = (3 3, 3). A = 2 x + 3y + 2z =
2 x + 2y + y + 2z = (2 x + 2y) + (y + 2z) = 2(x + y) + (y + 2z) =
2 5 + 5 = 10 + 5 = 15

13 ( )
:
2
2

x xy + y = 21

y2 2 xy = 15

http://www.mathematica.gr/forum/viewtopic.php?f=21&t=33297

( )
y , 0, y = 0
0 = 15, . :
2
2

x xy + y2 = 21

x xy + y = 21

y2 + 15

y2 2 xy = 15

x
=

2y

14 ( ) x, y R , : x2 +
2 x sin(xy) + 1 = 0.

!2

y2 + 15
y2 + 15

y + y2 = 21

2y
2y

y2 + 15

x=
2y
4

y
+
30
y2 + 225 2y2 (y2 + 15) + 4y2 y2 = 84y2

y2 + 15

x=
2y
4
2

3y 84y2 + 225
y = 25 or y2 = 3

y2 + 15
y2 + 15

x=
x=
2y
2y

http://www.mathematica.gr/forum/viewtopic.php?f=21&t=33912

( ) (x + sin(xy))2 + cos2 (xy) = 0, cos(xy) =


0 x = sin(xy). , x2 = 1,
x = 1 x = 1. cos y = 0

sin(y) = 1, y = + 2k k .
2

15

15 ( KARKAR) AKLN ABC , (AB = AC)


AA . : A LKN


.
16 ( KARKAR) ABCD M AB, N
DC , : DC = 3DN .
AN, DM S .

AS C

http://www.mathematica.gr/forum/viewtopic.php?f=22&t=32867

1 ( ) KNA L

KA2 NA2 = NA2 NL2


KB2 + BA2 KL2 = NC 2 + CA2 NL2
KB2 KL2 = NC 2 NL2
BKL LNC .
2 ( ) Q KL AA .
QL k AC QL AC .
A ABC ( ABC )
L : QCA
A LQC : (1).
KQA = QAC = KAA
KQA .
KQA NLC AKLN
KQ = NC . : (2).
KQCN (

http://www.mathematica.gr/forum/viewtopic.php?f=104&t=12191

1 ( )
ABCD AB = CD = , (ABCD) = a .

ZEAB, S .
AS M DS N :
1

DN
DS
2
DS
DS
=

= 3
= : (1).
1
S M AM S M
SM 3

)
(1) , (2) A LKN
16

3 ( ) L DM AC .

DS Z ES M
:

AL AM 1 DN
=
= =

LC
DC
2
NC
SL
LN CN 2
NL k AD
=
=
=
S D AD CD 3
2
SL 2
SL
=

=
SD+SL 3+2
DL 5
(S AC) 2
2
= (S AC) = (ADC)
(ADC) 5
5

AM k DC

SZ
DS (1) S Z
2
=
=

SE SM
SE 3
2
SZ 2
2
SZ
=

= S Z = : (2)

SE + SZ 3 + 2

5
5
(AS N) = (ANC) (S NC)
1
1
DC EZ NC S Z
2
2
1
1 2
2
(AS N) =
2
2 3
5
2
1
(AS N) = (ABCD) (ABCD).
2
15
1

(AS N) = (ABCD)
5

(AS N) =

2
(ABCD)
10
.
(ABCD)=2(ADC)

(S AC) =

(AS N) = 200/0 (ABCD)

4 ( )

CL

2( )
AB = 6k, k > 0.
AM = 3k, DN = 2k, NC = 4k.

LCDLAM

2LA,

(CAN) = (CDL) =

2(CAD)
,
3

(S AL) = (S DN) = x.

T S AC
E ABCD.

(S LC) = (S NC) = 2 x

AS
3
3
AS
AM 3
=
=
=

DN
2
SN 2
AN 5

(ADS )

T =
T =

(ANC)=2(ADN)

, 7, 5 x =

3 4
2 1
3
(ANC) = (ADC) = (ABCD)
5
5 6
5 2

3x =

1
E = 200/0E
5

17

(ABCD)
5

1, 5 x.

(ABCD)
2

x=

(ABCD)
15

(AS C) =

17 ( )
~ , ~0.

18 ( ) E ,
E :





~x
~ ~x = ~x + 8
~
~.

x2

y2
4

=1

http://www.mathematica.gr/forum/viewtopic.php?f=23&t=1990

PE
= 2,
PE

PEE .

( - )
~x =
~ , :

http://www.mathematica.gr/forum/viewtopic.php?f=23&t=1979

P , :

~0 = 9|~|
~ |~| = 0.

( )


~ = ~0, ,
~ , ~0.
~x =
~ , > 0.



x x = x + 8






~
~
~ =
~ + 8 ~
~





~ ,0

~
~
~ +8
~
~ =0

= 3, = 2 =

| 1| = | + 8|
2

= | + 8|

2 = + 8

( > 0)

(PE)
(PE)
(PE) 2
2
=2
=

= ,
(PE )
(PE ) + (PE) 2 + 1
6
3

= 8



E 5, 0


(PE) + PE = 6,

2 2 8 = 0

= | + 8|

5, 0 ,

( > 0)

= 8 ().

(PE) = 4

= 4 = 2 ()


PE = 2.

1 :

~x = 4
~,

( ) ( ) ( ).

PEE = 3 +

.
18

5,

( PE) ( PE ) ( )
q




51 1+ 5 3 5
=
3+ 5
r

 2   2
=
5
5 12
32

= 16 = 4

E=

3 : () P

PEE .
(x, y) P.


2

PE = 2 x + 5 + y2 = 4

2 :

P (x, y)

1
() () .
2

1 4
2 5 = 4.
2
5

x2


PE = 2.

(PE) = 4

x+

2
5

+ y2 = 4

= 16 EK .

E K = x, EK = 2 5 x :

4
=
5

2

x+

x
= 0

x = 3 5 (

x < 3 ) x = 3 , y = 4 .

4 E K 2 = 16 EK 2

2

= 1.

y2
+
=1
4
4


2
y2
x2 x +
5
x2
4 =1 9

=0
4
9

x + 5 x x + 5

PKE

4 x2 = 16 2 5 x

y2

PK . PKE

2 = 4 E K 2

P ,
5

... x = ,

PK

E=

16
4
= ,
5
5

19

P ,
4

1 4
2 5 = 4.
2
5

19 ( erxmer)

f (x) = 4 sx2 2 x x + 2012,

1
2

f (x) > 0 x > .

  f h

, 21 21 , + -

x s x > 0. C f (1, 2012)


:

x =

i)
.

1
= s x + 2012.
f
2

ii) ( ) f .
iii) 2 sx + s > x x >

1
2

iii)
1 f
1
x > f (x) > f
2
2

1
.
2

4 sx2 2 x x + 2012 > s x + 2012

iv) lim f (x) = 2012 x x

4 sx2 s 2 x x + x > 0


s 4 x2 1 x (2 x 1) > 0

.
v)
, (1, 2) (0.5, 1)
.

s (2 x 1) (2 x + 1) x (2 x 1) > 0

(2 x 1) (2 sx + s x) > 0

x> 21

2 sx + s > x

iv)

http://www.mathematica.gr/forum/viewtopic.php?f=18&t=32383

lim f (x) = 2012 4 s x2 2 x2 + 2012 = 2012

( )

x x

x2 (4 s 2) = 0

i) C f A (1, 2012)
:

s= ,

f (1) = 2012 4 s 2 x + 2012 = 2012


s 1
=
x 2
CV = 0, 5 > 0, 1

CV =

1
2

s
x

1
2

1
2

1
2

x = 1

v)

R  6 s = 3.

ii) x R f (x) = 8 sx 2 x,

f (x) = 0 8 sx 2 x = 0
x
x=
4s
x=

1
2

x>0


(1, 2) 34% + 13, 5% = 47, 5% (0, 5, 1) 34%

4CV
1
x=
2
20

) k = 2, n = 2 2 + 1 = 5, 0, 1, 2, 3, 4, 5.

20 ( )
( xi ): 0, 1, 2, ..., n , n N
2k + 1
d =
, k N.
2
) n .

x =

0+1+2+3+4+5
15
5
=
= .
6
6
2

) n = 2k + 1.
) yi :

k = 2,
) .

y = x

y = x + b (1 ) x = b

) yi = axi + b ,
ab < 0 x = y sy = sx .
a, b.

(1)

yi :
sy =s x

sy = || sx || = 1 = 1 = 1.
= 1 (1) b = 0,
ab < 0. = 1 (1)
b = 2 x b = 5.
= 1 b = 5.

http://www.mathematica.gr/forum/viewtopic.php?f=18&t=26302

1 ( )

2 ( ) )
.

,
m
m + 1 ,

) n + 1. n
, n+ 1 , d ,
1
d N . d = 2k+
< N , 2k + 1 .
2

n .
) n + 1,
.
n+1

2
n+1
+ 1. 2
1 = 0
= 1 = 1.

n+1 =

n+1

1 n+1 +1 =
2

d=


m m + 1 = k + 1
n + 1.
m + 1 = k + 1 ,
(n + 1) (k + 1) = n k.
( )

.
k + 1 = n k n =
2k + 1.

n+1
2

:
n+1

d=

1+
2

n+1
2

m + (m + 1)
2k + 1
=
k = m.
2
2

2k + 1
n
= n = 2k + 1.
2
2

21

21 ( ) m+2
+
z = m + (m + 2)i, w =
m+1

m = 0 [ (1)] m = 2 [ (2)].
4)

i, m , 1.
m+1

1) z, w.
2) m (OAB) = 2,
A z B
w.
3) m , |z + w| = 2 2.
4) m , |z w|
.



! 

m 
m+2
i
|z w| = m +
+ m+2

m+1
m+1

!
!


1
1
+ m+1+
i ,
= m+1

m+1
m+1

http://www.mathematica.gr/forum/viewtopic.php?f=51&t=31965

|z w| =

1 ( )
1) z = x + yi, x, y R x = m, y = m + 2,
y = x + 2
z () y = x + 2.
w = + i, , R
m
m+2
, =
, m , 1,
=
m+1
m+1
m+1+1
m+11
=
, =
m+1
m+1
1
1
, =1
,
= 1
m+1
m+1
= 2 = + 2
w y = x + 2.
2)

A(x, y), B(, ),


!

m+2
m

OA = (m, m + 2), OB =
,

m+1 m+1

(OAB) =
=
=

1
2
1
2
1
2

|x y|


m m + (m + 2) m + 2
m+1
1
2
m +

2m + 4m + 4 m2 + 2m + 2

=
.
m+1
m+1

(OAB) = 2,


2
m + 2m + 2
= 2

m+1


m2 + 2m + 2 = |2m + 2|
m2 + 2m + 2 = 2m + 2, (1)

m2 + 2m + 2 = 2m 2, (2)
22

2 m+1+


2 m + 1 +

!2

m+1

1
,
m + 1


1
.
f f (x) = x + 1 +
x + 1
1
, x > 1 f (x) = x + 1 +
x+1
x(x + 2)
1
=
f (0)
= 0
f (x) = 1
2
(x + 1)
(x + 1)2
f (x) < 0, x (1, 0), f (x) > 0, x (0, +), f (1, 0], [0, +), x = 0 f (0) = 2.

1
x < 1 f (x) = (x + 1)
,
x+1
x(x + 2)
1
=
, f (2) = 0
f (x) = 1 +
2
(x + 1)
(x + 1)2
f (x) < 0, x (, 2), f (x) > 0, x (2, 1),
f (, 2], [2, 1),
x = 2 f (2) = 2. f
2 x = 0, x = 2, min |z w| = 2 2
m = 0, m = 2.
2 (
)

3) |z
+
w|
=
2

m
m+2


+
i = 2 2
m + (m + 2) i

! m + 1 m + 1 



m
m+2
i = 2 2
+ m+2+
m


2 m + 1 2
m+1

m 2 m + 4m + 2

+
i = 2 2
m+1

m+1

!2
!2

m2 + 4m + 2
m2 2
+
=2 2

m+1
m+1
2

2 
2
2
m + 4m + 2
m 2
+
=8

(m + 1)2
(m + 1)2
2

2 
m2 2 + m2 + 4m + 2 = 8(m + 1)2
m4 4m2 + 4 + m4 + 16m2 + 4 + 8m3
+4m2 + 16m = 8m2 + 16m + 8 

2m4 + 8m3 + 8m2 = 0 2m2 m2 + 4m + 4 = 0, m = 0 m = 2.

:
(x ty) + (xt + y)i = 2 + 3ti (1).

ty
=
2

x 2 = ty

(2).

xt + y = 3t
y = t(3 x)

a = (x 2, y),
b = (y, 3 x) (2) :

a = t
b , t R

a ,
det(
b ) = 0 (x 2)(3 x) y2 = 0 .

3 ( )
m2 2 m2 + 4m + 2
+
i,
3) z + w = ... =
m+1
m+1
|z + w|2 = 8

!2

!2

5
1
(3).
x
+ y2 =
2
2
M(z) (3), A(3, 0),
t R 1 + ti , 0 2 + 3ti , 3(1 + ti).
4 ( )
(2 + 3ti) (1 ti)
2 + 3ti
z=
=
=
(1 + ti) (1 ti)
1 + ti
2 + 3t2
t
2 2ti + 3ti + 3t2
=
+
i.
=
2
2
1+t
1+t
1 + t2
2
2 + 3t
t
z = x + yi, x =
,y=
.
2
1+t
1 + t2
:

2

2 
m2 2 + m2 + 4m + 2 = 8(m + 1)2
. . . m2 (m + 2)2 = 0 m = 0 m = 2.

22 ( ) 2 + 3ti
t R.
z =
1 + ti
http://www.mathematica.gr/forum/viewtopic.php?f=51&t=32004

1 ( )
2 + 3it
z2
z=
,t R t =
, z , 3.
1 + ti
(3 z)i

z 2
z2
=
2zz 5z 5z + 12 = 0
(3 z)i i(3 + z)
x2 + y2 5 x + 6 = 0,

!
5
1
K , 0 , ,
2
2
(3, 0).
2 ( )

x +y

=
=
=

t
t2
+
i.
2
t
1 + t2
1 +

t = tan q, q , ,
z = 2 +

(z) = 2 +

2 2

tan q

1 + tan2 q

= 2 + sin2 q =

!2

t 2
1 + t2
4 + 12t2 + 9t4 + t2
2
1 + t2
9t4 + 13t2 + 4
2
1 + t2



t 2 + 1 9t 2 + 4
2
1 + t2
9t 2 + 4
.
1 + t2
2 + 3t2
1 + t2

A, B, G R :

1
1
5 1
cos 2q = cos 2q
2 2
2 2

tan q
1
(z) =
= sin 2q.
2
1 + tan2 q
, z = x + yi, x, y R
5
1
1
x = cos m, y = sin m, m (, ).
2
2
2

,
z
!
5
1
K , 0 , , 2
2
(3, 0).
3 ( )
z = x + yi x, y R :
2 + 3ti
(x + yi)(1 + ti) = 2 + 3ti
x + yi =
1 + ti

=2+

9t 2 + 4
1 + t2





A 2 + 3t2 + Bt + G 1 + t2
1 + t2

= A

2 + 3t
t
1 + t2
+
B
+
G
1 + t2
1 + t2
1 + t2

= Ax + By + G.


 t :


9t2 + 4 = A 2 + 3t2 + Bt + G 1 + t2

9t2 + 4 = (3A + G) t2 + Bt + (2A + G)


3A + G = 9 2A + G = 4 B = 0.
A = 5 , B = 0 , G = 6.


x2 + y2 = 5 x 6 x2 + y2 5 x + 6 = 0
23

5
x
2

!2

1
,
4
!
1
5
K , 0 , ,
2
2
A(3, 0) z = 3 3 + 3ti = 2 + 3ti
.
5 ( )
t2
2 + 3it
t
= ... = 2 +
z = x + yi =
+
i.
1 + it
1 + t2 1 + t2

+ y2 =

x 2 =

t4
t2
t4 + t2
+
=
=
(1 + t2 )2 (1 + t2 )2 (1 + t2 )2
t2 (1 + t2 )
t2
=
=
= x 2.
(1 + t2 )2
1 + t2
!2
1
5
2
2
+ y2 = ,

(x 2) + y = x 2 x
(x 2)2 + y2 =

2
4


!
5
1
K , 0 , ,
2
2
(3, 0).

t2
t
y =
.
2
1+t
1 + t2

24

:
h

x 15
, + . 8
: f (x) = f 1 (x), f .

23 ( KARKAR) 3
: 8 x3 4 8 x + 15 15 = 0.

( )
f (x) = x. :
8 x3 15
= x 8 x3 8 x 15 = 0
8
(2 x 3)(4 x2 + 6 x + 5) = 0, ( )
x = 32 (
).

http://mathematica.gr/forum/viewtopic.php?f=52&t=16142

1 ( )
h
,
+

15
8

8 x3 27 = 4

 3

8 x + 15 3

16(2 x 3)
. (2 x 3)(4 x2 + 6 x + 9) = p
3
3
(8 x + 15)2 + 3 8 x + 15 + 9
x = 32 .

24 ( Ardid) f ,

( )

16

4 x2 + 6 x + 9 = p
3

(8 x + 15)2 + 3 8 x + 15 + 9

f (x) f (x) = f 2 (x) (1)


x. f
.

(1).

f (x) = 4 x2 + 6 x + 9, x R ( )
.
27
4

g(x) = p
3

16

http://mathematica.gr/forum/viewtopic.php?f=52&t=17342

1 ( ) x R : f (x) f (x) = f 2 (x). x x


( x R x R)
f (x) f (x) = f 2 (x).
x R
2 f (x) f (x) = f 2 (x) + f 2 (x) ( f (x) f (x))2 = 0
f (x) = f (x) x R.

f .
2 ( ) f , x0 f (x0 ) , f (x0 )
(). f 2 (x) = f (x) f (x) ()
x x, : f 2 (x) = f (x) f (x), f 2 (x) = f 2 (x). : f 2 (x0 ) = f 2 (x0 )
f (x0 ) = f (x0 ), f (x0 ) = f (x0 ) ()
(). () x x0 , :
f 2 (x0 ) = f (x0 ) f (x0 ), () :
f 2 (x0 ) = f 2 (x0 ) f (x0 ) = 0, f (x0 ) = f (x0 ) = 0,
(), f .

(8 x + 15)2 + 3 8 x + 15 + 9

( ),
, 16
. 27
> 16
(1)
15
8
9
4
9
3
, 2 .

h
2 (KARKAR) x 15
,
+
,
8
:

3
8 x3 15
8 x3 15
= 8 x + 15
=
4
8

r
3

8 x + 15
8

8 x3 15
,
8
8 x3 15
, . y =
,
8
r
3 8 x + 15
f 1 (x) =
,
8

f (x) =

25

:
g(x1 ),0 f (x1 )

25 ( ) f : [0, 1] (0, +) [0, 1]


(0, 1)

f ( 1) =

1
2
f (0) f
f
2
3

f (x1 )
f (x2 )
f (x3 )
+3
+2
=0
f (x1 )
f (x2 )
f (x3 )

26 ( )
f : [a, ] R [a, ] f (a) = f () = 0.
(a, ) (a, )
2 f ()

http://www.mathematica.gr/forum/viewtopic.php?p=154549

( )


!
!
x+1
x+2
g (x) = f (x) f
f
, x [0, 1], 2
3
[0, 1]
x+1
2
x+2
1

1,

1
2
2
3
3
x [0, 1] f [0, 1] (0, 1)
x+1
2
x+2
1

1,

1
2
2
3
3
x [0, 1] f (0, 1)

x+1

g (x) = f (x) f
+
+

x+2

x+1
x+2
1
f (x) f
f
2
2
3
!

1
x+1 x+2
f (x) f
f
3
2
3

f () =

1 ( ) -

Rolle
h(x) = f (x) g(x)

h(x) = f (x) (x a)(x b)

f ()
( a)( b)

[a, ], [, b] k
h (k) = f (k) g (k) = 0, h (l) =

(a, ), l (, b) :
f (l) g (l) = 0.

h (x) = f (x) g (x), x


[k, l]
Rolle h (k) = h (l) = 0 h 2 f ()
h (x) = f (x) g (x) = f (x) (a)(b)

2
1
f
= ( f (1))3 =
2
3
g (1).
g Rolle [0, 1],
=
x1

!(0, 1) !g (x1 )

x
+
1
x
+
2
1
1

f
f

2
3
1
f (x1 ) 1
+
! +
! g (x1 ) =
0
f (x1 ) 2
3
x1 + 2
x1 + 2

f
f
3
3

f (0) f

(x a)(x b)

h(a) =
( a)( b)
f (a) g(a) = 0 h() = f () g() = f () f () = 0
h(b) = f (b) g(b) = 0
f ()
, h (x) = f (x) g (x) = f (x) (2 x a b) (a)(b)

g(x) = f ()

g (0) =

( a)( )

http://www.mathematica.gr/forum/viewtopic.php?p=151131

!
!

x+1
x+2

f
f

2
3
1
f (x) 1
! +
! g (x)
+
=
f (x) 2
3
x+2
x + 2

f
f
!

1 f (x2 )
1 f (x3 )
f (x1 )
+
= 0 6
+
2 f (x2 )
3 f (x3 )
f (x1 )

f (x1 )
f (x2 )
f (x3 )
3
+ 2
= 0, x1 (0, 1), x2 =
f (x2 )
f (x3 )
x1 + 2
x1 + 1
(x1 , 1) , x3 =
(x2 , 1). 2
3
x1 , x2 , x3 (0, 1) x1 < x2 < x3 ,
f (x1 )
f (x2 )
f (x3 )
6
+3
+2
= 0.
f (x1 )
f (x2 )
f (x3 )

x1 , x2 , x3 (0, 1) x1 < x2 < x3


6

2 f ()

(k, l) (a, b) : h () = f () (a)(b) = 0


2 f ()

f () = (a)(b)
2 ( ) f (x)( a)( b) 2 f () = 0
(a, b).
f (x)( a)( b) f ()(x a) f ()(x b) = 0
( f (x)( a)( b) f ()(x a) f ()(x b)) = 0.
26

Rolle

h(b) = 0 Rolle
[a, ], [ , b] 1 (a, ),
2 (, b) h (1 ) = h (2 ) = 0

f (x)( a)( b) f ()(x a) f ()(x b)

Rolle

f (x)(a)(b) f ()(xa)(x b) f ()(x a) (x


b) =( f (x)( a)( b) f ()(x a)(x b)) .

h(x) = f (x)( a)( b) f ()(x a)(x b),

h (x) = f (x)( a)( b) f ()(x a) f ()(x b)


[1 , 2 ] (1 , 2 ) h () = 0 f ()( a)( b) 2 f () = 0
.

x [a, b],

, h(a) = h() =

27

27 ( )

f : [0, 1] (0, +)

f (1)
=e
f (0)
Z1

f2

(x)

Z1

dx +


f (x) 2 dx 2

f (x)

22

( f (x))2

dx

( f (x))2 dx

f (x)

!2
f (x) dx 0

f (x) = 0 2 f (x) f (x) = 2, x [0, 1]

f (x)
dx =
f (x)

2x +

e2

f (x)dx = 2

, x [0, 1].

3/4

f (x)dx.

1/4

xo (0, 1) : f (xo ) = 0.

http://www.mathematica.gr/forum/viewtopic.php?f=54&t=33715

( )

a
f (x) f (x) = a f 2 (x) = 2ax + b.
f (x)
R1
R1
0 f 21(x) dx = 0 ( f (x))2 dx, a = 1.
f (x) =

f (1)
f (0)

f (x) =

= e b =

2x +

e2

e2 1

g (x) = f x +

f 2 (x)

1
2

21

g (x)dx = 2

1
4

Rx
g (t)dt 2 2x g (t)dt,
2
G (0) = 0. (1)
 ,

G 21 = 0. Rolle
h
i
G 0, 21
!
1
0,
G (x) =

g (x)dx (1) .

41

+ [ f (x)] dx 2

1
,
2

.
2 ( ) :

f (x)

28 ( )
f
:

- CauchySchwarz.

(1)

f (x) =

i1
= 2 ln f (x) = 2(ln f (1) ln f (0)) = 2

+ [ f (x)] 2

!
f (x) dx 0

1 ( ) -
Cauchy-Schwarz

( f 2 (x)) = (2 x) , x [0, 1] f 2 (x) = 2 x + c, x [0, 1]

http://www.mathematica.gr/forum/viewtopic.php?f=54&t=27156

f 2 (x)

f (x)
dx
f (x)

x
x

28

, G () = 0.

G (x) = g (x) + g (x) g

 x
2


    

0 = g () g
g g () =
2

 x
g ,
2

h
 i h  
i
0 = G () = g () g 2 g 2 g () .

i
h
i
h
g 2 , , 2 -

1 , 2 2



g (2 ) g (1 ) .

Rolle g


1 , 2 t0 (1 , 2 )


21 , 21 , g (t0 ) = 0. x0 = t0 + 21
(0, 1), f (x0 ) = 0 .


,

,
2

29

29 (
)

f (x) = x + x2 + 1

30 ( )
g : [0, a] R, a > 0 .
f : [0, a] R
k f (x)+m f (a x) = g(x) x [0, a]
k, m > 0 k , m .

() f .

() (x + x2 + 1)(y +
x + y = 0.

()

y2 + 1) = 1,

x2 + 1

dx

. f [0, a]
. (k + m)

f (x)dx =

( )

()

x R, x2 + 1 > x2
(1)

(1)

x < 0

x2 + 1 >

g(x)dx

1+e

1x x

dx =

1
2

x2

x2 + 1 > |x| (1)

http://www.mathematica.gr/forum/viewtopic.php?p=150127

x2 + 1 > x f (x) > 0

( )

x2 + 1 > x > x f (x) > 0


f (x)
x
=
> 0, x R, (2),
f (x) = 1 +
x2 + 1
x2 + 1
f .
x 0

http://www.mathematica.gr/forum/viewtopic.php?p=161300

. x a x

k f (x) + m f (a x) = g(x), x [0, a]


()

f (x) = x + x2 + 1




x + x2 + 1 x + x2 + 1
=

x + x2 + 1

k f (a x) + m f (x) = g(a x), x [0, a]


k2 f (x) + km f (a x) = kg(x)

f (x)

f (x) f (y) = 1

f (y) =

km f (a x) m2 f (x) = mg(a x)

= f (x)
f (x)
y = x x + y = 0

k2 f (x) m2 f (x) = kg(x) mg(a x)


kg(x) mg(a x)
f (x) =
k 2 m2

f 1-1, .
()

1
0

x2 + 1

dx

(2)

k, m > 0, k , m f [0, a] , g
.



f (x)
dx = ln | f (x)| 10
0 f (x)

ln(1 + 2)
1

30

. k f (x) + m f (a x) = g(x)

Za
0

(k f (x) + m f (a x))dx

Za

f (x)dx + m

Za

f (a x))dx =

Za

g(x)dx

g(x)dx

m)

f (a x))dx u = a x

x) =

f (a x))dx =

f (u)(du) =

f (x)dx + m

f (x)dx =

f (x)dx =

Z
0

1x x

e 1x

e 1x + e x
e 1x
= 1 = g(x)

e 1x

=
e

Za
0

f (a x))dx =

g(x)dx

31

f (x)dx =

1
2

+e

1x

f (1

1 f (x) + 1 f (1 x) =

Z1
0

Z1

g(x)dx (k +

g(x)dx

() (1 + 1)

Z
0

f (u)du

Za

Za
0

1+ e

e
Za

Za

. f (x) =

x = 0 u = a x = a
u = 0 du = dx

Z0

f (x)dx + m

Za

Za

f (x)dx =

Z1
0

dx

:
Z

31 ( )

f : [0, 1] [0, +) :

f (x)dx 1

1
3

f (x )dx 3

1
0

!2
x f (x)dx .

f (x)dx
Z

3
4

f (x)dx , 0

0
Z

!2
f (x)dx =

Z 1
 3 
f
x dx = 3
x2 f (x)dx

http://www.mathematica.gr/forum/viewtopic.php?f=56&t=28235

( )

2
2 x
x

Z
Z

h (x) = f (t) dt g (t) dt , x [a, b].

!2
Z 1
Z 1
 3 
x 3
x2 f (x)dx
x f (x)dx 3
f (x ) f
3

!2 Z 1
x2 f (x)dx
x f (x)dx

h (a) = h (b) = 0 h Rolle [a, b].

Rolle,
x1 (a, b) :

x f (x)dx = m,
2

g (t)dt

, f (x1 ) f (x2 ) = g (x1 ) g (x2 ).

, f

Rb

x1 , x2 (a, b) x1 , x2

( )

f (t)dt =

f (x) > 0, x [a, b].

4
f (x)dx .
3

Rb
a

2( ) :
1

(x m)2 f (x)dx = 0

[a, b]

f (x)dx = 0 = f 0.

32 ( ) f, g

!2
Z 1
Z
xdx
x f (x)dx 3

(x m)2 f (x)dx 0

f (x) = 0, x [0, 1]

f (x3 )dx

!2
f (x)dx

(x m)2 f (x) = 0, x [0, 1]

(x2 2mx + m2 ) f (x)dx 0

1 ( ) , Chebyshev,

x2 f (x)dx m2 ,

http://www.mathematica.gr/forum/viewtopic.php?f=56&t=33652

f (x)dx m , 2m

h (x1 ) = 0

x f (x)dx = 2m2
32

2 f (x1 )

Zx1

f (t) dt 2g (x1 )

f (x1 )

Zx1

Zx1

Rx1

g (t) dt = 0

f (x1 ) a

=
g (x1 ) Rx1

f (t) dt = g (x1 )

Zx1

g (t) dt .

g (x1 )

Zx1

Zx1

a
Rx1
a

g (t) dt
f (t) dt

Cauchy
( )

Ra

a
Ra

g (t) dt
.

f (t) dt

x2 (a, x1 ) :

Rx1

a
Rx1

g (t) dt > 0.

g (t) dt
=
f (t) dt

g (x1 ) , 0

f (t) dt

Rx1

f (t) dt > 0, f (x1 ) > 0

Zx1

a
x
Zx

f (t) dt
f (t) dt = f (x) > 0,

[a, b].

g (t) dt

g (t) dt , 0.

33

a
Rx1
a

g (t) dt
f (t) dt

Ra

a
Ra

g (t) dt
=
f (t) dt

g (x2 )
f (x2 )

f (x1 ) g (x2 )
=
f (x1 ) f (x2 ) = g (x1 ) g (x2 )
g (x1 )
f (x2 )

x2 < x1 .

Rx1

33 ( )
a, b, c R,
:
(a + b + c)2 9bc, (a + b + c)2 9ca

34 (

x + y = y + 2z = 5,
:
A = 2 x + 3y + 2z

(a + b + c)2 9ab .

http://www.mathematica.gr/forum/viewtopic.php?f=69&t=31590
http://www.mathematica.gr/forum/viewtopic.php?f=69&p=151590

1 (dr.tasos) ,
: (a + b + c)2 < 9bc (1)

1 (

)
A = 4z + 3y + 2z ( x + y = y + 2z
x = 2z)
A = 6z + 3y y + 2z = 5 3
A = 15
2 (T-Rex)
: y + 2z = 5 x + y = 5. :
A = 2 x + 3y + 2z = 2 x + 2y + y + 2z = (2 x + 2y) + (y + 2z) =
2(x + y) + (y + 2z) = 2 5 + 5 = 10 + 5 = 15

(a + b + c)2 < 9ac (2) , (a + b + c)2 < 9ab (3)



: a2 + b2 + c2 < ab + ac + bc
: a2 + b2 + c2 ab + ac + bc

, . , -

34

,
,


x y = 2 y {0, 2}

35 ( )
Z2 x3 y3 = 2 xy + 8.

(x, y) = (2, 0) , (0, 2)

http://www.mathematica.gr/forum/viewtopic.php?f=109&t=34871

36 (q )

( )

x +
2

x , y ( x = 4)
y = 0 x = 2

1 + {x} = 1

http://www.mathematica.gr/forum/viewtopic.php?f=109&t=28027

( )

y = 1, x3 2 x 9 = 0
.

x < 1, [x] 2

y = 1, x3 + 2 x 7 = 0
.

y , 0, 1 2 <

[x] +

2 xy + 8

x2

+ xy +

y2

1 + {x} < 2

1 + {x} < 0, , x 1.

[x] +

<4

[x] +

1 + {x} = 1 x,

1 x 0, x 1,

(x + y)2 + 4 > 0.

1 x 1 .

2 x2 + yx + 2y2 4 > 0

1 x < 0 [x] = 1

= 32 15y < 0 y , 0, 1.

1 +

2 < x y < 4.

1 + {x} = 2 {x}.

qp

1 + {x} 1 <

2 1 < 1

2 {x} > 1, .

,
x y .

0 < x < 1, [x] = 0,

, x, y
,
x3 y3 .

x+

35

qp

1 + {x} = 1

qp

1 + {x} = 1 x,

qp

x = 1

1 + {x} 1 0 < 1 x < 1.

x = 0 .

36

37 ( )
ABCD a. BC
E , BE =


3
a
DC F , CF = .
2
G AE BF , G
.

2( )
ABGC
= CAE.

, CBF
AKLN, KRML, RBE M .
+ AEN
= 45o .
AMN
CBF AMN
= AMN
.
CBF

= 45o

CBF + EAB
o
= 45 EAB
= CAE
.
CBF

http://www.mathematica.gr/forum/viewtopic.php?f=110&t=31734

1 ( KARKAR) AE
1
3

y = x + a , BF y = 2 x + 3a .
G

6a 3a
,
,
5 5

3( ) Q AG DC

AB k CQ ABE QCE

a
a2
a
(x )2 + (y )2 =
2

37

CF= a2
AB
BE BE= a3 EC= 23a 1 AB=a
=
=
CQ = 2a
CQ EC
2
ABkDQ
3a
FQ =
= DF F DQ
2
(AB, AF, AQ, AD) (B, K, L, D)
( B, K, L, D BD)
(FB, FK, FL, FD)
(B, H, Z, C)
( B, H, Z, C BC ). :

HZ
BC
BZ
BZ
: (1)
=

=
BC HC
HZ HC
AB k CF ABH FCH

38 ( ) O
(C)
ABC(AB , AC) . H ABC . AM
(C) N . AM
(C) P .
AP, BC OH AH = HN .

AB
BH CF= a2 = AB2 BH
a (1) BZ
=
= 2 HC =
=3

CF HC
HC
3
HZ
BZ
3

=
HZ + BZ 4
2a
[ 900
FCZ=
3 BH= 3
a
a
BZ
=
BZ = ZC = = CF
BH 4
2
2
[

45
[ = 450 LDF=
[ = 900 FLDB : (2)
LFD
DLF
AB k DF
AB

BG

AB DF=FQ AB ABkFQ FQ = GF
AK
=
=

KF DF
FQ
MGKBC
BG oo .
AK

KG k DF

=
KF GF
BM
2
BK
AB
a
=
=
= 3a =
MC KC DF
3
2
2 BM+MC=BC=a
BM
=

BM + MC 5
BM =
ME =

2a
2a a
: (3) & ME = BM BE =

5
5
3

a
15

: (4)

http://www.mathematica.gr/forum/viewtopic.php?f=110&t=34943

(B, K, L, D) (GB, GK, GL, GD)



B, M, E, Z ( Z GD AC )
:

1 ( )
.
:

ME
BZ
BE (4) 3
BE
= a =5
=

BZ
MZ
MZ
ME
15
(3)
a
BZ

= 5 BZ = = BZ Z Z
BZ BM
2

D, Z, G .
BDF FLBD, BCDF Z
( ) DZG

ABC (AB , AC) M


BC . P ( A)
(O) ABC
(K) AM , PM
H ABC


[ = 900 A, B, G, C, D
DGBF DGB

38

: OM, AP, BC ( T ) , AHT M ( H


ABC )
MHPAT ( AM (K))
H AT M (
)
T HO , T HOAN O
ABC
AN (O) T HO
AN HA = HN .
2 ( )
,

H PM AD (ADBC)

E PM (O).
MPA

900

EPA = 900 AE (O) ECAC : (1).

H1 MP AH AK = KH1 ,
KH1 MO

OMBC ( BC (O))
ADBC

OM
HE

M o BC

AD

AHE ( O o AE)

.

AH1 = 2KH1 = 2OM = AH H H1 T OAN

BHCE (

HA = HN.

AH = HN T
(1)

ADBC

OH, CB.

BH k EC BHAC H ABC .

AT M MH ,
P MH AT MPA = 90 .


P, H, M .

AH = HN

OA=ON=RO

AHNO OHAN ( ) OHAM : (1).


APMHP ( AM
(K) MBAH ( H
ABC ,
AP, OH, CB
AH M T .
39

, ,

: -

39 ( ) x, y, z > 0 xy + yz + zx = 1,

- ,

( x + y + z) (x + y)(y + z)(z + x) 2 2.

X
X
(a3 b + ab3 ) 2
a2 b2 = 2.
40 ( )
100.000.000 Fibonacci
( ).

http://www.mathematica.gr/forum/viewtopic.php?f=111&t=33945

( ) (x, y, z) 7 (a2 , b2 , c2 )

(a + b + c) (a2 + b2 )(b2 + c2 )(c2 + a2 ) 2 2

http://www.mathematica.gr/forum/viewtopic.php?f=111&t=23550

( )
Fibonacci F0 = 0, F1 = 1, F2 = 1.

(a + b + c) 2(a2 + b2 )(b2 + c2 )(c2 + a2 ) 4



(a + b + c)(

fn = Fn

[(a + b)2 + (a b)2 ][c2 (a b)2 + (ab + c2 )2 ] 4.

An = ( fn , fn+1 ) n 0.
{ fn }n0
[0, 9999], {An }n0 108
, m, n 108 .
Am = An , Am1 = An1
{An }n0 A0 = (0, 1).

{Fn }n0
104 .

Cauchy-Schwarz,
q

[(a + b)2 + (a b)2 ][c2 (a b)2 + (ab + c2 )2 ]

mod 10000

ab(a + b) 2abc.

(a + b + c)[(ab(a + b) + bc(b + c) + ca(c + a) 2abc] 4


a3 b + ab3 + b3 c + bc3 + c3 a + ca3 2.

40

:
(OP)(OM) = (OB)(OB) = (OA)(OA ) = (OS )(OQ)
= (OP)(OM) = (OS )(OQ) , (1)
(1), S PQM
(T )
PMQ = PS Q = 90o = QM A A , (2)
(L ), (2) MA = MA , S Q BB A S A

41 ( KARKAR.)
AA , BB, ABA B
O M, S , . AA BMB,
BB AS A ,
O AA , BB S , M , AA BMB BB
AS A , A, B, A , B ,
.

.
ABA B S PQM .
3 ( .) BM
B M AA ,
(K) ABA B B ,

http://www.mathematica.gr/forum/viewtopic.php?f=112&t=33906

1 ( .) K,
(K) ABA B K M AA
K M
M BMB , P BB K M O,
B, B .
, AA P (K),
PK AA .
Q,
(K) B, B BB
Q (K)
P, Q
AA , P (K).

, Q, O
A, A QS S O ( K, S , Q QK AA ),
S O BB AS A
.
, .
2 ( .) (L), (L ),
B MB, A S A
P, Q, A A, B B,
.
(L), MP BMB S BB,

B , N AA .
, BO B A BA , T,
(K), A, A .

, ABA B
, S T BB, S
BB, A S A
.
( KS S T S BB (L), AKA T
KT, K (K),
A S T = A KT = AKT = AS T

).
,
.
4 ( .) A MB =
A MB , (1)
B,
AA , (K)
ABA B Q.
QBA A
K M
AA , K (K),
A MB = AMQ AMQ = A MB , (2)
(1)
(2) B , M, Q
.
AA k QB MA = MA , Q. BA B A

PS BB

O (L), (K), (L ), (K)


ABA B , :
41

ABA B .

O1 O2 AG
O4 O5 GD, O1 O2 O4 O5 AD
.
2
, O3 O4 BG
O2 O6 GE , O3 O4
O2 O6
.
BE
2

KO4 LO2 , :

BB,

,
B A BA
A S A, S
BB .
, .
.
, ,
:
A MB = A MB
, (1)

A S B = AS B , (2), M, S
AA , BB K, .
K KB = KB,
(K), AA
X, X , A, A , ,

(KO4 LO2 ) =

AD
2

KO2 =

KO4 AD
=
KO2
BE

BE
2

KO4

, (1)

O1 KO3 = BGD, KO1 O3 =


AGF KO3 O1 = BGF ( ).
M, AD C BE .
, KO1 O3 MGC ( ).
, MGC AD, BE
2
CF , .
3
, MCD BGD
2
( -- ), MG = 2GD = AD
3
2
MC = BG = BE .
3
, :

AA .
(1), , BB
X S X (2)
X S A = XS A , (3), , (2), (3) ,
BS B , K M, K M
AA .
42 ( .) ABC AD, BE, CF
G.

O1 , O2 , O3 , O4 , O5 , O6 , AFG,
AGE, BGF, BGD, CGD, CGE ,
.

http://www.mathematica.gr/forum/viewtopic.php?f=112&t=34427

KO1
MG AD
, (2)
=
=
KO3
MC
BE
(1) (2) :
KO1
KO4
=
KO3 KO2
, (KO1 )(KO2 ) = (KO3 )(KO4 ) , (3)
(3) O1 , O2 , O3 , O4

( .) K , O1 O2 O3 O4 L,
O4 O5 O2 O6 .

.
, O1 , O3 , O4 , O6
O3 , O4 , O5 , O6
.

42

x O x
, 1
A2 (2 + 1)A + 2 I = O.

43 ( )

4
|a 3 b 5| >
4a + 5b
a, b.

http://www.mathematica.gr/forum/viewtopic.php?p=157900
http://www.mathematica.gr/forum/viewtopic.php?p=161557

( ) A2 = 3A 2I A3 = 3A2 2A = 9A 6I 2A = 7A 6I
A4 = 7A2 6A = 21A 14I 6A = 15A 14I .
, A = (2 1)A (2 2)I

( ) 3a2 5b2 .
r
5
2
2
. 3a 5b , 0
3
3a2 5b2 , 1
3a2 = 5b2 1. 0, 1 4 mod 5,
0, 3 2 mod 5.
1 mod 5,
.
|3a2 5b2 | 2.

|a 3 b 5| =
>

A A = (2 1)A2 (2 2)A

= (2 1)(3A 2I) (2 2)A

= 3(2 1)A 2(2 1)I (2 2)A

= (2+1 1)A (2+1 2)I


A+1 = (2+1 1)A (2+1 2)I ,
A = (2 1)A (2 2)I, 2.

2
|3a2 5b2 |

a 3+b 5 a 3+b 5
2
2a +

5
b
2

4
4a + 5b

2 =

A2 (2 + 1)A + 2 I = (( 1)A ( 2)I)2


( + 1) (( 1)A ( 2)I) + I = ( 1)2 A2

.
: 2

2( 1)( 2)A + ( 2)2 I

a, b N |3a 5b | = 2.
, a = 9, b = 7 |3a2 5b2 | =
|3 81 5 49| = | 2| = 2.

(2 1)A + ( + 1)( 2)I + I

= ( 1)2 A2 3(2 2 + 1)A




+ 2( 1)2 I = ( 1)2 A2 3A + 2I = O

44 ( parmenides51) x
A A2 3A + 2I = O I

43

45 ( Putnam 2012)
S : S S S , :

http://www.mathematica.gr/forum/viewtopic.php?f=59&t=32954

1 ( )
x, y S , z S x z = y.

P1 (x) = P( x) = x2 1

a c = b c = a = b a, b, c S .

Pn+1 (x) = P1 (Pn (x))


n.

P1 (x) > 1

http://www.mathematica.gr/forum/viewtopic.php?f=59&t=33418

( )
a S . e S a e = a.
e a = a.
c S . b S
c = b a.

Pn (x) = P2n1 (x) 1 > 1


n N n > 2 x R.
1: n y (0, +), Pn (x) = y
.

c e = (b a) e = b (a e) = b a = c

: . n = 1,

e c = c.

S ,
e.
x S . x S x x = e
x x = e. x
x (
).

o
n p
p
P1 (x) = y x2 1 = y x y + 1, y + 1 .


n. ,

Pn+1 (x) = y P2n (x) = y + 1


n p
o
p
Pn (x) y + 1, y + 1
p
Pn (x) = y + 1,

a b = a c a (a b) = a (a c)

(a a) b = (a a) c

eb=ec

y + 1 < 1 y > 0.
,
.

b=c

2: n,
Pn (x) = 0 n + 1 .

.
46 ( IMC 2004) P(x) = x2 1.

P(P( P(x)) )) = 0.

| {z }

: . n = 1 n = 2

P1 (x) = 0 x2 1 = 0 x {1, 1}

2004

44

() Pn (x) = 1 P2n1 (x) 1 = 1 Pn1 (x) = 0


n ,
.

n

2
o
P2 (x) = 0 x2 1 = 1 x 2, 0, 2 .

() Pn (x) = 1 P2n1 (x) = 2 Pn1 (x)

n o

2, 2 Pn1 (x) = 2


n 1. ,

, 1.

Pn+1 (x) = 0 P2n (x) = 1

, Pn+1 (x) = 0 n + 2 , .
, P2004 (x) = 0
2005 .

(Pn (x) 1) (Pn (x) + 1) = 0.

45

47 ( )
K 2.
f : K K 1-1

48 ( )
A, An , n N, m m .
An A,

(An ) (A)

() f (x + y) = f (x) + f (y) x, y K

: > 0, N N,
n > N

() f (x) f (x1 ) = 1 x K {0}.


f f K .

(An )

(B)
B, .

( )
f 2 (1) = 1. f (1) = 1, f
f .
, f (1) = 1. x
0, 1.
1

f (x2

http://www.mathematica.gr/forum/viewtopic.php?f=10&t=33208

= f 2
+ f (x)
+ x)
x +x
!
1
1
1
= f

=
x x+1
f (x)
=

( )
.
0 > 0 nk
Ank (nk )

f (x)+1

f 2 (x)

|(nk ) i | > 0

+ f (x)

(1)

i {1, . . . , m}.
|(nk )| |(nk )| 6 kAnk k
Ank .
Bolzano-Weierstrass , (nk p ) 0 .

f 2 (x) = f (x2 )

i=1


z C : |z i | < ,

(A) = {1 , . . . , m }.

http://www.mathematica.gr/forum/viewtopic.php?f=10&t=33892

f (x2 )

m
[


(1)

(1) x = 0 x = 1.
x, y K

f ((x + y)2 ) = f 2 (x + y)

det(Ank p (nk p )I) = 0.

2 f (xy) = 2 f (x) f (y)


(). 2, f (xy) = f (x) f (y).
f K .
: f . .. K = Z3 (t)
( Z3 )
f (k) = k3 .

det(A 0 I) = 0.
0 A.
(1) |0 i | > 0 i {1, . . . , m}
.
.

46

: -

49 ( china university)
:

x ln sin x ln cos x dx =

( ln 2)2
8

4
192

, f (z) = Logn (z) D = {z C : |z 1| < 1} (


Re (1 + z) > 0 ),
Gauss1

http://www.mathematica.gr/forum/viewtopic.php?f=9&t=31631

( )
1:

Z/2

1.

ln (sin 2 x) dx =

Z2 

f 1 + eix dx

1
f (1) =
2

f (1) = 0 ,

Z/2

ln (sin x) dx =

Z/2

Z2

ln (cos x) dx,

Logn (1 + cos x + i sin x) dx = 0,

Z/2
Z/2
2
ln2 (sin x) dx.
2.
ln (sin 2 x) dx =

, 1.:

Z/2

2 x=y

ln (sin 2 x) dx =

1
2

0=

ln (sin y) dy

y=x

1
2

/2

ln (sin x) dx +

1
2

ix/2

Log 2 cos e
2

y=x/2

dy =

y=x

Z/2

ln (sin x) dx

/2

Z/2

ix

Log 2 cos x e

Z/2


dx + Logn 2 cos x ei(x) dx
0

x/2x

Z/2
ln (cos x) dx,

Z/2
Z/2
n
(ln cos x + ln 2 + ix) dx+ (ln cos x + ln 2 ix)n dx = 0,

2: n = 1, 2, 3, ..

Z/2
Z/2
n
(ln cos x + ln 2 + ix) dx + (ln cos x + ln 2 ix)n dx = 0.
1

2. .



Logn 2 cos yeiy dy

Z/2
Z/2




n
ix
=
Log 2 cos x e dx + Logn 2 cos x eix dx

Z/2
Z/2
ln (sin 2 x) dx =
ln (sin x) dx

Z
0

Z/2
Z




n
ix
=
Log 2 cos x e dx + Logn 2 cos y eiy dy

Z
Z/2
1
ln (sin y) dy
ln (sin x) dx +
Z/2

1
=
2

Z2

( ).
:

http://mathworld.wolfram.com/GausssMean-ValueTheorem.html

47

Z/2

I=

Z/2
ln 2
ln sin x dx =
2

x ln (sin x) ln (cos x) dx

2 , n = 2

x=/2y

Z/2


y ln (cos y) ln (sin y) dy

Z/2
Z/2
2
(ln cos x + ln 2 + ix) dx+ (ln cos x + ln 2 ix)2 dx = 0
0

Z/2

ln (cos y) ln (sin y) dy I
=

Z/2

I=

Z/2

Z/2

ln (sin x) ln (cos x) dx


ln2 cos x + ln2 2 + 2 ln cos x ln 2 x2 dx = 0

Z/2
Z/2
3
ln2 2
2
+ 2 ln 2 ln cos x dx
=0
ln cos xdx +

Z/2


(ln (sin x) + ln (cos x))2 ln2 (sin x) ln2 (cos x) dx
=

Z/2
3
ln2 2
+ ln2 2 +

ln2 cos xdx =

!!2
Z/2

sin 2 x

2
2ln (sin x) dx
ln

.1

Z/2
ln2 2 3
ln2 sin x dx =
+


(ln (sin 2 x) ln 2)2 2ln2 (sin x) dx

.1

I=

2ln2 (sin x))dx

16

2 ln2 2
16

ln 2
8

2 ln2 2
16

ln 2

Z/2
Z/2

ln (sin x) dx
ln2 (sin x) dx.
0

2 , n = 1

ln 2
4

192

ln 2
2

ln2 2
8

3
24

( ) 1:
C f : C , .
n g : C , f (z) = gn (z)
z . , f (z) , 0 z .

http://mathematica.gr/forum/viewtopic.php?f=9&t=32640

Z/2
(ln cos x + ln 2) dx = 0
Z/2

Z/2
Z/2

ln (sin x) dx
ln2 (sin x) dx

Z/2
Z/2
(ln cos x + ln 2 + ix) dx+ (ln cos x + ln 2 ix) dx = 0
0

ln 2

50 ( .. ) f :
C ,
. f n
n , f -

I=

2 ln2 2


ln 2 2 ln 2 ln (sin x) ln (sin x) dx
2

24

Z/2
(ln2 (sin 2 x) + ln2 2 2 ln 2 ln(sin 2 x)
Z/2

24

Z/2

24

.1


(ln cos x + ln 2)2 x2 dx = 0

Z/2
ln cos x dx = ln 2dx
0

48

f
. , ,
f
Z
f (z)
dz = 0

f (z)

: z0 .
ordz0 ( f ) f z0 . f ,
ordz0 ( f ) N. n :
n

ordz0 ( f ) = ordz0 g

.
,
,

: [a, b] . : [a, b] C

= n ordz0 (g) .

ordz0 ( f ) n, n.

, ordz0 ( f ) = 0, f (z0 ) , 0.

f ( (s))

(t) = exp
(s) ds

f ( (s))

2:
C
f : C , f (z) , 0
z . , :
(1) g : C ,
f (z) = eg(z) z .
(2)

f
f

t [a, b]. ,

(t) = (t)

( f ) (t)
(t)

( f ) (t)
( f ) (t)

=0

t [a, b],

g(z)

: (1) (2): f (z) = e ,


f (z)
f
= g (z) g .
f (z)
f
f
(2) (1): F
.
f
h (z) = f (z) eF(z)
h (z) = 0 z . , a C
, h (z) = a, f (z) = aeF(z)
z . a = eb g (z) = b + F (z), f (z) = eg(z) z .

f (z)

exp
dz = (b) = (a) = 1
f (z)

f (z)
dz 2iZ.
f (z)

, n

C f : C
, . n g : C , f (z) = gn (z)
z . 1,
f (z) , 0 z . 2,

1
2i

f (z)
1
dz = n
f (z)
2i

g (z)
dz
g (z)

n.
Z

f (z)
dz = 0 .
f (z)

49

!
!
1 1
1
1
1 1
& C , ,
A (0, 0, 0) , B , ,
3 3 3 3
3 3 3 3

51 ( )
~r (t) = (t, t2 , t3 )
x + z = 1.

52 ( Jeronymo Simonstone) ,


, .

http://www.mathematica.gr/forum/viewtopic.php?f=11&t=3106

( ) ~r (t) = t, t2 , t3



~r (t) = 1, 2t, 3t2 ~t =

~r (t) = (0, 2, 6t) .

1 + 4t2 + 9t4

1, 2t, 3t2

http://www.mathematica.gr/forum/viewtopic.php?f=11&t=17664

( ) : .
1
2
~b =

3
t
,

3
t,
1
.
=
..
=

~r ~r
= 0 . p 1 + 9t 2 + 9t 4
,

~
N
p . ~
~ = ~b ~t = .. =
p ,
 
 
 

~ N
1

9t3 + 2t , 9t4 1 , 6t3 + 3t


2
4
2
4
(1+9t +9t )(1+4t +9t )
, kn = 0 ,
p : x + z = 1
.
~a = (1, 0, 1) BeltramiEnneper
p ~x (x, y) = (x, y, 1 x)
~x x = (1, 0, 1) & ~xy = (0, 1, 0) , 2 = K , K
p
Gauss .

~a = ~xx ~xy = .. = (1, 0, 1) .
K = 0 3
~ k p ~~a ~ ~a = 0 .. 3t t = 0
,
K , L , M , 1
1
t=0,t= &t= ,
.
3
3

~r

~r

50

z, x, y, w m .

53 ( dimtsig)

.

x = mz, y = m2 z, w = m3 z
:

http://www.mathematica.gr/forum/viewtopic.php?f=63&t=22945

( )
1 .
m

m3 z z = 20141 m3 =

20141 + z

,
m .
z
20141 + z . z = k3
20141 + k3 = n3 .

111...1 = m , n > 1.
|{z}
n

, m
1 9.
m

n3 k3 = 20141 (n k)(n2 + nk + k2 ) = 11.1831

m = 10a 1, a N .

n k = 11, n2 + nk + k2 = 1831

n k = 1, n2 + nk + k2 = 20141

111...1 = 100a 20a + 1,


|{z}

n k = 1831, n + nk + k = 11

n k = 20141, n2 + nk + k2 = 1

111...1 0 = 100a 20a,


|{z}

1 : n k = 11, n2 + nk + k2 = 1831.
n = k + 11

n1

111...1 = 10a2 2a.


|{z}
n1

(k + 11)2 + k(k + 11) + k2 = 1831 k = 19.

, ,
.

z = 193 = 6859 m3 =

54 ( )
, , . 20.141,
.

m =

30

19

27000
.
193

30
193 = 10830,
19
30
y = ( )2 193 = 17100,
19
30 3
w = ( ) 193 = 27000
19

x=

http://www.mathematica.gr/forum/viewtopic.php?f=63&t=24006

( ) x, y, z
x, y, w .

y
z
x
= = y2 = xw, x2 = yz.
y w x

51


- - , , , v
, :

55 ( )


.

sin = 0, 09 20m .
,
44m .
,
, .
, 100Kgr, 20Kgr, 60Kgr.

9 s.

. 80m

.

250N .

. ( g = 10m/s2 )

(M + )u = (M + + m)v v = 4m/s.
9 s 9v = 36m < 44m,
.
,
v

F
= 2, 5m/s2 .
M
1
, x = vt at2 . 2
t = 0 s ( )
2v
= 3, 2 s.
t=
a
2, 5 s , a=

,
1
y |y| = gt2 = 51, 2m < 80m,
2
, .
56 ( )
u :
R R R, :

http:// http://www.mathematica.gr/forum/viewtopic.php?f=47&t=32440

a) ux = 0,

b) ux = x,

c) uy = x,

d) uy = x + y

e) ux = e + y.

( )

x = 20m, s = 44m, M = 100Kgr,


http://www.mathematica.gr/forum/viewtopic.php?f=47&t=17633

= 20Kgr, m = 60Kgr, t1 = 9 s F = 250N.



, (M + )gxsin, -, 1
(M + )u2 , ,
2
u .
: u = 2gxsin = 6m/s.

(solars)

a) u (x, y) = f (y) .
x2
b) u(x, y) =
+ f (y).
2
c) u (x, y) = xy + g(x).
y2
+ g(x).
d) u(x, y) = xy +
2
x
e) u(x, y) = e + xy + f (y).

52

57 ( ) x, y, z ,

http://www.mathematica.gr/forum/viewtopic.php?f=13&t=14163

( )

.

||x + y|| + ||y + z|| + ||z + x|| ||x|| + ||y|| + ||z|| + ||x + y + z||.

A = {a1 , a2 , ..., an , ...}


am A.

http://www.mathematica.gr/forum/viewtopic.php?f=13&t=20760

( ) :

am = an1 .
A {a1 , ..., am } al = an2 .

(kx + y + zk + kxk + kyk + kzk kx + yk ky + zk kz + xk)


(kx + y + zk + kxk + kyk + kzk) =


k ar = ank

(kyk + kzk ky + zk) (kxk ky + zk + kx + y + zk) +

k + 1
A {a1 , ..., ar }

(kzk + kxk kz + xk) (kyk kz + xk + kx + y + zk) +


(kxk + kyk kx + yk) (kzk kx + yk + kx + y + zk) 0


ank ,
.

58 ( ) (n )nN .
(nk )kN

ni < n j i < j.

53

59 ( )
R a, , .
.

60 ( )
f, g : [a, b] R. f f (x) + f (x) g (x) + x =
f (x) + g (x) , x [a, b] f (a) = a, f (b) = b,
f .

http://www.mathematica.gr/forum/viewtopic.php?p=156229
http://www.mathematica.gr/forum/viewtopic.php?p=159292

(
)

:
S = 2 + 2 + 2.

, : 2 + 2 + 2 = 4R2 .

2 xy x2 + y2


: S 2 2 + 2 + 2 = 8R2 ,
= = .
,
R
2 3
R.
=
3

( )
. h(x) = f (x) x,

h (x) + h (x)g(x) = h(x) (1)


h(a) = h(b) = 0. h 0.
. h [a, b] x0 (a, b) .
h (x0 ) = 0, h (x0 ) 0. (1) h(x0 ) 0, . ,

x1 (a, b).

54

61 (
)
1

(an )

62 ( )
f :
R R f (x + y) = f (x + f (y)),

, |am+n am an | <
m+n
m, n. .

x, y R.

http://mathematica.gr/forum/viewtopic.php?f=61&t=30772
http://mathematica.gr/forum/viewtopic.php?f=61&t=28384

1 ( )
1 f : R R ,

( ) :

|am+n am an | <

m+n

T 1 , T 2 ,

: (1)

m, n. (1)
n N

T1
R \ Q, f T2

: A = {a + b

T1
/a, b Z}
T2

R (Kronecker), -

lim (am+n am an ) = 0,

x an + bn

m+

A lim(an + bn

lim (am+n am ) = an : (2) .

T1
T2

T1
x
) = .
T2
T2

f (0) = f (an T 2 + bn T 1 )

m+

f (0) = lim f (an T 2 + bn T 1 ) =

lim (am+1 am ) = a1

m+

= lim f (T 2 (an + bn

( n = 1),

T1
x
) = f (T 2 ) = f (x)
T2
T2

1 .
2 a f (a) ,
.
: (1) x
x f (a), y = a y a,

lim (am+2 am+1 ) = a1

m+

.......................
lim (am+n am+n1 ) = a1 .

m+

f (x a + f (a)) = f (x f (a) + f (a)) = f (x)


an = lim (am+n am ) =

2 .
:
.
. f
a a f (a) , 0,
a f (a) (
2). x f (x)
f .

m+

= lim

m+

n
X
k=1

(am+k am+k1 ) =

= a1 + a1 + ... + a1 = na1 , n N .

n N

an+1 = (n + 1) a1 = na1 + a1 = an + a1 .

1 g(x) =

{an } a1 .

x f (x)
, x R
a f (a)

, ,
55

[0, T ] ,
f .
(2) x = 0 y R

f (x) = x + c, x R. (1) c = 0, .

.

f ( f (y) y) = f (0) f (g (y)) = f (0) : (3) ,

http://www.mathematica.gr/forum/viewtopic.php?f=61&t=30088

g (y) = f (y) y y R.
g R , f ,

2 ( )
f

lim g (y) = lim ( f (y) y) =

f (x + y) = f (x + f (y)) : (1) .

y+

y+

(1) x x y
x, y R

lim g (y) = lim ( f (y) y) = +,

g (R) = R.
x R. x g (R) = R,
y R , g (y) = x (3)
f (x) = f (0) , x R, .

f (x) = f (x + f (y) y) : (2) .


:
1) f (y) y = 0, y R, f (y) = y, y R,
.
2) y0 R , f (y0 ) y0 , 0.
(2) y = y0 x R
f (x) = f (x + f (y0 ) y0 ), f
T = | f (y0 ) y0 | > 0. f f (R) = f ([0, T ])

1) f (x) = x, x R,
2) f (x) = c, x R,
c .

56

E, F, G . - UT, VW .
( FI , ,
UT ), K, P I, N . PK
VW B.

63 ( .-. ) , , ,
.

http://www.mathematica.gr/forum/viewtopic.php?f=62&t=32374

1 ( )

EF, FG, E, F, G ( ) AB = BC = CD,
S .
T, S F
B, C, S H, S I , T
S I, S B, H, I, EF, FG,
.
BC F
FB = FC, BTCS .
FB = BE FC = CG.
BE, CG,
E, G , A, D EA = EB GD = GC
.

64 ( ) , , A ,

3A = 3A 43 A.

http://www.mathematica.gr/forum/viewtopic.php?f=62&t=33367

.
2 ( ).

( )
57

AB
B = E A + E A.
E :
M EZ

E
Z

E = 2M M .
E .
Z
Z 2 = 2Z 2 2Z.E E = 1 2M 2

= 2A
= 2,
B

A AB, Z = = 1, M Z, E B,
A = M, A = M = MZ.
3 =

AB
B

E A + E A =
2M M A + (1 2M 2 ) A =

3 43 = 3A 4A3 .

2A (1 A2 ) + A 2A3 = = 3A 4A3 .

E A + E A = AE = AE

AEB B

AB
B

AE

AE =

AB
B ,

AB
B

= E A + E A = 3A 4A3

3 = 3 43 .

58

65 ( .. )
()

n
X

1
1 1
1, , , . . . , , . . . ,
2 3
n
() .

k3 =

k=1

n
 n(n + 1) 2 X
n(n + 1)(2n + 1)
,
,
k2 =

n
X

k=

n(n + 1)
2

k=1

http://www.mathematica.gr/forum/viewtopic.php?f=60&t=35164

a
( )

N N 1
2 1
,
,..., ,
N! N!
N! N!

n +
b

1
1 1
1, , , . . . , , . . .
2 3
n

c
2

n +

!
+ d n = n4 .

b
2

c
2

n2 +

f (x) = 4 x3 6 x2 + 4 x 1.

2 ( )

66 ( apotin)
a, b, c d

P (1) + P (2) + P (3) + + P (n) = n4


P (x) = ax3 + bx2 + cx + d

P (1) + P (2) + P (3) + + P (n 1) = (n 1)4

P (1) + P (2) + P (3) + + P (n) = n4 , n N

P(n) = n4 (n 1)4

http://www.mathematica.gr/forum/viewtopic.php?f=60&t=33227

an3 + bn2 + cn + d = 4n3 6n2 + 4n 1

1 ( )

k=1


a = 4, b = 6, c = 4, d = 1.

N ,

n
X

k=1

k +b

n
X
k=1

k +c

n
X
k=1

k+

n
X

( n, )
a = 4, b = 6, c = 4, d = 1.

=n

k=1

59

You might also like